You are on page 1of 280

Dr.

Sanjay Kumar

Electric Field and Potential

by
Dr. Sanjay Kumar
M.Tech, PhD

Managing Director: Quanta Classes Lucknow, Mo. 9453763058


Ex. Sr. Faculty of Physics: Imagine Point Kanpur, Jain Classes Jhansi, Bansal Classes
MP, TATA Aarambh Engineering and Medical Simplified Lucknow.

February 2, 2022
ii

SK Education
Quanta Classes: K 423 A Sector K Ashiyana Colony Lucknow (UP)
JEE (Main & Advanced) / SAT / NEET / Foundation
Mo. +919453763058

Email: spphysicsworld@gmail.com
Concepts and Problems in Physics

First edition 2022

© 2022, Sanjay Kumar


All rights reserved.
No part of this book may be reproduced or distributed in any form or by any means, electronic, mechanical,
photocopying, recording, or otherwise or stored in a database or retrieval system without the prior written
permission of the author.
iii

Dedicated to
My Mother Late Smt. Karuna Devi
and
My Father Late Shri RB Pandey.
This page intentionally left blank
Preface

This physics book is the product of more than sixteen years of teaching and innovation experience in physics for JEE main
and Advanced aspirants. Our main goals in writing this book are-

• to present the basic concepts and principles of physics that students need to know for JEE-advanced and other related
competitive exams.
• to provide a balance of quantitative reasoning and conceptual understanding, with special attention to concepts that have
been causing difficulties to student in understanding the concepts.
• to develop students problem-solving skills and confidence in a systematic manner.
• to motivate students by integrating real-world examples that build upon their everyday experiences.
What’s New?
Lots! Much is new and unseen before. Here are the big five:
1. 1 Every concept is given in student friendly language with various category based solved problems. The solution is provided
with problem solving approach and discussion.
2. Checkpoint questions with answer keys and solutions have been added to applicable sections of the text to allow students
to pause and test their understanding of the concept explored within the current section.
3. Special attention is given to all tricky topics (like- Coulomb’s law in a medium partially filled with a dielectric material
slab, Vector form of Coulomb’s Law, Distribution of charges on metal surfaces, Electric field from electric potential, Electric
potential from electric field, Image method etc.) so that students can easily solve them with fun.
4. At the end of the theory part, there are miscellaneous solved examples which involve the application of multiple concepts
of this chapter.
5. To test the understanding level of students, multiple choice questions, conceptual questions, practice problems with previous
years JEE Main and Advanced problems are provided at the end of the whole discussion. Number of dots indicates level
of problem difficulty. Straightforward problems (basic level) are indicated by single dot (•), intermediate problems (JEE
mains level) are indicated by double dots (••), whereas challenging problems (advanced level) are indicated by thee dots
(• • •). Answer keys with hints and solutions are provided at the end of the chapter.
We have kept these goals in mind while developing the main themes of our physics book.
Dr. Sanjay Pandey

v
vi

Online Physics Classes


by
Dr. Sanjay Kumar
JEE (Main & Advanced) / SAT / NEET / International Physics Olympiad /
Foundation (IX - XII)
Quanta Classes: K 423 A Sector K Ashiyana Colony Kanpur Road Lucknow
Mo. +919453763058
Email: spphysicsworld@gmail.com
Contents

Preface vii

1 Electric Charge and Field 1


1.1 Introduction . . . . . . . . . . . . . . . . . . . . . . . . . . . . . . . . . . . . . . . . . . . . . . . . . . . . . . . . 1
1.2 Electric Charge . . . . . . . . . . . . . . . . . . . . . . . . . . . . . . . . . . . . . . . . . . . . . . . . . . . . . . 1
1.2.1 Electric Fluid Model . . . . . . . . . . . . . . . . . . . . . . . . . . . . . . . . . . . . . . . . . . . . . . 1
1.2.2 Properties of Electric Charge . . . . . . . . . . . . . . . . . . . . . . . . . . . . . . . . . . . . . . . . . . 3
1.3 Effect of Charge on the Mass of a Given Object . . . . . . . . . . . . . . . . . . . . . . . . . . . . . . . . . . . . 5
1.4 Check Point 1 . . . . . . . . . . . . . . . . . . . . . . . . . . . . . . . . . . . . . . . . . . . . . . . . . . . . . . . 5
1.5 Conductors and Insulators . . . . . . . . . . . . . . . . . . . . . . . . . . . . . . . . . . . . . . . . . . . . . . . . 7
1.5.1 Conductors . . . . . . . . . . . . . . . . . . . . . . . . . . . . . . . . . . . . . . . . . . . . . . . . . . . . 7
1.5.2 Insulators . . . . . . . . . . . . . . . . . . . . . . . . . . . . . . . . . . . . . . . . . . . . . . . . . . . . . 7
1.5.3 Semiconductors . . . . . . . . . . . . . . . . . . . . . . . . . . . . . . . . . . . . . . . . . . . . . . . . . . 7
1.5.4 Superconductors . . . . . . . . . . . . . . . . . . . . . . . . . . . . . . . . . . . . . . . . . . . . . . . . . 7
1.6 Methods of Charging . . . . . . . . . . . . . . . . . . . . . . . . . . . . . . . . . . . . . . . . . . . . . . . . . . . 7
1.6.1 Charging by Friction . . . . . . . . . . . . . . . . . . . . . . . . . . . . . . . . . . . . . . . . . . . . . . . 7
1.6.2 Grounding a Conductor . . . . . . . . . . . . . . . . . . . . . . . . . . . . . . . . . . . . . . . . . . . . . 8
1.6.3 Charging by Direct Contact . . . . . . . . . . . . . . . . . . . . . . . . . . . . . . . . . . . . . . . . . . . 9
1.6.4 Charging by Induction . . . . . . . . . . . . . . . . . . . . . . . . . . . . . . . . . . . . . . . . . . . . . . 9
1.6.5 Field emission . . . . . . . . . . . . . . . . . . . . . . . . . . . . . . . . . . . . . . . . . . . . . . . . . . . 10
1.6.6 Charging by Thermionic Emission . . . . . . . . . . . . . . . . . . . . . . . . . . . . . . . . . . . . . . . 10
1.6.7 Charging by Photoelectric Effect . . . . . . . . . . . . . . . . . . . . . . . . . . . . . . . . . . . . . . . . 10
1.7 Coulomb’s Law . . . . . . . . . . . . . . . . . . . . . . . . . . . . . . . . . . . . . . . . . . . . . . . . . . . . . . 11
1.7.1 Vector Form of Coulomb’s Law . . . . . . . . . . . . . . . . . . . . . . . . . . . . . . . . . . . . . . . . . 13
1.8 Equilibrium of Charged Particles . . . . . . . . . . . . . . . . . . . . . . . . . . . . . . . . . . . . . . . . . . . . 19
1.8.1 Equilibrium of Three Point Charges . . . . . . . . . . . . . . . . . . . . . . . . . . . . . . . . . . . . . . 19
1.8.2 Equilibrium of Symmetric Geometrical Point Charged System . . . . . . . . . . . . . . . . . . . . . . . . 20
1.8.3 Equilibrium of Suspended Point Charge System . . . . . . . . . . . . . . . . . . . . . . . . . . . . . . . . 22
1.9 Check Point 2 . . . . . . . . . . . . . . . . . . . . . . . . . . . . . . . . . . . . . . . . . . . . . . . . . . . . . . . 28
1.10 The electric field . . . . . . . . . . . . . . . . . . . . . . . . . . . . . . . . . . . . . . . . . . . . . . . . . . . . . 32
1.11 The Electric Field of Point Charges . . . . . . . . . . . . . . . . . . . . . . . . . . . . . . . . . . . . . . . . . . . 34
1.11.1 The Electric Field due to a Point Charge . . . . . . . . . . . . . . . . . . . . . . . . . . . . . . . . . . . 34
1.11.2 Electric Field due to a group of Point Charges (Superposition of Electrostatic Fields) . . . . . . . . . . 35
1.11.3 Electric Dipole . . . . . . . . . . . . . . . . . . . . . . . . . . . . . . . . . . . . . . . . . . . . . . . . . . 41
1.11.4 Dipole Moment of a System of Discrete Charges . . . . . . . . . . . . . . . . . . . . . . . . . . . . . . . 41
1.11.5 Induced Dipole Moment . . . . . . . . . . . . . . . . . . . . . . . . . . . . . . . . . . . . . . . . . . . . . 41
1.11.6 Electric Field at any Point due to an Electric Dipole . . . . . . . . . . . . . . . . . . . . . . . . . . . . . 41
1.12 Check Point 3 . . . . . . . . . . . . . . . . . . . . . . . . . . . . . . . . . . . . . . . . . . . . . . . . . . . . . . . 43
1.12.1 Electric Field of a Continuous Charge Distribution . . . . . . . . . . . . . . . . . . . . . . . . . . . . . . 45
1.12.2 Electric Field Due to Finite Rod at Perpendicular Distance x from the wire . . . . . . . . . . . . . . . . 47
1.12.3 Electric field at the center of a charged circular arc . . . . . . . . . . . . . . . . . . . . . . . . . . . . . . 52
1.12.4 Electric Field at Any Point on the Axis of a Thin Charged Ring . . . . . . . . . . . . . . . . . . . . . . 53
1.12.5 Electric Field on the Axis of a Charged Disc . . . . . . . . . . . . . . . . . . . . . . . . . . . . . . . . . 55
1.13 The Shape of Lightning rods . . . . . . . . . . . . . . . . . . . . . . . . . . . . . . . . . . . . . . . . . . . . . . 56
1.14 Check Point 4 . . . . . . . . . . . . . . . . . . . . . . . . . . . . . . . . . . . . . . . . . . . . . . . . . . . . . . . 59
1.15 Electric Field Lines . . . . . . . . . . . . . . . . . . . . . . . . . . . . . . . . . . . . . . . . . . . . . . . . . . . . 63

vii
viii CONTENTS

1.16 Theorem on Circulation of Vector E . . . . . . . . . . . . . . . . . . . . . . . . . . . . . . . . . . . . . . . . . . 64


1.16.1 Deduction of Pattern of Field Lines . . . . . . . . . . . . . . . . . . . . . . . . . . . . . . . . . . . . . . 65
1.16.2 Properties of Electric Field Lines . . . . . . . . . . . . . . . . . . . . . . . . . . . . . . . . . . . . . . . . 65
1.16.3 Check Point 5 . . . . . . . . . . . . . . . . . . . . . . . . . . . . . . . . . . . . . . . . . . . . . . . . . . . 66
1.17 Action of the Electric Field on Charges . . . . . . . . . . . . . . . . . . . . . . . . . . . . . . . . . . . . . . . . 68
1.17.1 Motion of a charged Particle in a Uniform Electric Field . . . . . . . . . . . . . . . . . . . . . . . . . . . 68
1.18 Electric Dipole in an External Electric Field . . . . . . . . . . . . . . . . . . . . . . . . . . . . . . . . . . . . . . 70
1.18.1 Electric force and torque on electric dipole in a uniform external electric field . . . . . . . . . . . . . . . 70
1.18.2 The Angular Acceleration and Time period of a Dipole in an External Uniform Electric Field . . . . . . 71
1.18.3 Dipoles in a Nonuniform Field . . . . . . . . . . . . . . . . . . . . . . . . . . . . . . . . . . . . . . . . . 71
1.19 Earnshaw’s Theorem . . . . . . . . . . . . . . . . . . . . . . . . . . . . . . . . . . . . . . . . . . . . . . . . . . . 74
1.19.1 Check Point 6 . . . . . . . . . . . . . . . . . . . . . . . . . . . . . . . . . . . . . . . . . . . . . . . . . . . 75
1.20 Conductors in Electrostatic Equilibrium . . . . . . . . . . . . . . . . . . . . . . . . . . . . . . . . . . . . . . . . 76
1.20.1 Check Point 7 . . . . . . . . . . . . . . . . . . . . . . . . . . . . . . . . . . . . . . . . . . . . . . . . . . . 78
1.21 Solid Angle . . . . . . . . . . . . . . . . . . . . . . . . . . . . . . . . . . . . . . . . . . . . . . . . . . . . . . . . 82
1.22 Electric Flux . . . . . . . . . . . . . . . . . . . . . . . . . . . . . . . . . . . . . . . . . . . . . . . . . . . . . . . 84
1.22.1 Check Point 8 . . . . . . . . . . . . . . . . . . . . . . . . . . . . . . . . . . . . . . . . . . . . . . . . . . . 87
1.23 Gauss’s Law . . . . . . . . . . . . . . . . . . . . . . . . . . . . . . . . . . . . . . . . . . . . . . . . . . . . . . . . 91
1.24 Applications of Gauss’s law . . . . . . . . . . . . . . . . . . . . . . . . . . . . . . . . . . . . . . . . . . . . . . . 94
1.24.1 Cylindrical Symmetry . . . . . . . . . . . . . . . . . . . . . . . . . . . . . . . . . . . . . . . . . . . . . . 97
1.24.2 Spherical Symmetry . . . . . . . . . . . . . . . . . . . . . . . . . . . . . . . . . . . . . . . . . . . . . . . 100
1.24.3 Check Point 9 . . . . . . . . . . . . . . . . . . . . . . . . . . . . . . . . . . . . . . . . . . . . . . . . . . . 104
1.25 Conductors in Electrostatic Equilibrium . . . . . . . . . . . . . . . . . . . . . . . . . . . . . . . . . . . . . . . . 113
1.25.1 Field in a Substance . . . . . . . . . . . . . . . . . . . . . . . . . . . . . . . . . . . . . . . . . . . . . . . 113
1.25.2 Fields Inside and Outside a Conductor . . . . . . . . . . . . . . . . . . . . . . . . . . . . . . . . . . . . . 113
1.25.3 Mechanical Pressure (or Surface Density of Force) on the Surface of a Charged Conductor . . . . . . . . 116
1.25.4 Faraday Cage Protection . . . . . . . . . . . . . . . . . . . . . . . . . . . . . . . . . . . . . . . . . . . . 117
1.26 Faraday’s Ice Pail Experiment . . . . . . . . . . . . . . . . . . . . . . . . . . . . . . . . . . . . . . . . . . . . . . 120
1.26.1 Check Point 10 . . . . . . . . . . . . . . . . . . . . . . . . . . . . . . . . . . . . . . . . . . . . . . . . . . 123
1.27 Gauss’s Law for Gravity . . . . . . . . . . . . . . . . . . . . . . . . . . . . . . . . . . . . . . . . . . . . . . . . . 125
1.28 Exercises and Problems . . . . . . . . . . . . . . . . . . . . . . . . . . . . . . . . . . . . . . . . . . . . . . . . . 127
1.28.1 Conceptual Questions . . . . . . . . . . . . . . . . . . . . . . . . . . . . . . . . . . . . . . . . . . . . . . 127
1.28.2 Problems . . . . . . . . . . . . . . . . . . . . . . . . . . . . . . . . . . . . . . . . . . . . . . . . . . . . . 130
1.29 Answer Keys and Solutions . . . . . . . . . . . . . . . . . . . . . . . . . . . . . . . . . . . . . . . . . . . . . . . 152
1.29.1 Conceptual Questions . . . . . . . . . . . . . . . . . . . . . . . . . . . . . . . . . . . . . . . . . . . . . . 152
1.29.2 Problems . . . . . . . . . . . . . . . . . . . . . . . . . . . . . . . . . . . . . . . . . . . . . . . . . . . . . 155

2 Electric Potential 187


2.1 Electric Potential Energy and Electrostatic Potential in Fields . . . . . . . . . . . . . . . . . . . . . . . . . . . . 187
2.1.1 Potential and Potential Energy in a Uniform Field . . . . . . . . . . . . . . . . . . . . . . . . . . . . . . 189
2.1.2 Electrostatic Potential Energy and Potential Difference due to a Point Charge . . . . . . . . . . . . . . 190
2.1.3 Check Point 1 . . . . . . . . . . . . . . . . . . . . . . . . . . . . . . . . . . . . . . . . . . . . . . . . . . . 192
2.2 Electric Potential Energy with Several Point Charges . . . . . . . . . . . . . . . . . . . . . . . . . . . . . . . . 194
2.2.1 Potential Energy of a Dipole in a Uniform Electric Field . . . . . . . . . . . . . . . . . . . . . . . . . . . 197
2.3 Electric Potential . . . . . . . . . . . . . . . . . . . . . . . . . . . . . . . . . . . . . . . . . . . . . . . . . . . . . 197
2.3.1 The Electric Potential of a Point Charge . . . . . . . . . . . . . . . . . . . . . . . . . . . . . . . . . . . . 197
2.4 Potential Gradient . . . . . . . . . . . . . . . . . . . . . . . . . . . . . . . . . . . . . . . . . . . . . . . . . . . . 199
2.4.1 Electric Potential For a System of Charges . . . . . . . . . . . . . . . . . . . . . . . . . . . . . . . . . . 201
2.4.2 Check Point 2 . . . . . . . . . . . . . . . . . . . . . . . . . . . . . . . . . . . . . . . . . . . . . . . . . . . 203
2.4.3 Electric Potential at a Point P(r, θ) Due to an Electric Dipole . . . . . . . . . . . . . . . . . . . . . . . . 207
2.4.4 Calculation of Electric Field due to a Dipole from Potential Formula . . . . . . . . . . . . . . . . . . . . 208
2.4.5 Electric Potential due to a Charged Ring . . . . . . . . . . . . . . . . . . . . . . . . . . . . . . . . . . . 209
2.4.6 Electric Potential Due to a Charged Disc at a Point on it’s Geometric Axis . . . . . . . . . . . . . . . . 210
2.4.7 Electric Potential Due to a Shell . . . . . . . . . . . . . . . . . . . . . . . . . . . . . . . . . . . . . . . . 212
2.4.8 Electric Potential due to a Non-conducting Charged Sphere . . . . . . . . . . . . . . . . . . . . . . . . . 213
2.4.9 Potential on the edge of a Uniformly Charged Disc: . . . . . . . . . . . . . . . . . . . . . . . . . . . . . 214
2.5 Potential Energy in an External Field . . . . . . . . . . . . . . . . . . . . . . . . . . . . . . . . . . . . . . . . . 214
2.5.1 Potential energy of a single charge . . . . . . . . . . . . . . . . . . . . . . . . . . . . . . . . . . . . . . . 214
CONTENTS ix

2.5.2 Potential Energy of a System of Two Charges in an External Field . . . . . . . . . . . . . . . . . . . . . 215


2.6 Equipotential Surfaces and Field Lines . . . . . . . . . . . . . . . . . . . . . . . . . . . . . . . . . . . . . . . . . 216
2.6.1 Different Equipotential Surfaces . . . . . . . . . . . . . . . . . . . . . . . . . . . . . . . . . . . . . . . . 216
2.6.2 Properties of Equipotential surface . . . . . . . . . . . . . . . . . . . . . . . . . . . . . . . . . . . . . . 217
2.7 Equipotentials and Conductors . . . . . . . . . . . . . . . . . . . . . . . . . . . . . . . . . . . . . . . . . . . . . 218
2.7.1 Check Point 3 . . . . . . . . . . . . . . . . . . . . . . . . . . . . . . . . . . . . . . . . . . . . . . . . . . . 221
2.8 Connected Conducting Spheres . . . . . . . . . . . . . . . . . . . . . . . . . . . . . . . . . . . . . . . . . . . . . 223
2.9 Earthing . . . . . . . . . . . . . . . . . . . . . . . . . . . . . . . . . . . . . . . . . . . . . . . . . . . . . . . . . . 223
2.10 Corona Discharge . . . . . . . . . . . . . . . . . . . . . . . . . . . . . . . . . . . . . . . . . . . . . . . . . . . . . 224
2.10.1 Electric Potential Energy for Continuous Charge System . . . . . . . . . . . . . . . . . . . . . . . . . . . 224
2.11 The Van de Graaff Generator . . . . . . . . . . . . . . . . . . . . . . . . . . . . . . . . . . . . . . . . . . . . . . 226
2.12 The Millikan Oil-Drop Experiment . . . . . . . . . . . . . . . . . . . . . . . . . . . . . . . . . . . . . . . . . . . 227
2.13 Important Points . . . . . . . . . . . . . . . . . . . . . . . . . . . . . . . . . . . . . . . . . . . . . . . . . . . . . 228
2.13.1 Check Point 4 . . . . . . . . . . . . . . . . . . . . . . . . . . . . . . . . . . . . . . . . . . . . . . . . . . . 228
2.14 Image Method . . . . . . . . . . . . . . . . . . . . . . . . . . . . . . . . . . . . . . . . . . . . . . . . . . . . . . 229
2.15 Questions and Exercises . . . . . . . . . . . . . . . . . . . . . . . . . . . . . . . . . . . . . . . . . . . . . . . . . 232
2.15.1 Conceptual Questions . . . . . . . . . . . . . . . . . . . . . . . . . . . . . . . . . . . . . . . . . . . . . . 232
2.15.2 Problems . . . . . . . . . . . . . . . . . . . . . . . . . . . . . . . . . . . . . . . . . . . . . . . . . . . . . 235
2.16 Answer Keys and Solutions . . . . . . . . . . . . . . . . . . . . . . . . . . . . . . . . . . . . . . . . . . . . . . . 255
2.16.1 Conceptual Questions . . . . . . . . . . . . . . . . . . . . . . . . . . . . . . . . . . . . . . . . . . . . . . 255
2.16.2 Problems . . . . . . . . . . . . . . . . . . . . . . . . . . . . . . . . . . . . . . . . . . . . . . . . . . . . . 260
2.16.3 Multiple Choice Assignments . . . . . . . . . . . . . . . . . . . . . . . . . . . . . . . . . . . . . . . . . . 270
Chapter 1

Electric Charge and Field

1.1 Introduction There are two types of electric charge, as the following simple
experiments show. suppose a hard rubber rod that has been
Electromagnetism is a science of the combination of electrical vigorously rubbed on fur is suspended by a string as shown in
and magnetic phenomenon. Electromagnetism can be divided Figure 1.2. When a glass rod that has been rubbed on silk is
into 2 parts: brought near the rubber rod, the two attract each other (Fig.
1.2 a). On the other hand, if two charged rubber rods (or two
1. Electrostatics: The branch of physics which deals with charged glass rods) are brought near each other as shown in
electric effect of static charge is called electrostatics. Figure 1.2b, the two repel each other. This observation shows
2. Electrodynamics: It deals with the study of charges in that the rubber and glass have two different types of charge on
motion (discusses magnetic phenomenon). them. Each type of charge repels the same type but attracts
the opposite type. That is: unlike charges attract; like
charges repel.
1.2 Electric Charge
You have probably seen that on combing hair, on a dry day,
hair is attracted to the comb. When we rub objects together
such as wool against amber or silk against glass (Fig.1.1a), we
find that after rubbing, the objects acquire a property due to
which they attract to each other; a silk cloth is attracted to a
glass rod that it was rubbed against (Fig.1.1b). The, rubbed Rubber
objects can also attract other objects. For example, small bits Rubber
of paper are attracted to a comb that had been rubbed through S
F
hair (Fig.1.1c). – – ––
When objects behave in this way, they are said to be –– – S –
– –
S
F F
electrified (from the Greek word elektron meaning amber) or + + + Glass – – Rubber
electrically charged. + + + + – –– – S
– F
(a) (b)
You can easily electrify your body by vigorously rubbing
your shoes on a wool carpet. Evidence of the electric charge Figure 1.2: The electric force between (a) oppositely charged ob-
on your body can be detected by touching a metal doorknob. jects and (b) like-charged objects.
Under the right conditions, you will feel a shock when you
touch the metal doorknob. (Experiments such as these work
best on a dry day because an excessive amount of moisture in
the air can cause any charge you build up to “leak” from your 1.2.1 Electric Fluid Model
body to the Earth.)
According to Franklin1 , all objects are full of an electric fluid.
When you bring two objects close together, some electric fluid
may transfer from one object to the other. As a result of which,
one object gets a surplus of electric fluid and the other has a
deficit of electric fluid. Franklin called the object with a sur-
plus of electric fluid as positive or plus, and the object with
a deficit of electric fluid as negative or minus. According to
(a) (b) (c)
1 Benjamin Franklin (1706––1790) developed many of the concepts we

Figure 1.1: (a) Silk is rubbed against glass, and afterward (b) the study in this chapter. Franklin–one of the Founding Fathers of the United
States and a signer of the Declaration of Independence—was first a sci-
silk is attracted to the glass. (c) After running a comb through hair
entist and inventor. As we’ll discuss, Franklin made major contributions
on a dry day, the comb attracts paper. to the field of electricity.

1
silk cloth have some amount of electric fluid initially, and after they are rubbed

Andrew
A ne
together, some of the electric fluid is transferred from the silk to the glass of el
2 C. (Fig. 23.6A). The silk then has a deficit ofCHAPTER electric fluid, so FranklinCHARGE
1. ELECTRIC said the silk
AND FIELD
is negative. The glass has a surplus of electric fluid, so he said it is positive. Frank-
madeA.

Debora Katz
Figurelin23.5  anSilk is rubbed
arbitrary choice against
in this model; he assumed the electric fluid was trans-
Franklin, initially, both of them (the glass rod and the silk Silk Glass Silk
glass, and afterward
ferred from B.
the the
silk silk
to is
the attracted
glass. to
cloth) have some amount of electric fluid, and after they are There was no way for Franklin to know whether
the glass.
rubbed together, some of the that
C.was
electric
After running
true.
fluid is Because
a comb
transferred therethrough
fromwas no experimental evidence to determine which
hair on a dry
wayThe day,
thesilk the
electric comb attracts
fluida flowed, paper. p e
the silk to the glass (Fig.1.3a). then has deficit ofFranklin could have imagined that electric fluid was
electric fluid, so Franklin saidtransferred
the silk is from the glass
negative. to the silk. Subsequent scientists have kept his arbitrary
The glass p e
has a surplus of electric fluid,choice,
so he said however, and that has important implications for our contemporary model
it is positive.
Franklin’s choice was arbitrary of inelectric charge.
this model; he assumed the p e
electric fluid was transferred from the silk
Today wetohave the experimental
glass. There evidence supporting the model that all objects are
was no way for Franklin to know whether that was true. Rubbing causes electric fluid
Be- are made up of three types
made up of atoms, and atoms of particles—neutrons, Ru
cause there was no experimental evidence to flow from silk to glass
protons, and to determine
electrons. The which
neutrons and protons are tightly packed into the central to
way the electric fluid flowed, Franklin
region of could
thehave
atomimagined
known that as the nucleus. The electrons move rapidly outside the
electric fluid was transferred from the glass to the silk. Subse-
Debora Katz

nucleus, forming a cloud. According to this contemporary model, when two objects
quent scientists have kept his arbitrary choice in their model Deficit of fluid Surplus fluid Deficit of p
are brought near each other, it is possible for electrons that are loosely bound to the
of electric charge.
Today we have experimental atoms evidencein one object tothe
supporting move to the other object. The protons and neutrons are tightly
model
that all objects are made up of atoms, and atoms are made up they are not transferred. Therefore, when glass is
bound in the nucleus, and so
rubbed against
of three types of particles—neutrons, protons, silk,
andelectrons
electrons.are transferred from the glass to the silk. Afterward,
The neutrons and protons arethe glass packed
has a surplus
into theofcentral
protons and the silk has a deficit of protons (Fig. 23.6B).
Andrew Lambert /Leslie Garland Picture Library/Alamy

tightly
region of the atom known as theOur sign convention
nucleus. The electrons is based
move on the relative number of electrons and protons in a p e
rapidly outside the nucleus, given
forming object. An object
a cloud. that has
According to an equal number of electrons and protons is said to p e
this model, when two objects be areneutral
brought . An near each that
object other,hasthe
a deficit of electrons has a surplus of protons; such an e
electrons can move from atoms of the object in which these
object is said to be positive. An object that has a surplus of electrons has a deficit of p e
are loosely bound to atoms of protons;
the object in which
such electrons
an object areto be negative.
is said
relatively tightly bound. The protons and neutrons are tightly
“Negative” “Positive” “Nega
bound in the nucleus, and so they are not transferred. There-
fore, when glass is rubbed against silk, electrons are trans- A. Franklin’s model(a) B. Contemp
ferred from the glass to the silk. As a result of which, the glass
A neutral object has an equal number
has a surplus of protons and the silk has a deficit of protons
Figure of23.6  Comparison
electrons and protons. of Ben Franklin’s model and ou
(Fig.1.3b).
Our sign convention is based on the relative number of elec-
charge.
3.5  A.
trons
Silkand protons
is rubbed in a given object. An object that has an
against Unless otherwise
Glass p e
equalB.number
afterward the silk isofattracted to and protonsSilk
electrons
Glass
is said to be neutral.
Silk
C. After
Anrunning
object athat
comb hasthrough
a deficit of electrons has aCopyright
surplus 2017
of pro- p e
Cengage Learning. All Rights Reserved. e scanned, or duplicated, in whole o
May not be copied,
dry day, the comb attracts paper. p e p
tons; such an object is said to be positive. An object that has
p e e
a surplus of electrons has a deficit of protons; such an object p
p e
is said to be negative. e e p p
Rubbing causes electric fluid
Important Points to flow from silk to glass Rubbing causes electrons
to flow from glass to silk
1. Charge of a material body or particle is the property (ac-
quired or natural) due to which it produces and experiences
Deficit of fluid Surplus fluid Deficit of protons Surplus of protons
electrical and magnetic effects. Some of naturally occurring
charged particles are electrons, protons, α-particles etc.
e
2. A material is said to be charged if there are more of one p
kind of charge than the other. A negatively charged body e
p e p
has excess electrons over protons, while a positively charged p e e
body has excess protons over electrons. The protons are p
e p e
tightly bound in the nucleus, making them very difficult to
p e p
remove. Charging a body therefore involves the removal,
addition, and rearrangement of the orbital electrons“Positive”
“Negative” only. “Negative” “Positive”
A body becomes positively charged if it loses
A. Franklin’s model electrons, and B. Contemporary model
negatively charged if it gains electrons. (b)

3. Charge is a derived physical Figure 23.6 and


quantity Comparison
is measuredof Benin Franklin’s
Figure 1.3:model and our contemporary
Comparison of Ben Franklin’s model of electric
model and our contem-
charge. porary model of electric charge.
Coulomb in SI unit. 1 Coulomb is a very large amount
Unless otherwise noted, all content on this page is © Cengage Learning.
of charge, in practice, we use smaller units mC(10−3 C,
µC(10 −6
C), nC(10 C) etc.
−9
Copyright 2017 Cengage Learning. All Rights Reserved. May not be copied, scanned, or duplicated, in whole or in part. WCN 02-200-203
1.2. ELECTRIC CHARGE 3

4. CGS unit of charge is “electrostatic unit (esu)” at lead dioxide plate:


1C = 2997924579.9996 esu ≈ 3 × 10 esu 9
of charge. PbO2 + 4H+ + SO2−
4 + 2[ electrons ] → PbSO4 + 2H2 O
Dimensional formula of charge = [M L T A ]
0 0 1 1
charges: 0 + 4e + (−2e) + (−2e) → 0 +0
5. Charges are divided into two parts (i ) positive (ii ) negative. (1.2)
Like charges repel and unlike charges attract. The plates of such a battery are connected by an external

6. Magnitude of the smallest known charge is e = 1.6×10−19 C


(charge of one electron or proton).

Note: Mass of a body is always positive whereas a charge can


be either positive or negative. Particles are the substance and
charge happens to be one of their intrinsic properties, just as
mass is.

1.2.2 Properties of Electric Charge


1. Charge is a scalar quantity: It adds algebraically and rep-
resents excess or deficiency of electrons.

2. Charging a body implies transfer of charge (electrons) from


one body to another. Positively charged body means loss
of electrons i.e. deficiency of electrons. Negatively charged
body means excess of electrons. Since each electron has
mass me ≈ 9.1 × 10−31 kg. So, excess of electron means
excess of mass. Therefore, the mass of a negatively charged
body will always be greater than the original mass of the
neutral body. A positively charged body means loss of elec-
trons and hence loss of mass.
If n electrons are transferred from a body A to body B, then
the body A losses its mass by an amount of ∆m = nme ,
whereas body B gains the same amount of mass. Here me
is the mass of electron. Figure 1.4

3. Charge is conserved: In an isolated system, total charge circuit (a wire), and the electrons released by the reaction
(sum of positive and negative charges) remains constant (1.1) travel from one plate to the other, forming an electric
whatever change takes place in that system. Example of current (see Fig.1.4)
charge conservation occurs when an electron e− (charge −e)
and its antiparticle, the positron e+ (charge +e), undergo 4. Charge is quantized: Charge on a body is the integral
an annihilation process, transforming into two gamma rays multiple of the charge on an electron ⇒ Q = ± ne in
(high-energy light): e− + e+ → γ + γ (annihilation) which e, the elementary charge, has the approximate value
In pair production, the converse of annihilation, charge is 1.602 × 10−19 C and n = 0, 1, 2, ...
also conserved. In this process a gamma ray transforms into The fact that electric charge is always an integral multiple
an electron and a positron: γ → e− + e+ (pair production) of e is termed as quantization of charge and e is called
Charge is of course also conserved in chemical reactions. the quanta of charge. The elementary charge e is one of the
For instance, in a lead-acid battery (automobile battery), important constants of nature. The electron and proton
plates of lead and of lead dioxide are immersed in an elec- both have a charge of magnitude e.
trolytic solution of sulfuric acid. The reactions that take
place on these plates involve sulfate ions SO42− , where the No unit of charge smaller than e has been detected on a free
superscript 2 - indicates an ion with two extra electrons) particle; current theories, however, propose the existence of
and hydrogen ions (H+ ) ; the reactions release electrons at particles called quarks (the constituent particles protons
the lead plate, and they absorb electrons at the lead dioxide and neutrons) having charges −e/3 and +2e/3. Although
plate: there is considerable experimental evidence for such par-
at lead plate: ticles inside nuclear matter, free quarks have never been
detected. For this reasons, we do not take their charges to
Pb + SO2−4 → PbSO4 + 2[ electrons ] be the elementary charge.
(1.1)
charges: 0 + (−2e) → 0 + (−2e)
4 CHAPTER 1. ELECTRIC CHARGE AND FIELD

Note: The quantization of charge was first suggested by the EXAMPLE 1. Is it possible for a body to have an electric
experimental laws of electrolysis discovered by English ex- charge of 2.0 × 10−19 C ? 3.2 × 10−19 C?
perimentalist Faraday. It was experimentally demonstrated (A) Yes; yes (B) Yes; no
by Millikan in 1912. Other quantized quantities are energy, (C) No; yes (D) No; no
angular momentum. The quantum of energy is hv (i.e., pho-
APPROACH According to quantization of charge, the net
ton) and that of angular momentum is 2π h
. The quantum
charge on any charged object is given by-
of mass not known till date.
Number of Electrons in 1C of Charge: Q = ±ne
By quantization of charge, we have-
Here, n is an integer value. Substitute given values of Q and
Q = ne e = 1.6 × 10−19 C and solve for n. Acceptable values of n are
.
only the integer one, i.e. n = 0, 1, 2, . .
Therefore, the number of electrons in 1C charge is given by-
SOLUTION (C) In first case, 2.0 × 10−19 C, therefore-
Q 1C
n= = = 6.25 × 1018 2.0 × 10−19 C 2
e 1.6 × 10−19 n= =
−19
1.6 × 10 C 3
Thus 1C charge contains 6.25 × 1018 electrons, which is a
huge number. Thus, the step size is very small as compared Since, n = 2/3, is a fraction which is not acceptable.
to the charges usually found on many cases. At macroscopic In second case, 2.0 × 10−19 C, therefore-
level, we deal with charges that are enormous compared to
the magnitude of minimum charge i.e. e (1.6 × 10−19 C). In 3.2 × 10−19 C
n= =2
this case, the increase or decrease in units of e is not very 1.6 × 10−19 C
different from saying that charges are continuous. So at
macroscopic level we can ignore the quantization of electric Since, n = 2 is an integer vale so it is acceptable.
charge. Thus the first charge given is impossible, while the second
charge given, twice the fundamental charge, is possible - in-
As 1C charge contains 6.25×1018 electrons, which is a huge deed, it is common.
number. So “1 coulomb” is a huge amount of charge. Prac-
tically we use relatively smaller units for charge like ‘mili- EXAMPLE 2. We have an iron piece of mass 56 milligram
coulomb (mC = 10−3 C),’ micro-coulomb (µC = 10−6 C)’, (Atomic number- 26) if 10˘6 electron are removed from this
‘nano-coulomb (nC = 10−9 C)’ or ‘pico-coulomb (pC = body then calculate charge appear on this body.
10−12 C)’. −3
SOLUTION moles = 56×10 56 = 10−6 moles Number of
5. Charge is always associated with mass i.e. charge cannot atom = 10−3 × 6.022 × 1023 ≈ 6 × 1020 Number of elec-
exist without mass, though mass can exist without charge. tron removedn = 26 × 6 × 1020 × 10−8 Charge on body, Q=ne
Particles such as neutrino or photon have no rest mass, so = 156 × 1012 × 1.6 × 10˘19 = 25µC
they can have no charge.
EXAMPLE 3. How much negative charge and how much
6. Charge is transferable from one charged body to another positive charge are there on the electrons and the protons in a
body which may be charged or uncharged, if they are put in cup of water (0.25 kg)?
contact. The process of charge transfer is called conduction.
Whole of the charge cannot be transferred by conduction APPROACH To find total negative and positive charge in
from one body to another except in case when a charged 250 g of water, we first calculate total number of electrons and
body is enclosed by a conducting body and connected to it protons in it. Then by using the expression, Q = −N e we can
(it will be discussed later in this chapter). find the total negative charge in it. Q = +N e will give total
positive charge in it.
7. An accelerated charge always radiates energy in the form SOLUTION The “molecular mass” of water is 18 g; therefore,
of electromagnetic waves. number of moles in 250 g of water = 250/18 moles.
Since, each mole has NA , i.e., 6.02 × 1023 molecules, therefore,
8. Charge is invariant i.e., charge on a body does not change,
number of molecules in 250/18 moles of water
whatever be its speed, however specific charge (q/m) de-
pends on speed as mass depends on speed. = (250/18) × 6.0 × 1023
Specific charge = q/m
m = q m0v2 where m is the dynamic mass and m0 is the Each molecule consists of two hydrogen atoms (one electron
1− c2
rest mass. for each one) and one oxygen atom (eight electrons). Thus,
there are 10 electrons in each molecule. Therefore, total
Note: We sometimes speak of the fundamental unit of charge number of electrons in 250 g of water-
e as the charge of an electron but e is a positive quantity. The
charge of an electron, properly, is −e. N = (250/18) × 6.0 × 1023 × 10
1.3. EFFECT OF CHARGE ON THE MASS OF A GIVEN OBJECT 5

p n
Total negative charge on 250 g of water,
Q = −N e = −(250/18) × 6.02 × 1023 × 10 e u u u d
= −(250/18) 6.02 × 1023 × 10 × 1.60 × 10−19 C
 
d d
7
= −1.3 × 10 C
(a) Proton (uud), q = +e (b) Neutron (udd), q = 0
The positive charge on the protons is the opposite of this. You
can also calculate it by using Q = +N e. Figure 1.5: Quark composition of proton and neutron

1.3 Effect of Charge on the Mass of a 1.4 Check Point 1


Given Object
1. • If you charge a pocket comb by rubbing it with a silk
Let M0 be the mass of a neutral object, m be the mass of an scarf, how can you determine if the comb is positively or
electron and −e be the charge on an electron. If the object is negatively charged?
given a positive charge Q by taking away n = Q/e electrons
from it, then the mass of the positively charged object will be- 2. • Why does a shirt or blouse taken from a clothes dryer
sometimes cling to your body?
Mpos = M0 − mn = M0 − mQ/e
Now, if another similar object is given a negative charge −Q 3. • Explain why fog or rain droplets tend to form around ions
by putting n = Q/e electrons on it, then, the mass of the or electrons in the air.
negatively charged object-
4. • A positively charged rod is brought close to a neutral
Mneg = M0 + mn = M0 + mQ/e piece of paper, which it attracts. Draw a diagram show-
ing the separation of charge in the paper, and explain why
Quarks attraction occurs.
Quarks are truly elementary particles which carrycharges that 5. • Why does a plastic ruler that has been rubbed with a cloth
are fractions of electronic charge ± 23 e and ± 13 e . have the ability to pick up small pieces of paper? Why is
There are six types of quarks (referred to as six flavours of this difficult to do on a humid day?
quarks) and these are: (i) up u, (ii) charmed c, (iii) top or truth
t, all having charge +(2/3)e, and (iv) down d, (v) sideways or 6. • Contrast the net charge on a conductor to the “free
strange s (vi) bottom or beauty b, all having charge −(1/3)e. charges” in the conductor.
The up, charmed, and top quarks have electric charges of + 23
that of the proton, whereas the down, strange, and bottom 7. • Figures 1.6 (a) and (b) show how a charged rod placed
quarks have charges of − 13 that of the proton. near an uncharged metal object can attract (or repel) elec-
trons. There are a great many electrons in the metal, yet
Table 1.1: Quarks and their charges
only some of them move as shown. Why not all of them?
Quark Symbol Charge
Up u +(2/3)e
Down d −(1/3)e
Strange s −(1/3)e
Charmed c +(2/3)e
Bottom b −(1/3)e
Top t +(2/3)e
(a) Charging by induction
Two kinds (or flavors) of quarks, up (u) and down (d), are
sufficient to describe normal matter. The electric charges are
+ 32 e and − 13 e for the u and d quarks, respectively. The proton
consists of two u and one d valence quarks, and the neutron of
one u and two d valence quarks.
2e 2e e
p=+ + − = +e i.e., uud
3 3 3
2e e e
n=+ − − =0 i.e., udd (b) Inducing a charge on an object con-
3 3 3 nected to ground.
There is firm experimental evidence of the existence of all six
quarks and their six antiquarks within the nucleus, but free Figure 1.6: Comparison of Ben Franklin’s model and our contem-
quarks have not been detected yet. Current theory implies that porary model of electric charge.
direct detection of quarks may, in principle, be impossible.
6 CHAPTER 1. ELECTRIC CHARGE AND FIELD

8. • The fact that the electron has a negative charge and the 5. A plastic ruler that has been rubbed with a cloth is charged.
proton has a positive charge is due to a convention estab- When brought near small pieces of paper, it will cause sep-
lished by Benjamin Franklin. Would there have been any aration of charge in the bits of paper, which will cause the
significant consequences if Franklin had chosen the opposite paper to be attracted to the ruler. On a humid day, po-
convention? Is there any advantage to naming charges plus lar water molecules will be attracted to the ruler and to
and minus as opposed to, say, A and B? the separated charge on the bits of paper, neutralizing the
charges and thus eliminating the attraction.
9. • Small bits of paper are attracted to an electrically charged
comb, but as soon as they touch the comb they are strongly 6. The net charge on a conductor is the difference between
repelled. Explain this behavior. the total positive charge and the total negative charge in
the conductor. The “free charges” in a conductor are the
10. Find the total electric charge of 2.5 kg of (a) electrons and electrons that can move about freely within the material be-
(b) protons. cause they are only loosely bound to their atoms. The “free
11. • A container holds a gas consisting of 2.85 moles of oxygen electrons” are also referred to as “conduction electrons.” A
molecules. One in a million of these molecules has lost a conductor may have a zero net charge but still have sub-
single electron. What is the net charge of the gas? stantial free charges.

12. • A copper penny (Z = 29) has a mass of 3.10 grams. What 7. Most of the electrons are strongly bound to nuclei in the
is the total charge of all the electrons in the penny? metal ions. Only a few electrons per atom (usually one or
two) are free to move about throughout the metal. These
are called the “conduction electrons.” The rest are bound
Answer Key and Solutions
more tightly to the nucleus and are not free to move. Fur-
1. Rub a glass rod with silk and use it to charge an electro- thermore, in the cases shown in Figures 1 and 2, not all
scope. The electroscope will end up with a net positive of the conduction electrons will move. In Figure 1, elec-
charge. Bring the pocket comb close to the electroscope. If trons will move until the attractive force on the remaining
the electroscope leaves move farther apart, then the charge conduction electrons due to the incoming charged rod is
on the comb is positive, the same as the charge on the elec- balanced by the repulsive force from electrons that have al-
troscope. If the leaves move together, then the charge on ready gathered at the left end of the neutral rod. In Figure
the comb is negative, opposite the charge on the electro- 8, conduction electrons will be repelled by the incoming rod
scope. and will leave the stationary rod through the ground con-
nection until the repulsive force on the remaining conduc-
2. The shirt or blouse becomes charged as a result of being tion electrons due to the incoming charged rod is balanced
tossed about in the dryer and rubbing against the dryer by the attractive force from the net positive charge on the
sides and other clothes. When you put on the charged ob- stationary rod.
ject (shirt), it causes charge separation within the molecules
of your skin, which results in attraction between the shirt 8. No, the basic physics of electric charges would not have
and your skin. been affected at all by an opposite assignment of positive
and negative labels. The use of + and – signs, as opposed
3. Fog or rain droplets tend to form around ions because water to labels such as A and B, has the distinct advantage that
is a polar molecule, with a positive region and a negative it gives zero net charge to an object that contains equal
region. The charge centers on the water molecule will be amounts of positive and negative charge.
attracted to the ions (positive to negative).
9. Initially, the bits of paper are uncharged and are attracted
4. The negatively charged electrons in the paper are attracted
to the comb by polarization effects. When one of the bits of
to the positively charged rod and move towards it within
paper comes into contact with the comb, it acquires charge
their molecules. The attraction occurs because the negative
from the comb. Now the piece of paper and the comb have
charges in the paper are closer to the positive rod than
charge of the same sign, and hence there is a repulsive force
are the positive charges in the paper, and therefore the
between them.
attraction between the unlike charges is greater than the
repulsion between the like charges.
10. (a)−4.4 × 1011 C (b) 2.4 × 108 C

11. 0.275C

12. APPROACH Since, total charge on a body having N


electrons is given by, Q = N e. So, first of all, find the total
number of electrons (N ) in 3.10 g of copper and then use
above relation.
Figure 1.7: Polarization in the paper atoms. SOLUTION As, the number of electrons in a copper
atom is 29 (the atomic number of copper). Therefore,
1.5. CONDUCTORS AND INSULATORS 7

total number of electrons in 3.10 gram of copper is given by- became negatively charged. The two bodies acquire opposite
signs of electricity; one gets positively charged, while the other
N = number of copper atoms in3.10gram of copper 688  CHAPTER 23  Electric Forces
becomes negatively charged (Fig.1.8 ). When two bodies are
× 29 charged by friction, they acquire the same magnitude of charge.
Given mass of copper Furthermore, the bodies retain these excess charges even when
i.e., N = × NA × 29 they are separated from each other.
molarmassof copper
Here, NA is avogadro’s number and it’s value is 6.02 × 1023 ,
3.10g Neutral objects: Conservation
molar mass of the copper is 63.5 g, therefore, N = × no + or − drawn equal number
63.5g
6.02 × 10 × 29 = 8.53 × 10 electrons
23 23

So, the total charge on the copper penny, −



Q = N e = 8.53 × 1023 × 1.6 × 10−19 C = 1.37 × 105 C −

Since, electrons are negatively charged particles, so the na- −
ture of this charge will be negative, i.e., Q = −1.37 × 105 C
(a) (a) (b

Figure 23.7  When youofare


Conservation sketching, indicate only the excess charged
charge:
1.5 Conductors and Insulators B. The rod equal
and cloth areofcharged
number by rubbing. When charge is transferre
+’s and −’s
number of excess positive particles on one object must equal the numbe
were charged independently. (The cloths used to charge these rods are n
An object can be classified in either of the following four cat-
egories: (i) Conductors −

(ii) Insulators − Instead, we represent
+
(iii) Semiconductors − +
+
actual number of sign
(iv) Superconductors −
+
+ work. A sketch, howev
charge is transferred f
B
C

(b) of pluses on one D


obje
1.5.1 Conductors (Fig. 23.7B). Charge
Greater number of +’s means minuses must be the s
These are the materials that allow flow of charge through them. red rod carries a greater charge. positive charge than
Examples include metals (such as copper in common lamp object (Fig. 23.7C).
wire), tap water, and the human body((which is similar to
tap water). This category generally comprises of metal but
may sometimes contain non-metals too. (Ex. Carbon in form
CONCEPT EXER
of graphite). +
+
+
+
+ + Initially a glass rod and
+ +
+ + rod, the glass rod has a
1.5.2 Insulators +
+
+
These are the materials which do not allow movement of charge +
+
through them. Examples include rubber (such as the insulation +
23-3 A Quali
+
on common lamp wire), plastic, glass, and chemically pure
water. Franklin learned that s
B
C
D
(c)
another. Suppose you
two silk handkerchiefs
Figure 1.8: When you are sketching, indicate only the excess
1.5.3 Semiconductors tween any of these obje
charged particles. (a) Neutral objects are shown without any +
chiefs and the other ro
These are materials that are intermediate between conductors or – signs. (b) The rod and cloth are charged by rubbing. When
both glass rods attract t
and insulators; Examples include silicon and germanium in charge is transferred from one neutral object to another, both
repel each be-(Fig.
other
computer chips. come charged. ELECTROSTATIC
The number ofFORCE 
excess positive particles onsimple
This one object
experimen
must equal the number of excess negative
! Underlying Principle particles force
on the exerted
other ob-by one c
ject. (c) The two rods were charged independently. electrostatic
(The clothsforce depe
1.5.4 Superconductors used to charge these rods are not shown in the sketch.)
1. Neutral objects
These are materials that are perfect conductors, allowing initially uncharge
Rubbing causes a transfer of electrons from one object tocharg
2. Oppositely
charge to move without any hindrance. In this chapter we
another. If some electrons are transferred from an object silk isA to
attracted t
discuss only conductors and insulators.
another object B, then A becomes positively charged 3. Twoandnegatively
B c
negatively charged. (Fig. 23.8C). Tw
glass rods (Fig. 2
1.6 Methods of Charging Note: Charging involves transformation of mass. sign are mutually

1.6.1 Charging by Friction Insulator


Copyright 2017 Cengage Learning. All Rights Reserved. May not

Charging by friction is the oldest form of charging. It was The electrons in some materials—such as rubber—do not move
found that when an amber rod is rubbed with fur, the rod freely. Such materials are known as insulators. When a surplus
all this rubber
GROUND protect ★ him
Majorfrom getting shocked? The Earth often serves as a charge reservoir known as a
Concept

K. Miri Photography/Shutte
The electrons in some materials—such as rubber—do
If you try to charge a copper rod in the same accept not
way,oryou
move
provide freely.
electrons
will find
Such
freely,
that you
ma-
can- and it is so large that the
UCTOR  ★ Major Concept terials are known as insulators . When a surplus
not build up charge on the copper. Copper isCHAPTER of ofcharged
electronsparticles
has a (positive
negligible
an example1.of ELECTRIC or nega-
A con- onAND
effect
a conductor. CHARGE it. So, the ground rem
8 FIELD
tive) builds up isona some
ductor partinofwhich
material an insulator,
the charged theparticles
excess remains
at all times. there.
The
(usually copperSo,rod
electrons) if you
can in
flowhold 23.11
Figure is connected
one endfreely.of When
a rubber
you holdrodonewhile far your
thecopper
end of the end
rod andbody.
is rubWhen
being something
rubbed
the other issilk,
with
end with connected
silk to the ground by a
of charged particles (positive electrons
or negative) builds up on some to the itcopper
isa grounded.
(Fig. 23.10A), arefar
the transferred
end of the from thewill
rod silk acquire surplus ofEvery
rod, and those building
excessand
electrons
Copper
(with
thosea contemporary
elec-

electri
part of an insulator, the excess remains
trons are there.
free to So,
flow.if you hold likes repel, the
Because connected
electronsbetween
move the
away electrical
from one system and −−
a copper
− − pip
electrons
one end of a rubber rod will
while the farremain at that
end is being end, with
rubbed never flowing into your hand (Fig. 23.10B). − −−− −− − − −
does this technician another, which means they travel through the rod into your hand (Fig. 23.11). − − − − −

silk (Fig.1.9a),
se electrical lines be-the far end ofThe thehuman
rod will acquire a surplus
body is also a conductor, so charged particles move freely through −−
ators and Conductors
of electrons and those electrons yourwillbody
remain at that
toward theend, never
Earth. If there are no insulators between you and the −

This part of the rod Silk stay on the
Excess electrons − Exce
flowing into your hand (Fig. 1.9b).
ground—such as when
sheets you are barefoot—the −
epair work, a technician covers electrical lines with rubber remains neutral. charge willfarcontinue
side of thetorubber
flow into
rod. − −flow−
also Concept
ajor wear a rubber suit or rubber sheets over
the his arms.
Earth. As Why does you and the copper rod remain neutral (a)
a result, despite the rod − −into
ect him from getting shocked? being rubbed with silk. −
Copper − −− − −
some materials—such
ND  ★ Major Concept as rubber—do not
Rubber move freely. Such ma-
The Earth often serves as a charge reservoir −
known
−−− −−as−a − − −. −
ground A ground
− − − can − −
A.− − − − − − − − − − −

− B. −
s insulators. When a surplus of charged particles (positive or nega- − − − − − −
ome part of an insulator, the excess remains
accept or provide electrons freely, and it is so large
there. So, if you hold
− that−the

−− − −−−−−
addition or subtraction
− −
of electrons has a negligible effect on it. So, the −ground
Figure
− remainsA.essentially
23.11  When youneutral
rub a copper rod with a cloth, − −B.
ubber rod while the far end is being rubbed with silk − connected to the ground through −
at all times. The copper Silk rod in Figure 23.11 the is− Earth. Excess electrons quickly −
far end of the rod will acquire a surplus of electrons SilkWhen and those − −− −
your body. something is connected to the ground by a conductor, we say that
flow through copper rod − −
in at that end, never flowing into your hand (Fig. 23.10B). − −
A. it is grounded. Every building B.(with a contemporary electrical intosystem)
your hand.has a wire − − − −
(a)
connected between the electrical system and a copper pipe. The copper Copyright pipe is
2017 Cengage Learning. All Rights Reserved.

Figure
This part 23.10  A.Excess
of the rod A.When electrons the is rubbed with a silkB.cloth, B. the excess (b)
stay onrod
a rubber charge stays on the C.
remains neutral. far side of the rubber rod.
part of the rod that was in contact with the cloth.
Figure 23.11  A. When you rub a copper rod with a cloth, B. the excess charge moves all over the rod and C.
the Earth. −

If you − you− will
−−−try − to charge a copper rod in the same way, find
− that you can-
− −−− −− − − −
−−− −− Unless otherwise noted, all co

Major Concept
Copper not build up charge
− on the copper. Copper is an example
− of a conductor . A con-
−−−− − − −Reserved. May not be copied, scanned, or duplicated, in whole or in part.
ductor is −−a −− material
− − −in−which
− − − − − − −the
− − − − − charged
Copyright 2017 Cengage Learning. All Rights
− −− − −
− particles− (usually
− electrons)
Excess electrons
can flow

B. freely. When −−
(b) you hold one end of the copper rod and rub − − the other continue end with silk,
to flow
− − − through your body elec-
ubber rod is rubbed Silk electrons −
with a silk cloth, B. the − are transferred
excess chargeExcessstays on from
the
electrons
the silk
quickly
to the copper − − rod, and those excess
1.9: (a) Whentrons − −− − and into Earth.
ontact Figure
with the cloth. arerodfree
a rubber to flow.
is rubbed with Because (b) repel, the −
a silk cloth,likes
flow through copper rod electrons
− − − − move away from one
the excess charge stays on the part of the rodinto that was in contact − − −
another, which meansyour they
hand.travel through the rod into your hand (Fig. 23.11).
arge awith the cloth.
copper rod in the same way, you will find that you can-
The human
e on the copper. Copper is an example B. of abody is also a conductor, soC.charged particles move
conductor. A con- (b)
freely through
your body toward
al in which the charged particles (usually electrons) can flow the Earth. If there are no insulators between you and the
re 23.11 
hold one A. When you rub
end of the copper rod
Conductor a copper rod
ground—such with
and rub the other a cloth,
as end
when B. the
withyouexcess charge moves all over the rod
silk, are barefoot—the charge will continue to
Figure 1.10: (a) and
When C. through
you rub a your
copper bodyflow
rodinto
withinto
a cloth, (b) the
rth. excess charge moves all over the rod and (c) through your body into
ferred from the silk to the the copper rod,
Earth. and those
Asthea same excess
result, elec-
flow. If you trylikes
Because to charge
repel, athecopper rod in
electrons move away from way,youyou and
one
the copper
will find the Earth.
rod remain neutral despite the rod
Unless otherwise noted, all content on this page is © Cengage Learning.
that travel
eans they you cannot
throughbuild
being rubbed
up charge
the rod into your onhand withcopper.
the silk. Copper is an
(Fig. 23.11).
rs Concept
example of a conductor. The
A Earth
conductor
also a conductor, so charged particles move freely often
is a serves
material
through as ina charge
which reservoir known as a ground. A ground can
Copyright 2017 Cengage Learning. All Rights Reserved. May not be copied, scanned, or duplicated, in whole or in part. WCN 02-200-203

the charged particles accept


(usually or provide
electrons)
the Earth. If there are no insulators between you and the electrons
can flow freely,
freely. When and the
it issilk a negative
so large that the charge. addition If silk orissubtraction
rubbed against amber,
when you you hold one end of the
are barefoot—the copper
charge rod
will and
continue rub to the
flow other
into end with however, the
of electrons has a negligible effect on it. So, the ground remains essentially neutralsilk becomes positively charged, as electrons in
sult, silk,
you electrons are transferred
and the copper rod remainfrom neutralthe silk
despite to the
the copper
rod rod, this case pass from the silk to the amber.
at all times. The copper rod in Figure 23.11
silk.and those excess electrons are free to flow. Because like charges
is connected to the ground through
Table1.1, shows the relative charging due to rubbing for a
serves as a the
repel, charge reservoir
electrons
your
known
move
body.
away asfromWhen
a ground something
one. another,
A canis connected
ground which to the ground by a conductor, we say that
means variety of materials. This relative charging is known as tri-
lectrons freely, and it is so it is
large grounded.
that the addition Every
or building
subtraction
they travel through the rod into your hand (Fig.1.10). The boelectric charging. (with a contemporary electricalThe more system) plus has signsa associated
wire with a
egligible effect on it. So, the
human body is also connected ground remains
a conductor,between essentially
so charged neutral
the particles
electrical movesystem and athecopper
material, more readilypipe. The it gives copper pipe isand becomes
up electrons
opperfreely
rod inthrough
Figure 23.11
your isbody
connected
towardto the
the ground
Earth. through
If there are no positively charged. Similarly, the more minus signs for a ma-
omething is connected to the ground by a conductor, we say that terial, the more readily it acquires electrons. For example,
insulators between you and the ground—such as when you are
ery building (with a contemporary electrical system) has a wire
we know that amber becomes negatively charged when rubbed
n the barefoot—the charge
electrical system and awill continue
copper pipe. to flow
The into the
copper pipeEarth.
is As a
result, you and the copper rod remain neutral despite the rod against fur, but a greater
− negative charge is obtained if rubber,
being rubbed with silk. PVC, or Teflon − is − − with fur instead. In general, when
rubbed
− − −−
two materials in − Table
− 1 are rubbed together, the one higher
• The human body consists −−largely of salt water. Pure water − positively charged, and the one lower in the
−− −− −− in the list becomes
− −
is not a very good − −− −
conductor,− −but salt water,
− −with − −its−N a
+

− − −−− −− − list becomes negatively
− − −
and Cl ions, is.−Consequently,
− − − − − − − − − − − −
humans are reasonably − − charged.Excess The greater
electronsthe separation on
− the list, the greater the magnitude
− − − − continuethe
of charge.
to flow
− − − − − − conductors.− − − −
good − −
− − − − − − − − − Excess electrons through your body
Silk • Charging by − friction
− can be

− −
− Excess electrons
continue to flow quickly
− − − − andapplied only if at least one
into Earth.
through
flow your copper
through body rod body is−insulators.−
electrons quickly
Triboelectric −
Charging− − − − − − −
− − − −
and into Earth.
rough copper rod − − into your hand.
− −
ur hand. − − −are
In general, when two materials − rubbed together, the magni-
1.6.2 Grounding a Conductor
tude and sign of theB.charge that each material acquires depends C.
C.
on how strongly it holds onto its electrons. For example, if silk Earth is a conductor because of the presence of ions and mois-
When
e excessisyou rubmoves
rubbed
charge aagainst
copper rodthewith
glass,
all over the a cloth,
glass
rod and B. the
C. acquires
through a excess
your positive charge
body into charge.moves all over
It ture in it.the
It rod
is also C. through
and large enoughyour
thatbody into purposes it
for many
means electrons have moved from the glass to the silk, giving can be considered as a limitless reservoir of charge (i.e., the
Unless otherwise noted, all content on this page is © Cengage Learning.
Unless otherwise noted, all content on this page is © Cengage Learning.
y not be copied, scanned, or duplicated, in whole or in part. WCN 02-200-203
Copyright 2017 Cengage Learning. All Rights Reserved. May not be copied, scanned, or duplicated, in whole or in part. WCN 02-200-203
Triboelectric Charging
In general, when two materials are rubbed together, the magnitude and692   CHAPTER
sign of the 23 
charge that eachElectric
material Forces
acquires
depends on how strongly it holds onto its electrons. For example, if silk is rubbed against glass, the glass acquires a
1.6. positive
METHODS charge.OF
It means electrons have moved from the glass to the silk, giving the silk a negative charge. If silk is
CHARGING 9
rubbed against amber, however, the silk becomes positively charged, as electrons in this case pass from the silk to the
amber. −−
TableTable 1.2: Triboelectric
1. Triboelectric series series −−
−− −−

Material Relative charging
Conductor − −


with rubbing −
Air Most positive
Asbestos Insulating
Rabbit fur pedestal
Glass
Human hair, oily skin
mica A. (a) B.
Nylon

Wool (no charge) Figure 23.14 −−− A. −
− −
A conductor rests on top of an insulating pedes
Lead is transferred −−
from
− − the rod to the conductor. C. The excess charge
Cat Fur − −
Silk
pedestal prevents −the excess charge

Electronsfrom
are flowing into the Earth.
transferred to
Aluminum conducting spheres.
Paper Least positive
Charging by In
Cotton Least negative
Wood (b) There is another w
Amber equipment as in Fi
Rubber
Vinyl (PVC) −
Electrons quickly rod near but not t
spread out so they
Teflon − − are uniformly sphere are free to
Ebonite Most negative
− − distributed. side of the conduct
Table 1, shows the relative charging due to rubbing for a variety of materials. This relative charging is known as will flow to the gr

triboelectric charging. The more plus signs associated with a material, the more readily it gives up electrons and charge of the sphe
addition or subtraction of electrons has a negligible effect on
becomes positively charged. Similarly, the more minus signs for a material, the more readily it acquires electrons. For
it). example,
So, the we ground
know remains
that amberessentially neutral charged
becomes negatively when rubbed against fur, but a greater negative charge is electrons are lost f
at all times.
Grounding
obtainedaif conductor
rubber, PVC,means to provide
or Teflon is rubbedawith
conducting
fur instead. In general, when two materials in Table(c)1 are rubbed fact, you could re
path
together,
between the one
it and thehigher
Earth. in the list becomes
The copper rod positively charged, and
in Figure1.10 isthe one lower in the list becomes negatively charged. having to recharge
The greater
connected theground
to the separation on theyour
through list, the greater
body. the magnitude
When something of the charge.
Figure 1.11: (a) A conductor rests on top of an insulating In this process—
pedestal.
is connected to the ground by a conductor, we say that it is (b) When the conductor is touched by a charged rod, some charge
Grounding a Conductor The charge that is
grounded.
Earth isEvery building
a conductor (withof athe
because contemporary
presence of ionselectrical sys- inisit.transferred
and moisture from
It is also large the rod
enough thattofor
the conductor.
many purposes(c) The excess charge
tem)it has a wire
can be connected
considered between
as a limitless the electrical
reservoir of charge system and quickly
(i.e., the addition distributes
or subtraction over thehas
of electrons surface This
of the conductor.
a negligible effect Theis true whe
insulating
on it). pipe.
a copper So, theThe
ground remains
copper pipeessentially neutral through
is connected at all times. Grounding
other a conductor
pedestal preventsmeans to provide
the excess chargea from
conducting Figure
flowing into 23.14, in w
the Earth.
path between it and the Earth. The copper rod in Figure 3 is connected to the ground through your body. When ends up being nega
copper pipes to the Earth. This ensures that the third con-
something is connected to the ground by a conductor, we say that it is grounded. Every building (with a contemporary
nection on electrical
electrical sockets
system) has a wire is grounded.
connected Thethe
between connection to and a copper pipe. The copper pipe is connected through direct cont
electrical system
the ground keeps electrical appliances from building up excess • The uncharged body acquires the same sign of charge as
same sign as a resu
the charged body. The total charge is distributed between
charge.
the two bodies.
Quanta Classes: K-423A Sector K Ashiyana Colony Kanpur Road Lucknow. Mo 9453763058 7
− Polarized sphere
1.6.3 Charging by Direct Contact 1.6.4 −
Charging by Induction

− − ++−−
−− + −
You can charge the conductor by touching it with a charged There is another way to build up − charge+on a conductor.
− Sup-
pose you use the same equipment− −as in Figure1.11b.
+ − ThisElectrons
time, in
object such as a negatively charged plastic rod (Fig.1.11b). −
+
When the rod is in contact with the sphere, some of the elec- however, you hold the negatively charged −− near but
++rod notaway
move
trons are transferred from the rod to the sphere. Remember touching the sphere (Fig.1.12). Because the electrons negatively
in the c
that charge cannot flow through the rod because it is an in- sphere are free to move, they flow to the side opposite the
sulator, so you may need to roll the rod around the surface rod due to repulsion of negatively charged rod. If you ground
of the sphere in order to transfer charge from many parts of the side of the conducting sphere where Groundthere is a surplus of
the rod. If we bring a negatively charged conducting rod in electrons, those electrons will flow to the ground. When you
A.
contact of an uncharged conducting sphere, then, off course, remove the connection to the ground, keeping the charged rod
rolling is not required. In this case electron can easily trans- near the sphere, the net charge of the sphere is positive and
fer from charged rod to uncharged sphere. Because electrons the ground remains essentially neutral. No electrons

− are lost
No electrons −
move freely throughout the conductor and because likes repel, from the rod; it has the same charge
have been lost throughout
− the
++ process. C

− + conductor
the electrons quickly redistribute themselves in the conductor, In fact, you could reuse therod.
from rod to charge − another po
− +
moving as far apart as possible. For a spherical conductor, “as by same method without recharging the rod − again.
− +
far apart as possible” means that the electrons are uniformly This process is known as induction. In this++ process
+ the
distributed on the outside surface of the sphere. If you try charged rod never touches the sphere. The charge that is in-
to charge a conductor of another shape, the charge is again duced on the sphere has the opposite sign as the charged rod.
distributed on the outside surface, although for nonspherical Note:You can also charge two bodies by induction as
shapes the charge distribution is not uniform. follows- − − − − − − − −

Excess e
absorbed
10 CHAPTER 1. ELECTRIC CHARGE AND FIELD

− Polarized sphere connect another neutral conductor with it. Due to repulsion


− ++−−
of the negatively charged rod, some free electrons get trans-

−−
+
+ − ferred from the left conductor to the right conductor and due
− −

− + − Electrons in sphere to deficiency of electrons positive charge appears on the left
+ −
++ −− move away from conductor and on the right conductor, there is an excess of
negatively charged rod. electrons due to transfer from left conductor. Now disconnect
the connecting wire and remove the rod. The first conductor
will get negatively charged and the second conductor will get
Ground positively charged.
(a)



− Conducting wire
− ++
−− +
− + Electrons

− + Electrons flow transfer
+ − to ground.
++ −




− This is the

symbol for
ground. (a)
Charged spheres
(b)


No electrons −
have been lost −− ++ Conductor is
−− +
from rod. − + positive.

− +
+
++

Copyright 2017 Cengage Learning. All Rights Reserved. May not be copied, scanned, or duplicated, in whole or in part. WCN 02-2

− − − − − − − −
(b)
Excess electrons
absorbed by ground. Figure 1.13: Charging by induction

(c)
+ 1.6.5 Field emission
Electrons redistribute. + +
Surplus positive charge + + When electric field of large magnitude is applied near the metal
is uniform. + + surface then some electrons come out from the metal surface
+ due to electric force applied by external electric field and hence
the
Copyright 2017 Cengage Learning. All Rights Reserved. May not be copied, scanned, or metalingets
duplicated, wholepositively charged.
or in part. WCN 02-2

00-203
− − − − − − − − 1.6.6 Charging by Thermionic Emission
When the metal is heated at a high temperature then some
(d)
electrons of metals are ejected and the metal becomes posi-
Figure 1.12: (a) When a charged rod is held near a neutral con- tively charged.
ductor, the conductor becomes polarized. (b) The far side of the
conductor is connected to the ground, and some electrons flow out 1.6.7 Charging by Photoelectric Effect
of the conductor into the Earth. (c) The connection is removed.
(d) When the rod is removed, the electrons redistribute, leaving the When light of sufficiently high frequency is incident on metal
surplus positive charge uniformly distributed over the surface of the surface then some electrons gain energy from light and come
conductor. out of the metal surface and remaining metal becomes posi-
tively charged.
Important Points:
00-203
Take an isolated neutral conductor and then bring a charged 1. Neutral does not mean “chargeless”. A neutral body always
rod near it. In Fig.1.13 the the rod is negatively charged. Due has equal amount of positive and negative charge, i.e., there
to the charged rod, charges will induce on the conductor. Now, is a microscopic balance of –ve and +ve charge.
1.7. COULOMB’S LAW 11

r
2. To charge the bodies through friction one of them has to
be insulator.
q=0 q=0
3. Since charge cannot flow through insulators, neither con- F= 0
duction nor induction can be used to charge insulators, so (a)
in order to charge an insulator friction is used. Whenever r
an insulator is rubbed against a body, exchange of electrons
takes place between the two. This results in appearance of F
−q +q
equal and opposite charges on the insulator and the other Attracted
body. Thus, the insulator is charged. For example, rubbing
(b)
of plastic with fur, silk with glass causes charging of these r
things.
F F
+q +q
Repelled
1.7 Coulomb’s Law
(c)
Coulomb’s law gives the electric force acting between two r
point charges quantitatively. A point charge is a point like
F F
object with a non zero electric charge. Note that a point −q −q
like object is small enough that its internal structure is of Repelled
no importance. The electron can be treated as a point
(d)
charge, since there is no experimental evidence for any inter-
nal structure. The proton does have internal structure—it Figure 1.14: (a) There is no electrostatic force between neutral
contains three particles called quarks bound together—but, objects. (b) The electrostatic force between two oppositely charged
since its size is only about 10−15 m, it too can be treated objects is attrac-tive. (c) The electrostatic force between two posi-
as a point charge for most purposes. A charged metal tively charged objects is repulsive. (d) The electro-static force be-
sphere of radius 10 cm can be treated as a point charge tween two negatively charged objects is repulsive.
if it interacts with another such sphere 100 m away, but
not if the two spheres are only a few centimeters apart.
force is proportional to the product of masses of each of two
Coulomb’s Experiments Charles Augustin Coulomb (1736- interacting objects.
1806) measured electrical attractions and repulsions quantita- Mathematically, we can write,
tively and deduced the law that governs them. His apparatus, F ∝ |q1 ||q2 | and F ∝ r2
shown in Fig. 1.15. One small metal sphere is charged and On combining above two statements, we get-
|q1 ||q2 |
fixed in place. A pair of small metal spheres is attached to a F ∝
lightweight insulated rod. The rod is suspended from a torsion r2
spring of known torsion spring constant. |q1 ||q2 |
or F =k (1.3)
Imagine that the pair of spheres is initially uncharged. The r2
rod is twisted, bringing one of those spheres in contact with Here, k is a proportionality constant, its numerical
the charged fixed sphere and thus transferring charge. If the value depends on the system of units used. its value is
spheres are identical, half of the charge on the fixed sphere 8.99 × 109 N.m2 /C 2
will be transferred to the movable sphere. So these spheres re- In one significant figure, k = 9 × 109 N.m2 /C 2
pel each other, causing the rod to twist. The amount of twist
is measured. Because Coulomb knew the torsion spring con-
stant, he could calculate the force exerted between the spheres. Value of k in CGS Units
In many trials, Coulomb varied the amount of charge on the In SI units, k = 9.0 × 109 N.m2 /C 2
spheres as well as the torsion spring constant. From these tri- Here, we use the method of unit conversion. If n1 , u1 are nu-
als, he found that- meric value and unit respectively in SI, whereas n2 u2 in CGS,
“The electrostatic force acting between two charged spheres is then
directly proportional to the of product of magnitude of charges n1 u1 = n2 u2 , gives
and inversely proportional to the square of the distance between n = n u1 = 9.0 × 109 N.m2 /C 2
2 1 u2 dyne.cm2 /esu2
them.” 5 4 2 9 2
9 10 dyne.10 (cm) /(3×10 esu)
= 9.0 × 10 dyne.(cm)2 /(esu)2 =1
Like gravity, the electric force is an inverse square law force. Therefore, in CGS, k = 1 dyne(cm) /(esu)2 2

That is, the strength of the force decreases as the separation in-
creases such that the force is proportional to the inverse square Generally, Eq.(1.3) holds only for charged objects whose
of the separation r between the two point charges (F ∝ 1/r2 ). sizes are much smaller than the distance between them. We
The strength of the force is also proportional to the product often say that it holds only for point charges.
of magnitudes of charges i.e., (|q1 q2 |) just as the gravitational Direction of Electric Force The directions of the forces the
12 CHAPTER 1. ELECTRIC CHARGE AND FIELD

cant figures)

1
k= = 8.99 × 109 N · m2 /C2
4π0

For simplicity, we’ll often use the approximate value

1
Torsion k= = 9.0 × 109 N · m2 /C2
spring 4π0

With this choice of the constant k, Coulomb’s law can be


written as-

1 |q1 ||q2 |
F = (1.5)
4πo r2
Fixed
Electric force between two point charges of 1C each
Movable Scale for separated by 1 m apart in free space
measuring By Coulomb’s law, we have
rotation
|q1 ||q2 |
F =k
r2
Figure 1.15: Torsion balance designed by Coulomb. Two (blue)
spheres are attached to a rod suspended from a torsion spring. The On putting, the values of k, q1 , and r, we get- F = (9.9 ×
fixed (red) sphere is positively charged, and then one of the movable |1C||1C|
109 N.m2 /C 2 ) = (9.9 × 109 N
spheres is brought in contact with the charged, fixed one. Because (1m)2
they are both conductors, charge is shared between the spheres, which is a very large value. So, practically we use smaller
and the movable (blue) sphere becomes positive. Both spheres have charges, such as- micro coulomb (1µC = 10−6 C), nano
charge of the same sign, so they are repelled, which causes the rod coulomb (1nC = 10−9 C) and pico coulomb (1pC = 10−12 C)
to rotate. The rod’s rotation is measured using a scale (shown in Coulomb’s law in a dielectric medium: A nonconduct-
yellow). ing material (for example, air, glass, paper, or wood) is called
a dielectric. If two point charges q1 and q2 are placed at sepa-
ration r in a dielectric medium of electric permittivity , then
two charges exert on each other are always along the line join- the electric force acting between them is given by-
ing them. When the charges q1 and q2 have the same sign,
either both positive or both negative, the forces are repulsive; 1 q1 q2
when the charges have opposite signs, the forces are attractive. F = (1.6)
4π r2
In the SI system, the constant k is expressed in the following
form: If r or K denotes relative permittivity or dielectric constant
of the medium, then-
1 
k= (1.4) r or K = 0 or  = 0 K
4π0 On substituting this value of  in Eq.1.6, we get-
Although the choice of this form for the constant k appears
1 q1 q2
to make Coulomb’s law needlessly complex, it ultimately re- F = (1.7)
sults in a simplification of formulas of electromagnetism that 4π 0 K r2
are used more often than Coulomb’s law.
Here, 0 is called the electric permittivity of free space. Its For free space or air,  = 0 , therefore, K = 0 = 1
0

value, determined by the adopted value of the speed of light, Now, Eq.1.7 can also be written as
is 0 = 8.854 × 10 C /N · m
−12 2 2
1 q1 q2
F = √ ·
4πε0 (r K)2
Electric permittivity is the measure of resistance that is
encountered when forming an electric field (1.10) in a particu- which is similar to an expression for electric √ force between
lar medium. More electric permittivity means more resistance two charged particles placed at a separation r K in vacuum.
is offered by the medium in forming electric field inside the So, the separation r of charged particles placed in a dielectric
medium, i.e., the medium is more insulating. medium produces same electric
√ force if these charged particles
Inside a conductor the electric field is always zero, therefore, are placed at separation r K in vacuum.
its electric permittivity is infinite. Now, we calculate the electric force between two point
charges when the vacuum separation between the charges is
The constant k has the corresponding value (to three signifi- partially filled with a dielectric medium of dielectric constant
1.7. COULOMB’S LAW 13

Figure 1.16

(a)
K (Fig.1.16). Suppose two charges q1 and q2 are separated by
distance r in vacuum. If, we insert a dielectric slab of dielectric
constant K and thickness t(< r) between the charges, then, the
width of vacuum part between the charges will become (r − t)
and effective vacuum√ width corresponding to the thickness t of
dielectric slab is t K. Therefore, effective vacuum separation
between the charges can be written as-

rvacuum = (r − t + t K)
(b)
So, in this case, Coulombs law can be written as-
Figure 1.17: (a) Two point charges q1 and q2 of the same sign
F = 1 q1 q√
4πε0 (r−t+t K)2 ·
2
(1.8) exert equal and opposite repulsive forces on one another. The vector
~r12 locates q1 relative to q2 , and the unit vector r̂12 points in the
~12 is parallel to ~r12 . (b) The two charges
direction of ~r12 . Note that F
now have opposite signs, and the force is attractive. Note that F ~12
1.7.1 Vector Form of Coulomb’s Law
is antiparallel to ~r12
So far we have considered only the magnitude of the force
between two point charges determined according to Coulomb’s
law. Force, being a vector, has directional properties as well. F~21 = −F~12 (1.11)
In the case of Coulomb’s law, the direction of the force is
determined by the relative sign of the two electric charges. Thus, electric forces form an action-reaction pair.
As illustrated in Fig.1.17, suppose we have two like point The significance of Coulomb’s law goes far beyond the de-
charges q1 and q2 at positions ~r1 and ~r2 respectively. scription of the forces acting between charged particles. This
The position vector of charge q1 with respect to q2 is given by- law, when incorporated into the structure of quantum physics,
correctly describes-
~r12 = ~r2 − ~r1
1. the electrical forces that bind the electrons of an atom to
The unit vector in the direction of q1 from q2 is- its nucleus

~r12 ~r2 − ~r1 2. the forces that bind atoms together to form molecules, and
r̂12 = =
r12 |~r2 − ~r1 |
3. the forces that bind atoms and molecules together to form
Here, r12 represents the magnitude of the vector ~r12 , i.e., the solids or liquids.
separation between q1 and q2 .
Thus most of the forces of our daily experience that are not
The repulsive electric force exerted by charge q2 on q1 is given
gravitational in nature are electrical. Moreover, unlike New-
by-
ton’s law of gravitation, which can be considered a useful ev-
eryday approximation of the more basic general theory of rel-
1 q1 q2
~
F12 = 2 r̂12 (1.9) ativity, Coulomb’s law is an exact result for stationary charges
4π0 r12 and not an approximation from some higher law. It holds not
only for ordinary objects, but also for the most fundamental
Similarly, electric force on charge q2 due to charge q1 is given
“point" particles such as electrons and quarks. Coulomb’s law
by-
remains valid in the quantum limit (for example, in calculating
1 q1 q2
~
F21 = 2 r̂21 (1.10) the electrostatic force between the proton and the electron in
4π0 r21 an atom of hydrogen). When charged particles move at speeds
close to the speed of light, such as in a high-energy acceler-
Here r̂21 is a unit vector that points from charge q1 to charge
ator, Coulomb’s law does not give a complete description of
q2 ; that is, it would be the unit vector in the direction of poit
their electromagnetic interactions; instead, a more complete
charge q2 if the origin of coordinates were at the location of
analysis based on Maxwell’s equations2 must be done.
charge q1 . 2 Maxwell’s equations will be discussed in chapter “Electromagnetic

Since, r̂21 = −r̂12 , therefore- Waves"


14 CHAPTER 1. ELECTRIC CHARGE AND FIELD

Superposition Principle 6. The net Coulomb’s force on two charged particles q1 and
The vector form of Coulomb’s law is useful because it carries q2 separated by distance r in free space and in a dielectric
within it the directional information about F~ and whether the medium, are-
force is attractive or repulsive. Using the vector form is of
critical importance when we consider the forces acting on an 1 q1 q2
(i) F = (in vacuum or free space)
assembly of more than two charges. In this case, Eq.(1.11) 4π0 r2
would hold for every pair of charges, and the total force on 1 q1 q2
(ii) F 0 = (in the dielectric medium)
any one charge would be found by taking the vector sum of 4π0 K r2
the forces due to each of the other charges. For example, the F 
Therefore, 0 = =K
force on point charge q1 in an assembly would be- F 0
So, the dielectric constant (K) of a medium, is nu-
merically equal to the ratio of the force on two point
F~1 = F~12 + F~13 + F~14 + · · · (1.12)
charges in free space to that in the fully filled dielec-
tric medium.
where F~12 is the force on point charge q1 due to point charge
q2 , F~13 is the force on point charge q1 due to point charge q3 , 7. The law expresses the force between two point charges at
and so on. Equation 1.12 is the mathematical representation rest. In applying it to the case of extended bodies of finite
of the principle of superposition applied to electric forces. size care should be taken in assuming the whole charge of
It permits us to calculate the force due to any pair of charges a body to be concentrated at its ‘centre’ as this is true only
as if the other charges were not present. For instance, the for spherical charged body, that too for external point.
force F~13 that point charge q3 exerts on q1 is completely Although net electric force on both the charged particles
unaffected by the presence of point charge q2 . changes in the presence of dielectric but force due to one
charged particle on another charge particle does not depend
Note that- if the electromagnetic force were proportional to on the medium between them.
the square of the total source charge, then, the principle of
2
superposition would not hold, since (q1 + q2 ) 6= q12 + q22 (there 8. Coulomb’s law resembles Newton’s inverse square law of
would be an extra term “2q1 q1 " to consider). Superposition is gravitation, F = Gm1 m2 /r2 . Both are inverse square
not a logical necessity, but an experimental fact. laws, and the charge q plays the same role in Coulomb’s
The electric force on test charge q1 , due to the point source law that the mass m plays in Newton’s law of gravitation.
charge qi (say) (i 6= 1), not only depends on the position of qi , Main differences between the two forces are-
it also depends on both velocities and on the acceleration.
• Gravitational forces, as far as we know, are always
Since, an electromagnetic signal or effect produced by the
attractive, while electrostatic forces can be repulsive
source charge qi travels at the speed of light, so to find the
or attractive, depending on whether the two charges
effect of qi on q1 , we need the position, velocity, and accelera-
have the same or opposite signs.
tion of qi at some earlier time, when the signal left.
Hence, we can say that the principle of superposition does not • Electric force depends on the nature of medium be-
hold in many situations when the source charge is moving. In tween the charges while gravitational force does not.
this chapter, we will be considering only the stationary source • There is another important difference between the
charges, therefore, the principle of superposition is valid. two laws. In using the law of gravitation, we were
able to define mass from Newton’s second law, F =
ma, and then by applying the law of gravitation to
Key Points Regarding Coulomb’s Law
1. This is a fundamental law and is based on physical obser- known masses we could determine the constant G. In
vation. It is not logically derivable from any other concept. using Coulomb’s law, we take the reverse approach—
Experiments till today reveal its universal nature. we define the constant k to have a particular value,
and we then use Coulomb’s law to determine the ba-
2. Electric forces always form action-reaction pairs, i.e., the sic unit of electric charge as the quantity of charge
force which one charge exerts on the other is equal and that produces a standard unit of force.
opposite to the force which the other charge exerts on the For example, consider the force between two equal
first. charges of magnitude q. We could adjust q until the
force has a particular value, say 1 N for a separa-
3. The direction of force is always along the line joining the tion of r = 1 m, and define the resulting q as the
two charges. basic unit of charge. It is, however, more precise to
measure the magnetic force between two wires carry-
4. Electrostatic force between two point charges is indepen- ing equal currents, and therefore the fundamental SI
dent of presence or absence of other charges. electrical unit is the unit of current, from which the
unit of charge is derived.
5. The electric force is conservative, i.e., the work done in
moving a point charge once round a closed path under the EXAMPLE 4. If a charged body is placed near a neutral con-
action of Coulomb’s force, is zero. ductor, will it attract the conductor or repel it?
1.7. COULOMB’S LAW 15

SOLUTION If a charged body (+ve) is placed left side near


me mp
a neutral conductor, (−ve) charge will induce at left surface Fg = G 2
r
and (+ve) charge will induce at right surface. Remember that
× 10−11 N · m2 /kg2

the magnitude of induced charge produced on each side is al- = 6.67
ways less than the magnitude of the source charge, i.e., |+q| 9.11 × 10−31 kg 1.67 × 10−27 kg
 

(See EXAMPLE 59). Due to positively charged body nega- × 2


(5.3 × 10−11 m)
tive induced charge will feel attraction and the +ve induced
= 3.6 × 10−47 N
charge will feel repulsion. But as the negative induced charge
is nearer, so the attractive force will be greater than the repul- and from Eq. ii, the magnitude of the electric force is-
sive force. So, the net force on the conductor due to positively 1 e×e
charged body will be attractive. Similarly, we can chow that Fe =
4π0 r2
a negatively charged body also attracts a neutral body. From
= 8.99 × 109 N · m2 /C2

2
1.60 × 10−19 C
× 2
(5.3 × 10−11 m)
= 8.2 × 10−8 N
+q
The ratio of these forces is 8.2 × 10−8 N / 3.6 × 10−47 N =
 
Force of Force of
attraction repulsion 2.3×1039 . Thus the electric force overwhelms the gravitational
force. Since the gravitational force is insignificant compared
with the electric force, it can be neglected. The acceleration
of the electron is then
F 8.2 × 10−8 N
Figure 1.18 a= = = 9.0 × 1022 m/s2
m 9.11 × 10−31 kg

the above example, we can conclude that. “A charged body This is a very large acceleration. If it occurred along the elec-
can attract a neutral body.” tron’s motion instead of centripetally, such an acceleration
If there is attraction between two bodies then one of them could boost the electron’s velocity close to one-third of the
may be neutral. But if there is repulsion between two bodies, speed of light in only a femtosecond 10−15 s .
both must be likely charged. COMMENTS: Note that for the ratio of the electric force
So, “repulsion is the sure test of electrification” whereas at- and the gravitational force between the proton and electron,
traction is not. we would obtain the same immense value 2.3 × 1039 whatever
the separation between the two particles, since both are in-
EXAMPLE 5. Compare the magnitudes of the gravitational verse square forces. Also notice that for the given atomic-scale
force of attraction and of the electric force of attraction between distance, the electric force has a measurable value, the same as
the electron and the proton in a hydrogen atom. According to weighing an 8 µg mass, whereas the gravitational force is far
Newtonian mechanics, what is the acceleration of the electron? below the current limits of detection (the highest sensitivity
Assume that the distance between these particles in a hydrogen attained by a measurement of force is near 10 N ).
−20
−11
atom is 5.3 × 10 m. EXAMPLE 6. In Example 3, what is the magnitude of the
attractive force exerted by the electrons in a cup of water on
APPROACH Gravitational force between two masses m1
the protons in a second cup of water at a distance of 10 m?
and m2 separated by distance r, is given by-
APPROACH Substitute the values of charges, obtained
m1 m2 q q
Fg = G 2 (i) in Example 3, in Coulomb’s force formula F = k 1 2 and
r r2
simplify for Fe .
with G = 6.67 × 10 N.m /kg
−11 2 2
SOLUTION According to the preceding example, the charge
Electrostatic force between two point charges q1 and q2 on the electrons in the cup is −1.3 × 107 C and the charge on
separated by distance r, is given by- the protons is +1.3 × 107 C. If we treat both of these charges
q1 q2 as point charges, the force on the protons is
Fe = k 2 (ii)
r 1 q1 q2
F =
with k ≈ 9.0 × 10 N.m /C
9 2 2
4π0 r2
For electron-proton system- m1 = me = 9.11 × 10 kg, −31
 −1.3 × 107 C 1.3 × 107 C
 
9 2 2
q1 = −e = −1.6 × 10−19 C, m2 = mp = 1.67 × 10− 27kg and = 8.99 × 10 N · m /C
−16
(10 m)2
q2 = +e = +1.6 × 10 C 22
Substitute these values in Eq.i and ii and then compare Fg = −1.5 × 10 N
and Fe . This is approximately the weight of a billion billion tons! This
SOLUTION From Eq.i, we get- enormous attractive force on the protons is precisely canceled
16 CHAPTER 1. ELECTRIC CHARGE AND FIELD

by an equally large repulsive force exerted by the protons in between the centers of charge of the two spheres, decreases.
one cup on the protons in the other cup. Thus, the cups exert So, force of electrostatic attraction between the charges, in-
no net forces on each other. creases. But, if the spheres have identical charges, then due
to mutual repulsion between the charge of one sphere and the
EXAMPLE 7. The force of electrostatic repulsion between the other, the charge of of each sphere moves away from other
two fixed point charges separated by distance of 1m is F . Now sphere. So, in this case, the effective distance between the cen-
if we replace the two point charges by two metallic charged ters of charges of the two sphere, increases and hence force of
spheres each of radius 25cm having the same charges as that electrostatic repulsion between the charged conducting sphere
of given point charges. then compare the force of repulsion in decreases.
two cases. SOLUTION In 2nd case due to mutual repulsion, the
APPROACH The electrostatic force acting between two effective distance between their centre of charges will be
point charges separated by distance r is given by increased (rf > ri ) so force of repulsion decreases (as F ∝ r12 )

q1 q2
F =k
r2 EXAMPLE 8. Five Styrofoam balls A, B, C, D and E are
So, if magnitudes of charges q1 and q2 remains fixed, then used in an experiment. Several experiments are performed on
F ∝ r12 . the balls and the following observations are made:
In case of charged conducting spheres, due to mutual inter- (i) Ball A repels C and attracts B.
(ii) Ball D attracts B and has no effect on E.
Centre of charge of (iii) A negatively charged rod attracts both A and E.
an isolated charged For your information, an electrically neutral Styrofoam ball is
metal sphere very sensitive to charge induction and gets attracted consider-
ably, if placed nearby a charged body. What are the charges, if
any, on each ball?
Metal sphere
A B C D E
Insulated stand (A) + − + 0 +
(B) + − + + 0
(C) + − + 0 0
(a) Isolated charged metal sphere (D) − + − 0 0
ri APPROACH Balls can be considered like extended
rf bodies unlike just point particle. Also keep in mind that a
neutral body is attracted towards both positive as well as
negative charges. Now observe the given observations and
decide the sign of the balls.
Effective centres SOLUTION (C) From (i), as A repels C, so both A and C
of charges
must be likely charged. Either both are +ve or both are −ve.
As A also attract B, so charge on B should be opposite of A
(b) Centres of charges shift towards each or B may be uncharged conductor.
other From (ii) as D has no effect on E, so both D and E should
rf be uncharged and as B attracts uncharged D, so B must be
ri charged and D must be an uncharged conductor.
From (iii), a negatively charged rod attracts the charged ball
A, so A must be +ve and from experiment (i) C must also be
+ve and B must be −ve
Effective centres .
of charges
EXAMPLE 9. A particle of mass m carrying charge ‘+q1 ’
is revolving around a fixed charge ‘−q2 ’ in a circular path of
(c) Centres of charges shift away from radius r. Calculate the period of revolution.
each other
APPROACH To revolve the charged particle q1 of mass
Figure 1.19: Shifting of effective centers of charges when two m around the charge −q2 , with a constant angular speed
charged metal spheres placed close to each other
ω in a circular orbit of radius r, the required centripetal
force Fc = mrω 2 is provided by the  electrostatic force of

action between the charges, the effective distance (r) between
center of charges get altered, so the electrostatic force also get attraction of charge −q2 on charge q1 Fe = 1 q1 q2
4π0 r 2 . Here,
altered. If the charges of spherical conductors have opposite ω= T .

T is the time period of q1 around the charge −q2 .
nature, then due to electrostatic attraction, the charge of each So, the condition for the charged particle q1 to revolve around
sphere moves towards the other and hence the effective distance −q2 in a circular orbit of radius r, is Fe = Fc , i.e.,
1.7. COULOMB’S LAW 17

1 q1 q2
= mrω 2 (1.13)
4π0 r2
C
SOLUTION From the above condition (1.13), we have- A B
1 q1 q2
= mrω 2
4π0 r2
Since, angular speed ω = 2π T, therefore, we have
2
1 q1 q2 2π
= mr
4π0 r2 T
1 q1 q2 4π 2 mr
or =
4πε0 r2 T2
2 2

(4πε ) r 4π mr πε0 mr
r
0 (a) (b)
or T = 2
or T = 4πr
q1 q2 q1 q2
here the vector ~r is drawn from source charge (−q2 ) to charge Figure 1.21
q1

EXAMPLE 10. A ring of radius R has uniformly distributed to tension of the ring is 2∆T sin dθ2 . In mechanical equilib-
charge q. A point charge Q is placed at the centre of the rium, this force will be equal to electrostatic repulsion Fe of
ring[Fig.1.20]. Q at center on the segment of the ring under consideration.
(a) Find the increase in tension in the ring after the point Therefore,-  
charge is placed at its center. dθ
2∆T sin = Fe (i)
(b) Find the increase in force between the two semicircular 2
parts of the ring after the point charge is placed at the cen- If λ is the linear charge density of the ring, then charge on the
ter. segment ACB will be-
(c) Using the result found in part (b) find the force that the dq = lengthACB × λ = (Rdθ)λ
point charge exerts on one half of the ring. kQdq kQRdθ
Therefore, Fe = 2
= , here R is the radius of the
R R2
ring.
Since, dθ/2 is a very small quantity, therefore, sin dθ/2 = dθ/2
.
On substituting these values in Eq.i, we get-

dθ kQλRdθ
or 2∆T · =
2 R2
kQλ kQq
∆T = =
R 2πR2
Figure 1.20 (b)From Fig.1.21b it is clear that the answer is 2∆T = kQq
πR2
(c) Answer to (b) must be the answer to (c) also.

EXAMPLE 11. Three charges q1 = 1µC, q2 = −2µC and


APPROACH If there is no charge at the center, there is some
q3 = 3µC are placed on the vertices of an equilateral triangle
tension in the ring due to repulsion of charge present on the
of side 1.0 m [Fig.1.22]. Find the net electric force acting on
ring. (a) When Q is placed at the center the tension increases
charge q1 .
by ∆T . The direction of ∆T is tangential to the ring at each
point on it. Now, consider any infinitesimally small portion
ACB(say) of the ring and resolve components of ∆T , at both
ends, in to two mutually perpendicular directions- one compo-
nent towards center of the ring whereas other perpendicular
to it. In equilibrium, the magnitude of net component of ∆T
towards center of the ring will be equal to the magnitude of
outward electrostatic force on the segment ACB.
SOLUTION Let the infinitesimally small segment ACB of Figure 1.22
the ring subtends a very small angle dθ at the center of the
ring (see Fig.1.21a) . From Fig.1.21a, it is clear that hori-
zontal components of ∆T at each end A and B is ∆T cos dθ 2 .
APPROACH Charge q2 will attract charge q1 (along the
These components are opposite in directions so get canceled. line joining them) and charge q3 will repel charge q1 [Fig.1.23].
Vertical components of ∆T at each end A and B is ∆T sin dθ 2 .
Therefore, two forces will act on q1 , one due to q2 and another
These two components are directed towards the center of the due to q3 . Since, the force is a vector quantity both of these

→ −

ring. Therefore, net inward force on the segment ACB due forces (say F 1 and F 2 ) will be added by vector method.
18 CHAPTER 1. ELECTRIC CHARGE AND FIELD

Now you can apply any one of the following methods of vector
addition.
SOLUTION Method 1. In the figure 1.23, the magnitude
of force between
− q1 and q2 1
→ q1 q2
F 1 = F1 = ·
4πε0 r2

Figure 1.24

1 q1 q3
F~2 = · (~r1 − ~r3 )
4πε0 |~r1 − ~r3 |3
9.0 × 109 1.0 × 10−6 3.0 × 10−6
  
= ×
(1.0)3
[(0−0.5)î+(0−0.87)ĵ +(0−0)k̂]
Figure 1.23 ⇒ F~ = (−1.35î − 2.349ĵ) × 10−2 N
Therefore, net force on q1 is-
9.0 × 109 1.0 × 10−6 2.0 × 10−6
  
= F~ = F~1 + F~2
(1.0)2
= 1.8 × 10−2 N = (0.45î − 2.349ĵ) × 10−2 N

Similarly, the magnitude of force between q1 and q3 . Note: Once you write a vector in terms of î, ĵ and k̂, there
− is no need of writing the magnitude and direction of vector
→ 1 q1 q3 separately.

F 2 = F2 = ·
4πε0 r2
9.0 × 109 1.0 × 10−6 3.0 × 10−6
   EXAMPLE 12. Four identical particles, each having charge
= +q, are fixed at the corners of a square of side L. A fifth point
(1.0)2
charge −Q lies a distance z along the line perpendicular to the
= 2.7 × 10−2 N plane of the square and passing through the center of the square
q (Fig.1.25). (a) Show that the force exerted by the other four
Now, F~net = F12 + F22 + 2F1 F2 cos 120◦ charges on −Q is

s !

→ 4ke qQz

1
= (1.8)2 + (2.7)2 + 2(1.8)(2.7) − × 10−2 N F =− 3/2

2 [z 2 + (L2 /2)]
= 2.38 × 10−2 N
Note that this force is directed toward the center of the square
F2 sin 120◦ whether z is positive (−Q above the square) or negative (−Q
and tan α =
F1 + F2 cos 120◦ below the square). (b) If z is small compared with L, the above
2.7 × 10−2 (0.87) expression reduces to F~ ≈ −( constant )z k̂. Why does this

= imply that the motion of the charge −Q is simple harmonic,
(1.8 × 10−2 ) + (2.7 × 10−2 ) − 12

and what is the period of this motion if the mass of −Q is m ?
or α = 79.2◦
APPROACH 1 In first approach we break the forces applied
Thus, the net force on charge q1 is 2.38 × 10−2 N at an angle by all positive charges +q on −Q into horizontal and vertical
α = 79.2◦ with a line joining q1 and q2 as shown in the figure. components and then add these components to find the net
Method 2. In this method let us assume a coordinate axes force on Q .
with q1 at origin as shown in figure. The coordinates of q1 , q2 SOLUTION (a) The distance from√each corner to the center
and q3 in this coordinate system are (0, 0, 0), (1 m, 0, 0) and of the square of side length L, is L/ 2
(0.5 m, 0.87 m, 0 ) respectively. From Fig. 1.26, the distance from each positive charge to −Q
Now, force on charge q1 due to charge q2 is- is-
1 q1 q2
F~1 = · (~r1 − ~r2 ) r
4πε0 |~r1 − ~r2 |3 L2
r = z2 +
9.0 × 109 1.0 × 10−6 −2.0 × 10−6
  
2
= ×
(1.0)3 Each positive charge exerts a force directed along the line join-
  [(0 − 1)î + (0 − 0)ĵ + (0 − 0)k̂] ing +q and −Q, of magnitude
⇒ ~
F = 1.8 × 10 î N−2
Qq kQq
and force on q1 due to charge q3 is given by- F0 = k = 2
r2 z + L2 /2
1.8. EQUILIBRIUM OF CHARGED PARTICLES 19

Figure 1.27

q1 = q2 = q3 = q4 = +q, ~r1 = Lĵ, r~2 = Lî r~3 = −Lî; ~r4 = −Lĵ


Figure 1.25 ~rp = z k̂
∴ ~rp − ~r1 = z k̂ − Lî,
p 1/2
|~rp − ~r1 | = z 2 + L2 = z 2 + l2

→ 1/2 and
r − ~r = z k̂ − Lĵ, |−
p 2

r − ~r | = z 2 + L2
p 2

→r p − ~r3 = z k̂ + Lî, |~rp − ~r3 | = z 2 + L2
1/2
1/2
r~p − ~r4 = z k̂ + Lĵ, |~rp − ~r4 | = z 2 + L2

→ − → −→ −
→ −

F = F1+ F2+ F3+ F4
q1 q0 (r~p − r~1 ) q2 q0 (r~p − r~2 )
⇒ ~F = 3 + 3
4πε0 |r~p − r~1 | 4πε0 |~rp − r~2 |
q3 q0 (r~p − r~3 ) q4 q0 (r~p −~r4 )
+ 3 + 3
4πε0 |~rp − r~3 | |~rp − r~4 |
Putting the values, we get

→ qQ
F =− 3/2
{(z k̂ − Lî) + (z k̂ − Lĵ) + (z k̂ + Lî)
Figure 1.26 4πε0 (z 2 + L2 )
+ (z k̂ + Lĵ)}
The line of force makes an angle θ with the z-axis. From qQ
=− (4z k̂)
Fig.1.26 4πε0 (z 2 + L2 )
3/2

z Since, 4πε
1
0
= k, therefore, above expression can also be
cos θ = p written as-
z 2 + L2 /2
The four charges together exert forces whose horizontal com-
ponents F 0 sin θ add to zero, while the vertical z-components −
→ 4kqQz
F =− 3/2
(k̂)
F 0 cos θ add to (z 2 + L2 )


F = 4F 0 cos θ(−k̂)


1.8 Equilibrium of Charged Particles
4kQqz
⇒ F =− k̂
(z + L2 /2)
2 3/2
If in a system of charged particles, the net electric force on
every particle is zero, then they are said to be in equilibrium.
APPROACH 2 In second approach [Fig.1.27], we use In 1842, British mathematician Samuel Earnshaw proved that
Coulomb’s law in vector form and then apply the superpo- a collection of point charges cannot be maintained in a stable
sition principle of electric force stationary equilibrium configuration solely by the electrostatic
F~ = F~1 + F~2 + E ~ 3 + F~4 interaction of the charges. Discussion of this statement is
q1 q0 (~rp − − →
r 1) q2 q0 (~rp − ~r2 ) q3 q0 (r~p − ~r3 ) given in "Earnshaw’s theorem" in section 1.19.
= −
→ 3 + −
→ 3 +
4πε0 |−→ 3
4πε0 | r p − ~r1 | 4πε0 | r p − r~2 | r p − ~r3 |
q4 q0 (~rp − ~r4 )
+ 3 1.8.1 Equilibrium of Three Point Charges
4πε0 |~rp − ~r4 |
SOLUTION The arrangement of charges is shown in Fig. Suppose, three positive point charges q1 , q2 and q are placed
1.27. along a line as shown in adjoining figure. r is the separation
According’ to given problem, between charges q1 and q2 . Charge q is placed at distance x
20 CHAPTER 1. ELECTRIC CHARGE AND FIELD

from charge q1 . EXAMPLE 13. Two particles A and B having charges 8 ·


For the equilibrium of all the three charges, the net electro- 0 × 10−6 C and −2 · 0 × 10−6 C respectively are held fixed with a
separation of 20 cm. Where should a third charged particle be
[ placed so that it does not experience a net electric force?
T T T
U APPROACH Since, the net electric force on charge C should
be equal to zero, the force due to A and B must be equal and
Figure 1.28 opposite in direction. Hence, the particle should be placed on
the line AB. As A and B have charges of opposite signs, C
static force on each of the above charges should be zero. cannot be between A and B. Also, A has larger magnitude
Now, for the equilibrium of charge q, the net electrostatic of charge than B. So, we expect the charge C to be closer to
force on it should be zero, i.e., Fq = 0 B, the the charge that has smaller magnitude, than A. The
situation is shown in figure 1.29
q1 q q2 q
or k 2 − k =0 (1.14)
x (r − x)2
Two charges must be of like nature as Similarly, for the equi-
librium of charge q1 ,we have Figure 1.29
Fq1 = 0, i.e., −k qr1 q22 − k qx12q = 0
SOLUTION Suppose BC = x and the charge on C is Q.
q1 q2 q1 q −6
or k +k 2 =0 (1.15) The force due to A = (8·0×10 C)Q2 .
r2 x 4πε0 (20 cm+x)
(2·0×10−6 C)Q
The force due to B = 4πε0 x2 .
From Eq. (1.14), we have They are oppositely directed and to have a zero resultant, they
should be equal in magnitude. Thus, or,
 2
q1 q2 r−x q2
2
= 2
⇒ =
x  (r −x) x q1 8 2
r−x
r
q2 2
= 2
⇒ =± (20 cm + x) x
x r q1 20 cm + x
r  q2 = 2, ⇒ x = 20 cm.
⇒ −1 =± x
x q1
r q2
r
⇒ =1± 1.8.2 Equilibrium of Symmetric Geometrical
x q1 Point Charged System
r
⇒x= q (1.16) Figure 1.30 shows an equilateral triangle and a square of side
1 ± qq21 length a. Point charges, each of magnitude q, are placed at
each corner and a point charge Q at their centers. If the
From Eq. (1.16), it is clear if q1 and q2 are unlike charges, then
systems are in electrostatic equilibrium, then net electrostatic
x will be imaginary. So, for a real value of x, both q1 and q2
force on each point charge will be zero.
must be like charges, i.e., either both of them must be positive
To determine the value of charge Q placed at the center,
or both of them must be negative.
Now, from Eq.(1.15), we have q
q1 q2 x2 q2 2
q=− 2 = − 2x
r q1 r
on substituting the value of x, we get
a Q a
 2

q2 r
q=− 2 q  (1.17)
r 1 ± qq21 q a q
(a) (b)
Negative sign in above Eq.(1.17) shows that, q and q2 will be
opposite in nature. But q2 and q1 are like charges, therefore, Figure 1.30
the nature of q is also opposite to q1 .
So, the result is that the nature of q1 and q2 will be similar always apply the condition of equilibrium (i.e., ΣF = 0) at
where as that of q is opposite to q1 and q2 any corner charge.
(i ) Calculation of Q Placed at the Center of Triangle:
Eq. (1.16)
√ and (1.17) can be written as- In Fig.1.31, three similar charges, each of magnitude q are
q1 −q1 q2 placed at three vertices of an equilateral triangle ABC of side
x= √ √ r and q = √ √ 2
q1 ± q2 q1 ± q2 a. A fourth charge Q is placed at the center O of the triangle.
respectively. From Fig.1.31, the distance OA = (a/2) sec 30◦ = √a3 .
1.8. EQUILIBRIUM OF CHARGED PARTICLES 21

2F cos30° r jˆ r
FCO
r
FAC
r r
FAO FAB FCB
°
r
FAB
r
FAC F
y 45° F
r
30° 30
D a C 45r° CA iˆ
A F sin30° q q
F sin30
a/2
q FCD
a a
30° sec 30° =
2 3
a a
O Q a O a
Q
Bq a qC
q
Figure 1.31 q
A a B x
The electrostatic forces acting on charge q at A are- Figure 1.32
1. The force F~AB (along BA) due to charge q at B.
2. The force F~AC (along CA) due to charge q at C. q √
In equilibrium, ΣF~ ⇒ √ +q+Q 2=0
3. The force F~AO (along OA) due to charge Q at center O. 2 2
2
Note that, |F~AB |= |F~AC |= k aq 2 = F (say) √ 
The horizontal components of each of these two forces are −q 2 2 + 1
⇒ Q= (1.20)
F sin 30◦ and are oppositely directed, therefore they get can- 4
celled with each other. However, the vertical components due
Here, again negative sign shows that Q will be opposite in
to each force is F cos 30◦ and directed along OA, therefore the
nature to q.
get added. So, the resultant of these forces will be 2F cos 30◦
Students are advised to keep results 1.18 and 1.20 in their
(along OA)
memory for solving such types of problems in a quick way.
The electrostatic force applied by the charge Q at center O on
charge q at A is- EXAMPLE 14. Two particles A and B having charges
and |F~AO |= k (a/Qq

3)2
= k 3Qq
a2 8 × 10−6 C and −2 × 10−6 C respectively are held fixed with
Forces FAO and 2F cos 30◦ both are directed along OA, a separation of 20 cm. Where should a third charged particle
therefore, net force on A- √ be placed so that it does not experience a net electric force?
3Qq q2 3
ΣF = FAO + 2F cos 30◦ = k 2 + k 2
a a
In equilibrium, ΣF = 0

q
⇒ Q = −√ (1.18)
3
Figure 1.33
Here, negative sign indicates that Q will be opposite in nature
to q.
(ii ) Calculation of Q Placed at the Center of Square: APPROACH As the net electric force on C should be equal
Four equal charges, each of magnitude q, are placed at the to zero, the force due to A and B must be opposite in direction.
vertices of square a ABCD of side a and a charge Q is placed Hence, the particle should be placed on the line AB. As A and
at it’s center O (Fig.1.32). Considering AB along the positive B have charges of opposite signs, C cannot be between A and
direction of x axis and AD along the positive direction of y B.
axis.
Also A has larger magnitude of charge than B. Hence, C
The electrostatic forces acting on charge q at C are-
should be placed closer to B than A. The situation is shown
1. Force F~CA , due to charge q at A
in adjoining figure.
2. Force F~CB , due to charge q at B
SOLUTION Suppose BC = x and the charge on C is Q
2. Force F~CD , due to charge q at D −
→ −6
1 (8.0×10 )Q
2. Force F~CO , due to charge Q at O F CA = 4π∈ 0 (0.2+x)2 î
Net electrostatic force on charge q at C, is- −
→ −6
−1 (2.0×10 )Q
and F CB = 4π∈ 0 x2 î

ΣF~ = F~CA + F~CB + F~CD + F~CO (1.19) F~C = vecF CA + F~CB


(8.0 × 10−6 )Q (2.0 × 10−6 )Q
 
2
  1
Here,F~CA = k (a√
q ~CB = k q22 ĵ, F~CD
√ ĵ , F
î+ 2
= k aq 2 î, = − î
2)2 2 a  4π∈0 (0.2 + x)2 x2
and F~CO = qQ î+ĵ 2qQ
k (a/√2)2 √2 = k a2 î+ √ ĵ
2 But |F~C |= 0 h
On substituting these values in
 Eq.(1.19), we get-   (8.0×10−6 )Q (2.0×10−6 )Q
i
q

q √

q √ Hence 4π∈ 1
(0.2+x)2 − x2 =0
ΣF~ = k 2
0
√ + q + Q 2 î + √ + q + Q 2 ĵ which gives x = 0.2m
a 2 2 2 2
22 CHAPTER 1. ELECTRIC CHARGE AND FIELD

1.8.3 Equilibrium of Suspended Point We can eliminate the tension from the problem by taking the
Charge System ratio of these equations, yielding

EXAMPLE 15. A simple electroscope for the detection F = mg tan 30◦


and measurement of electric charge consists of two small
foil-covered cork balls of 1.5 × 10−4 kg each suspended by From Fig.1.34a, we see that the distance between the balls is
threads 10 cm long (see Fig.1.34). When equal electric charges r = 2l sin 30◦ , so Coulomb’s Law tells us
are placed on the balls, the electric repulsive force pushes
them apart, and the angle between the threads indicates the 1 q2
F =
magnitude of the electric charge. If the equilibrium angle 4π0 (2l sin 30◦ )2
between the threads is 60◦ , what is the magnitude of the charge?
Equating these two expressions for F , we find

1 q2
mg tan 30◦ =
4π0 (2l sin 30◦ )2

and

q = 4π0 mg tan 30◦ × 2l sin 30◦


p

= [ (4π) 8.85 × 10−12 C2 /N · m2 1.5 × 10−4 kg


 

1/2
× 9.81 m/s2 (tan 30◦ ) ] × (2)(0.10 m) (sin 30◦ )


=3.1 × 10−8 C

EXAMPLE 16. Two identically charged spheres are sus-


pended by strings of equal length L. The charge on each sphere
is q and mass is m. Find the separation between them in
equilibrium- (a) if the the system is located in gravitational
field of earth, (b) if the system is located in gravity free space.
(a)

(b)
Figure 1.35
Figure 1.34: (a) Two equal charged balls suspended by threads.
(b) “Free-body” diagram for the right ball.
APPROACH Make FBD of any one sphere. Resolve all
forces acting on it in to two mutually perpendicular compo-
APPROACH Write equilibrium conditions for FBD of any nents along x and y direction (say). If ΣFx and ΣFy are the net
one ball in air and liquid. Simplify for dielectric constant K. forces along x and y directions, then, in equilibrium, ΣFx = 0
SOLUTION Figure 1.34b shows a “free-body” diagram for and ΣFy = 0.
one of the balls. The electric force F acts along the line join- If whole setup is placed in gravity free space, then there will be
ing the two charges and is thus horizontal. In equilibrium, no downward gravitational force on the sphere. In this case,
the vector sum of the electric repulsion F, the weight w, and the angle between the strings becomes 180◦ .
the tension T of the thread must be zero. Accordingly, the SOLUTION FBD of left sphere is shown in Fig.1.36. The
horizontal component of the tension must balance the electric forces acting on the left sphere are-
repulsion, and the vertical component of the tension must bal- 1. Electrostatic force Fe (along negative x direction)
ance the weight: 2. Gravitational force mg (along negative y direction)
F = T sin 30◦ 3. tension applied by string T (away from sphere along the
mg = T cos 30◦ string)
1.8. EQUILIBRIUM OF CHARGED PARTICLES 23

x and y components of tension T are T sin θ and T cos θ respec- and in liquid and then solve for dielectric constant.
tively. x is the separation between the spheres in equilibrium. SOLUTION Each ball is in equilibrium under the following
Net force on the sphere, along x axis is- three forces:
ΣFx = T sin θ − Fe (i) tension,
Net force on the sphere, along y axis is- (ii) electric force and
ΣFy = T cos θ − mg (iii) weight So, Lami’s theorem can be applied.
In equilibrium, The weight of ball in air, W = mg = V ρg
The upthrust applied by liquid is, Fth = V σg
ΣFx = 0 ⇒ T sin θ = Fe (i) Therefore, the weight of the ball in liquid,
W 0 = W − Fth = V ρg − V σg = V (ρ − σ)g
and ΣFy = 0 ⇒ T cos θ = mg (ii) The electrostatic force between the balls in liquid-
Fe
On dividing Eq. (i) by (??), we get- Fe0 =
Fe K
tan θ = where, Fe is the electrostatic force between the balls in air and
mg K = dielectric constant of liquid.
q2
Here, electrostatic force, Fe = k 2 , therefore,
x
kq 2
tan θ =
mgx2
If θ is small then-

(a) In vacuum (b) In liquid

Figure 1.38

Applying Lami’s theorem in vacuum


W Fe
Figure 1.36 =
sin (90◦ + θ) sin (180◦ − θ)
W Fe
x x kq 2 ⇒ = (1.21)
tan θ ≈ sin θ = ⇒ = 2 cos θ sin θ
2L 2L x mg
2kq 2 L h 2 i 13 Similarly in liquid,
3 q L
⇒ x = ⇒ x = 2π∈ 0 mg
mg W0 F0
(b) If whole set up is taken into a gravity free space or in an = e (1.22)
cos θ sin θ
artificial satellite (gef f ' 0), then from FBD shown in Fig.1.37
T = Fe = 4L kq 2 Dividing Eq.(1.21) by Eq. (1.22), we get-
2
F W V ρg
0
= 0 = (V = volume of ball )
F W V (ρ − σ)g
ρ
or K=
ρ−σ
Note: In the liquid Fe and W have changed. Therefore, T will
also change.
Figure 1.37
EXAMPLE 18. For the system shown in Fig., find Q for
which resultant force on q is zero.
EXAMPLE 17. Two identical balls each having a density ρ
are suspended from a common point by two insulating strings APPROACH Make FBD of charge q. Write force balance
of equal length. Both the balls have equal mass and charge. In equations and simplify for required value of q.
equilibrium each string makes an angle θ with vertical. Now, SOLUTION For force on q to be zero, charges q and Q
both the balls are immersed in a liquid. As a result the angle must be of opposite nature. Net attraction force on q due to
θ does not change. The density of the liquid is σ. Find the charges Q = Repulsion force on q due to q
dielectric constant of the liquid. Net attraction force on q due to charges Q = Repulsion force
on q due to q.
APPROACH Make FBD of any one ball in air as well as in √ √ kQq kq 2
2FA = FR ⇒ 2 2 = √
liquid. Write the force balance equations for equilibrium in air a ( 2a)2
24 CHAPTER 1. ELECTRIC CHARGE AND FIELD

Figure 1.39


⇒ q = 2 2Q. (a) In air (b) In liquid

Therefore, the required charge = −2 2Q
Figure 1.41

EXAMPLE 19. Two identically charged spheres are sus-


pended by strings of equal length. The strings make an angle 3. Tension in the string T 0 , acting upward along the string.
of 30◦ with each other in air (Fig.1.40). When suspended in a 4. Upthrust of liquid Fth , acting in upward direction
liquid of density 0.8 g/cc the angle remains same. What is the In equilibrium, all forces and their components acting on left
dielectric constant of liquid. Density of sphere = 1.6 g/cc. charged sphere, are shown in Fig.1.41(b)
In horizontal direction we have-
T 0 sin 15◦ = Fe0 (iv)
In vertical direction we have-
T 0 cos 15◦ = mg − Fth (v)
On dividing Eq. (iv ) by Eq. (v ), we get-
Fe0
tan 15◦ = (vi)
mg − Fth
From Eq. (iii) and (vi), we can write-
Fe Fe0
=
mg mg − Fth
Fe Fe /K
Figure 1.40 ⇒ = [∵ Fe0 = Fe /K]
mg mg − Fth
mg
⇒ K=
APPROACH Make FBD of any one ball in air and in liquid. mg − Fth
Write translational equilibrium conditions in both case. Now mg
⇒ K= [∵ Fth = V ρl g]
simplify for dielectric constant K of liquid. mg − V ρl g
SOLUTION In air: The forces acting on the left charged 1 1
sphere are- ⇒ K= V ρl
= [∵ m = V ρs ]
1− m 1 − VV ρρsl
1. Gravitational force mg, acting in downward direction,
2. Electrostatic repulsive force Fe , acting towards left, 1
⇒ K= 0.8 = 2
3. Tension in the string T , acting upward along the string. 1 − 1.6
In equilibrium, all forces and their components acting on left
EXAMPLE 20. Two small equally charged spheres, each of
charged sphere, are shown in Fig.1.41
mass m, are suspended from the same point by silk threads of
In horizontal direction we have-
length l. In equilibrium, the distance between the spheres is
T sin 15◦ = Fe (i) x (x << l). Find the rate dq/dt with which the charge leaks√
In vertical direction we have- off each sphere if their approach velocity varies as v = a/ x,
T cos 15◦ = mg (ii) where a is a constant.

On dividing Eq. (i ) by Eq. (ii ), we get- APPROACH Initially, the charged spheres were in equilib-
Fe rium when they were separated by distance x. Now consider
tan 15◦ = (iii) only left sphere in equilibrium. The forces acting on this
mg
sphere are-
In liquid: When set up is immersed in the liquid medium as 1. Gravitational force m~g - acting downwards.
shown in Figure 1.41(b), the forces on the sphere are- 2. Tension applied by thread (T )- acting away along the
1. Gravitational force mg, acting in downward direction, thread.
2. Electrostatic repulsive force Fe0 , acting towards left, 3. Electrostatic repulsion Fe applied by second sphere- acting
1.8. EQUILIBRIUM OF CHARGED PARTICLES 25

leftwards.
If ΣFx and ΣFy are the net electrostatic forces acting on
the left sphere along x and y direction respectively, then, in
equilibrium-
(i) ΣFx = T sin θ − Fe = 0 ⇒ T sin θ = Fe , and
(ii) ΣFy = T cos θ − mg = 0 ⇒ T cos θ = mg
On dividing condition (i) by (ii), we get-

Figure 1.43
l
T cosθ r
APPROACH All charges ( 1 to 7) are applying electrostatic
x forces simultaneously on charge at A. Therefore by applying
T sin θ superposition principle, you can find the net force on tharge at
2x
mg A. SOLUTION The net force on particle A can be given by
vector sum of force experienced by this particle due to all the
Figure 1.42 other charges on vertices of the cube. For this we use vector

→ −
→ −

form of coulomb’s law F = − → −
kq1 q2
→ 3 ( r 1 − r 2 )
r 1− r 2
Fe
tan θ = (1.23) From


the figure the different forces acting on A are given as-


mg 2
F A1 = kq (−ak)
2
, F A2 = kq ((−a√
ĵ−ak̂)
,
a3 2a)3
But from the Figure1.42, −
→ kq 2 (−aî−aĵ−ak̂) −→ kq 2 (−aî−ak̂)
F A3 = √
( 3a)3
, F A4 = (√2a)3
x ∼ x 2 2 2
tan θ = q = as x << l (i) F~A5 = kq (−a î)
, F~A6 = kq ((−a
√ , FA7 = kq (−a
î−aĵ) ~ ĵ)
x
 2 2l a3 2a)3 a3
2 l2 − 2 By superposition principle, the net force experienced by A can
q2 be given as-
and Fe = (ii) ~
4πε0 x2 Fnet = F~A1 +  F~A2 + F~A3 + F~A4 + ~ ~ ~
 FA5 + FA6 + FA7
−kq 2 1 1
Now, substitute the values obtained in Eq.i and ii in Eq. 1.23 = √ + √ + 1 (î + ĵ + k̂)
and simplify for q. From here you will get a relation between a2 3 3 2
q and x, i.e., q = f (x) in equilibrium. EXAMPLE 22. Five point charges, each of value +q are
Differentiation of this relation with respect to time ‘t’, will give placed at five vertices of a regular hexagon of side L m. What
you dq/dt, i.e., the rate of leakage of charge. is the magnitude of the force on a point charge of value −q
SOLUTION On substituting the values tan θ and Fe from coulomb placed at the center of the hexagon?
Eq.i and ii in Eq. 1.23, we get-
x q2 APPROACH If there had been a sixth charge +q at the
= 2 remaining vertex of hexagon force due to all the six charges on
2l 4π0 x mg
−q at O will be zero (as the forces due to individual charges
2πε0 mgx 3 will balance each other). Now if f~ is the force due to sixth
or q2 = (iii)
l
On differentiating Eq.(iii) with respect to time ’t’, we get-

dq 2πε0 mg 2 dx
2q = 3x
dt l dt
dx a
According to the problem = v = √ ( approach velocity
dt x
is dx so,

dt
Figure 1.44

1/2 −

charge and F due to remaining five charges. F~ + f~ = 0 ⇒

2πε0 mg 3 dq 3πε0 mg 2 a
x = x √
l dt l x F~ = −f~ ⇒ F = f
Now, by using Coulomb’s law, find f
Hence, q1 q2 1 q2
r SOLUTION f = k 2 =
dq 3 2πε0 mg r 4π0 L2
= a
dt 2 l 1 q×q q2
= =
4πε0 L2 4πε0 L2
EXAMPLE 21. Given a cube with point charges q on each of
its vertices. Calculate the force exerted on any of the charges EXAMPLE 23. A particle A having a charge of 5 · 0 × 10−7 C
due to rest of the 7 charges. is fixed in a vertical wall. A second particle B of mass 100 g
26 CHAPTER 1. ELECTRIC CHARGE AND FIELD

and having equal charge is suspended by a silk thread of length ⇒ θ + α + α = 180◦


30 cm from the wall. The point of suspension is 30 cm above ⇒ 2α = 180◦ − θ
the particle A. Find the angle of the thread with the vertical ⇒ α = 90◦ − θ/2
when it stays in equilibrium. Again in right angle ∆BLA, α + 90◦ + 6 LBA = 180◦
⇒ 6 LBA = 90◦ − α = 90◦ − 90 − θ2
APPROACH The situation is shown in figure 1.45. Suppose
= θ2 = 6 CBX (opposite angles)
the point of suspension is O and let θ be the angle between the
All angles between forces on the particle are shown in Fig.1.45.
thread and the vertical. Forces on the particle B are
By Lami’s theorem at B, we have-
(i) weight mg downward Fe mg
=
sin(180 − θ) sin(θ + 90◦ − θ2 )
Fe mg F mg
⇒ = ◦ θ
⇒ e =
sin θ sin 90 + 2 sin θ cos θ2
Fe mg
⇒ 2 sin θ2 cos θ2
= cos θ
2
θ Fe
⇒ sin = ... (i )
2 2mg
From sine rule, we have-
AB OA AB OA
=  θ
 ⇒ =
sin θ sin 90 − 2 sin θ sin α
OA sin θ
⇒ AB =
sin α
OA 2 sin θ2 cos θ2 2OA sin θ2 cos θ2

AB = ◦ θ
=
cos θ2

Figure 1.45 sin 90 − 2
θ
(ii) tension T along the thread and ⇒ AB = 2OA sin
(iii) electric force of repulsion F along AB. 2
Electrostatic force between A and B is,
For equilibrium, these forces should add to zero. q2 q2 q2
SOLUTION In this case, we solve the problem by using Fe = k (AB)2 = k 2 =k
2OA sin θ2 4OA sin2 θ2
2

Lami’s theorem.3 The distance between A and B can be ob-
tained by sine rule4 Substituting this value in Eq.(i ), we get-
SOLUTION The FBD of problem is shown in Fig.1.45. Let sin θ = k q2
B is the equilibrium position of ball and 6 AOB = θ. Let us 2 4(OA) sin2 θ2 (2mg)
2
q2
draw a perpendicular BL on side OA. ⇒ sin3 θ2 = k 8(OA) 2 mg
Note that OA = OB, so ∆OAB is an isosceles triangle. On substituting given values in above expression, we get-
Therefore, 6 OAB = 6 OBA = α(say) 3 θ 9 × 109 × 25 × 10−14 N
So, in ∆OAB, AOB + OAB + OBA = 180
6 6 6 ◦ sin =
2 4 × 9 × 10−2 × 2 × (100 × 10−3 kg) × 9 · 8 m s−2
3 Lami’s Theorem: If P ~, Q~ and R~ are three concurrent forces such = 0 · 0032
θ
~ ~ ~ ~
that the angle between Q and R is α, between R and P it is β and between ⇒ sin = 0 · 15 ⇒ θ = 17◦
~ ~
P and Q the angle is γ (Fig.1.46a), then in equilibrium- 2
P Q R EXAMPLE 24. A particle ‘A’ having a charge of 2 × 10−6 C
= =
sin α sin β sin γ and a mass of 100 g is fixed at the bottom of a smooth inclined
4 Sine Rule: In any ∆ABC(Fig.1.46b) , we have-
plane of inclination 30◦ . Where should another particle B,
having same charge and mass be placed on the incline so that
it may remain in equilibrium?

(a) Lami’s Theorem (b) Sine Rule


Figure 1.47

Figure 1.46
APPROACH Make FBD for B and apply the condition of
a b c translational static equilibrium along and perpendicular to the
= = inclined plane of Fig.1.47
sin A sin B sin C
here a, b and c are sides against angles A, B and C respectively. SOLUTION Suppose, the equilibrium distance of particle B
1.8. EQUILIBRIUM OF CHARGED PARTICLES 27

from A is x. FBD of particle B, is shown in Fig.1.47.


The forces acting on the particle B are-
1. Gravitational force: mg (downwards)
2. Normal reaction of the plane: N (perpendicular to the plane
in upward sense)
3. Electrostatic repulsion of A: Fe (along the plane in upward
sense)
We resolve the gravitational force mg into components— one Figure 1.49
(mg sin 30◦ ) along the plane and other (mg cos 30◦ ) perpendic-
ular to it.
In stastic equilibrium, we have- and on q2 to get
kq 2
Along the plane: mg sin 30◦ = Fe = 2
x N2 + Fe cos α = mg cos 60◦ + T cos α, (1.26)
mg kq 2
⇒ =
2 s x2 Fe sin α + mg sin 60◦ = T sin α (1.27)
2kq 2
⇒ x= On simplifying equations (1.25) and (1.27), we get α = 60◦
mg
and T = mg + Fe
On substituting the given values- q = 2 × 10−6 C, m = 0.1 kg
Now, substitution√of these values in equations (1.24) and
and k = 9.0 × 109 N.m^2/C^2, we get -
(1.26) gives N1 = 3mg and N2 = mg.
x = 0.27 m = 27 cm
The tension becomes zero when the cord is cut. Substituting
EXAMPLE 25. A rigid insulated wire frame in the form of T = 0 in T = mg + Fe gives Fe = −mg i.e., q1 q2√= −4π0 l mg
2

a right angled triangle ABC, is set in a vertical plane as shown Ans. (a) (i) 60 (ii) 4π0 l2 + mg (iii) 3mg, mg (b)
◦ 1 q1 q2

in the figure. Two beads of equal masses m each and carrying q1 q2 = −4π0 mgl2
charges q1 and q2 are connected by a cord of length l and can
slide without friction on the wires. Considering the case when
the beads are stationary determine. EXAMPLE 26. Three particles, each of mass 1 g and carry-
(a) (i) The angle α. (ii) The tension in the cord. (iii) The ing a charge q, are suspended from a common point by insulated
massless strings, each 100 cm long. If the particles are in equi-
A librium and are located at the corners of an equilateral triangle
of side length 3 cm, calculate the charge q on each particle.
[Take g = 10 m/s2 .
q1  q2
l APPROACH Make FBD of any one particle and apply the
condition of translational equilibrium
 60  SOLUTION Let the three charged particles, each of charge q,
30
B C be located at the three corners of an equilateral triangle of side
a = 3 cm in the horizontal plane. The distance of the charges
Figure 1.48
from the centroid O is r. Apply law of cosines in triangle OBC
to get √ √
normal reaction on the beads.
r = a/ 3 = 3 cm .
(b) If the cord is now cut what are the values of the charges for
which the beads continue to remain stationary? Consider the electrostatic force on the charge at A due to
APPROACH Make FBDs of both the beads. Write trans-
lational equilibrium conditions along and perpendicular to
their respective planes and then simplify them for required
variables.
SOLUTION The forces acting on the charge q1 are its weight
mg, normal reaction N1 , string tension T and Coulomb’s
force Fe = q1 q2 / 4π0 l2 . Similarly, forces on the charge
q2 are its weight mg, normal reaction N2, string tension
T and Coulomb’s force Fe = q1 q2 / 4π0 l2 The forces are
shown in the figure. In triangle ABC, 6 A = 90◦ . Resolve
the forces on q1 and q2 in the directions parallel and perpen-
dicular to the sides of the frame. Balance the forces on q1 to get

N1 + Fe sin α = mg cos 30◦ + T sin α (1.24)



Fe cos α + mg sin 30 = T cos α (1.25) Figure 1.50
28 CHAPTER 1. ELECTRIC CHARGE AND FIELD

the charges at B and C. The magnitude of forces due to each 2. • Can two similarly charged bodies attract each other?
charge is
q2 3. • Does in charging the mass of a body change?
|F~ |=
4πε0 a2 4. • Why a third hole in a socket provided for grounding?
and their directions are as shown in the figure. Resolve F~ along
5. • Two balls, separated by some distance, carry equal
and perpendicular to AO. The resultant electrostatic force on
electric charges and exert a repulsive electric force on each
charge at A is given by
other. If we transfer a fraction of the electric charge of one
√ 2
3q ball to the other, will the electric force increase or decrease?
~ ◦
Fe = 2F cos 30 ı̂ = ı̂
4πε0 a2
The charges are connected with strings of length l = 100 cm. 6. •• What would be the interaction force between two copper
spheres, each of mass 1g, separated by the distance 1 m, if
the total electronic charge in them differed from the total
charge of the nuclei by one percent?

7. •• Two point charges A and B, halving charges +Q and


−Q respectively, are placed at certain distance apart
and force acting between them is F . If 25% charge of A
is transferred to B, then force between the charges becomes-
(A) 4F
3 (B)F (C) 9F
16 (D) 16F
9

8. •• Suppose the charge of a proton and an electron differ


slightly. One of them is −e, the other is (e + ∆e). If the net
of electrostatic force and gravitational force between two
hydrogen atoms placed at a distance d (much greater than
atomic size) apart is zero, then ∆e is of the
 order of [Given:
mass of hydrogen mh = 1.67 × 10−27 kg
(A) 10−23 C (B) 10−37 C (C) 10−47 C (D) 10−20 C
9. •• Two identical charged spheres suspended from a com-
Figure 1.51 mon point by two massless strings of lengths l, are initially
at a distance d(d << l) apart because of their mutual
Let the other end of each string is hanging from the point P , repulsion. The charges begin to leak from both the spheres
which is vertically above O (see figure). In triangle OAP, at a constant rate. As a result, the spheres approach each
√ other with a velocity v. Then v varies as a function of the
r 3 distance x between the spheres, as
tan θ = √ =√ = 0.017
l2 − r 2 1002 − 3
The forces on the charge at A are Fe , tension T , and weight (A)v ∝ x−1/2 (B)v ∝ x−1 (C) v ∝ x1/2 (D) v ∝ x
mg. Resolve T in the horizontal and the vertical directions.
10. •• Three identically charged, small spheres each of mass
The equilibrium condition on charge at A gives
√ 2 m are suspended from a common point by insulated light
3q strings each of length l. The spheres are always on vertices
T sin θ |= Fe = 2
, of an equilateral triangle of length of the sides x(<< l).
4π0 a
T cos θ = mg The rate dq/dt with which charge on each sphere increases,
if length of the sides of the equilateral triangle increases
Divide equation (1) by (2) and simplify to get slowly according to law dx dt =
√a , is-
x
q q
3πε 2 πε0 mga2
4π0 a2 mg tan θ
1/2
(A) 0 mga
(B)
 
l l
q= √ q q
πε0 mga2 2πε0 mga2
3 (C) 3 (D) l
" #1/2
2 −3 Six charges are kept at the vertices of a regular hexagon

(0.03) 10 (10)(0.017) 11. ••
= = 3.17 × 10−9 C. as shown in the Fig.1.52. If magnitude of force applied by
9 × 109 (1.73)
+Q on +q charge is F, then net electric force on the +Q is
nF. Find the value of n.
1.9 Check Point 2
1. • Suppose that the electric force between two point charges 12. •• Two pith balls carrying equal charges are suspended
is attractive. What can you conclude about the signs of from a common point by strings of equal length, the
these point charges? equilibrium separation between them is r. Now the strings
1.9. CHECK POINT 2 29

result of the three electric force contributions from the


vertex charges, the fourth charge
(A) Is in equilibrium and remains at rest
(B) Is pushed toward the center of the triangle
(C) Is pushed outside the triangle
(D) Is in unstable equilibrium

19. •• Two stiff non conducting rods have length L each and
Figure 1.52 have small balls connected to their ends. The rods are
placed parallel to each other and the balls are connected
by two identical springs as shown. When each ball is given
are rigidly clamped at half the height. The equilibrium a charge q, the system stays in equilibrium when it is in the
separation between the balls now become [Fig.1.53] shape of a square. If natural relaxed length of each spring
is L/2 find the force constant (k) for them.
   2  
(A) √ 2r
(B) 2r (C) √12 (D) √ r

3 3 3
2

Figure 1.53

13. •• Two positive ions, each carrying a charge q, are sepa- Figure 1.54
rated by a distance d. If F is the force of repulsion between
the ions, the number of electrons missing from each ion
will be (e being the charge on an electron) 20. •• A charge Q is to be divided on two objects. What
2
q
2 should be the values of the charges on the objects so that
(A) 4πεe02F d (B) 4πεd02F e the force between the objects can be maximum?
q
4πε0 F d2 4πε0 F d2
(C) e2 (D) q2

14. •• When air is replaced by a dielectric medium of constant 21. •• Will a point charge have a position of stable equilibrium
K, the maximum force of attraction between two charges when it is at the mid-point between two other equal point
separated by a distance charges, the signs of which are either the same or opposite
(A) increases K times (B) remains unchanged to that of the first charge?
(C) decreases K times (D) increases K times.
−1

15. •• A charge q is placed at the centre of the line joining two 22. •• Three equal charges q of the same sign are located at
equal charges Q. The system of the three charges will be the vertices of an equilateral triangle. What charge Q of
in equilibrium if q is equal to opposite sign must be placed at the center of the triangle in
(A) −Q/4 (B) Q/4 (C) −Q/2 (D) Q/2 order for the resultant of the forces acting on each charge
to be zero?
16. •• Point charges +4q, −q and +4q are kept on the X -axis
at point x = 0, x = a and x = 2a respectively. Then
(A) only −q is in stable equilibrium Answer Key and Solutions
(B) all the charges are in stable equilibrium
(C) all of the charges are in unstable equilibrium 1. For point charges, the two charges must have opposite signs.
(D) none of the charges is in equilibrium.
2. Yes, when the charge on ope body (q1 ) is much greater than
that on the other (q2 ) and they are close enough to each
17. •• A positively charged body ‘A’ attracts a body ’B’ then other so that force of attraction between q1 and induced
charge on body ’B’ may be: charge on the other exceeds the force of repulsion between
(A) positive (B) negative (C) zero (D) can’t say q1 and q2 . However, two similar point charges can never
attract each other because no induction will take place here.
18. Three identical point charges are at the vertices of an
equilateral triangle. A fourth, identical point charge is 3. Yes, as charging a body means addition or removal of elec-
placed at the midpoint of one side of the triangle. As a trons and electron has a mass.
30 CHAPTER 1. ELECTRIC CHARGE AND FIELD

4. All electric appliances may end with some charge due to


faulty connections. In such a situation charge will be ac-
cumulated on the appliance. When the user touches the
appliance, he may get a shock. By providing the third hole
for grounding all accumulated charge is discharged to the
ground and the appliance is safe.

5. Consider any two equal charges q, with an electric force Figure 1.55
F = kq 2 /r2 . If we transfer a charge δ from one to the
other, the charges become (q + δ) and (q − δ), so the force
becomes F 0 = k(q+δ)(q−δ)/r2 . But (q+δ)(q−δ) = q 2 −δ 2 , will act between two hydrogen atoms. The gravitational
which is less than q 2 , so the force decreases. force between two hydrogen atoms is given as
Gmh mh
6. Total number of atoms in the sphere of mass 1 gram Fg = (1.31)
d2
mass of copper sphere Since, the net force on the system is zero, Fe = Fg Using
N= × Avogadro number
molar mass of copper Eq. (1.30) and (1.31), we get
1 (∆e)2 Gm2
= × 6.023 × 1023 = d2 h or, (∆e)2 = 4πε0 Gm2h
4πεo d2
63.54 −11
2 
× 1.67 × 10−27 1/ 9 × 109

= 6.67 × 10
So the total nuclear charge ∆e ≈ 10−37 C

Q1 = number of atoms × atomic number 9. (B) T cos θ = mg (1.32)


i.e., Q1 = N × 29
kq 2
6.023 × 1023 T sin θ = (1.33)
= × 1.6 × 10−19 × 29 x2
63.54
Now the charge on the sphere = total nuclear charge − total
electronic charge

6.023 × 1023 29 × 1
= × 1.6 × 10−19 ×
63.54 100
= 4.298 × 102 C

Hence, force of interaction between these two spheres,


2
4.398 × 102

F =k· N
12
= 9 × 109 × 104 × 19.348 N = 1.74 × 1015 N

7. (C) : In case I : Figure 1.56


1 Q2
F =− (1.28)
4πε0 r2
kq 2
Q Q From Eqns. (1.32) and (1.33), tan θ =
In Case II : QA = Q − , QB = −Q + x2 mg
4 4 x

Q

Q
 Since θ is small, ∴ tan θ = sin θ =
1 Q− 4 −Q + 4 2l
∴ F0 = x kq 2 mg
4πε0 r2 ∴ = 2 ⇒ q 2 = x3
2l x mg 2lk
3 −3
 
1 4Q 4 Q 1 9 Q2
⇒ F0 = =− (1.29) or q ∝ x3/2
4πε0 r 2 4πε0 16 r2
dq 3 √ dx 3√
From equations (1.28) and (1.29), F 0 = 9 ⇒ ∝ x = xv.
16 F dt 2 dt 2
dq 1
8. (B): A hydrogen atom consists of an electron and a pro- Since, = constant ∴ v ∝ √
ton. ∴ Charge on one hydrogen atom = qe + qp = dt x
−e + (e + ∆e) = ∆e Hence, the correct answer is option (B).
Since a hydrogen atom carries a net charge ∆e ∴ Elec-
trostatic force, 10. (A) Solve as above problem
1 (∆e)2
Fe = (1.30) 11. Fnet = 9 F n=9
4πεo d2
1.9. CHECK POINT 2 31

Divide Eq.(1.37) by (1.36), we get


2r0 r2 r3 r
= 02 ⇒ r3 = ⇒ r0 = √
r r 2 3
2

13. (C): According to Coulomb’s law, the force of repulsion


between the two positive ions each of charge q, separated
by a distance d is given by-
2
1 (q)(q)
F = 4πε 0 d2 or F = 4πεq 0 d2 q 2 = 4πε0 F d2
p
Figure 1.57 or q = 4πε0 F d2 (1.38)
Since, q = ne where, n = number of electrons missing from
each ion, e = q
magnitude of charge on electron
q 4πε0 F d2
∴ n= e = e2

14. (C): Fm = F0
K i.e., decreases K times
15. (A) The situation is as shown in the Fig.1.60.
Let two equal charges Q each placed at points A and B at

Figure 1.58
Figure 1.60

12. (D) : Let m be mass of each ball and q be charge on each a distance r apart. C is the centre of AB where charge q is
ball. Force of repulsion, placed.
1 q2 For equilibrium, net force on charge Q = 0
F = 1 QQ 1 Qq
∴ 4πε 2 + 4πε (r/2)2 = 0
4πε0 r2 0 r
2
0
1 4Qq
or 1 Q
= − 4πε
In equilibrium, we have 4πε0 r 2 0 r
2

or Q = −4q or q = −Q
4
T cos θ = mg (1.34) 16. (C) Net force on each of the charge due to the other charges
T sin θ = F (1.35) is zero. However, disturbance in any direction other than
along the line on which the charges lie, will not make the
Divide Eq.(1.34) by (1.34), we get,
1 q charges return.
F 4πε0 r 2
tan θ = = 17. A. B, C
mg mg
From Fig.1.58,
1 q 18. (C) Is pushed outside the triangle. Like charges repel.
r/2 2
The forces from the two charges at vertices adjacent to the
= 4πε0 r (1.36)
y mg fourth charge are equal but opposite and cancel. The net
From Fig.1.59, we have force is thus the same as the contribution from the charge
at the opposite vertex; that repulsion will push the fourth
charge away, out of the triangle.
19. Consider the equilibrium of charge at A F = force due to
2
other charge at distance L = kq
L2
2
Force due to diagonally opposite charge √kq
( 2L)2
= F2
The
 resultant
 force in X direction Fx = F + F2 cos 45◦ =
1
F 1 + 2 √2
 
This gets balanced by the spring force. k · L2 = F 1 + 1

2 2
√ !
Figure 1.59 q2 2 2+1
kL = √
4π ∈0 L2 2
2 2 √ !
1 q
r0 /2
1 q q2 2 2+1
0
tan θ = 4πε0 r 02
⇒ = 4πε0 r 02
(1.37) ∴ k= √
mg y/2 mg 4π ∈0 L3 2
32 CHAPTER 1. ELECTRIC CHARGE AND FIELD

2 2+1
22. Q = e (the equilibrium is unstable).
4
e
23. Q = √ (the equilibrium is unstable).
3

1.10 The electric field


The concept of a field was developed by Michael Faraday. An
Figure 1.61 electric field is said to exist in the region of space around a
charged (source) object, if another test charge (q0 ) experiences
an electric force (Fe ) in it.
20. APPROACH 1. Suppose the charge on one object is Every position in a field is assigned a particular value of some
q and on the other is Q − q. Since, the electric force quantity. For example, a temperature field is a scalar field
between these charges depends on the magnitude of q, and every position is associated with a temperature. If the
therefore, for maximum force between the charges, we have- field is a vector field (for example electric field), each
position in the field is associated with a particular magnitude
dF and direction. Always remember that-
=0
dq
1. The source charge sets up an electric field, that influences
So, first find the expression for electric force F and then its surroundings.
use above expression and solve for q.
2. When a test charge placed in the electric field of source
SOLUTION . The force between the objects is
charge, it interacts with the electric field of the source
q(Q − q) (qQ − q 2 ) charge. The force exerted on the test charge depends on
F =k =k properties of the field and the test charge.
r2 r2
If we place a point test charge (q0 ) in a field of charged metal
where r is the separation between them.
dF d

(qQ − q 2 )
 
(Q − 2q)
 plate, then the charged metal plate is called source and test
Now, = k = k charge is the object.
dq dq r2 r2
In analogy with the gravitational field, we define the electric
For maximum force, dq = 0, i.e.,
dF
~ associated with a certain collection of charges in terms
  field E
k (Q−2q)
r 2 = 0 or q = Q/2 Thus, the charge should be of the force exerted on a positive test charge q0 at a particular
divided equally on the two objects. point, as

F
E= (1.39)
21. The equilibrium will be unstable. q0
Explanation We take the case when the central charge is i.e., the electric force per unit test charge is called the electric
of opposite sign to the two other charges. If this charge is field at the position of test charge. The direction of the
moved slightly along the line joining the three charges, the vector E ~ is the same as the direction of F~ , because q0 is a
attraction due to the nearer charge increases, whilst that positive scalar. Dimensionally, the electric field is the force
due to the more distant charge diminishes, with the result per unit charge, and its SI unit is the newton/coulomb (N/C),
that the charge moves still further from the equilibrium although it is more often given in the equivalent unit of
position. Its equilibrium is therefore unstable. volt/meter (V/m). The term volt will be discussed in next
If the central charge is of the same sign as the other two, chapter “Electric Potential”.
and it moves slightly along the line joining the charges, Note the similarity with the gravitational field, in which g
forces will arise that tend to return it to its equilibrium (which is usually expressed in units of m/s2 can also be ex-
position. However, if it moves at right angles to the line pressed as the force per unit mass in units of newton/kilogram.
joining the charges, the resultant of the repulsions will no Both the gravitational and electric fields can be expressed as a
longer be zero and will act in the direction in which it has force divided by a property (mass or charge) of the test body.
moved. As a result the charge will tend to move further In Fig.1.62, the charge Q, which is producing the electric
from its equilibrium position. The equilibrium is thus field, is called a source charge and the charge q0 , which tests
unstable. the effect of a source charge, is called a test charge. Note
This result, which we have obtained for an elementary case, that the source charge Q must remain at its original location.
is always valid. If only Coulomb forces of interaction are However, if a charge q0 is brought at any point around Q,
present in a system of free electric charges, the equilibrium Q itself is bound to experience an electrical force due to q0
is always unstable. and will tend to move. To sort out this difficulty, take q0
negligibly small. The force F~ is then negligibly small but the
1.10. THE ELECTRIC FIELD 33

conductor. It produces an induced charge over the surface of


the conductor and this surface induced charge also produces
an electric field in space. If we move this point charge q,
continuously, near the surface, the amount and distribution
both of the induced surface charge changes continuously with
time. So electric field produced by it also changes with time.
Therefore, this induced electric field is time dependent and
hence it is non-conservative.
Figure 1.62

EXAMPLE 27. Suppose someone discovers that blue and



→ −

yellow objects attract each other, that two blue objects repel
ratio E = lim F
q0 is finite and defines the electric field.
q0 →0 each other, and that two yellow objects repel each other. The
This limit in actuality cannot be taken to 0 because the test strength of this “chromatic interaction” is found to depend on
charge can never be smaller than the elementary charge e. If color depth: The deeper the color, the greater the magnitude
we are calculating (rather than measuring) the electric field of the interaction. How would you define the magnitude and
due to a specified collection of charges at fixed positions, direction of the “chromatic field” of an object?
neither the magnitude nor the sign of q0 affects the result.
Also, note that an electric field is a property of its source, the SOLUTION The gravitational field is defined as the grav-
presence of the test charge is not necessary for the field to itational force per unit of mass, with the field direction the
exist. The test charge is used to detect the electric field. same as the direction of the force. The electric field is defined
When using E = q0 we must assume that the test charge q0 is as the electric force per unit of charge, with the field direction
F

small enough that it does not disturb the charge distribution parallel to that of the force exerted on a positively charged
responsible for the electric field. If vanishingly small test particle. Therefore, the chromatic field can be defined as
charge q0 is placed near a uniformly charged metallic sphere, the chromatic force per unit of color, with the field direction
as in Fig. 1.63a (a) the charge on the metallic sphere, which parallel to that of the force exerted on a particle carrying
produces the electric field, remains uniformly distributed. If some chosen color.
the test charge is great enough (q00 >> q0 ), as in Fig. 1.63b
the charge on the metallic sphere is redistributed and the
EXAMPLE 28. What happens when a high energy X-ray
ratio of the force to the test charge is different. If in this case,
beam falls on a small metal ball suspended in a uniform electric
the electric force on the test charge q00 is F 0 , then-
F 0
F field with the help of an insulated thread?
=
6
q0 0 q0 SOLUTION Since, the high energy X-rays cause ejection
That is, because of this redistribution of charge on the metallic of the electrons from the metal ball (photoelectric effect),
sphere, the electric field it sets up is different from the field it therefore, the ball gets positively charged. Now, this positively
sets up in the presence of the much smaller test charge q0 . charged ball is deflected in the direction of electric field.
Note: 1. It is the surrounding charges (source) that create

Why concept of electric field necessary?


Modern understanding of electric interaction between two
charges is visualized in terms of the electric field concept.
A charge produces an electric field around itself; this field
then exerts force on the other charge. Thus, the interaction
between two charges is a two step process.
For two charges, the measurable quantity is the force on a

Figure 1.63

an electric field at a given point. Any positive or negative


charge placed at the point interacts with the field and, as a Figure 1.64
result, experiences a force.
2. Since, electric force is conservative in nature, therefore charge which can be directly determined using Coulomb’s law.
the static electric field produced by any point charge is also Why then introduce this intermediate quantity called
conservative. A conservative force or field must be a function the electric field?
of only spatial coordinates. The work done by such a force When charges are stationary, the concept of electric field
or field depends only on start and ending location in the is convenient, but not really necessary. Electric field in
field, and not on the actual path taken between those points. electrostatics is an elegant way of characterizing the electrical
Now, suppose we place a point charge q near the surface of a environment of a system of charges.
34 CHAPTER 1. ELECTRIC CHARGE AND FIELD

Electric field is a characteristic of the system of charges and inward (i.e., opposite to r̂ ) if q is negative.
is independent of the test charge that you place at a point to If q is negative, as in Figure1.65(c), the force on the test
determine the field. The term field in physics generally refers
to a quantity that is defined at every point in space and may q0
vary from point to point. Electric field is a vector field, since
r
r
F E
force is a vector quantity. P P
The true physical significance of the concept of electric field, r r
however, emerges only when we go beyond electrostatics q r̂ q r̂
and deal with time dependent electromagnetic phenomena. + +
Suppose we consider the force between two distant charges q1 , (a) (b)
q2 in accelerated motion. Now the greatest speed with which
a signal or information can go from one point to another is q0
c, the speed of light. Thus, the effect of any motion of q1
on q2 cannot arise instantaneously. There will be some time r P r P
F E
delay between the effect (force on q2 ) and the cause (motion q r̂ r q r̂ r
of q1 ). For example, if a charge is suddenly moved, the force it
exerts on a second charge a distance r away does not change
- -
immediately. It takes time r/c. (c) (d)
The field picture is this: the accelerated motion of charge q1
Figure 1.65
produces electromagnetic waves, which then propagate with
the speed c, reach q2 and cause a force on q2 . The notion
of field elegantly accounts for the time delay. Thus, even charge is toward the source charge, so the electric field at P is
though electric and magnetic fields can be detected only by directed toward the source charge, as in figure (d).
their effects (forces) on charges, they are regarded as physical Above figure shows the magnitude and direction of the electric


entities, not merely mathematical constructs. They have an field E at a point P due to positive and negative point charges.
independent dynamics of their own, i.e., they evolve according
to laws of their own. They can also transport energy. Thus,
EXAMPLE 29. (a) Does an electrically neutral particle that
a source of time dependent electromagnetic fields, turned on
has mass interact with an electric field? (b) Does a charged
briefly and switched off, leaves behind propagating electro-
particle interact with a gravitational field?
magnetic fields transporting energy. The concept of field was
first introduced by Faraday and is now among the central SOLUTION (a) No. Uncharged particles don’t interact with
concepts in physics. electric fields. (Remember that a particle has no extent and
therefore cannot be polarized.). [Here, always remember that
uncharged body is a different case. It has equal amount of
positive and negative charge. So it is always attracted towards
1.11 The Electric Field of Point a charged body or charged particle.] (b) Yes, because any
Charges particle which is charged would certainly have mass. so it will
be, interacted with a gravitational field.
In this section we consider the electric field of point charges,
first a single charge and then an assembly of individual charges.
Later we generalize to continuous distributions of charge. EXAMPLE 30. A water droplet of mass 3.00 × 10−12 kg
is located in the air near the ground during a stormy day.
An atmospheric electric field of magnitude 6.00 × 103 N/C
1.11.1 The Electric Field due to a Point points vertically downward in the vicinity of the water droplet.
Charge The drople remains suspended at rest in the air. What is the
Let a positive test charge q be placed a distance r from a point electric charge on the droplet?
0
charge q. The magnitude of the force acting on q0 is given by
Coulomb’s law. APPROACH Since, the droplet is hovering at rest in the
qq0
F~ = k 2 r̂ air in a downward atmospheric electric field and gravitational
r
The magnitude of the electric field at the position of the test field of earth, so there will be a downward gravitational force of
charge is, from Eq.(1.39) earth on the droplet and to keep it in equilibrium, there must
be an equal upward force on it. This upward force is provided


~
F q 1 q by downward electric field E .
E~ = = k 2 r̂ = r̂ (1.40)
q0 r 4πε0 r2 SOLUTION Suppose, the mass of droplet is m and it’s


charge is q. If the downward electric field is E , then The
where r̂ is a unit vector directed from q towards q0 . The forces acting on the droplet are-

→ −

direction of E is the same as the direction of F , along a radial
line from q, pointing outward(i.e., along r̂), if q is positive and 1. Downward gravitational force m− →
g
1.11. THE ELECTRIC FIELD OF POINT CHARGES 35

qE EXAMPLE 31. Two point charges are separated by a dis-


tance of 10.0 cm. One has a charge of −25µC and the other
+50µC. (a) Determine the direction and magnitude of the elec-
tric field at a point P between the two charges that is 2.0 cm
from the negative charge (Fig.(1.67)a). (b) If an electron (mass
mg = 9.11 × 10−31 kg ) is placed at rest at P and then released,
what will be its initial acceleration (direction and magnitude)?
E

Figure 1.66: FBD of water droplet

−→
2. Upward electrostatic force q E
The free body diagram (FBD) of the droplet is shown in Fig.
(1.66).
In equilibrium, we have
mg
qE = mg ⇒ q = Figure 1.67
E
On substituting the given values, we get APPROACH The electric field at P will be the vector sum
of the fields created separately by Q1 and Q2 . The field due to
3.00 × 10−12 kg 9.80 m/s2
 
q= = 4.90 × 10−15 C the negative charge Q1 points toward Q1 , and the field due to
6.00 × 103 N/C the positive charge Q2 points away from Q2 . Thus both fields
point to the left as shown in Fig. 1.67b, and we can add the
Since, the direction of electrostatic force is opposite to the magnitudes of the two fields together algebraically, ignoring


electric field E , therefore the nature of charge on the drop is the signs of the charges. In (b) we use Newton’s second law
negative. So, the charge on the droplet, is −4.90 × 10−15 C P − → P−→ −

F = m− →

a to find the acceleration, where F = qΣ E .
SOLUTION (a) Each field is due to a point charge as given
1.11.2 Electric Field due to a group of Point by Eq. (1.40), E = kq/r2 . The total field points to the left
Charges (Superposition of Electro- and has magnitude
static Fields)
 
E = kQ 1
r12
+ k Q2
r22
= k Q1
r12
+ Q2
r22

→  
To find E for a group of N point charges, the procedure is = 9.0 × 109 N · m2 /C2 25×10−6 C
+ 50×10−6 C
(2.0×10−2 m)2 (8.0×10−2 m)2
as follows: (1) Calculate Ei due to each charge i at the given
= 6.3 × 108 N/C
point as if it were the only charge present. (2) Add these
(b) The electric field points to the left, so the electron will feel
separately calculated fields vectorially to find the resultant
a force to the right since it is negatively charged. Therefore
field E at the point. In equation form,
the acceleration a = F/m (Newton’s second law) will be to the

→ − → −
→ −
→ right. The force on a charge q in an electric field E is F = qE
E = E1 + E2 + E3 + ···
X− (1.41) (Eq. 16 − 5). Hence the magnitude of the electron’s initial

= E i (i = 1, 2, 3, . . . , N ) acceleration is
(1.60×10−19 C)(6.3×108 N/C)
a= m F
= qEm = = 1.1 × 1020 m/s2
The sum is a vector sum, taken over all the charges.Eq. (1.41) 9.11×10−31 kg − →
is an example of the application of the principle of superposi- Note By considering the directions of each field E 1
tion, which states that at a given point the electric fields due −

and E 2 ) before doing any calculations, we made sure our
to separate charge distributions simply add up (vectorially) or
calculation could be done simply and correctly.
superimpose independently. This principle may fail when the
magnitudes of the fields are extremely large, but it will be
valid in all situations we discuss in this text. EXAMPLE 32. Calculate the total electric field at point A
Above equation can also be written as- in Fig. 1.68 due to both charges, Q1 and Q2 .

→ X qi
E =k r̂i (1.42) APPROACH Ignore signs of charges and determine direction
ri2
i physically, showing directions on diagram. The electric field at


where ri is the distance of point P from i source charge qi . point A is the vector sum of the fields E A1 due to Q1 , and
th


The unit vector rb is directed from qi toward P . E A2 due to Q2 . We find the magnitude of the field produced
If some more charges are added, more terms are added to the by each point charge, then we add their components to find
summation. However, there is no change to the terms that the total field at point A.
were already there, provided that the original charges do not SOLUTION The magnitude of the electric field produced
move. at point A by each of the charges Q1 and Q2 is given by E =
36 CHAPTER 1. ELECTRIC CHARGE AND FIELD

Figure 1.68: Calculation of the electric field at point A,

kQ/r2 , so

(9.0×109 N·m2 /C2 )(50×10−6 C)


EA1 = (0.60 m)2 = 1.25 × 106 N/C
(9.0×109
N·m2 /C2 )(50×10−6 C) Figure 1.69
EA2 = (0.30 m)2 = 5.0 × 106 N/C


The direction of EA1 points from A toward Q1 (negative E 2 = ke b2|q+y
2| |q2 |
2 cos θ î − ke b2 +y 2 sin θ ĵ

charge), whereas EA2 points from A away from Q2 , as shown;



→ The X and Y components of the net electric field vector:
so the total electric field at A, E A , has components
Ex = E1x + E2x = ke a2|q+y
1| |q2 |
2 cos φ + ke b2 +y 2 cos θ

EAx = EA1 cos 30◦ = 1.1 × 106 N/C Ey = E1y + E2y = ke a2|q+y 1| |q2 |
2 sin φ − ke b2 +y 2 sin θ
EAy = EA2 − EA1 sin 30◦ = 4.4 × 106 N/C
(b) APPROACH Fig.1.70 shows the situation when |q1 | =

→ |q2 | and a = b. Such symmetrical charge distribution is called
Thus the magnitude of E A is an electric dipole.
p Because Fig.1.70 is a special case of the general case shown in
EA = (1.1)2 + (4.4)2 × 106 N/C = 4.5 × 106 N/C
Fig. 1.69, So, we can take the result of part (a) and substitute
the appropriate values of the variables.
and its direction is φ (Fig. 1.68) given by
SOLUTION Evaluate the values of Ex and Ey obtained in
tan φ = EAy /EAx = 4.4/1.1 = 4.0,
−1 ◦ from part (a) with a = b, |q1 | = |q2 | = q, and φ = θ:
∴ φ = tan 4 = 76 q q q
Ex = k a2 +y 2 cos θ + k a2 +y 2 cos θ = 2k a2 +y 2 cos θ
q q
EXAMPLE 33. Charges q1 and q2 are located on the x axis, Ey = k a2 +y2 sin θ − k a2 +y2 sin θ = 0
at distances a and b, respectively, from the origin as shown in From the geometry in Fig.1.70, evaluate cos θ :
Fig.1.69 cos θ = ar = (a2 +ya2 )1/2
(a) Find the components of the net electric field at the point Now substitute this value of cos θ in the above expression for
P , which is at position (0, y). Ex : h i
(b) Evaluate the electric field at point P in the special case that Ex = 2k 2 q 2 a 2aq
= k (a2 +y
a +y (a2 +y 2 )1/2 2 )3/2
|q1 | = |q2 | and a = b.
(c) In the solution to part (b), because y >> a, neglect
(c) Find the electric field, in part (b), when point P is at a
a2 compared with y 2 and write the expression for E in this case:
distance y >> a from the origin.
|q1 | = |q2 | and a = b. 2aq
E≈k 3
y
(a) APPROACH Here, we add electric field vectors to find
the net electric field at a point in space. EXAMPLE 34. Two identical positive point charges q are
SOLUTION Find the magnitude of the electric field at P due placed on the axis at x = −a and x = +a, as shown in adjoin-
to charge q1 : E1 = k |qr12 | = k a2|q+y1|
2
ing figure.
1
(a) Find an expression of net electric field on x axis and plot
Find the magnitude of the electric field at P due to charge
|q2 | |q2 | the variation of electric field E along the x-axis.
q2 : E2 = k r2 = k b2 +y2
2 (b) Find an expression of net electric field on y axis and plot
the variation of E along the y-axis.
Write the electric field vectork for each charge in unit-vector
form: (a) APPROACH Let E ~ 1 and E ~ 2 be the electric fields due

→ |q1 | |q1 | ~ 1 points
E 1 = ke a2 +y2 cos φî + ke a2 +y2 sin φĵ to q1 and q2 respectively. Because q1 is positive, E
0
1.11. THE ELECTRIC FIELD OF POINT CHARGES 37



(ii) When x → +∞, then, E = 0î
(b) Calculate E ~ at point B, where r1B = |xB + a| and
r2B = |xB − a| :

~ =E
E ~1 + E~ 2 = kq1 r̂1B + kq2 r̂2B
2
r1B 2
r2B
kq1 kq2
= 2 î + 2 (−î)
(xB + a) (xB − a)
" #
1 1
= kq 2 î + 2 (−î) (∵ q1 = q2 = q)
(xB + a) (xB − a)


Special Positions: (i) When x → a, then, E = −∞î


(ii) When x = 0, then, E = 0î


(iii) When x → −a, then, E = +∞î

(c) If we consider another point C in the left of charge q1


(not shown in the diagram,), the calculation will be similar to
solution step 2(a) except that the net field is directed towards
the negative direction of x-axis (students are advised to think
this situation and draw diagram themselves). In this case-

~ =E
E ~1 + E~ 2 = kq1 r̂1C + kq2 r̂2C
2
r1C 2
r2C
Figure 1.70: When the charges in Fig 1.69 are of equal magnitude
kq1 kq2
and equidistant from the origin, the situation becomes symmetric = 2 (−î) + 2 (−î)
as shown here. (xC − a) (xC + a)
" #
1 1
= kq 2 + 2 (−î)(∵
q1 = q2 = q)
away from q1 everywhere, and because q2 is also positive, so, (xC − a) (xC + a)
~ 2 also points away from q2 everywhere(here, q1 = q2 = q).
E −

~ =E
~1 + E
~ 2. Special Positions: (i) When x → a, then, E = −∞î
We calculate the resultant field using E −

(ii) When x → −∞, then, E = 0î
SOLUTION (a) 1. Draw the charge configuration and place
the field point A on the x axis at the appropriate place. Draw E vs x graph: The resultant electric field at source points
~
vectors representing the electric field at A due to each point close to q1 is dominated by the field E1 due to q1 . There is
charge. Repeat this procedure for field point B (Fig.1.71): one point between q1 and q2 where the resultant electric field
is zero. Since, q1 = q2 = q, therefore it is the origin, where
net electric field due to both the charges is zero. A test charge
xA
placed at this point would experience no electric force.
a a
A sketch of E versus x for this charge configuration is shown
E2 E1 E1 E2 in Fig.1.72.
+ + A x
q1 O
x
B q 2
(b)APPROACH We’ll assume that q is positive when draw-
B

~ 1 points away from


Figure 1.71: Because q1 is a positive charge, E E
q1 , at both point A and point B. Because q2 is a positive charge,
~ 2 points away from q2 at both point A and point B.
E

~ at point A using, r1A = |xA + a|


2. Calculate E (-a, 0)
and r2A = |xA − a|: +q
+ +
+q (a, 0) x
O
~ =E
E ~1 + E~ 2 = kq1 r̂1A + kq2 r̂2A
2
r1A 2
r2A
kq1 kq2
= 2 î + 2 î
(xA + a) (xA − a)
" #
1 1 Figure 1.72: The variation of electric field E along the x axis.
= kq 2 î + 2 î (∵ q1 = q2 = q)
(xA + a) (xA − a) Electric field directed along +ve x -axis is taken as poisitive


Special Positions: (i) When x → a, then, E = +∞î ing pictures, but the solution should allow for the possibility
38 CHAPTER 1. ELECTRIC CHARGE AND FIELD

that q is negative. The question does not ask about any spe- The other two components of E ~ net are zero, hence the electric
cific point, so we will be looking for a symbolic expression in field of the two charges at a point on the y -axis is
terms of the unspecified position y. Fig. shows the charges, the
coordinate system, and the two electric field vectors E ~ 1 andE
~ 2. −
→ 2qy 1 2qy
E net = k 3/2
ĵ = ĵ
Each of these fields points away from its source charge because (y 2
+ a2 ) 4π0 (y + a2 )3/2
2

of the assumption that q is positive. We need to find the vector h


1
i
sum ∵ k = 4π 0
~ net = E
E ~1 + E~2 This is the electric field only at points on the y -axis.
Before rushing into a calculation, we can make our task much Furthermore, this expression is valid only for y > 0. The
easier by first thinking qualitatively about the situation. For electric field to the downward of the charges points in the
example, the fields E ~ 1 and E
~ 2 both lie in the xy -plane, hence opposite direction. Our expression is valid for both positive
we can conclude without any calculations that (Enet )z = 0. and negative q. A negative value of q makes (Enet )y negative,
Next, look at the x -components of the fields. The fields E ~1 which would be an electric field pointing in negative direction
~
and E2 have equal magnitudes and are tilted away from the y of y-axis.
-axis by the same angle θ. Consequently, the x -components Tracing E vs y Graph
of E~ 1 and E~ 2 will cancel when added. The only component we
need to calculate is (Enet )y . 1 Maximum and Minimum values of Electric Field:
SOLUTION The y -component of the field is For maximum value of E, on y axis, we have
dE
=0
y dy

θ θ −

P
We have already obtained the expression for E , on y axis
as-

→ 2qy
E net = k 3/2

(y 2 + a2 )
θ θ
+ + x Therefore,
q1 q2
dE d y
Figure 1.73 = 2kq
dy dy (y 2 + a2 )3/2
" 3/2 1/2 #
y 2 + a2 1 − y 23 y 2 + a2 2y
(Enet )y = (E1 )y + (E2 )y = 2 (E1 )y = 2kq 3
(y 2 + a2 )
" #
where we used the fact that fields E ~ 1 and E ~ 2 have equal y - 2 2 1/2
 a2 − 2y 2
= 2kq y + a
components. (y 2 + a2 )
3
~ 1 is at angle θ from the x -axis, so its y -component is
Vector E
2 1/2 a −2y
2 2
Therefore, dE 2

q1 dy = 0 ⇒ 2kq y + a (y 2 +a2 )3
=0
(E1 )x = E1 sin θ = k 2 sin θ
r or y=± 2 √a

So, the value of electric field will be maximum at y = ± √a2


where r is the distance from q1 . This expression for (E1 )y
Substituting this value of y in the expression for E, we get
is correct, but it is not yet sufficient. Both the distance r
and the angle θ vary with the position y and need to be .√
a 2
r
expressed as functions of y. From the Pythagorean theorem, −
→ 16 kq
1/2 E max = ±2kq 2 ĵ = ± ĵ
r = y 2 + a2 . Thus a
+ a2
 3/2 27 a2
2
y y −
→ −

sin θ = = 1/2 2 From the equation for E , at y = 0, the electric field E =
r (y + a2 )
2
0.
By combining these pieces, we see that (E1 )y is −

3 When y → ±∞, then, again E → 0
q y
(E1 )y = k Keeping these points in mind, the variation of E with y is
y 2 + a2 (y 2 + a2 )1/2
given in Fig.1.74
qy EXAMPLE 6. A charge +q is at x = a and a second
⇒ (E1 )y = k 3/2 charge −q is at x = −a (Figure 21 − 17).(a) Find the electric
(y 2 + a2 )
field on the x axis at an arbitrary point x > a. (b) Find
2qy the limiting form of the electric field for x  a (c) Plot the
∴ (Enet )y = 2 (E1 )y = k
(y 2 + a2 )
3/2 variation of E along the x-axis.
1.11. THE ELECTRIC FIELD OF POINT CHARGES 39

Between the charges, the contribution from each charge is in




the negative direction. An expression for E at distance x(< a),
is
E~ = − kq î − kq
î − a < x < a
(x − a)2 (x + a)2
In the range, −a ≤ x ≤ +a, corresponding to x → ±a, above
equation gives-


E → −∞î
Figure 1.74
At origin, i.e., at x = 0, above equation gives-

APPROACH We calculate the electric field at point P ~ = − kq î − kq î = − 2kq î


E
using the principle of superposition, E ~p = E ~ 1P + E
~ 2P . For (−a)2 (a)2 a2
~ + due to the positive charge is in the
x > a, the electric field E In right side of charge +q, if the point is very close to it, i.e,
+xdi− rection and the electric field E ~ − due to the negative x → a, then point (3) of part (a), gives
charge is in the −x direction. The distances are x − a to the
positive charge and x − (−a) = x + a to the negative charge. −
→ 4ax
E = kq lim 2 î = +∞î
SOLUTION (a) 1. Draw the charge configuration on a x→+a (x2 − a2 )
coordinate axis and label the distances from each charge to
In right side of charge +q, if the point is very far from it, i.e,
the field point (Fig.1.75):
~ due to the two charges for x > a: (Note: The x → ∞, then point (3) of part (a), gives
2. Calculate E

→ 4ax
y E = kq lim 2 î = 0î
x→∞ (x2 − a2 )
In left of charge −q, the net electric field at distance x from
x+ a the origin is given by
x kq kq
~ =E
E ~+ + E ~− = î + î
[|x|+a] 2 [|x|−a]2
a a x− a  
E– E+ 1 1
− + = kq 2
+ î
x (|x|+a) (|x|−a)2
−q +q P

→ 4a|x|
or E = kq 2 î
Figure 1.75 (|x|2 − a2 )
when, x → a, then
equation on the right holds only for x > a.)

→ 4a|x|
~ =E~+ + E ~− = kq kq or E = kq lim 2 î = +∞î
E 2
î + (−î) x→−a (|x|2 − a2 )
[x − a] [x − (−a)]2
and when x → −∞, then
 
1 1
= kq 2
− î
(x − a) (x + a)2

→ 4a|x|
On simplifying, we get- E = kq lim 2 î = 0î
x→−∞ (|x|2 − a2 )
(x + a)2 − (x − a)2
 

→ 4ax
E = kq î = kq 2 î x>a Keeping these points in mind, the variation of E with x is given
(x + a)2 (x − a)2 (x2 − a2 ) in Fig.1.76
(b) In the limit x  a, we can neglect a2 compared with x2
in the denominator: EXAMPLE 35. Three point charges lie along the x axis as

→ 4ax 4ax 4kqa shown in Fig.1.77. The positive charge q1 = 24.0 µC is at
E = kq 2 î ≈ kq î = 3 î xa x = 3.00 m, the positive charge q2 = 6.00 µC is at the origin,
(x2 − a2 ) x4 x
and the resultant force acting on q3 is zero. What is the x
The answer approaches zero as x approaches infinity, which is coordinate of q3 ?
as expected.
(c) Fig. 1.76 shows Ex versus x for all x, for q = 1.0 nC and APPROACH Since the charges q1 and q2 are of same nature
a = 1.0 m. For |x| a (far from the charges), the field is given (here both ar positive), therefore the null point lies between
by them near the smaller charge.
If x is the distance of null point from smaller charge q2 , then
E~ = 4kqa î |x| a
|x|3 net electric force on any charge q3 , placed at this point will be
40 CHAPTER 1. ELECTRIC CHARGE AND FIELD

O
–_ + x
(-a, 0) q +q (a, 0)
Figure 1.78

APPROACH Apply superposition principle of electric field-



→ − → −→ −→ −

Figure 1.76: A plot of E versus x on the x axis for the given E = E1 + E2 + E3 + E4
charge distribution
q1 (−
→r p−− →
r 1) q2 (−

r p−− →
r 2) q3 (−

rp − −→
r 3)
= −
→ −
→ + −
→ −
→ +
4πε0 | r p − −

→ →
3 3 3
4πε0 | r p − r 1 | 4πε0 | r p − r2 | r 3|
q4 (−→
r p−− →r 4)
+
4πε | r − −

→ → 3
0 r |
p 4
SOLUTION The arrangement of charges is shown in Fig.

Figure 1.77

zero, i.e.,

|F~q3 q2 |= |F~q3 q1 | (i)

here F~q3 q2 is the electrostatic force of charge q2 on the charge


q3 and F~q3 q1 is the electrostatic force of charge q1 on the charge
q3 . Figure 1.79
Now, find the forces F~q3 q2 and F~q3 q1 by using Coulomb’s law
and substitution in Eq.?? gives the required value of x.
1.79.
Caution: be carefull that Eq.?? is the relation between mag-
According’ to given problem,
nitudes only. So, for each value of x obtained from Eq.??, the
q1 = +q, ~r1 = lĵ, q2 = +q, r~2 = lî
magnitudes of forces on charges q3 will be equal but it does
not tell anything about directions of forces on q3 . Since the q3 = −q, r~3 = −lî; q4 = −q, ~r4 = −lĵ
position of null point should be btween q1 and q2 , therefore, ~rp = xk̂
only +ve values of x will be acceptable. ∴ ~rp − ~r1 = xk̂ − lî,
|q2 | |q3 | |q1 | |q3 |
SOLUTION k =k 1/2
p
x2 (3.00 − x)
2 |~rp − ~r1 | = x2 + l2 = x2 + l2
Noting that k and |q3 | are common to both sides and so can −
→r p − ~r2 = xk̂ − lĵ, |−→r p − ~r2 | = x2 + l2
1/2
be dropped, we solve for x and find that

→r p − ~r3 = xk̂ + lî, |~rp − ~r3 | = x2 + l2
1/2
(3.00 − x)2 |q2 | = x2 |q1 | and − →
rp − ~r4 = xk̂ + lĵ, |~rp − ~r4 | = x2 + l2
1/2

→ − → −→ −→ −

On solving this quadratic equation for x, we find that E = E1 + E2 + E3 + E4

→ −
→ −

q1 ( rp − r1 ) q2 (−

rp − − →
r2 )
the positive root is x = 1 m. There is also a second root, ⇒ E = +

→ −
→ 3
4πε0 | r p − −

→ → 3
x = −3 m. This is another location at which the magnitudes 4πε0 | rp − r1 | r2 |
of the forces on q3 are equal, but both forces are in the same q3 (−

rp − −→
r3 ) q4 (−

rp − − →r 4)
direction at this location. + −
→ −→ 3 + −
→ −
→ 3
4πε | r − r |
0 p 3 |r −r |
p 4
Putting the values, we get

→ q
EXAMPLE 36. Point charges q and −q are located at the E = 3/2
{(xk̂ − lî) + (xk̂ − lĵ) − (xk̂ + lî)
4πε0 (x + l2 )
2
vertices of a square with diagonals 2I as shown in Fig. 3.1.
Find the magnitude of the electric field strength at a point lo- − (xk̂ + lĵ)}
cated symmetrically with respect to the vertices of the square q
= 3/2
(−2lĵ − 2lî)
at a distance x from its centre. 4πε0 (x + l2 )
2
1.11. THE ELECTRIC FIELD OF POINT CHARGES 41

q
tend to be shifted in a direction opposite the field. This shift
p
∴ E= 3/2
(−2l)2 + (−2l)2
4πε0 (x2 + l2 ) sets up a dipole moment − →
p that points in the direction of the

2 2lq ql field. This dipole moment is said to be induced by the field,
⇒ E= 3/2
=√ 3/2 and the atom or molecule is then said to be polarized by the
4πε0 (x2 + l2 ) 2πε0 (x2 + l2 )
field (that is, it has a positive side and a negative side). When
the field is removed, the induced dipole moment and the po-
1.11.3 Electric Dipole larization disappear.
Two equal and opposite point charges +q and −q placed a short
distance (2l, say) apart, form an electric dipole. The particles The electric field shifts the positive and
are attracted to each other, but their separation is maintained
negative charges, creating a dipole.
so that the distance between them remains constant (Fig.1.80):

E
-q r
p +q
O + p
+
2l

Figure 1.80
(a) (b)

Figure 1.81
Dipole Moment
The product of the magnitude of either charge and the
distance between the charges is called the electric dipole
1.11.6 Electric Field at any Point due to an
moment. The electric dipole moment–also called the dipole
moment–is a vector quantity whose direction is along the axis Electric Dipole
of dipole pointing from negative towards the positive charge Electric field at an Axial Point, i.e., in End on Position
in a dipole.

Magnitude of dipole moment p = q.(2l) = 2ql y


The S.I. unit of dipole moment is Coulomb-metre −q l l
O +q P
The practical unit of dipole moment is Debye − +
1 debye (D) = 3.3 × 10−30 Coulomb-meter. The di- A p x
B EA EB
r
rection of p~ identifies the orientation of the dipole, and the
dipole-moment magnitude p = 2ql determines the electric field Figure 1.82
strength.
Let EA and EB are the magnitudes of electric fields at point
1.11.4 Dipole Moment of a System of Dis- P due to charges at A and B respectively
crete Charges

→ q −→
Since dipole moment is a vector quantity, hence in case of two EA = k 2 (Along P O) (1.43)
(r + l)
or more than two dipoles, the resultant dipole moment will be
the vector sum of the dipole moments of individual dipoles, ~B = k q −→
E 2 (Along OP ) (1.44)
i.e., (r − l)
p~ = p~1 + p~2 + p~3 + . . .
Net intensity of electric field at P
The real dipoles are the chemical dipoles such as H2 O and CO2
~ = EB − EA (Along − →
. H2 O has some dipole moment but CO2 being linear has zero E  OP ) −→
dipole moment. = kq (r−l)1
2 −
1
(r+l)2
(Along OP )
 2
−(r−l)2
 −→
= kq (r+l) 2
(r −l )2 2 (Along OP )
1.11.5 Induced Dipole Moment   −→
= kq (r24rl
−l2 )2
(Along OP )
Many molecules, such as water, have permanent electric dipole
moments. In other molecules (called nonpolar molecules) and 2(2ql)r −→
=k (Along OP )
in every isolated atom, the centers of the positive and nega- (r2 − l2 )
2
tive charges coincide (Fig.1.81a) and thus no dipole moment is 2pr −→
set up. However, if we place an atom or a nonpolar molecule =k 2 (Along OP )
(r2 − l2 )
in an external electric field, the field distorts the electron or-
bits and separates the centers of positive and negative charge 2pr −→
⇒ E=k 2 (Along OP ) (1.45)
(Fig.1.81b). Because the electrons are negatively charged, they (r2 − l2 )
42 CHAPTER 1. ELECTRIC CHARGE AND FIELD

In vector form, Therefore, net electric field

2~
pr −→ E = E1 cosθ + E2 cosθ
⇒ ~ =k
E (Along OP ) (1.46)
2 = 2E1 cosθ
(r2 − l2 )
  
q l
As r  l, therefore = 2k √
r 2 + l2 r 2 + l2

2p −→
E=k (AlongOP (1.47)
r3 2ql
⇒ E=k 3/2
(1.51)
In vector form (r2 + l2 )

~ = k 2~
p −→ Since, 2ql = p (dipole moment), therefore Eq.(1.51) gives-
E (AlongOP (1.48)
r3
p

→ E=k (1.52)
In this case, E is in the direction of −

p. (r2 + l2 )
3/2

Note: In Fig. 1.76, if we replace a by l and x by r, we get In vector form-


E vs r variation graph along the axis of the dipole. p~
~ = −k
E (1.53)
3/2
(r2 + l2 )
At a Point on its Perpendicular Bisector, i.e. in Broad-


side on Position In this case, the direction of E is opposite to that of −

p.
If r  l , then,
2 2

p
E=k 3 (1.54)
r
E1 sin q
In vector form-
E = E1 cos q + E2 cos q
E~ = −k p~ (1.55)
r3
Thus, we see that the intensity of electric field in axial position
E1 sin q of dipole is double in magnitude to that of intensity at the same
distance on perpendicular bisector.
r2 + l2 Special Cases: (i) At r = 0, the electric field E = k lp3 . It
is the maximum value of electric field in broadside on position
(ii) At r → ±∞, the electric field E → 0.
A B Keeping these point in mind, the variation of E with r is plot-
l O p l
r ted in Fig.1.84.
-q +q
E
Figure 1.83

Let us consider mid point O of dipole as the origin. Let P


be any point at distance OP = r on the perpendicular bisector

→ −

of the electric dipole. E 1 and E 2 are the intensities at P due O r
to charges +q and −q respectively.
Figure 1.84: Variation of E with r in broad side on position
~1 = k q −→
E (along BP direction) (1.49)
r 2 + l2
q −→ At any General Point
~2 = k
E (along P A direction) (1.50)
r2 + l2 Let P (r, θ) be any general point where intensity of electric
From Eq.1.49 and 1.50, it is clear that E1 = E2 field due to electric dipole is to be determined. The direction

→ −
→ −

Now to get the resultant of E 1 and E 2 we resolve E 1 and of dipole moment is along axis A to B.


E 2 into components: Now, resolving vector − →p into two components
−→
(i) Vertical components E1 sin θ, and E2 sin θ are equal in mag- (i) p1 = p cos θ (along the direction OP ),
−→
nitude and oppositely directed to each other, therefore they (ii) p2 = p sin θ (perpendicular to OP )
get cancelled. Point P is now at end on position of a dipole of dipole mo-
(ii) Horizontal components E1 cosθ and E2 cosθ are directed ment p1 and it is on perpendicular bisector position of dipole
along same direction, so they get added of dipole moment p2 .
1.12. CHECK POINT 3 43

and E = k rp3 3cos2 0 + 1 = k 2p
r3
 
tan θ
φ = tan−1 = tan−1 0 = 0
2
2. θ = 90◦ . In this case P is on the perpendicular bisector of
the dipole axis.
2pcos90◦
E1 = k =0
r3
psin90◦ p
E2 = k =k 3
r3 r

and E = k rp3 3cos2 90◦ + 1 = k rp3
 
−1 tan90 π
φ = tan = tan−1 ∞ =
2 2
Physical significance of dipoles: In most molecules, the
centres of positive charges and of negative charges lie at the
same place. Therefore, their dipole moment is zero. These
molecules are called non-polar molecules. CO2 and CH4 are
of this type of molecules. However, they develop a dipole mo-
Figure 1.85 ment when an electric field is applied. But in some molecules,
the centres of negative charges and of positive charges do not
coincide. Therefore, they have a permanent electric dipole mo-


Electric field E 1 at P due to dipole of dipole moment p1 ment, even in the absence of an electric field. Such molecules
−→ −→
will be along OP vector. The magnitude of E 1 is given by are called polar molecules. Water molecules, H2 O, is an ex-
E1 = k 2p
r2 = k
1 2pcosθ
3
ample of this type. Various materials give rise to interesting
→ r
− properties and important applications in the presence or ab-
Electric field E 2 at P due to dipole of dipole moment p2 will
−→ sence of electric field.
be perpendicular to OP and opposite to the direction of − →
p2


vector. The magnitude of E 2 is given by
E2 = k 2p
r2 = k
2 psinθ
r3 1.12 Check Point 3
Resultant electric fieldpat P ,
E = E12 + E22 = k rp3 4cos2 θ + sin2 θ 1. •• Shown in Figure 1.86 is an array of six charges. The top
p

⇒E = k rp3 4cos2 θ + (1 − cos2 θ )


p charges are all positive and of the same magnitude, +Q, and
there is a distance a between the charges. At a distance 2a
below this line of charges are three other charges, with the
pp
⇒ E = k 3 3cos2 θ + 1 (1.56) outer two being +Q and the central one −Q. What is the
r net electric field at a point P at the centre of the charge
The angle between the resultant field and the radial direction array?
−→
OP (O is the centre point of the dipole axis and P is that at
which electric field is being calculated) is φ. From Fig.1.85,
we can write
Er
tan φ =

 
tan θ
⇒ φ = tan−1 (1.57)
2

Special cases:
1. θ = 0◦ . In this case P is on the axis of the dipole. This
position is called an end-on position. Figure 1.86: Charge array
In this case
2pcos0◦ 2p (A) upward direction and magnitude 4πε
1 Q
0 a
2
E1 = k =k 3 1 2Q
r 3 r (B) downward direction and magnitude 4πε0 a2

1 2 2Q
◦ (C) downward direction and magnitude 4πε0 a2
psin0
E2 = k =0 (D) downward direction and magnitude 1 √Q
r3 4πε0 2a2
44 CHAPTER 1. ELECTRIC CHARGE AND FIELD

2. •• A charge q = 1µC is placed at point (1 m, 2 m, 4 m).


Find the electric field at point P (0, −4m, 3 m).
3. •• A positive point charge 50 µC is located in the plane xy
at the point with radius vector − →r 0 = 2î + 3ĵ, where î and
ĵ are the unit vector of the x and y axis. Find the vector of


the electric field strength E and its magnitude at the point
with radius vector r̂ = 8î − 5ĵ. Here r0 and r are expressed
in metres.
Figure 1.89
4. •• Four identical positive charges (each of magnitude q)are
fixed at the corners of a square of side l. Find electric
field at point P which is at a distance z lying on the line
perpendicular to the plane of the square passing through
the centre of square. Now, if we place a test charge q0 at
point P , find the net electric force on it. Figure 1.90

9. •• Infinite charges√ are placed on√x-axis such that +q at


x = a,
√ −q at x = 2a, +q at x = 3a, −q at x = 2a, +q at
x = 5a etc. Calculate electric field intensity at origin.
Multiple Choice Questions
10. •• A small electric dipole is placed at origin with
its dipole moment directed along positive √ x -axis.
The direction of electric field at point (2, 2 2, 0) is
(A) along z-axis
(B) along y -axis
(C) along negative y -axis
(D) along negative z -axis
Figure 1.87
11. •• What is the angle between electric dipole moment and
the electric field strength due to it on the axial
5. •• Charges Q1 and Q2 are at point A and B of a right angle (A) 0◦ (B) 90◦
triangle OAB [Fig.1.88]. The resultant electric field at point (C) 180 ◦
(D) none of these
O is perpendicular to the hypotenuse, then find the value
Q1
of . Answer Key and Solutions
Q2
1. (B) We can use symmetry to significantly reduce the num-
ber of actual calculations. If we place a small test charge at
point P, the forces from the top left and the bottom right
will be of equal magnitude but opposite direction and will
cancel. The same is true for the charges at the top right
and the bottom left. Therefore, we only need to consider
the two central charges. Since the top one is positive and
the bottom one is negative, a positive test charge placed
at P would experience downward forces from both charges.
Figure 1.88 We simply calculate te the equal electric fields using the
point charge relationship and then use the superposition
6. •• A positively charged ball hangs from a long silk thread. principle to combine.
We wish to measure E at a point in the same horizontal
2. Here, r − r0 = (8î − 5q ĵ) − (2î + 3ĵ) = 6î − 8ĵ
plane as that of the hanging charge. To do so, we put
a positive test charge q0 at the point and measure F/q0 Therefore, |r − r0 |= 6î − 8ĵ = 10 m So, Required field
[Fig.1.89]. Will F/q0 be less than, equal to, or greater than strength,
E at the point in the question?

→ 1 q 50 × 10−6
|E | = = 9.0 × 109
7. •• In Fig.1.90, find distance from 4Q, where total electric 4πε0 |~r − ~r0 |2 102
field is zero. = 4.5 × 103 V /m = 4.5 kV /m
8. •• Infinite charges of equal magnitude 20µc each are placed
at x = 1 cm, 2 cm, 4 cm, 8 cm, 16 cm, . . . ∞. Calculate in- 3. APPROACH follow the approach of Example 36. If E ~ is
tensity of electric field at origin. the electric field at point P , then, the electric force on test
1.12. CHECK POINT 3 45


→ −

charge placed at point P , is given by F =√q0 E . given by-
→ qz.2√2 −
− → −
→ qq0 z.2 2
Answer E = ql3 k̂, F = q0 E = ql3 k̂ kq kq kq kq kq kq
EO = − + 2 − 2 − 2 + 2 − ...
a2  2a2 3a 3a  4a 5a
4. Given: Charges Q1 and Q2 are located on vertices of a right kq 1 1 1
angled triangle OAB, = 2 1 − + − + ...
a 2 3 4
The resultant electric field at O due to Q1 and Q2 is per-
kq
= 2 [ln 2]
a
Multiple Choice Questions


8. (B) tan θ = y
= 2; cot θ = √1 [Fig.1.92]
x 2
Also, tan α = tan θ
= √1 ~ is
= cot θ ⇒ θ + α = 90 i.e., E
2 2

Figure 1.91

pendicular to side AB.


Q1
To find .: From the Fig.1.91,
Q2
x1 F2
tan θ = =
x2 F1 Figure 1.92
Q2
2
x1 x2 Q2 x1
⇒ x2 = 2
Q1 = Q1 . x22 along positive y -axis.
x2
1
Q1 x1 9. (D) Direction of dipole moment will be opposite to the elec-
⇒ =
Q2 x2 tric field

5. Net intensity of electric field at origin is given by-


1.12.1 Electric Field of a Continuous Charge
kq kq kq Distribution
EO = 2 + 2 + 2 + . . . ∞
r1 r2 r So far we have dealt with only charged particles because
 3
Coulomb’s law applies only to charged particles. However,

4 1 1 1 1
= kq × 10 + + + + ...∞
1 4 16 64 most charged charged objects— like charged comb, charged

1
 sphere, charged sheet, line charge etc. — are not particles. In-
9 −6 4
= 9 × 10 × 20 × 10 × 10 × stead, they are extended bodies. Although every macroscopic
1 − 1/4
8 object consists of very large numbers of charged particles-
= 24 × 10 N/C protons and electrons-it is not practical to calculate the in-
dividual field of each of these particles and then add them vec-
6. Due to repulsion of test charge q0 (assumed not too small), torially. Instead, we shall treat any macroscopic charged object
placed at point P , the suspended charged ball repel away as having a continuous charge distribution(Fig.1.93). The field
from q0 , therefore the separation between the two charges set up by a continuous charge distribution can be computed by
increases. As a result of it, the measured value of electric dividing the distribution into infinitesimal elements dq. Each
field Emeasured = qF0 will be less than the actual value Eact . element of charge establishes a field dE ~ at a point P , and the
resultant field at P is then found from the superposition princi-
7. APPROACH Apply the principle of superposition of ple by adding (that is, integrating) the field contributions due
electric fields and the expansion- to all the charge elements, i.e.,
Z
x 2
x 3
x 4 ~
E = dE ~ (1.58)
ln(1 + x) = x − + − + ...
2 3 4
For, x = 1, above expression gives- In Fig.1.93, the electric field produced by small charge
1 1 1
ln 2 = 1 − + − + . . . element dq at point P is,
2 3 4
SOLUTION Net intensity of electric field at origin is dE~ = k dq r̂ (1.59)
r2
46 CHAPTER 1. ELECTRIC CHARGE AND FIELD

s
dA

dq = sdA
Figure 1.95

Q
σ≡
A
The dimensions of σ are charge per area, with SI units C/m2 .
The amount of charge dq in a small area dA can be written in
terms of the surface charge density:

charge contained charge per


   
= × ( area )
in small area unit area
Figure 1.93
dq = σ dA (1.63)
Copyright 2017 Cengage Learning. All Rights
Therefore, net electric field at point P , will be given as-
3. Linear charge density λ (lowercase Greek letter
dq lambda) is the amount of charge per unit length (Fig.1.96).
Z Z
~ = dE
E ~ =k r̂ (1.60)
r2 For a uniform charge distribution,

In order to evaluate this integral, we must express dq, 1/r2 ,


l dL
and r̂ in terms of the same coordinate(s). To do so, it is nec-
essary to express the charge on the object in terms of a charge
density. There are three types of charge densities:
1. Volume charge density ρ (lowercase Greek letter rho) dq = ldL
is the amount of charge per unit volume (Fig. 1.94). For a C.
uniform charge distribution, Figure 1.96

Q
λ≡ (1.64)
L
r
The dimensions of λ are charge per length, with SI units C/m.
dV The amount of charge dq in a small lengthCopyright
element2017 can beLearning. All R
dLCengage
dq = rdV written in terms of the linear charge density:

charge contained in charge per


   
= × ( length )
small length element unit length
Figure 1.94
dq = λ dL (1.65)
Q
ρ≡ (1.61) Problem solving tactics for calculating the electric
V
field from continuous charge distributions:
Volume charge density has the dimensions charge per volume,
with SI units C/m3 . The amount of charge dq in a small 1. Identify the type of charge distribution and compute the
volume dV can be written in terms of the volume charge charge density λ, σ or ρ.
density:
2. Divide the charge distribution into infinitesimal charges
charge contained charge per dq, each of which will act as a tiny point charge.
   
= × ( volume )
in small volume unit volume
3. The amount of charge dq, i.e., within a small element dl,
dq = ρ dV (1.62) dA or dV is
dq = λdl (charge distributed along length)
2. Surface charge density σ (lowercase Greek letter sigma)
is the amount of charge per unit area (Fig. 1.95). For a uniform dq = σdA (charge distributed over a surface)
charge distribution, dq = ρdV (charge distributed throughout a volume)
1.12. CHECK POINT 3 47

4. Draw at point P the dE vector produced by the charge


dq. The magnitude of dE is
1 dq
dE =
4πε0 r2
~ is along radial line joining dq to P , dE
Vector dE ~ is di-
rected away for positive charge dq while directed towards
dq for negative dq.
5. Resolve the dE vector into its components. Identify any
special symmetry features to show whether any compo-
nent(s) of the field that are not get cancelled by other
components.
6. Write the distance r and any trigonometric factors in
terms of given coordinates and parameters.
7. The electric field is obtained by summing over all the in-
finitesimal contributions.
dq
Z Z
~ =
E ~ =
dE
4πε0 r2 Figure 1.97: Geometry for the calculation of the electric field at
field point P due to a uniformly charged rod.
8. Perform the indicated integration over limit of integration
that include all the source charges.
3. First we solve for Ex . Express dEr using Equation (1.59),
1.12.2 Electric Field Due to Finite Rod at where r is the distance from the source point S to the
Perpendicular Distance x from the field point P . From Fig.1.97 cos θ = − |xs | /r = −x/r. In
wire addition, use dq = λdx:
kdq −x
Let us consider a rod of length L as shown in Fig.1.97. suppose dEr = and cos θ =
r2 r
a positive charge Q is uniformly distributed over the surface of
kdq k cos θλdx
the rod along it’s length. P is an arbitrarily positioned point so dEx = 2 cos θ =
where, the electric field /E~ is to be calculated. r r2
APPROACH Choose the x axis so the rod is on the x axis 4. Integrate the step-3 result:
between points x1 and x2 , and choose the y axis to be through
the field point P . Let y be the perpendicular distance of P from x2
k cos θλdxs x2
cos θdxs
Z Z
~ at P , we separately
the x axis. To calculate the electric field E dEx = = kλ
x1 r2 x1 r2
calculate Ex and Ey . Using Equations (1.59) and (1.60), first
find the electric field increment dE ~ at P due to an arbitrary 5. Now, change the integration variable from x to θ. From
increment dq of the charge distribution. Then integrate each Figure 1.97, find the relation between x and θ and between
component of dE ~ over the entire charge distribution. (Because r and θ.
Q is distributed uniformly, the linear charge density λ equals y y y
Q/L ) tan θ = = , so x = − = −y cot θ
− |xs | −x tan θ
Step wise Calculation y y
sin θ = , so r =
1. Sketch the charge configuration and the field point P . In- r sin θ
clude the x and y axes with the x axis lying along the line 6. Differentiate the step 5 result to obtain an expression for
of charge and the y axis passing through P. In addition, dx (the field point P remains fixed, so y is constant):
sketch an arbitrary increment of the line charge at point
S (at x = xs ) that has a length dxs and a charge dq, and d
the electric field at P due to dq. Sketch the electric field dx = −y cot θ = y cosec2 θdθ

vector dE~ as if dq is positive (Figure 1.97)


2. E = Ex î + Ey ĵ. Find expressions for dEx and dE y in 7. Substitute y cosec2 θdθ for dx and y/sin θ for r in the in-
~ tegral in step 4 and simplify:
terms of dEr and θ, where dE r , is the component of dE Z x2 Z θ2
in the direction away from S toward P : cos θ dx cos θ y cosec2 θ dθ
~ = dEr r̂ = sin2 θ
dE x1 r2 θ1 y2
so dEx = dEr cos θ 1 θ2
Z
= cos θ dθ (y 6= 0)
dEy = dEr sin θ y θ1
48 CHAPTER 1. ELECTRIC CHARGE AND FIELD

8. Evaluate the integral and solve for Ex :


1 θ2 kλ
Z
Ex = kλ cos θ dθ = (sin θ2 − sin θ1 ) E ER
y θ1 y
 
kλ y y
= − P
y r2 r1 Ez
 
1 1
= kλ − (r1 > 0 and r2 > 0) E = Ez kˆ + ER R
ˆ
r2 r1
From this expression, it is clear that Ex > 0 at all points
on the x axis in the region x > x2 . R̂
9. Similar to Ex , use steps steps 3-7 to get Ey : r1
kλ R
Ey = − (cos θ2 − cos θ1 ) r2
y
 
cot θ2 cot θ1
= −kλ − (y 6= 0)
r2 r1 Q
If point P lies on the x-axis, then y = 0 and in this case, λ=Q k̂
θ1 L
θ1 = θ2 = 0, therefore, Ey = 0 θ
+ + + + + + + + + 2
10. Combine steps 8 and 9 to obtain an expression for the z
electric field at P :
L


E = Ex î + Ey ĵ Figure 1.98: The electric field due to a uniformly charged thin
Making Sense of the Result: Consider the plane that rod.
is perpendicular to and bisecting the rod. At points on this
plane, symmetry dictates that E ~ points directly away from the
center of the rod. That is, we expect that Ex = 0 throughout Electric Field due to a Finite Line Charge For a Point
this plane. At all points on this plane r1 = r2 . The step- 8 on the Extension of it
result gives Ex = 0 if r1 = r2 , as expected. EXAMPLE 37. A charge Q is uniformly distributed along
Comment: The first expression for Ey in the step 9 result the z axis, from z = − 12 L to z = + 12 L. Show that for large
is valid everywhere in the xy plane but not on the x axis. The values of z the expression for the electric field of the line charge
two cotangent functions in the expression for Ey are given by on the z axis approaches the expression for the electric field of
−x1 −x2 a point charge Q at the origin.
cot θ1 = and cot θ2 = −

y y APPROACH To find E , at an axial point P , use Eq. (1.66).
and neither of these functions is defined on the x axis (where Step wise Calculation
y = 0). The second expression for Ey in the step- 9 result is
obtained using Equation (1.59). By recognizing that on the 1. The electric field on the z axis has only a z component,
x axis r̂ = ±î, we can see that Equation (1.59) tells us that given by Equation (1.66):
dE~ = ±dE î, which implies Ey = 0. 
1 1

The electric field at point P due to a thin uniformly charged Ez = kλ −

→ r2 r1
rod (1.98) located on the z axis is given by E = Ez k̂ + ER R̂,
where 2. Sketch the line charge. Include the z axis, the field point
kλ P, and r1 and r2 (Figure 1.99):
Ez = (sin θ2 − sin θ1 )
R  (1.66) Q
1 1
or Ez = kλ − (r1 6= 0) and (r2 6= 0) + + + + + + + + +
P E
r2 r1 z
kλ L/2 L/2 r2
ER = − (cos θ2 − cos θ1 )
R z
 (1.67)
cot θ2 cot θ1
or ER = −kλ − (R 6= 0) r1
r2 r1 r1 = z + −12 L r2 = z − −12 L
These equations are already derived in the last article. The
expressions for Ez (Equation (1.66)) are undefined at the end Figure 1.99: Geometry for the calculation of the electric field on
points of the thin charged rod and the expressions for ER the axis of a uniformly charged rod.
(Equation (1.67) are undefined at all points on the z axis
(where R = 0). However, ER = 0 at all points where R = 0.
Here, it is to be noted that θ1 and θ2 should to be used with 3. Substitute with r1 = z + 12 L and r2 = z − 12 L into the
proper sign. step 1 result and simplify:
1.12. CHECK POINT 3 49

  Step wise Calculations


1 1
Ez = kλ − 1. Choose the first expression for the electric field in each of
z − 12 L z + 12 L
Equations (1.66) and (1.67):
kQ L
= kλ
L z2 − 1 L 2

2
Ez = (sin θ2 − sin θ1 )
  R
kQ 1 kλ
= 2 z > L ER = − (cos θ2 − cos θ1 )
z 2 − 12 L 2 R

This is the required expression for electric field at an axial 2. Take the limit as both θ1 → 0 and as θ2 → π
position of a uniformly charged rod. kλ kλ
Note that the result is valid for the region L/2 > z > ∞ only. Ez = (sin π − sin 0) = (0 − 0) = 0
R R
The result is not valid in the region −L/2 < z < +L/2? kλ kλ kλ
For z  L, the above result gives- ER = − (cos π − cos 0) = − (−1 − 1) = 2
R R R
kQ 3. Express the electric field in vector form:
Ez ≈ (z  L) −
→ 2kλ 2kλ
z2 E = Ez k̂ + ER R̂ = 0k̂ + R̂ = R̂
This expression is the same as the expression for the electric R R
field of a point charge Q located at the origin. −
→ 2kλ
⇒ E = R̂ (1.68)
R
EXAMPLE 38. What is the electric field at any point on
the axis of a uniformly charged rod of length ‘L’ having linear Thus, the magnitude of electric field decreases inversely with
charge density ‘λ’ ? The point is separated from the nearer end the radial distance from the line charge.
by distance ‘a[Fig.1.100]. Special Results: semi infinite wire

Figure 1.100

Note: Although this problem has been already discussed


in above article, here we are providing an alternative method
by using integration.
SOLUTION Consider an elementary length dx of the rod Figure 1.101
at distance x, from point P , where electric field E is to be
determined. θ1 = π/2, θ2 = π
The elemental charge, dq = λdx
Therefore,
λdx
Then, dE = k. 2 kλ
x Ez = (sin θ2 − sin θ1 )
Z a+L  a+L   R
1 1 −1 1 kλ  π
or E = kλ dx = kλ − = kλ + = sin π − sin
a x2 x a a+L a R 2
λ

1 1
 
1
 kλ kλ
Thus, E= − ∵k= = (0 − 1) = −
4π ∈o a L + a 4πε0 R R

and ER = − (cos θ2 − cos θ1 )
Electric Field Due to an Infinite Line Charge R
kλ  π
In this section, we find the electric field due to a uniformly = − cos π − cos
R 2
charged line that extends to infinity in both directions and kλ kλ
has linear charge density λ. =− (−1 − 0) =
R R
In this case, net electric field,
APPROACH If L → ∞, then from the figure, we see that
θ1 → 0 and θ2 → π. Now, put these values in Equations (1.66)
q
E = Ez2 + ER 2
and (1.67) and solve for Ez and ER . The resultant electric
field at point P can be obtained by using the expression-
p
= (−kλ/R)2 + (kλ/R)2

→ kλ √
E = Ez k̂ + ER R̂ = 2
R
50 CHAPTER 1. ELECTRIC CHARGE AND FIELD

~ and length of the


The angle between net electric field vector E
rod, i.e., z-axis, is given by-
ER π E
θ = tan−1 = tan−1 1 =
Ez 4
P

Approximating Equations (1.66) and (1.67) on the Sym-


metry Plane
r1 r2
EXAMPLE 39. A charge Q is uniformly distributed along R
the z axis, from z = − 12 L to z = + 12 L.(a) Find an expression Rˆ
for the electric field on the z = 0 plane as a function of R, the Q
radial distance of the field point from the z axis. (b) Show that θ1 θ2
for R  L, the expression found in Part (a) approaches that + + + + + + + + + z
of a point charge at the origin of charge Q · (c) Show that for
R  L, the expression found in Part (a) approaches that of an −L/2 0 +L/2 kˆ
infinitely long line charge on the z axis with a uniform linear
charge density λ = Q/L. Figure 1.102
B
APPROACH The charge distribution is uniform and the 1
linear charge density is λ = Q/L. Sketch the line charge on 2L
cos θ1 = q
the z axis and put the field point in the z = 0 plane. Then use 1
2
R2 + 2L
Equations (1.66) and (1.66) to find the electric field expression 1
for part (a). The electric field due to a point charge decreases 2kλ 2L kλL
so, ER = = q
inversely with the square of the distance from the charge.
q
R 2 2
R2 + 12 L R R2 + 21 L

Examine the part (a) result to see how it approaches that of a
point charge at the origin for R  L. The electric field due to 5. Express the electric field in vector form, and substitute Q
a uniform line charge of infinite length decreases inversely with for λL :
the radial distance from the line (Equation (1.68)). Examine
the Part (a) result to see how it approaches the expression ~ = Ez k̂ + ER R̂ = 0k̂ + ER R̂
E
for the electric field of a line charge of infinite length for R  L.
~ = ER R̂ = q kQ
so E 2 R̂
SOLUTION (a) Step wise solution is given below- R R2 + 12 L

(b) The step wise solution is given below-


1. From Eq.(1.66) and (1.66), we have-
kλ 1. Examine the step-5 result of part (a). If R  L then
Ez = (sin θ2 − sin θ1 ) 2 2
R R2 + 12 L ≈ R2 . Substitute R2 for R2 + 21 L :
kλ ~ ≈ kQ
ER = − (cos θ2 − cos θ1 ) E √
R R2
R̂ = kQ
R2 R̂ (R  L)
R
2. This (approximate) expression for the electric field
2. Sketch the charge configuration with the line charge on decreases inversely with the square of the distance from
the z axis from z = − 12 L to z = + 12 L. Show the field the origin, just as it would for a point charge Q at the
point P in the z = 0 plane a distance R from the origin origin.
(Figure 1.102):
~ ≈ kQ R̂
E(R  L)
3. From the Figure1.102, r1 = r2 , therefore, the interior R2
angle at the right end of the rod will also be θ1 . There- (c) Examine the Part (a), step-5 result. If R  L then
fore, θ2 + θ1 = π, so sin θ2 = sin (π − θ1 ) = sin θ1 and 2 2 2 2
R2 + 12 L ≈ 12 L . Substitute 12 L for R2 + 12 L .
cos θ2 = cos (π − θ1 ) = − cos θ1 .Substitution of these
This (approximate) expression for the electric field falls off
values in equations of step 1 gives:
inversely with the radial distance from the line charge, just as
the exact expression for an infinite line charge ((1.68)) would.

Ez = (sin θ1 − sin θ1 ) = 0
R ~ ≈ qkλL R̂ = 2kλ R̂ (R  L)
E
kλ 2kλ 1
2 R
ER = − (− cos θ1 − cos θ1 ) = cos θ1 R 2L
R R
EXAMPLE 40. Figure 1.103 shows a non-conducting rod
4. Express cos θ1 in terms of R and L and substitute into the that has a uniform positive charge density +λ and a total
step-3 result: charge Q along its right half, and a uniform negative charge
1.12. CHECK POINT 3 51

density −λ and a total charge −Q along its left half. What is Thus:
the direction and magnitude of the net electric field at point P Z Z x−−L/2
that shown in Fig. 1.103? E = dE = kλ (x2 + a2 )−3/2 (2xdx)
x=0
u=L/2
(u2 + a2 )−1/2

= kλ
−1/2 ιu=0
" #
−2 −2
= kλ p −
(L/2)2 + a2 a
" #
1 1
= 2kλ −p
a (L/2)2 + a2

When we use the fact that the magnitude of the charge Q is


given by Q = λL/2, we get:
Figure 1.103
" #
4kQ 1 1
SOLUTION When we consider a segment dx on the right E= −p
L a (L/2)2 + a2
side of the rod, the charge on this segment will be dq = λdx,
see Fig. 1.104. When P is very far away from the rod, i.e. a  L, we can ne-
The electric field dE~ + at P due to this segment is directed glect (L/2)2 in the denominator of this equation and hence get
outwards and away from the positive charge dq and has a mag- E ≈ O. In this situation, the two oppositely charged halves of
nitude: the rod would appear to point P as if they were two coinciding
dq λdx point charges and hence have a zero net charge.
dE+ = k 2 = k 2
r r
EXAMPLE 41. An infinite sheet of charge is lying on the xy-
A symmetric segment on the opposite side of the rod, but with plane as shown in Fig. 1.105. A positive charge is distributed
~
a negative charge, creates an electric field dE-that is directed uniformly over the plane of the sheet with a charge per unit area
inwards and toward this segment and has the same magnitude σ. Calculate the electric field at a point P located a distance a
as dE~ + , i.e. dE+ = dE− . The resultant electric field dE~ from
from the plane.
both symmetric segments will be a vector to the left, see Fig.
1.104, and its magnitude will be given by:
z
dE = dE+ cos θ + dE+ cos θ = 2dE+ cos θ

λdx x
= 2k = kλ(x2 + a2 )−3/2 (2x)dx
r2 r

dE x dE z

P
+ +
a
+ +
r
O y

+ θ +
x

+ dx +

x
Figure 1.104
Figure 1.105

The total electric field at P due to all segments of the rod SOLUTION Let us divide the plane into narrow strips par-
is found by integrating dE from x = 0 to only x = L/2,since allel to the y-axis. A strip of width dx can be considered as an
the negative charge of the rod is considered in evaluating dE. infinitely long wire of charge per unit length λ = σdx. From
52 CHAPTER 1. ELECTRIC CHARGE AND FIELD

~
Eq. (1.68), at point P , the strip sets up an electric field dE
lying in the xz-plane of magnitude:
λ σdx
dE = 2k = 2k
r r
This electric field vector can be resolved into two components
dE~ x and dE
~ z . By symmetry the components dE ~ x will sum to
zero when we consider the entire sheet of charge. Therefore,
the resultant electric field at point P will be in the z-direction,
Figure 1.106: Finding the electric field due to two oppositely
perpendicular to the sheet. From Fig. 1.105, we find the fol-
charged infinite sheets. The sheets are seen edge-on; only a portion
lowing: of the infinite sheets can be shown!
dEz = dE sin θ
and hence:
same magnitude at all points, independent of distance from
+∞
sin θdx either sheet:
Z Z
E = dEz = 2kσ σ
−∞ r E1 = E2 =
20
To perform the integration of this expression, we must first − → −

relate the variables θ, x, and r. One approach is to express θ E 1 is everywhere directed away from sheet 1, and E− 2 is ev-

and r in terms of x. From the geometry of Fig.1.105, we have: − erywhere directed toward sheet 2. Between the sheets, E 1 and
√ a a →
r = x + a and sin θ = = √
2 2 E 2 reinforce each other; above the upper sheet and below the
r x2 + a2 lower sheet, they cancel each other. Thus the total field is
Then, from the table of integrals in Appendix B, we find
that: Z +∞

 0 above the upper sheet
dx −
→ − → −

E = 2kσa E = E 1 + E 2 = σ
̂ between the sheets (1.70)
−∞ x + a
2 2  0
 +∞ 0 below the lower sheet
1 −1 x
= 2kσa tan Discussion: Because we considered the sheets to be infinite,
a a −∞
 −1 −1
 our result does not depend on the separation d. Our result
= 2kσ tan (∞) − tan (−∞) shows that the field between oppositely charged plates is es-
hπ πi 1
= 2kσ + = 2πkσ = 2π σ sentially uniform if the plate separation is much smaller than
2 2 4π0 the dimensions of the plates.
σ important Point Electric fields are not some kind of phys-
=
2o ical substance that "flows". So, it is not correct to say that


σ the field E 1 of sheet 1 would be unable to "penetrate" sheet 2,
i.e., E= (1.69) −

2o and that field E 2 caused by sheet 2 would be unable to "pen-

→ −

etrate" sheet 1. Infact, the electric fields E 1 and E 2 depend
We note that the distance a from the plane to the point P
on only the individual charge distributions that create them.
does not appear in the final result of E. This means that the −

electric field set up at any point by an infinite plane sheet of The


total field at every point is just the vector sum of E 1 and
charge is independent of how far the point is from the plane. E 2 .
In other words, the electric field is uniform and normal to the
plane. 1.12.3 Electric field at the center of a charged
Also, the same result is obtained if the point P lies below
the xy-plane. That is, the field below the plane has the same
circular arc
magnitude as that above the plane but as a vector it points in Let us consider a thin circular arc of radius R. Suppose, total
the opposite direction. positive charge Q is uniformly distributed over it. This arc
forms central angle of α rad (Fig.1.107a). To find the electric
EXAMPLE 42. Two infinite plane sheets with uniform sur-
field at the center P of this arc, we place coordinate axes such
face charge densities +σ and −σ are placed parallel to each
that the axis of symmetry of the arc lies along the x axis and
other with separation d (Fig.1.106). Find the electric field be-
the origin is at the arc’s center. If λ represent the linear charge
tween the sheets, above the upper sheet, and below the lower
density of this arc which has a length Rα, then:
sheet.

APPROACH Equation (1.69) gives the electric field due to Q


λ=
a single infinite plane sheet of charge. To find the field due to Rα
two such sheets, we combine the fields by using the principle
of superposition (Fig.1.106). For an arc element ds subtending an angle dθ at P, we have:

→ −

SOLUTION From Eq.(1.69), both E 1 and E 2 have the ds = Rdθ
1.12. CHECK POINT 3 53

follows:
Z +α/2
kQ
Z
Q R E= dEx = cos θdθ
R2 α −α/2
y
kQ +α/2
= 2 |sin θ|−α/2
R α
P x kQ h α  α i
= 2 sin − sin −
R α 2 2
kQ h α αi
R ⇒ E = 2 sin + sin
R α 2 2

kQ sin α/2
or E= (1.73)
(a) R2 α/2

ds So, the total electric field at P will be along the x -axis and
dq it’s magnitude given by Eq.(1.73).
Q dθ
y
There are three special cases to Eq. (1.73):
s dE
dEy 1. α = 0 (Point charge)
θ
R x When we apply the limiting case
θ P dEx dEx
lim [sin(α/2)/(α/2)] = 1, we get:
dEy α→0
R dE
s
dθ kQ
E=
dq
R2
ds
2. α = π (A Semicircular arc )
(b) When we substitute with sin(π/2)/(π/2) = 2/π, we get:
Figure 1.107: (a) A circular arc of radius R, central angle α, and 2kQ
center P has a uniformly distributed positive charge Q(b). The E=
~ at P due to lower and upper arc πR2
figure shows the electric fields dE
elements (each of length ds and charge dq). From symmetry, the
3. α = 2π (A ring of radius R )
vertical components of all elements cancel out and the total field is
along the x-axis When we substitute with sin 2π
2 = 0, we get:

E=0
Therefore, the charge dq on this arc element will be given This is an expected result, since we shall see that Eq.
by: 20.50 gives E = 0 when P is at the center of the ring, i.e.
Q Q
dq = λds = Rdθ = dθ when a = 0.
Rα α
To find the electric field at point P , we first calculate the mag- 1.12.4 Electric Field at Any Point on the
nitude of the electric field dE at P due to this element of charge
Axis of a Thin Charged Ring
dq, see Fig. 1.107b, as follows:
EXAMPLE 43. Fig.?? shows a ring-shaped conductor with
dq kQ
dE = k 2 = 2 dθ (1.71) radius a carries a total charge Q uniformly distributed around
R R φ it. Determine the electric field at a point P that lies on the
axis of the ring at a distance x from its center.
This field has a vertical component dEy = dE sin θ along
the y -axis and a horizontal component dEx = dE cos θ APPROACH This is a clear case of the superposition of elec-
along the negative x -axis, as shown in Fig.1.107b. The y tric fields. Note that the charge is distributed continuously
-component created at P by any charge element dq is canceled around the ring rather than in a number of point charges.
by a symmetric charge element on the opposite side of the The field point is an arbitrary point on the x-axis in Fig.??.
arc. Thus, the perpendicular components of all of the charge Our target variable is the electric field at such a point as a
elements sum to zero. The horizontal component will take the function of the coordinate x.
form: Suppose, the ring divided into infinitesimal segments of length
ds, Each segment has charge dQ and acts as a point-charge
kQ −

dEx = dE cos θ = 2 cos θdθ (1.72) source of electric field (Fig.??). Let d E be the electric field
R α from one such segment; the net electric field at P is then the


Consequently, the total electric field at P due to all elements sum of all contributions d E from all the segments that make
of the arc is given by the integration of the x -component as up the ring.
54 CHAPTER 1. ELECTRIC CHARGE AND FIELD

Now, Consider two segments at the top and bottom of the ring: Since x does not vary as we move from point to point around
−→
The contributions d E to the field at P from these segments the ring, all the factors on the right side except dQ are constant
have the same x -component but opposite y -components. and can be taken outside the integral. The integral of dQ is
Hence the total y -component of field due to this pair of seg- just the total charge Q, and we finally get
ments is zero. When we add up the contributions from all
kx
Z


such pairs of segments, the total field E will have only a com- Ex = 2 dQ
(x + a2 )3/2
ponent along the ring’s symmetry axis (the x -axis), with no
component perpendicular to that axis (that is, no component
kQx
in yz-plane). So the field at P is described completely by its x or Ex = (1.79)
-component Ex . (x2 + a2 )3/2
CALCULATION OF E ~ AT POINT P: If r be the dis-
As Q is positive, therefore, field is directed away from the
center of the ring. In vector form Eq.(1.79) can be written as-

~ = Ex ı̂ = k Qx
E 3/2
ı̂ (1.80)
(x2 + a2 )

1
Here, k =
4π0
Special Cases:
1. When x = 0 i.e., the field point P is at the center of the
ring, the Eq. (1.80) gives

~ =0
E

2. When x  a, i.e., the field point P is much farther from


the ring than its size, the denominator in Eq. (1.80) be-
Figure 1.108: The total electric field at P is along the x axis. comes approximately equal to x3 , and the expression be-
The perpendicular component of the field at P due to upper charge comes approximately
element segment dQ is cancelled by the perpendicular component
due to lower charge element segment dQ. ~ = 1 Q
E ı̂
4π0 x2
tance of point P from the upper charge segment dQ, then from
Fig.??, we have In other words, when we are so far from the ring that its size
p a is negligible in comparison to the distance x, its field is the
r = x +a2 2 (1.74)
same as that of a point charge.
and,
x
cos α = (1.75) Tracing E vs x Graph
r
The magnitude of the electric field at P due to the upper seg- 1. Maximum and Minimum values of Electric Field:
ment ds of charge dQ is For maximum value of E, on x axis, we have

dQ dE
dE = k (1.76) dx
=0
r2
The x-component of this field- We have already obtained the expression for E, on x axis
as-
dEx = dE cos α (1.77)
Qx
E=k
Substituting, the values of dE, cos α and r from Eq.(1.76), (x2 + a2 )
3/2

(1.75) and (1.74) respectively in Eq.(1.77), we get


Therefore,
 
dQ x kx
dEx = k 2 = 2 dQ (1.78) dE d x
r r (x + a2 )3/2 = kQ
dx dx (x2 + a2 )3/2
All elements of the ring make the same contribution to the field " 3/2 1/2 #
x2 + a2 1 − x 23 x2 + a2 2x
at P because they are all equidistant from this point. Thus, = kQ 3
we can integrate to obtain the total field at P (x2 + a2 )
" #
2 2
Z
kx 1/2 a − 2x
= kQ x2 + a2

Ex = dQ 3
(x2 + a2 )3/2 (x2 + a2 )
1.12. CHECK POINT 3 55

2 1/2 a2 −2x2 the ring-


Therefore, dE 2

dx = 0 ⇒ kQ x + a (x2 +a2 )3
=0 s
or a
x = ± √2 2π ma3
T = = 2π
So, the value of electric field will be maximum at x = ± √a2 ω kQQ0
Substituting this value of x in the expression for E, we get
Note: Always look for the condition of SHM as F ∝ −x
.√
a 2 2 kQ
Emax = ±kQ 2 3/2 = ± √ a2 1.12.5 Electric Field on the Axis of a
a
+a 2 27
2 Charged Disc
2. We have already shown that, at x = 0, the electric field Let us consider a point P on the axis of a uniformly charged
E = 0. disc of radius R at a distance x from the center of it. Suppose

→ the surface charge density of it is σ We have already calculated
3. When x → ±∞, then, again E → 0
Keeping these points in mind, the variation of E with x is given
in Fig.1.109

Figure 1.109
Figure 1.110: A uniformly charged disk of radius R. The electric
EXAMPLE 44. Suppose a negative point charge −Q0 is field at an axial point P is directed along the central axis, perpen-
placed at the center of the ring and displaced slightly by a dis- dicular to the plane of the disc.
tance x  a along x-axis. When released, what type of motion
does it exhibit? the electric field at an axial point of a uniformly charged ring.
Now, the disc can be considered as made of number of concen-
SOLUTION In the expression for the field ((1.80)) due to a tric very thin continuous rings with continuously varying radii
ring of charge, we let x  a, which results in from r = 0 to r = R. To find the electric field on the axis of the
kQ disc, we first consider an elementary concentric ring of radius
Ex = 3 x r and thickness dr as shown in Fig.1.110. The surface area of
a
this ring is 2πrdr. The charge dq on this ring is 2πrdrσ. Here,
Thus, from Fx = Q0 Ex the force on a charge −Q0 placed near σ is the areal charge density of the disc. We can easily find the
the center of the ring is electric field at point P due to this elementary charged ring
and then integration from r = 0 to r = R gives the electric
QQ0
Fx = −k 3 x field at P due to the complete uniformly charged disc of radius
a R.
Negative sign shows that the force is restoring in nature. Be- Now, from previous result (Eq.(1.79)), the electric field at point
cause this force has the form of SHM, the motion will be simple P due to this elementary charged ring
harmonic.
If mass of the charged particle is m and it’s acceleration to- kx
dEx = 3/2
(2πσrdr)
d2 x
wards the center of the ring is dt2 , then above equation can (x + r2 )
2

also be written as-


To obtain the total field at P , we integrate this expression
d2 x QQ0 over the limits r = 0 to r = R. Here, x is a constant.
m 2 = −k 3 x
dt a Z R
d2 x kQQ0 2rdr
or 2
= − 3
x Ex = πkσx 3/2
dt ma 0 (x2 + r 2 )
2
d x !
or = −ω 2
x x
dt2 = 2πkσ 1 − 1/2
q (x2 + R2 )
here, ω = kQQ ma3 is the angular frequency of SHM.
0
1
Time period of oscillation of the particle about the center of ∵ k=
4π0
56 CHAPTER 1. ELECTRIC CHARGE AND FIELD

After conducting his kite-


!
σ x
∴ Ex = 1− 1/2
(1.81) flying experiments that
20 (x2 + R2 ) supported his theory of
This result is valid for all values of x > 0 and x < 0, but for lightning as a giant spark,
x = 0 the answer is not valid as there is discontinuity at x = 0. Franklin came up with a plan
We can calculate the field close to the disk along the axis by to avoid lightning strikes.
assuming that R >> x; thus, the expression in bracket reduces He knew that objects could
to unity to give us the near-field approximation. be charged by rubbing. He
also knew that you cannot
charge a metal object—a
conductor—by rubbing it if
you connect the conductor
to ground. Today we know
why: The Earth acts as a Figure 1.112: Franklin’s kite
source and sink of electrons. experiment showed that lightning
If you build up a few excess is a giant electrical spark. His
electrons on a grounded son William was outside, assisting
him in flying the kite.
conductor, these electrons
quickly travel through the conductor and along the pathway
to ground (Fig.1.113).
Franklin reasoned that during a storm, the atmosphere builds
Figure 1.111

In this case, just right side of the center of disc, the electric
field is directed towards the positive direction of x-axis and is
given by- Air is a
conductor.
σ
Ex = 2πkσ = ( if x → 0+ )
20
Lightning
Just left side of the center of disc, the electric field is directed
rod
towards the negative direction of x-axis and is given by-
σ
Ex = −2πkσ = − ( if x → 0− )
20
(a) (b)
In this case, the disc can be regarded as a plane sheet of charge.
A lot of excess
positive charges
1.13 The Shape of Lightning rods ++ + + + + + + + + + +

According to Benjamin Franklin, the lightning was a giant Air is a conductor. Lightning strikes
electrical spark. To support his theory, he suggested that a when a lot of charge moves through air.
metal rod be placed on top of a tall structure to capture “elec-
tric fluid”—what we call charged particles. While Franklin was −
− − A lot of excess
waiting for the completion of Christ Church in Philadelphia −
− −
− negative charges
− −
(there were no other tall structures in Philadelphia at the
time), he came up with another way to do his experiment. He
used a kite with a metal wire attached (Fig.1.112). The wire
was connected to a string, which when wet would act as a (c)
conductor. At the end of the string was a metal key connected
to a Leyden jar—a device used to store charge. (Leyden jars Figure 1.113: (a) Franklin knew that a grounded person cannot
were among the props normally used in demonstrations of charge a conducting rod by rubbing. (b) According to Franklin,
electricity at that time.) Franklin held a piece of dry silk, lightning strikes could be prevented if a town put up a lot of light-
which insulated him, and then proceeded to collect charge ning rods. Then the atmosphere would be connected to ground, so
from his fying kite. Franklin showed that a Leyden jar charged it could not build up charge. (c) Without a lightning rod, Franklin
reasoned, the atmosphere discharges in a violent strike.
by clouds produced all the same effects as Leyden jars charged
in the home. So, he concluded that lightning is an electrical
phenomenon, like a giant spark. up excess charge much as a glass rod builds up excess charge
when it is rubbed with silk. He knew from his experience with
1.13. THE SHAPE OF LIGHTNING RODS 57

charged objects that a spark can make its way through the Wilson’s design Franklin’s design
air. His indoor experiments demonstrated that if he used a
pointed object (like a knitting needle) to draw charge from an
object through the air, only a small spark occurred compared
Lightning if a lot of No lightning because very
to the larger spark drawn to a blunt object (like his thumb). charge builds up. little charge builds up.
Franklin reasoned that when a small charge builds up in
the atmosphere, a pointed lightning rod will draw the small
What is the charge here if
charge through the air and into the Earth continuously (Fig. E is near breakdown of air?
1.113b). If only a small charge travels through the air, it will
not be visible and there will be no giant spark of lightning.
Without a lightning rod, charge still builds up in the atmo-
sphere. Because buildings are connected to the Earth, they
attract charge of the opposite sign (Fig. 1.113c). When Figure 1.114: If there is a ball at the end of the lightning rod, a
sufficient charge builds up, the air acts as a conductor and a lot of charge builds up before the atmosphere discharges. If there
large charge is transferred in a giant lightning spark. Such a is no ball at the end of the lightning rod, little charge builds up
because the atmosphere continually discharges through the rod
violent spark causes great damage to buildings and can be very
dangerous. Franklin published the following recommendations
for lightning rods:
sphere connected to ground. If the electric field on the surface
of the ball is E = 3 × 106 N/C (breakdown field strength for
1. Just outside each building an iron rod should be planted air), what is the charge on the ball?
3 feet to 4 feet into the moist ground.
APPROACH The ball  is a sphere, so we can use the magni-
2. The rod should extend 6 feet to 8 feet above the tallest tude of ~
E(r) = kQ/r 2
r̂ to find the charge on the ball.
part of the structure. SOLUTION For the charge Q on the surface of the sphere,
where r = R, we have-
3. On top of the rod should be a foot of brass wire sharp- Q
E(R) = k 2
ened to a fine point. R
3 × 106 N/C (0.1m)2

ER2
Benjamin Wilson—a contemporary of Franklin’s—believed ⇒ Q= =
k (8.99 × 109 N · m2 /C 2 )
that lightning rods should be blunt. He argued that a pointed ⇒ −6
Q = 3.3 × 10 C = 3.3µC
rod would draw down lightning that might have just passed Result This is not very much charge, especially compared to
harmlessly overhead. Wilson argued that pointed lightning the amount of charge involved in a lightning strike, which is
rods were more dangerous than having no rods at all. on the order of hundreds of coulombs. However, our job is not
In this section, we explore both types of rods (Fig.1.114) in done. We need to see how Franklin’s pointed rod would do
following two solved examples. In order for air to breakdown5 under the same conditions.
and become a conductor, the electric field in the air must be
3 × 106 N/C. Let’s assume that in order for a lightning rod to EXAMPLE 46. Let’s model the end of a Franklin rod as
work, the electric field at its surface must equal to breakdown a tiny ball of radius R = 2mm -something like the end of a
electric field. We will calculate the amount of charge on the knitting needle. If the electric field on the surface of the ball is
surface of each conductor. The one with the least amount E = 3 × 106 N/C, what is the charge on the ball?
of charge is the better design because a smaller amount of
charge on the surface of the conductor means a smaller amount APPROACH Repeat the calculation from above example
of charge travels through the air. Calculating the charge on for the Wilson rod, this time for a much smaller ball. Solve
each rod gives us a way to compare their effectiveness. A rod for the charge q on the surface of the sphere, where r = R.
that does not require much charge to have a strong electric SOLUTION E(R) = k q
field on its surface will not generate a big spark to discharge R2
3 × 106 N/C (0.002m)2

2
the atmosphere. Because both designs use conductors, we will ER
⇒ q= =
assume the excess charge is uniformly spread over the surface k (8.99 × 109 N · m2 /C 2 )
−9
of the lightning rod. ⇒ q = 1.3 × 10 C = 1.3nC
So, the charge on Franklin’s rod is about 2500 times lower
than on Wilson’s.
EXAMPLE 45. At the end of Wilson’s rod was a cannonball From above two examples, we can say that an ideal Franklin
(Fig.1.115) with a radius of R = 0.1m. Because the ball is rod (one that is infinitely thin) would be better than the blunt
so much larger than the thickness of the supporting rod, we Wilson rod.
will ignore that rod and model Wilson’s device as a suspended
5 If the magnitude of an electric field in air is so great, that the air

becomes ionized and begins to conduct electricity, then the electric field is
EXAMPLE 47. Find an expression for the electric field at
called the breakdownfield for air and the phenomenon is called dielectric the tip of Franklin’s pointed lightning rod. Evaluate Franklin’s
breakdown of air design by considering a point very close to the end of the rod.
58 CHAPTER 1. ELECTRIC CHARGE AND FIELD
h i
APPROACH Fig. 1.115 shows a vertical rod of length L ⇒E ~ =k λL
d(d+L) ̂
and P is a point at a distance d from the upper end of the The total charge on the rod is q = λL. Therefore, on
rod. For a limiting case, when P is just above the upper tip substituting λL = q in above expression, we get-
of the rod, we have d → 0. As the charge will get distributed
uniformly on the metal rod, therefore, we cannot find the ~ = kq
E ̂ (1.82)
electric field at point P due to this charged rod by directly d(d + L)
using the formula of electric field at any point due to a point
If point P is far from the rod (d  L), then d + L ≈ d. In this
charge. To find electric field, we have to consider a differential
case, the electric field at P is given by
charge element dq(= λdl) of length dl of the rod at distance
l from the point P . Here λ is the linear charge density of ~ = kq ̂
the rod. Since, each elementary piece of the rod produces E
d2
electric field, at point P , in the same direction, therefore, the
~ approaches the electric field due to a charged particle.
So, E
integration from l = d to l = d + L will give the net electric
field at P due to complete length of the charged rod. For the point, just above the rod, d → 0, therefore
Electric field at P due to elementary charge aq, is given by-  
−→ dq ~ = lim kq
d E = k 2 ̂, with r = l E =∞
d→0 d(d + L)
r
To derive an expression for dE, ~ integrate this expression from So, it is clear that, as point P gets closer to the end of the rod,
l = d to l = d + L the electric field increases. At the tip of the rod, the electric
For the electric field just above the tip of the rod, we have field approaches infinity. So, in principle, even a very tiny
d → 0. charge on the rod would lead to a very high electric field at
SOLUTION Electric field at P , due to the infinitesimal its end. Franklin’s rod would continually draw a small amount
of charge from the atmosphere and prevent a large lightning
strike.
y
EXAMPLE 48. A thin non-conducting ring of radius R has
a linear charge λ = λ0 cos θ, where λ0 is the value of λ at
dE θ = 0. Find the net electric dipole moment for this charge
distribution.
P
d

q = lL l

L+d
L dl
Figure 1.116
dq = l dl
APPROACH The charge density of the given ring is λ =
λ0 cos θ. The value of cos θ is positive for −π
2 < θ < 2 and
π

negative for 2 < θ < 2 . So, the charge of the ring is positive
π 3π
x in first and fourth quadrants whereas, it is negative in second
and third quadrants. Since, |cos θ|= |cos(2π ±θ)|= |cos(π ±θ)|.
Therefore, corresponding to each positive elementary charge
Figure 1.115
in first and fourth quadrant, there will be an equal and oppo-
site charge in third and fourth quadrant. So, each elementary
charge dq = λdl, is- charge is able to form an electric dipole.

→ dq Let us consider a small part of length dl of the ring as shown
d E = k 2 ̂ in Fig.1.117a. If this part subtends angle dθ at the center of
l

→ λdl the ring, then
⇒ d E = k 2 ̂
l dl = Rdθ
Now, on integrating over the entire length of the rod from
l = d to l = d + L, we get The small charge on this part of ring is given as-
~ = l k λdl ~ = kλ − 1 d+L ̂
R R  
dE ̂ ⇒ E dq = λ dl
0 hl2 i l h
d i
⇒E ~ = −kλ 1 − 1 ̂ = −kλ −L ̂
d+L d d(d+L) dq = λ0 cos θRdθ
1.14. CHECK POINT 4 59

The symetrically opposite part to this part has charge −dq. to +π/2, we get-
So these parts form a dipole. The location of charge −dq Z +π/2
may be considered in quadrant fourth or in quadrant second
px = 2R2 λ0 cos2 θdθ
(Fig. 1.117). For, Fig.1.117a, dipole length is 2R, whereas for −π/2
dipole shown in Fig.1.117b, its lemgth is 2R cos θ.
Now, for the dipole considered in Fig.1.117(a), the magnitude
Z +π/2
of dipole moment of this elementary dipole is given by- and py = 2R2 λ0 cos θsin θdθ
−π/2

dp = 2Rdq Z +π/2
⇒ px = 2R2 λ0 cos2 θ dθ
= 2Rλ0 cos θRdθ −π/2

The direction of this dipole moment is from −dq to +dq. +π/2


Z
x and y components of this dipole moment are, = R2 λ0 (1 + cos 2θ) dθ
−π/2
" +π/2 #
(1.83a)

dpx = 2Rλ0 cos θRdθ cos θ 2 +π/2 sin2θ
= R λ0 [θ]−π/2 + = πR2 λ0
2 −π/2
dpy = 2Rλ0 cos θRdθ sin θ (1.83b)
⇒ px = πR2 λ0
To get, total electric dipole moment of the ring, we have to Z +π/2
integrate these expressions from θ = −π/2 to +π/2. For the and py = R2 λ0 sin 2θ dθ
−π/2
 +π/2
cos 2θ
= R2 λ0 − =0
2 −π/2

Method 2. In this method, we integrate Eq.(1.84) from limit


θ = −π/2 to θ = +π/2
On integrating, we get-
Z π/2
2
p = 2R λ0 cos2 θRdθ
−π/2

(a) (b) Simplifying as like we have done for px , we get

Figure 1.117: Charged ring p = πR2 λ0

dipole shown in Fig.1.117b, the dipole length is 2R cos θ. It’s Direrction of p is along +ve x direction.
dipole moment is given by-

dp = 2R cos θdq
= 2R cos θλ0 cos θRdθ 1.14 Check Point 4
1. •• In Fig.1.118, a thin glass rod forms a semicircle of
dp = 2R2 λ0 cos2 θdθ (1.84)
radius R = 5.00 cm. Charge is uniformly distributed
The direction of this dipole moment is from −dq to +dq along the rod, with +q = 4.50pC in the upper half and
(Fig1.117b), i.e., along +ve x- direction. In this case, y- com- −q = −4.50pC in the lower half. What are the (a) mag-
ponent of dipole moment is zero. nitude and (b) direction (relative to the positive direction
To get net dipole moment of the ring, we have to integrate it ~ at P , the center of the
of the x axis) of the electric field E
from θ = −π/2 to +π/2. semicircle?
Now, we find the net dipole moment of the ring by using
both above methods, i.e., by considering both the dipoles of y
Fig.1.117. +Q
SOLUTION Method 1. Considering, the dipole shown in
Fig.1.117, From, Eq.(1.83) the x y components of dipole mo-
ment are- x
P
R
dpx = 2R2 λ0 cos2 θdθ
–Q
and dpy = 2Rλ0 cos θsin θR dθ
→ Figure 1.118
On integrating both of the above expressions, from θ = −π/2
60 CHAPTER 1. ELECTRIC CHARGE AND FIELD

2. •• In Fig.1.119, two curved plastic rods, one of charge +q


and the other of charge −q, form a circle of radius R =
8.50 cm in an xy plane. The x axis passes through both
of the connecting points, and the charge is distributed
uniformly on both rods. If q = 15.0 pC, what are the
(a) magnitude and (b) direction (relative to the positive


direction of the x axis) of the electric field E produced at
P , the center of the circle?
Figure 1.122
y
+q
6. •• A positive point charge, Q, is located at a distance h
directly above the center of a charged thin non-conducting
x circular plate of radius R. The plate carries a total positive
P charge, Q, spread uniformly over its surface area. What
will be the electrical force on the point charge?
–q
7. • • •A thin nonconducting ring of radius R has a linear
Figure 1.119
charge density λ = λ0 cos φ, where λ0 is a constant, φ is
the azimuthal angle. Find the magnitude of the electric
field strength (a) at the center of the ring; (b) on the axis
3. ••Find the electric dipole moment of a non-conducting of the ring as a function of the distance x from its center.
ring of radius a, made of two semicircular rings having Investigate the obtained function at x >> R.
linear charge densities −λ and +λ as shown in figure ??.
8. •• A thread carrying a uniform charge λ per unit length
has the configurations shown in Fig. 1.123a and b. As-
suming a curvature radius R to be considerably less than
the length of the thread, find the magnitude of the electric
field strength at the point O.

Figure 1.120

4. ••Find the electric dipole moment of a non-conducting


ring of radius a, having linear charge densities +λ and −λ
and arranged as shown in figure. Figure 1.123

Multiple Choice Questions

9. ••Supposing that the earth has a surface charge density of


1 electron/m^2; calculate electric field just outside earth’s
surface. The electronic charge is −1.6 × 10−19 C and
earth’s radius is 6.4 × 106 m. ε0 = 8.9 × 1012 C 2 /N.m2
(A) 1.8 × 10−8 N/C (B) +1.8 × 10−8 N/C
(C) 1.8 × 10 N/C
−9
(D) +1.8 × 10−9 N/C

Figure 1.121 10. ••A circular wire of radius R carries a total charge Q
distributed uniformly over its circumference. A small
length of the wire subtending angle θ at the center is
5. ••Fig.1.122 shows two disks and a flat ring, each with the cut off. Find the electric field at the center due to the
same uniform charge Q. Rank the objects according to remaining portion.
the magnitude of the electric field they create at points P (A) = Q/(4π 2 ε0 R2 ) sin θ (B) = Q/(4π 2 ε0 R2 ) sin θ/2
(which are at the same vertical heights), greatest first. (C)= Q/(4π 2 ε0 R2 ) sin θ (D) = Q/(4π 2 ε0 R2 ) sin θ/8
1.14. CHECK POINT 4 61

Answer Key and Solutions The electric field at center P , due to each halves, can be
obtained by using expression-
1. APPROACH From Eq.(1.73), the total electric field at
the center of the uniformly charged circular arc of radius −
→ kq sin α/2
R, subtending an angle α at the center, is given by- E =− 2 ĵ (with α = π)
R α/2
kQ sin α/2 Therefore, the net electric field at point P due to both
E= (1.85) halves can be given as-
R2 α/2
Here, Q is the charge on circular arc. For positive charge, −
→ −
→ kq sin α/2
the direction of field is away from the circular arc along the E net = 2 E = − 2 ĵ
R α/2
symmetry axis i.e., net electric field vector passing through
Now, substitute the values in above expression and solve
the mid point of the arc at at angle α/2 at the center. −


→ for E net .
Similarly, for negative charge, the direction of E is towards
SOLUTION (a) With R = 8.50 × 10−2 m, α = π and
the circular arc at angle α/2 from the line dividing the ~ net |= 23.8N/C.
circular arc. q = 1.50 × 10−8 C, |E
(b) The net electric field E~ net points in the −ĵ direction, or
In this problem, each charged quarter-circle produces a field
of magnitude −90 counterclockwise from the +x axis.
0

3. 4λa2
Hint: Follow the approach used in solved Ex. 48.

4. 2 2λa2
Hint: Follow the approach used in solved Ex.48
5. a, b, c.
1 2Q2 h 1
 
1
6. Fz = − √ The direction of this
4π0 R2 h R2 + h2
force is directly away from the charged plate. Hint: The
electric force on the point charge Q placed on the axis of
Figure 1.124 charged disk is given by-

→ −

F = QE
√ −

kQ sin π/4 2 2kQ here, E is the electric field at axial position of charge disc.
E= 2 =
R π/4 πR2
7. The angle swept by radius vector is called azimuthal angle.
From Fig.1.124, it is clear that the x components of these Since, λ is cosine function of φ, so, ring is not uniformly
electric fields are opposite to each other and hence get can- charged. A part of ring is positively charged and remain-
celed with each other. The components along negative di- ing part of ring is negatively charged. The distribution of
rection of y-axis add up to each other and give the net charge on ring is shown in Fig. 1.125. Since, λ = λ0 cos φ,


electric field E net .
SOLUTION Net electric field at point P ,

Enet = 2E cos π/4


√ !
2 2kQ 4kQ
=2 2
cos π/4 =
πR πR2

On substituting the given values in above expression, we


get-

9.0 × 109 N · m2 /C2 4 4.50 × 10−12 C


 
Figure 1.125
E= 2
π (5.00 × 10−2 m)
therefore, first and fourth quadrants are positively charged
= 20.6 N/C
but second and third quadrants are negatively charged.
(b) By symmetry, the net field points vertically downward The given charge distribution is shown in the figure. The
in the −ĵ direction, or −90◦ counterclockwise from the +x −

symmetry of this distribution implies that vector E at the
axis. point O is directed to the right, and its magnitude is equal
2. APPROACH From symmetry, we see that the net field to the sum of the projection onto the direction of E of vec-


at P is twice the field caused by the upper semicircular tors d E from elementary charges dq. The projection of

→ −

charge +q (and that it points downward). vector d E onto vector E is
62 CHAPTER 1. ELECTRIC CHARGE AND FIELD

1 dq For x  R,
dE cos φ = cos φ ... (1)
4π0 R2 p
where dq = λRd φ = λ0 R cos φ dφ Ex = , where p = λ0 πR2
Integrating Eq. (1) over φ between 0 and 2π we find the 4πε0 x3
−→ Alternate: Take an element S at an azimuthal angle φ
magnitude of the vector E as-
from the x -axis, the element subtending an angle dφ at the
λ0 2π
λ0 center. The elementary field at P due to the element is
Z
E= cos2 φ dφ =
4π0 R 0 40 R
λ0 cos φ dφR
(along SP with components)
4πε0 (x2 + R2 )
(b) Let us take differential length element of the ring at an λ0 cos φ dφR
azimuthal angle φ from the x -axis, the element subtends × {cos θ along OP, sin θ along OS}
4πε0 (x2 + R2 )
an angle dφ at the center, and carries charge dq = λRdφ =
(λ0 cos φ) R dφ x
where cos θ =
Taking the plane of ring as x−y plane and center of the ring (x2 + R2 )
1/2


as origin O, locations of field point −

r , of charge element R R
and of field point relative to charge element − →r are shown and sin θ = 1/2
(x2 + R2 )
in the figure.
The component along OP vanishes on integration as-
Z 2π
cos φ dφ = 0 .
0
The component along OS can be broken into the parts
along Ox and Oy and given by
2R
λ0 R 2
Z
So, Ex = 3/2
cos2 φ dφ
4πε0 + (x2 R2 )0
Z 2R
Figure 1.126
∵ cos2 φ sin φ dφ = 0
0

Electric field strength at the field point due to considered Therefore, on integration, the part along Oy vanishes.
R2
charge element Finally E = Ex = 4ε (xλ20+R 2 )3/2
0

→ 1 dq − → √
dE = r1
4πε0 r13 8. (a) E = 4πε λ 2
0R
; (b) E = 0 ]
−→
Using −→
r1=− →
r − R = xk̂ − (R cos φ î + R sin φ ĵ)

→ 3 3/2 Multiple Choice Questions
and r 1 = R2 + x2

→ 1 (λ0 cos φ)R dφ
d E = 4πε {x k̂ − (R cos φ î + R sin φ ĵ)}
0 (R2 +x2 )3/2 9. (A) The electric field E just outside the earth’s surface is
So, the net electric field strength same is if the entire charge q were concentrated at this
center. Thus,

→ −

Z
1 q 1 4πR2 σ
E = dE E = 4πε 0 R
2 = 4πε
0 R2 = εσo
Substituting the given value:
(−1.6×10−19 )C/m2
E = 8.9×10−12 C 2 /N −m2 = 1.8108N/C
" Z

λ0 R
= 3/2
xk̂ cos φ dφ The minus sign indicates that E is radially inward.
4πε0 (R2 + x2 ) 0
Z 2π Z 2π # 10. Electric field due to an arc at its center is kλ/R2 sin θ/2,
− Rî 2
cos φ dφ − Rĵ sin 2φ dφ where k = 4πε 1
0
0 0 θ = angle subtended by the wire at the center, λ = Linear
Taking into account density of charge. Let E be the electric field due to remain-
ing portion. Since intensity at the center due to the circular
wire is zero. Applying principle of superposition.
Z 2π Z 2π Z 2π
cos φ dφ = 0, sin 2φ dφ = 0 and cos2 φ dφ = π
0 0 0 kλ θ
2 sin − E = 0
R 2

→ 2
1 πλ0 R (− î) 1 Q θ
We get E = 4πε 0 (R2 +x2 )
3/2 ⇒E= 2 sin
4πε0 R 2πR 2
R2
Hence, E = 4ε (Rλ20+x 2 )3/2 Q θ
0
= sin
4π 2 ε0 R2 2
1.15. ELECTRIC FIELD LINES 63

1.15 Electric Field Lines


9>7FJ
The picture of field lines (sometimes called lines of force) was
invented by Faraday to develop an intuitive non-mathematical
way of visualizing electric fields around charged configurations.
An electric field line is, in general, a curve drawn in such a
way that the tangent to it at each point is in the direction
of the net field at that point (Fig.1.127). An arrow on the
curve is obviously necessary to specify the direction of electric
field from the two possible directions indicated by a tangent to
the curve. A field line is a space curve, i.e., a curve in three
dimensions.
To understand the dependence of the field lines on the area,
š
Electr ic field at Figure 1.128
point Q š
Q

Electr ic field
at point P Electric field line

P P R
š

Figure 1.127: The direction of the electric field at any point is the
tangent to the electric field line at this point Figure 1.129: The magnitude of the electric field at point P is
larger than the magnitude at R.
or rather the solid angle subtended by an area element, let
us try to relate the area with the solid angle, a generalization
of angle to three dimensions. Recall how a (plane) angle is • At large distances from a system of charges that has a
defined in two-dimensions. Let a small transverse line element nonzero net charge, the field lines are equally spaced and
∆l be placed at a distance r from a point O. Then the angle radial, as if they emanated from (or terminated on) a sin-
subtended by ∆l at O can be approximated as ∆θ = ∆l/r. gle point charge equal to the total charge of the system.
Similarly, in three-dimensions the solid angle subtended by a
• Electric field lines cannot cross each other. The electric
small perpendicular plane area ∆S, at a distance r, can be
field at any point has a unique direction; if two field lines
written as ∆Ω = ∆S/r2 . We know that in a given solid angle
crossed, the field would have two directions at the same
the number of radial field lines is the same. In Fig.1.128, for
point (Fig.1.130)
two points P1 and P2 at distances r1 and r2 from the charge,
the element of area subtending the solid angle ∆Ω is r12 ∆Ω
at P1 and an element of area r22 ∆Ω at P2 , respectively. The
number of lines (say n) cutting these area elements are the 1
P
same. The number of field lines, cutting unit area element
2
is therefore n/(r12 ∆Ω) at P1 and n/(r22 ∆Ω) P2 , respectively.
Since n and ∆Ω are common, the strength of the field clearly Two tangents corresponding
has a 1/r2 dependence. to curves 1 and 2 at
The electric field is strong where field lines are close together intersection point P and
and weak where they are far apart (Fig.1.129). (More specifi- hence two directions of
E at P, which is not posible.
cally, if you imagine a small surface perpendicular to the field
lines, the magnitude of the field is proportional to the number Figure 1.130
of lines that cross the surface divided by the area.)
Rules for Drawing Electric Field Lines: The rules for
drawing electric field lines are as follows: We choose the number of field lines starting from any positively
charged object to be Cq and the number of lines ending on any
• The lines must begin on a positive charge and terminate
negatively charged object to be C|q|, where C is an arbitrary
on a negative charge. In the case of an excess of one type
proportionality constant. Once C is chosen, the number of
of charge, some lines will begin or end infinitely far away.
lines is fixed. For example, if object 1 has charge Q1 and
• The number of field lines drawn emerging from a positive object 2 has charge Q2 , then the ratio of number of lines is
charge or ending at a negative charge is proportional to N2 /N1 = Q2 /Q1 . The electric field lines for two isolated point
the magnitude of the charge. charges of magnitude +q and |−2q| are shown in Fig.1.131.
64 CHAPTER 1. ELECTRIC CHARGE AND FIELD

Because the magnitudes of charges are in the ratio of 1 : 2, the Z f


Wif = ~ · d~l
E (1.86)
number of field lines that originated from +q and terminated
i
on −2q are also in the ratio of 1 : 2. For example, 8 field line
are originated from +q and 16 lines are terminated on −2q, so This integral is taken along a certain line (path) and is
N1 : N2 = 8 : 16, i.e., 1 : 2. therefore called the line integral.
We shall now show that from the independence of line integral
(1.86) of the path between two points it follows that when
E field lines E field lines taken along an arbitrary closed path, this integral is equal
to zero. Integral (1.86) along a closed path is called the
circulation of vector E~ and is denoted by .
H

Thus, we state that circulation of vector E ~ in any electrostatic


field is equal to zero, i.e.
I
+q ~ · d~l = 0
E (1.87)
-2q

~
This statement is called the theorem on circulation of vector E.

In order to prove this theorem, we break an arbitrary closed


(a) (b) path into two parts iaf and
Z fbi (Fig.1.132). Since line integral

Figure 1.131: Electric field lines for a point charge (a) near (1.86), we denote it by , does not depend on the path
12
a positive charge the field lines point radially away from the charge. Z (a) Z (b)
The lines start on the positive charge and end at infinity. (b) near between points i and f , we have = . On the other
a negative charge the field lines point radially inward. They start if if
Z (b) Z (b) Z (b)
at infinity and end on a negative charge and are more dense where
the field is more intense. notice that the number of lines drawn for hand, it is clear that =− , where is the integral
if fi fi
part (b) is twice the number drawn for part (a), a reflection of the
over the same segment b but taken in the opposite direction.
relative magnitudes of the charges
Therefore,
I Z (a) Z (b)
E~ · d~l = E~ · d~l + E~ · d~l
if fi
Z (a) Z (b)
= E~ · d~l − E~ · d~l = 0,
1.16 Theorem on Circulation of Vec- if if
A field having property (1.87) is called the potential field.
tor E Hence, any electrostatic field is a potential field. The term
‘potential’ will be discussed in next chapter.
In mechanics we have studied that any stationary field of cen- The theorem on circulation of vector E ~ makes it possible to
tral forces is conservative, i.e. the work done by the forces of draw a number of important conclusions without resorting to
this field is independent of the path and depends only on the calculations. Let us consider two examples.
position of the initial and final points. This property is inher-
ent in the electrostatic field, viz. the field created by a system ~
EXAMPLE 49. The field lines of an electrostatic field E
of fixed charges. If we take a unit positive charge for the test
cannot be closed.
charge and carry it from initial position point i of a given field
E~ to final position point f , the elementary work of the forces
SOLUTION Suppose, the opposite is true and some lines
of the field done over the distance d~l is- ~ are closed like Fig.1.133.The arrows on the contour
of field E
dW = F~e · d~l = E~ · d~l
indicate the direction of circumvention as well as the direction
[ because, for unit positive test charge, F~e = q0 E~ = 1E~ =E ~ ]
of electric field.
and the total work of the field forces over the distance between Taking the circulation of vector E ~ along the line shown in this
points i and f is defined as figure, we get-
I Z (a) Z (b)
~ ~
E · dl = ~ ~
E · dl + ~ · d~l
E
if fi
For both the terms of right hand side of above expression,
~ and d~l are directed in the same direction. Therefore, the
E
scalar product ~ · d~l > 0. So,
E
I
~ · d~l > 0
E
It contradicts the theorem 1.16. This means that electric field
Figure 1.132 lines cannot form any closed curve in an electrostatic field: the
1.16. THEOREM ON CIRCULATION OF VECTOR E 65

Two field lines leave 2q for every


one that terminates on !q.

Figure 1.133

lines emerge from positive charges and terminate on negative


ones (or go to infinity).

2q ! !q
EXAMPLE 50. Is the configuration of an electrostatic field
shown in Fig.1.134 possible?

Figure 1.135

4. Null point: There is one point at which E = 0. No lines


Figure 1.134 should pass through this point.

SOLUTION No, it is not. It can be easily shown by applying 5. Number of lines: Twice as many lines leave +2Q as
the theorem on circulation of vector E ~ to the closed contour entire −Q. (Number of lines is directly proportional to
shown in the figure1.134 by the dashed line. The arrows on magnitude of charge.)
the contour indicate the direction of circumvention. With
such a special choice of the contour, the contribution to the The field lines around a dipole [Fig.1.136a], show clearly a
circulation from its vertical parts is equal to zero, since in vivid pictorial description of the mutual attraction between
this case E~ ⊥ d~l and E
~ · d~l = 0. It remains for us to consider the two charges. The total charge of the dipole is zero, but
the two horizontal segments of equal lengths. The figure1.134 because the charges are separated, the electric field does not
shows that the contributions to the circulation from these vanish. Instead, the field lines start from positive charge and
regions are opposite in sign, and unequal in magnitude (the terminate at negative charge.
contribution from the upper segment is larger since the field The field lines around a system of two positive charges (q, q)
lines are denser, and hence the value of E~ is larger). Therefore, [Fig.1.136b] give a vivid pictorial description of their mutual
~
the circulation of E differs from zero, which contradicts to repulsion. Fig.1.137 shows the electric field lines for a config-
(1.87). uration of two positive and two negative charges, all of equal
magnitude.

1.16.1 Deduction of Pattern of Field Lines 1.16.2 Properties of Electric Field Lines
Fig.1.135 shows the sketch of field lines for two point charges The field lines follow some important general properties:
2q and −q.
The pattern of field lines can be deduced by considering the 1. Field lines start from positive charges and end at negative
following points: charges. If there is a single charge, they may start or end
at infinity.
1. Symmetry: For every point above the line joining the
two charges there is an equivalent point below it. There- 2. In a charge-free region, electric field lines can be taken to
fore, the pattern must be symmetrical about the line join- be continuous curves without any breaks.
ing the two charges.
3. Tangent to the lines of force at any point gives the direc-
2. Near field: Very close to a charge, its own field pre- tion of the electric field.
dominates. Therefore, the lines are radial and spherically
symmetric. The high density of lines near the charges 4. Two field lines can never cross each other. (If they did,
indicates a region of strong electric field. the field at the point of intersection will not have a unique
direction, which is absurd.)
3. Far field: Far from the system of charges, the pattern
should look like that of a single point charge of value (2Q− 5. Electrostatic field lines do not form any closed loops. This
Q) = +Q, i.e., the lines should be radially outward. follows from the conservative nature of electric field.
66 CHAPTER 1. ELECTRIC CHARGE AND FIELD

E field lines
>
E (+q)
>
> Enet
E(-q)
1

2 2
++q
q -–qq

>
E
>
E
Figure 1.137: Electric field lines for a configuration of two positive
(a) and two negative charges, all of equal magnitude. Note that the field
lines always start on a positive charge and end on a negative charge
E field lines when there are equal numbers of both charges.

of charge 2? Explain.

+q +q

1 2

(b)

Figure 1.136: Electric field lines for systems of charges (a)


The electric field lines for a dipole form closed loops that become
more widely spaced with distance from the charges. note that at
~ is tangent to the field
each point in space, the electric field vector E
lines. (b) All of the field lines in a system with charges of the same Figure 1.138
sign extend to infinity

2. Rank the magnitudes of the electric field at points A, B,


6. Lines of force originate or terminate perpendicular to the and C shown in Figure1.139 (greatest magnitude first).
metal surface
3. • For the drawing shown in Figures1.140, write the ratio
7. The density of the lines at any point (the number of lines of charges q1 and q2 .
per unit area through a surface element normal to the
lines) is proportional to the magnitude of the field there. 4. • The electric field lines surrounding three charges are
shown in Figure1.141 (a) Which of the charges qA , qB ,
and qC are positively charged, and which are negatively
1.16.3 Check Point 5 charged? (b) Rank the charges in order of increasing mag-
1. • The electric field lines for a system of two charges are nitude.
shown in Figure1.138 (a) What is the sign of charge 1?
5. • Sketch the electric field lines for the system of charges
(b) What is the sign of charge 2? (c) Is the magnitude of
shown in Fig. 1.142.
charge 1 greater than, less than, or equal to the magnitude
1.16. THEOREM ON CIRCULATION OF VECTOR E 67

A
C
1 1

Figure 1.143

Figure 1.139

+ + −

q2 (a) (b)

q1
+ +
+ −

(a) (b)

Figure 1.140 (c) (d)

Figure 1.144

qA Multiple Choice Questions


8. • In the Figure1.145, rank points 1 − 4 in order of
increasing field strength.
(A) 2, 3, 4, 1 (B) 2, 1, 3, 4
(C) 1, 4, 3, 2 (D) 4, 3, 1, 2
qB
qC 2
E
Copyright 2017 Cengage Learning. All Rights Reserved. May not be copied, scanned,

2 3
1
Figure 1.141
4
Copyright 2017 Cengage Learning. All Rights Reserved. May not be copied, scanned
2

Figure 1.142 Figure 1.145

Copyright 2017 Cengage Learning. All Rights Reserved. May not be copied, scanned, or duplicated, in whole or in part. WCN 02-200-203

6. • Sketch the electric field lines for the system of charges Answer Keys and Solutions
described in Figure1.143.
1. (a) Charge 1 is negative. (b) Charge 2 is positive. (c)
7. • Which lines in Figure1.144 cannot represent an electric The magnitude of charge 1 is greater than the magnitude
field? Explain. of charge 2. 00-203
68 CHAPTER 1. ELECTRIC CHARGE AND FIELD

2. A > B > C 1.17.1 Motion of a charged Particle in a Uni-


form Electric Field
3. (a). 2 : 1 (b) 1 : 3
When a particle of charge q and mass m is in an external
4. (a) charges A and C are positive and charge B is negative electric field of strength E, ~ a force q E~ will be exerted on this
(b) |qA | : |qB | : |qC | = 5 : 10 : 15 = 1 : 2 : 3 i.e., |qA | < particle. If q ~
E is the only acting force on the particle, then
|qB | < |qC | according to Newton’s second law, Σ ~
F = m~a, the acceleration
of the particle will be given by:
5. See Fig.1.146
~
~a = q E/m (1.88)

Motion of a Charged Particle Along an Electric Field


Consider a particle of positive charge q and mass m in a
uniform horizontal electric field E~ produced by two charged
plates that are separated by a distance d as shown in Fig.1.148.
If the particle is released from rest at the positive plate and

v
Figure 1.146 ° 0
qE qE
t 0 t
6. See Fig.1.147

d
Figure 1.148

~ is the only force that acts on the particle, then the particle
qE
will move horizontally along the x -axis with an acceleration
~
~a = q E/m. In such a case, we can apply the kinematics
equations on the initial and final motion as follows:

• The particle’s time of flight t:


1
x = v◦ t + at2
2
1 qE 2
⇒d=0+ t
Figure 1.147 2 m
s
2md
⇒ t= (1.89)
7. (b), (d) qE

• The speed of the particle v:


8. (B) The electric field depends on the density of electric field
v = v◦ + at
lines. The field will be greater where the density of field
lines is greater.
s
qE 2md
⇒v =0+
m qE

1.17 Action of the Electric Field on r


2qEd
⇒ v= (1.90)
Charges m

A uniform electric field can exert a force on a single charged • The kinetic energy of the particle K :
particle and can exert both a torque and a net force on an 1
electric dipole. K= mv 2
2
1.17. ACTION OF THE ELECTRIC FIELD ON CHARGES 69

⇒ K = qEd (1.91) by charged particle in reaching to a general point P (x, y) in


~
The last result can also be obtained from the application electric field E is given by-
of the work-energy theorem W = ∆K because W = (qE)d x
and ∆K = Kf − Ki = K. t= (1.92)
v0
EXAMPLE 51. In Fig.1.148, assume that the charged par- The upward component of the velocity of the charged particle
ticle is a proton of charge q = +e. The proton is released from as it emerges from the field region is
rest at the positive plate. In this case, each of the two oppo-
qEx
sitely charged plates which are d = 2 cm apart has a charge per vy = uy + ay t = 0 +
unit area of σ = 5µC/m2 . (a) What is the magnitude of the mv0
electric field between the two plates? (b) What is the speed of qEx
the proton as it strikes the second plate? or vy = (1.93)
mv 0
SOLUTION (a) The electric field arises from two infinite At point P , The horizontal component of velocity remains vx =
plates, Thus: v0 . The angle θ made by the resultant velocity with the original
direction is given by
σ σ σ 5 × 10−6 C/m2
E= + = = = 5.65×105 N/C vy qEx
2◦ 2◦ ◦ 8.85 × 10−12 C/N · m2 tan θ = =
vx mv02
(b) We first find the proton’s acceleration from Newton’s sec-
ond law: Thus, the charged particle deviates by an angle,

1.6 × 10−19 C 5.65 × 105 N/C


 
F eE qEx
a= = = = 5.41×1013 m/s2 θ = tan−1 (1.94)
m m 1.67 × 10−27 kg mv02

Then, using x = v◦ t + 12 at2 , we find that d = 12 at2 . Thus: Vertical displacement y can be given by-
s 1
y = uy t + ay t2
r
2d 2(0.02 m)
t= = = 2.72 × 10−8 s 2
a 5.41 × 1013 m/s2 1 qEx2
=0+
Finally, we use v = v◦ + at to find the speed of the proton as 2 mv02
follows:
1 qEx2
⇒ y= (1.95)
v = at = 5.41 × 1013 m/s2 2.72 × 10−8 s = 1.47 × 106 m/s
 
2 mv02
This is an expression for path of a charged particle in a uni-
Motion of a Charged Particle Perpendicular to an
form electric field. Since Eq.(1.95) is of the form of a parabola,
Electric Field
therefore the path of charged particle will be parabolic in uni-
~ is created between two parallel, form electric field.
Let a uniform electric field E
charged plates as shown in Fig.1.149. A charged particle of When, charged particle particle emerges from the field, x = l,
charge q enters the field symmetrically between the plates with so from Eq.(1.95) the vertical displacement can be written as-
an initial speed v0 . The length of each plate is l.
1 qEl2
y1 = (1.96)
+ 2 mv02
y t1
y2
h The deviation of charged particle at x = l, is given by-
α
t 0 E +
y1 y1 qEl
P(x, y) D α = tan−1 (1.97)
q+ x mv02
O
The extra vertical distance y2 that the charged particle will
move before hitting the screen, which is located at a horizontal
l distance D from the plates, is given by:
D

Figure 1.149: The effect of an upward force q E~ exerted on charged qEl


y2 = D tan α = D
particle projected horizontally with speed v0 into a downward uni- mv◦2
~
form electric field E
Finally, the total vertical distance h that the charged particle
will move is:
The acceleration of the electron is a = in upward qE
m
direction. The horizontal velocity remains v0 as there is no
 
qEl l
acceleration in horizontal direction. Thus, the time taken h = y1 + y2 = + D
mv◦2 2
70 CHAPTER 1. ELECTRIC CHARGE AND FIELD

EXAMPLE 52. An electron enters the region of a uniform


electric field as shown in Fig.1.149, with v0 = 3.00 × 106 m/s
and E = 200 N/C. The horizontal length of the plates is
l = 0.100 m.
(A) Find the acceleration of the electron while it is in the elec-
tric field.
(B) If the electron enters the field at time t = 0, find the time
at which it leaves the field. Figure 1.151
(C) If the vertical position of the electron as it enters the field
is y0 = 0, what is its vertical position when it leaves the field?
acting on the block are-
SOLUTION (A) The charge on the electron has an abso- 1.Gravitational force m−→g : acting downwards.
lute value of q = 1.60 × 10−19 C, and me = 9.11 × 10−31 kg. −

2. Normal reaction N : acting upwards.
Therefore, acceleration of electron- −

3. Electric force F e (Fe = qE): acting towards right


1.60 × 10−19 C (200 N/C) 4. Spring force F s (Fs = kx): acting towards left.

eE
~a = − ĵ = − ĵ For vertical equilibrium of the block, N = mg. For horizontal
me 9.11 × 10−31 kg
equilibrium, we have-
= −3.51 × 1013 ĵm/s2
Fs = Fe ⇒ kx0 = qE
(B) The horizontal distance across the field is l = 0.100 m.
Using Equation (1.92) with x = l, we find that the time at qE
or x0 =
which the electron exits the electric field is k
Now, if we turnoff the electric field, the block begins to oscil-
l 0.100 m late about mean position x = 0 with amplitude of x0 = qE
t= = = 3.33 × 10−8 s k .
v0 3.00 × 106 m/s
(C) Vertical displacement of electron is given by-
1 1
y = 0 + at2 = − 3.51 × 1013 m/s2 3.33 × 10−8 s
 2 1.18 Electric Dipole in an External
2 2
= −0.0195 m = −1.95 cm
Electric Field
If the electron enters just below the negative plate in Figure 1.18.1 Electric force and torque on electric
1.149 and the separation between the plates is less than the dipole in a uniform external electric
value we have just calculated, the electron will strike the
field
positive plate.
Fig.1.152 shows an electric dipole AB of length 2l in a uniform


external electric field E . The charges at A and B are −q and
EXAMPLE 53. A block of mass m having a charge q is
+q respectively. The direction of dipole moment vector − →
p,
placed on a smooth horizontal table and is connected to a wall
is from −q to +q i.e., from point A to B. The angle between
through an unstressed spring of spring constant k as shown in −→
Fig.1.150. A horizontal electric field E parallel to the spring dipole moment vector −→p (which is along AB) and electric field


is switched on. Find the extension in the spring. Now, if we vector E is θ. The electric forces on −q and +q have same
turn off the field, find the amplitude of the resulting SHM of
the block.

Figure 1.150

Figure 1.152: (a) An electric dipole has an electric dipole moment


SOLUTION Suppose, the initial position of the block is at p
~ in an external uniform electric field E.~ The angle between p ~
x = 0. At this instant, the spring has it’s natural length. ~
and E is θ. The line connecting the two charges represents their


Now, if we turn on the horizontal electric field E as shown in rigid connection and their center of mass is assumed to be midway
Fig.1.151, it applies an electric force Fe = qE on the block in between them. (b) Representing the electric dipole by a vector p ~ in
~ and showing the direction of the torque
the direction of electric field that causes the spring to stretch the external electric field E
to an equilibrium length x = x0 . In equilibrium, the forces ~τ into the page by the symbol ⊗
1.18. ELECTRIC DIPOLE IN AN EXTERNAL ELECTRIC FIELD 71

2 2
magnitude (= qE) but their directions are opposite to each then torque can also be written as τ = I ddt2θ , here ddt2θ is the
other (Fig.1.152a) The electric force on −q is opposite to E ~ angular acceleration of the dipole. For small angular displace-
~
and on +q, it is parallel to E. ment, sin θ ≈ θ, therefore, we can write above equation as-
The net force on the dipole is Fnet = −qE + qE = 0 Thus, net
d2 θ
force on an electric dipole in a uniform external electric field I = −pEθ
dt2
is zero, so the dipole will have no translational acceleration.
Since, the lines of action of these forces, are not same, so they d2 θ pE
produce torque about the center of mass of the dipole. This ⇒ =− θ (1.100)
dt2 I
torque tends to rotate the dipole to bring − →
p into alignment which is of the form of angular SHM


with E .
If ~τ1 and ~τ2 are the respective torques produced by forces on d2 θ
= −ω 2 θ
+q and −q about the center of mass of the dipole, then from dt2
Fig.1.152, the magnitude of net torque on the dipole- r
pE
with ω =
τ = τ1 + τ2 = qE(l sin θ) + qE(l sin θ) I
Thus, if in uniform electric field the dipole is rotated by a
= 2qlE sin θ = pE sin θ small angular displacement θ and released, the electric dipole
⇒ τ = pE sin θ (1.98) performs angular SHM with time period-
In vector form, torque on dipole due to external uniform elec- s

→ 2π I
tric field E can be written as- T = = 2π (1.101)
ω pE
~
~τ = p~ × E (1.99)

which is consistent with the directional relationships for the 1.18.3 Dipoles in a Nonuniform Field
cross product, as shown by the three vectors in 1.152b Suppose that a dipole is placed in a nonuniform electric field
Unit: Unit of torque is “newton-meter”. in which the electric field is changing along x-axis. Let E and
Maximum and Minimum Values of Torque: E + dE are magnitudes of electric fields at positions A and
(i) Maximum torque applied by electric field on the dipole is
B respectively. The first response of the dipole is to rotate
given by- until it is aligned with the field, with the dipole’s positive end
pointing in the same direction as the field. Now, however, there
τmax = pE(sin θ)max is a slight difference between the forces acting on the two ends
Since, (sin θ)max = 1 and corresponding to it θ = π/2, of the dipole. This difference occurs because the electric field
therefore- is not same at positions A, and B. The net force exerted by
external non-uniform electric field on the dipole is given by-
τmax = pE
(ii) Minimum torque applied by electric field on the dipole is
given by-

τmax = pE |(sin θ)min |


Since, |(sin θ)min | = 0 and corresponding to it θ = 0, therefore-

τmin = 0
~ zero
The torque is greatest when p~ is perpendicular to E,
~
when p~ is aligned with or opposite to E.

1.18.2 The Angular Acceleration and Time Figure 1.153


period of a Dipole in an External Uni-
form Electric Field F = q (E + dE) − qE = qdE
Since, the torque applied by electric field on the dipole is restor- dE dE
⇒F =q dx = qdx (1.102)
ing, therefore we can write equation (1.98) as follows- dx dx
From Fig.1.153, dx = AC = 2l cosθ, therefore, on substituting
τ = −pEsinθ this value of dx in above equation, we get-
If I is the moment of inertia of dipole about an axis passing dE
through its centre and perpendicular to the plane of the Fig.??, F = q (2l cos θ)
dx
72 CHAPTER 1. ELECTRIC CHARGE AND FIELD

When the dipole is aligned in the direction of field, θ = 0, sense.


therefore, cos θ = 1. In this case, Eq.(1.102) becomes- 4. Electrostatic force of upward electric field E: ~ On charge
−q, it is −q E~ and directed opposite to E ~ i.e., in downward
dE dE direction and on charge +q, it is +q E ~ in the direction of E,
~
F = (2ql) =p (1.103)
dx dx i.e., in upward direction (Fig.1.154). These two forces are
equal in magnitude (qE) but opposite in direction. So, the
So, once the dipole is aligned, the rightward force on its
net electrostatic force on the wheel due to dipole attached to
positive end is slightly stronger than the leftward force on its
it, is zero. However, the lines of action of these electrostatic
positive end. This causes a net force towards the stronger
forces are not passing through the center of mass (CM) of the
region of the field. For any nonuniform electric field, the net
wheel, so it will definitely provide a mechanical torque about
force on a dipole is toward the direction of the strongest field.
CM.
Because any finite-size charged object, such as a charged rod
Conditions for Static Equilibrium for the Wheel-
or a charged disk, has a field strength that increases as you
Dipole System
get closer to the object, we can conclude that a dipole will
Let us consider positive direction of x axis along the inclined
experience a net force toward any charged object.
plane in downward direction and positive direction of y axis
perpendicular to it in upward sense. In static equilibrium, we
EXAMPLE 54. A wheel having mass m, charge +q and −q have-
on diametrically opposite points, is in equilibrium on a rough 1. Conditions for Translational Equilibrium: For translational
inclined plane in the presence of uniform vertical electric field equilibrium along x and y direction-

→ ~ ΣFx = 0, ΣFy = 0
E . Find the magnitude of E.
2. Condition for Rotational Equilibrium: In rotational
equilibrium, the net torque on the dipole-wheel system about
center of mass (CM),
ΣτCM = 0
Now, apply these conditions and simplify for minimum
required value of electric field.
SOLUTION If µ is the coefficient of static friction between
the wheel and the inclined plane, then for translational
equilibrium of the wheel, we have-
1. ΣFy = 0 ⇒ N − mg cos θ = 0

⇒ N = mg cos θ (i)
2. ΣFx = 0 ⇒ (fs )max − mg sin θ = 0
Figure 1.154

⇒ (fs )max = mg sin θ (ii)


APPROACH A wheel can be considered as a rigid body.
The system containing wheel, attached with charges +q and In limiting translational equilibrium, we have-
−q at it’s diametrically opposite ends, is shown in Fig.1.154. (fs )max = µs N
The forces acting on the wheel are- Substituting this value of (fs )max in Eq.(ii ), we get-

µs N = mg sin θ (iii)
From Eq.(i) and (iii), we get-

µs = tan θ (iv)
And for rotational equilibrium, the net torque on the wheel
about it’s center of mass should be zero, i.e.,

Στ = τelectric field + τmg + τfriction = 0 (v)


Considering, perpendicularly outward to the plane of figure as
a positive direction of torque, the torque of electric field about
the center of mass of the wheel
Figure 1.155
τelectric field = pE sin θ = +2qrE sin θ (vi)
1. Gravitational force m~g : acting in downward direction
2. Normal reaction N ~ : acting perpendicular to the inclined The direction of this torque is normally outward to the plane
plane of the figure.
3. Force of static friction f~s : acting along the plane in upward Torque of weight mg about the center of mass of the wheel is
1.18. ELECTRIC DIPOLE IN AN EXTERNAL ELECTRIC FIELD 73

zero, because it passes through the center of mass of the wheel. SOLUTION The electric field at an axial position of charged
ring is given by-
τmg = 0 (vii) kQx
E= 3/2
(1.105)
Torque of friction about the center of mass of the wheel- (R + x2 )
2

On substituting this value in Eq.(1.104), we get-


τf riction = −r × µs N (viii) !
Z ∞
The direction of this torque in perpendicularly inward to the kQλx
F = 3/2
λdx
plane of the figure. Substituting the value of µs N , from Eq. 0 (R2 + x2 )
(iii) in Eq. (viii), we get- Z ∞ !
x
or F = kQλ 3/2
λdx (1.106)
τf riction = −rmg sin θ (ix) 0 (R2 + x2 )
Using, Eq.(vi), (vii) and (ix) in Eq. (v), we get- To solve right hand integration, we let-
2qrE sin θ − rmg sin θ = 0
R 2 + x2 = t
mg
⇒ E= (x) 1
2q ⇒ xdx = dt
2
This is the required electric field needed for equilibrium of the when x → 0, then t → R2
wheel. and when x → ∞, then t → ∞
EXAMPLE 55. A system consists of a thin uniformly Now, substituting these values in Eq.(1.106), we get
charged wire ring of radius R and a very long uniformly charged kQλ ∞ dt
Z
thin rod oriented along the axis of the ring, with one of its ends F = 3/2
2 R2 t
coinciding with the centre of the ring. The total charge of the Z ∞  ∞
ring is equal to Q. The linear charge density of the rod is λ. kQλ 1
= t−3/2 dt = kQλ − √
Find the interaction force between the ring and the thread. 2 R2 t R2
kQλ
Note: To determine the total electric force between =
R
the rod and ring, we can use following two approaches-
APPROACH 1. Fig.1.156 shows a thin charged wire ring of So, the force on the rod is-
radius R carries a total charge Q uniformly distributed around
it. A very long uniformly charged wire having uniform charge kQλ
F = (1.107)
density λ is placed along it’s axis in such a way that it’s one R
end coincides with the center of the ring. By, Newton’s third law, a force of equal in magnitude but
Now, consider an elementary length dx of the rod at distance opposite in direction, will act on the ring.
APPROACH 2. In second approach, we first calculate the
electric field of the rod at any point on the circumference of
R ring. Since the ring is symmetrical about the rod, therefore,
Q the magnitude of electric field of the rod will be equal for each
dq position on the circumference of the ring.
O x
x dx Now, calculate the force on an elementary segment of ring due
to this field. Break it in component form and use symmetrically
opposite segment to know which component will get cancelled
and which will sum to provide the net force on the ring. Now
integration of this component for the total circumference gives
Figure 1.156 us the net force on ring due to charged rod.
SOLUTION Electric field of the charged thin rod, at any
x from the center of the ring and find the electric field E of point on the circumference of the ring is given by-
the ring at the position of the elementary charge of rod. If the √
kλ 2
elementary charge of the rod is dq(= λdx) then electric force Erod =
on this elementary charge due to electric field of ring is- R
This field is directed at angle 4 from the negative direction of
π
dF = Edq x-axis (see Fig.)
or dF = Eλdx Now consider an elementary part of ring having charge dQ.
The electric force on this part of the ring is given by-
The integration of this expression from x = 0 to x = ∞,

will give the net force F on the rod exerted by charged ring,i.e., kλ 2
dF = Erod dQ = dQ
Z ∞ R
F = Eλdx (1.104) This force has two components-
0
74 CHAPTER 1. ELECTRIC CHARGE AND FIELD

example, imagine three negative charges at the corners of an


equilateral triangle in a horizontal plane. Now, put a positive
charge at the center of the triangle and ignore gravity for
the moment (although including it would not change the
R results). The net electrostatic force on the positive charge is
Q zero at the center. So the charged particle is in equilibrium.
O x Now, suppose we displace it from its equilibrium position and
observe the direction of net force at new position, we find that
it is not restoring, i.e., not towards the center of the triangle,
so the equilibrium will be unstable.

This theorem also holds for a complicated arrangement of


charges held together in fixed relative positions—with rods.
For example, consider two equal charges fixed on a rod. This
Figure 1.157
combination too cannot be in a stable equilibrium in some elec-
trostatic field, because, the total force on the rod cannot be
1. dFx = dF cos π4 : acting along the negative direction of the restoring for displacements in every direction. The extension
axis of the rod. of the argument shows that no rigid combination of any num-
ber of charges can have a position of stable equilibrium in an
2. dFy = dF sin π4 : acting perpendicular to the axis of the electrostatic field in free space.
ring, i.e., acting along the outward radius vector of the If want to make equilibrium stable, we have to use pivots or
ring. other mechanical constraints. For example, consider a hollow
tube in which a charge can move back and forth freely, but
Now, Consider two segments at the top and bottom of the not sideways. Now, produce inward electric fields at both the
ring: The perpendicular force component on the top segment ends by fixing two identical positive charges at the ends of tube.
dF sin π4 get cancelled by equal and opposite perpendicular Near the center of the tube, electric fields also directed towards
force component of bottom segment. Hence the total perpen- the sidewall of the tube[Fig.1.158]. This time, the equilibrium
dicular component of force due to this pair of segments is zero. is a stable one. The sidewall restricts its lateral motion by ap-
When we add up the components of all such pairs of segments, plying nonelectrical normal force to it whereas inward electric


the total force F will have only a component along the nega- fields provide restoring force to it. A theoretical proof of the
tive direction of ring’s symmetry axis (the −x -axis), with no theorem, by applying Gauss’s law, is given in ’EXAMPLE 67’
component perpendicular to that axis (that is, no component
in yz-plane).
So, net force on the ring is given by-
π
Z Z
F = dFx = dF sin   
4
Z Q √
kλ 2 π +ROORZ
= dQ sin 7XEH
0 R 4
Z Q
kλ Figure 1.158: A charge can be in equilibrium if there are mechan-
= dQ ical constraints.
0 R

kQλ
or F = (1.108) EXAMPLE 56. Earnshaw’s theorem says that no particle
R
can be in stable equilibrium under the action of electrostatic
As expected, Eq. (1.107) and (1.108) are same. If the charge forces alone. Consider, however, point P at the center of a
density on the ring is also same as that of rod, then- square of four equal positive charges, as in Figure1.159. If you
Q = λ(2πR) put a positive test charge there it might seem to be in stable
Substituting this value and k = 4π 1
0
in Eq.(1.107) or (1.108) equilibrium. Every one of the four external charges pushes it
gives us the result- toward P . Yet Earnshaw’s theorem holds. Can you explain
how?
λ2
F = (1.109) SOLUTION The equilibrium of positive test charge at the
20
point P is unstable. Because, on shifting it from mean position
(including along diagonal) there is no net restoring force on
1.19 Earnshaw’s Theorem it towards the position P . So, the equilibrium is unstable.
For a stable equilibrium, the total force on the test charge
According to this theorem, A charged particle cannot be held should always be restoring for displacements in every direction.
in a stable equilibrium by electrostatic forces alone. As an
1.19. EARNSHAW’S THEOREM 75

1.19.1 Check Point 6


Multiple Choice Questions
1. • A charged drop of mass m flots in air in electric field
E. The magnitude of charge on the drop is-
(A) mg
E (B) mgE

(C) Eg
m
(D) none of these
2. •• A point charge q moves from point P to S along the


path PQRS in a uniform electric field E pointing parallel
Figure 1.159
to the positive direction of the x-axis. The coordinates of
the point P, Q, R and S are (a, b, 0), (2a, 0, 0), (a, −b, 0)
EXAMPLE 57. Earnshaw’s theorem states that a point and (0, 0, 0) respectively. The work done by the field in
charge cannot be in stable equilibrium while purely electrostatic the above process is given by the expression-
forces act on the point charge.
Consider a ring that is uniformly positively charged, with a
positive charge at the center. It appears that the center charge
suffers an identical repulsive force from every direction. How
can the theorem be true?
SOLUTION If we shift the positive charge along the sym-
metric axis of the ring, there will be a repulsion of ring charge
on it. As a result of it, the charge moves away from the center.
So, the equilibrium at the center of the ring is unstable. For
a stable equilibrium, there must be a restoring force on the
charge.
Figure 1.162

EXAMPLE 58. If a dielectric in the form of a sphere is


(A) qaE (B) −qaE
introduced into a homogeneous electric field. A, B and C are
(C)q (a2 + b2 )E (D) 3qE (a2 + b2 )
p p
three points as shown in Fig.1.160, then find the points where
the intensity of electric field increases and where it decreases. 3. •• A simple pendulum consists of a small sphere of mass
m suspended by a thread of length l. The sphere carries
a positive charge q. The pendulum is placed in a uniform
electric field of strength E directed vertically upwards.
With what period will the pendulum oscillate if the
electrostatic force acting on the sphere is less that the
Figure 1.160 gravitational force? Assume the oscillations to be small
 1/2  1/2
(A) T = 2π gl (B) T = 2π qE ml

SOLUTION The intensity of electric field at points A, C will


 1/2  1/2
increase and at B it will decrease. (C) T = 2π g−lqE (D) T = 2π g+lqE
( m) ( m)
EXAMPLE 59. Show that the charges of each sign induced 4. • • • A simple pendulum of mass m and length l carries a
on a conductor B, when it is approached by a point charge charge q. Find it’s time period when it is suspended in a
charge A of magnitude +q (Fig.1.161), is always less than q. uniform electric
r field region as shown in figure1.163-
r
(A) T = 4π √ 2 l (B) T = 2π √ l
g +(Eq/m)2 g 2 +(Eq/m)2
r
l2
(C) T = 2π 2 √ l
2
(D) T = 2π √ 2
g 2 +(Eq/m) g 2 +(Eq/m)

Figure 1.161

SOLUTION Since the sum of the charges in the system as


a whole is not zero, and the conductor A does not enclose
the charges +q, part of the lines of forces from +q must go
to infinity (or terminate on other conductors), and only a
fraction of the lines of force from +q terminate in the induced
charge of B. The induced charge (qind ) is thus less than q. Figure 1.163
76 CHAPTER 1. ELECTRIC CHARGE AND FIELD

5. •• An electric dipole, made up of a positive and a negative Time period


charge, each of magnitude 1 µC and placed at a distance #1/2
2 cm apart, is placed in an electric field 105 N/C. Com-
s "
l l
pute the maximum torque which the field can exert on T = 2π = 2π
g0 g − qE
the dipole. m

(A) 6 × 10−3 N.m


(B) 3 × 10−3 N.m
q
4. (B) Time period of the pendulum= 2π l
gef f
(C) 4 × 10−3 N.m
(D) 2 × 10−3 N.m Tension in the string in equiblirium position
Here, gef f = mass of the bob
6. •• An electric dipole of dipole moment p is placed in a q
uniform electric field E in stable equilibrium position. Its 2
(mg) + (Eq)
2 q
2
moment of inertia about the centroidal axis is I. If it is = = g 2 + (Eq/m)
m
displaced slightly from its mean position find the period
of small oscillation. r
Therefore, T = 2π .
q q l
(A) T = 2π PIE (B) T = 2π PIE √
g 2 +(Eq/m)2
q q
(C) T = 4π PIE (D) T = 4π 2 PIE
5. (D) The torque exerted by an electric field E on a dipole
of moment p is given by
Answer Keys and Solutions
1. (A) As the charged particle is floating in air (neglecting τ = pE sin θ
the buoyant force due to air we obtain-
where θ is the angle which the dipole is making with the
mg = qE electric-field.
mg Corresponding to maximum torque, θ = 90◦ , therefore-
or q=
E
τmax = pE
2. (B) Since, the electric field is a conservative field, there-
fore, the work done by electric field is independent of the Here, p = q (2l) = 1 × 10−6 × 0.02 C/m and E = 105 N/C
path followed and is equal to- Therefore, τmax = 1 × 10−6 × 0.02 × 105 = 2 × 10−3 N.m
 
W = qE ~ .~r
6. (B) When displaced at an angle
where ~r = displacement from P to S.
Here, −
→ ~ = E î
r = −aî − bĵ, while E θ
Therefore, work = (−qE î). (aî + bĵ) = −qaE
, from its mean position, the magnitude of restoring
3. (C) Let x be the small displacement given to the pendulum torque is
such that the angle θ is small. The forces acting at A are
[Fig.1.164]- τ = −pE sin θ
For small angular displacement sin θ ≈ θ, therefore-

τ = −pEθ
The angular acceleration is,
α = τIθ = − pE −cos2 θ Where ω 2 =
I θ =q
pE
I
Therefore, T = 2π pE I

Figure 1.164
1.20 Conductors in Electrostatic
Equilibrium
(i) tension T along the thread A good electric conductor like copper, although electrically
(ii) weight mg acting vertically downwards. neutral, contains charges (electrons) that aren’t bound to any
(iii) electrical force qE vertically upwards. atom and are free to move about within the material. When no
The resultant force vertically down wards is (mg˘qE). net motion of charge occurs within a conductor, the conductor
Therefore, effective acceleration is said to be in electrostatic equilibrium. An isolated con-
qE ductor (one that is insulated from ground) has the following
g0 = g − properties:
m
1.20. CONDUCTORS IN ELECTROSTATIC EQUILIBRIUM 77

which would exert a force on free electrons at the surface,


causing the electrons to move along the surface until they
reached positions where no net force was exerted on them
parallel to the surface-that is, until the electric field was
perpendicular to the surface. This is the reason, why E, ~
at each point, is normal to the conductor’s surface.

Figure 1.165: The electric field inside e a charged conductor.

1. The electric field is zero everywhere inside the conducting


material. In terms of field lines, we can say that, there are
no field lines within the conducting material [Fig.1.165]. Figure 1.167: This situation is impossible if the conductor is in
~ had component par-
electrostatic equilibrium. If the electric field E
Reason: If the electric field within a conducting material
is nonzero, it exerts a force on each of the mobile charges allel to the surface, an electric force would be exerted on the charges
along the surface and they would move to the left.
(usually electrons) and makes them move preferentially in
a certain direction. With mobile charge in motion, the
conductor cannot be in electrostatic equilibrium.
4. On an irregularly shaped conductor, the charge accumu-
2. Any excess charge on an isolated conductor resides entirely lates at sharp points, where the radius of curvature of the
on its surface. surface is smallest. In terms of field lines this can be
Reason: This Property is a direct result of the 1/r2 re- stated that the field lines are much denser near the more
pulsion between like charges described by Coulomb’s law. sharply curved surface than the less sharply curved sur-
If by some means an excess of charge is placed inside a con- face.[Fig.1.168b]
ductor, the repulsive forces between the like charges push Reason: Think of the charges as being constrained to
them as far apart as possible, causing them to quickly mi-
grate to the surface. So, the electric field is zero within the
conducting material, but is not necessarily zero outside.

3. The electric field just outside a charged conductor is per-


pendicular to the conductor’s surface (Fig.1.166). In
terms of field lines, we can say that- the field lines that
start or stop on the surface of a conductor are perpendic-
ular to the surface where they intersect it.
Reason: This property can be understood by con- Figure 1.168: (a) Repulsive forces on a charge constrained to move
along a curved surface due to two of its neighbors. The parallel
components of the forces Fk determine the spacing between the
E field lines charges.(b) For a conductor in electrostatic equilibrium, the surface
charge density is largest where the radius of curvature of the surface
is smallest and the electric field just outside the surface is strongest
+– there.
– +
– +
– E=0 + move along the surfaces of the conductor. On flat sur-
– +
– + faces, repulsive forces between neighboring charges push
– + parallel to the surface, making the charges spread apart
evenly. On a curved surface, only the components of the
repulsive forces parallel to the surface, Fk , are effective at
Figure 1.166: Electric field lines meet a conducting surface at making the charges spread apart (Fig.1.168a). If charges
~ vanishes inside the conductor
right angles and the electric field E were spread evenly over an irregular surface, the paral-
lel components of the repulsive forces would be smaller
sidering what would happen if it were not true. If the for charges on the more sharply curved regions (since, at
electric field in were not perpendicular to the surface, more sharply curved surface, the angle between E ~ and
it would have a component along the surface(Fig.1.167), Fk is larger that that of less sharply curved surface) and
78 CHAPTER 1. ELECTRIC CHARGE AND FIELD

charge would tend to move toward these regions. 2. •• A toy car with charge q moves on a frictionless
The electric field lines just outside a conductor are densely horizontal plane surface under the influence of a uniform
packed at sharp points because each line starts or ends on ~ Due to the force q E,
electric field E. ~ its velocity increases
a surface charge. Since the density of field lines reflects from 0 to 6 m s−1 in one second duration. At that instant
the magnitude of the electric field, the electric field out- the direction of the field is reversed. The car continues
side the conductor is largest near the sharpest points of to move for two more seconds under the influence of this
the conducting surface. The electric field near very sharp field. The average velocity and the average speed of the
points may be strong enough to ionize the air around it. toy car between 0 to 3 seconds are respectively-
(A) 2 m s−1 , 4 m s−1 (B) 1 m s−1 , 3 m s−1
EXAMPLE 60. Fig.1.169 shows a positively charged metal (C) 1 m s , 3.5 m s
−1 −1
(D) 1.5 m s−1 , 3 m s−1
sphere above a conducting plate with a negative charge. Sketch
the electric field lines. 3. •• A particle of mass m and charge q is placed at rest in
a uniform electric field E and then released. The kinetic
energy attained by the particle after moving a distance y is
(A) qEy (B) qE 2 y
(C) qEy 2
(D) q 2 Ey
4. •• A simple pendulum has a bob of mass m which carries
a charge q on it. Length of the pendulum is L. There
is a uniform electric field E in the region. Calculate the
time period of small oscillations for the pendulum about
its equilibrium position in following cases:

(a) E is vertically down having magnitude E = mg


q
Figure 1.169 2mg
(b) E is vertically up haying magnitude E = q
APPROACH Field lines start on positive charges and end (c) E is horizontal haying magnitude E = mg
q
on negative charges. Thus we draw the field lines from the −
→ √
positive sphere to the negative plate, perpendicular to both (d) E has magnitude ofE = 2mg/q and is directed
surfaces, as shown in Fig.1.170. The single field line that goes upward making an angle of 45◦ with the horizontal.
upward tells us that there is a field above the sphere, but that 5. •• In a region of space an electric field line is in the shape
it is weak. of a semicircle of radius R. Magnitude of the field at
all point is E. A particle of mass m having charge q is
constrained to move along this field line. The particle is
released from rest at A [Fig.1.171].

Figure 1.170: Drawing field lines from sphere to plate. Figure 1.171

(a) Find its kinetic energy when it reaches point B.


1.20.1 Check Point 7
(b) Find the acceleration of the particle when it is at
1. ••An electron falls from rest through a vertical distance h midpoint of the path from A to B.
in a uniform and vertically upward directed electric field
E. The direction of electric field is now reversed, keeping 6. ••Electric field E = −bx + a exists in a region parallel
its magnitude the same. A proton is allowed to fall from to the X direction (a and b are positive constants). A
rest in it through the same vertical distance h. The time charge particle having charge q and mass m is released
of fall of the electron, in comparison to the time of fall of from the origin X = 0. Find the acceleration of the
the proton is particle at the instant its speed becomes zero for the first
(A) smaller (B) 5 times greater time after release.
(C) 10 times greater (D) equal
1.20. CONDUCTORS IN ELECTROSTATIC EQUILIBRIUM 79

7. •• Two identical blocks resting on a frictionless, hori-


zontal surface are connected by a light spring having a
spring constant k = 100 N/m and an unstretched length
Li = 0.400 m as shown in Figure 1.172a. A charge Q is
slowly placed on each block, causing the spring to stretch
to an equilibrium length L = 0.500 m as shown in Figure
1.172b. Determine the value of Q, modeling the blocks as
charged particles.
Figure 1.174

Figure 1.172
Figure 1.175

8. •• A charged particle (mass m and charge q ) moves


along X axis with velocity V0 . When it passes through
the origin it enters a region having uniform electric


field E = −Eĵ which extends upto x = d [Fig.1.173].
Equation of path of electron in the region x > d is:
(A) y = mqEd
V20
x (B) y = mqEd
V20
(x − d)
qEd2
(B) y = m V2 2 − x
qEd d
(D) y = m V2 x

0 0

Figure 1.176

11. A point Charge q is placed inside the cavity of a metal-


lic shell. Which one of the diagram [Fig.1.177], correctly
represents the electric lines of force.

Figure 1.173

9. •• A small point mass carrying some positive charge on


Answer Key and Solutions
it, is released from the edge of a table. There is a uni-
form electric field in this region in the horizontal direction
[Fig.1.174]. Which of the following options then correctly 1. (A): Force experienced by a charged particle in an electric
describe the trajectory of the mass [Fig.1.175]? (Curves field, F = qE
are drawn schematically and are not to scale) As F = ma ∴ ma = qE ⇒ a = qE m As electron and
proton both fall from same height at rest. Then initial
10. •• A ring has charge Q and radius R. If a velocity = 0 From s = ut + 21 at2 (∵ u = 0) ∴ h =
charge q is placed at its centre then the in-
q √
1 2
at ⇒ h = 1 qE 2
t [ Using (i) ] ∴ t = 2hm
qE ⇒ t ∝ m
crease in tension in the ring is [Fig.1.176] 2 2 m
as ’ q ’ is same for electron and proton. As electron has
(A) 4πεQq (B) zero
0R
2
smaller mass so it will take smaller time.
(C) 4π2 ε0 R2
Qq
(D) 8π2 ε0 R2
Qq

Multiple Choice Questions


2. (B) See Fig.1.178
80 CHAPTER 1. ELECTRIC CHARGE AND FIELD

here, gef f is the effective gravitational acceleration. Now,


find the value of gef f in given situations and by using Eq.
1.110, calculate the value of time period T .

(a) In this case, from FBD [Fig.1.179a], the effective value


of g can be given by

(A) (B)

(C) (D)

Figure 1.177
(a) (b)

(c) (d)
Figure 1.178
Figure 1.179: Different situations of pendulum

6−0
Acceleration, a = = 6 m s−2
1
1
 
mg
For t = 0 to t = 1 s, s1 = × 6(1)2 = 3 m mgef f = mg + qE = mg + mg ∵ E=
2 q
1
For t = 1 s to t = 2 s, s2 = 6 × 1 − × 6(1)2 = 3 m ⇒ gef f = 2g
2
1 On substituting this value of gef f in Eq.1.110, we get
For t = 2 s to t = 3 s, s3 = 0 − × 6(1)2 = −3 m
2 s
Total displacement s = s1 + s2 + s3 = 3 m L
T = 2π
2g
3
Average velocity = = 1 m s−1
3 (b) In this case, from FBD (1.179b), the effective acceler-
Total distance travelled = 9 m ation is given by-
9 
2mg

Average speed = = 3 m s−1 mgef f = mg − qE = mg − 2mg ∵ E=
3 q
⇒ gef f = −g
 
3. (A) Since, v 2 = 02 + 2ay = 2(F/m)y = 2 qE m y
Here, −ve sign just says that the effective gravitational
1 acceleration is directed in upward direction. So, the
K.E. = mv 2
2   pendulum get inverted and oscillates about the fixed
1 (qE) point as shown in adjoining figure. On substituting
∴ K.E. = m 2 y ⇒ K.E. = qEy
2 m this value of gef f in Eq.1.110, we get

4. APPROACH Time period of a simple pendulum is given


s
L
by- T = 2π
g
s
T = 2π
L
(1.110) (c) Equilibrium position is shown in figure 1.179c
|gef f | In this case, the effective gravitational acceleration is
1.20. CONDUCTORS IN ELECTROSTATIC EQUILIBRIUM 81

given by-

 
mg
q
mgef f = (mg)2 + qE 2 = mg 2 ∵ E=
q

⇒ gef f =g 2
On substituting this value of gef f in Eq.1.110, we get
Figure 1.180
s
L
T = 2π √
g 2 This acceleration arises due to constraining forces.
Tangential acceleration, at = Fmt = qE
m
(d) In this case, the equilibrium is shown in figure1.179d ∴ Resultant acceleration,
(thread is horizontal). From, figure, we have-
qE p
q
a = a2r + a2t = 1 + π2
m
q
2 2
mgef f = (mg − qE sin 45◦ ) + (qE cos 45◦ )
6. Electrostatic force,
s 2  2
qE qE
= mg − √ + √
2 2 F = qE
ma0 = q(−bx + a) [a0 = accleration ]
s 2  2
qE qE
= mg − √ + √ = mg dv bq aq
2 2 V =− x+
 √  dx m m
E = 2mg
Z V
∵ bq x aq x
Z Z
q
V dv = − xdx + dx
0 m 0 m 0
⇒ gef f = g
V2 bqx2 aqx
On substituting this value of gef f in Eq.1.110, we get =− +
2 2m m
2
Now V = 0 when bqx = aqx ⇒ x = 2a at X = 2a
s
L b
T = 2π 2m m b
g q
a0 = (−bx + a)
m q
5. (a) At every point on the path, the electric force on the acceleration = m (−2a + a) = − qa m
particle and it’s displacement always remains tangential to
7. Charge Q resides on each of the blocks, which repel as
the path, i.e., both remains in the same direction. So, work
point charges. In equilibrium, we have-
done by the electric force on the particle-
B Q2
−→ − → F = ke = k (L − Li )
Z
W = q E · dl = qE · πR L2
A
here, ke = 4πε
1
0
= 9.0 × 109 N.m2 /C2
Now, by Work-Energy theorem, we have- Solving for Q, we find
s
W = KB − KA k (L − Li )
Q=L
Therefore, ke
s
KB − KA = qEπR (100 N/m)(0.500 m − 0.400 m)
= (0.500 m)
⇒ KB = πqER (∵ KA = 0) 9.0 × 109 N · m2 /C2
= 1.67 × 10−5 C
(b) Suppose, at mid point the speed of the particle is v
[Fig.1.180], then by work-energy theorem, we have 8. (C) Given: Mass of a particle is m, charge on particle is
q, initial velocity of particle before entering the region of
1 πR −

mv 2 − 0 = qE electric field is v0 î, electric field E = −Eĵ exists in the
2 2 region x = 0 to x = d. To find : The equation of the
v2 πqE particle for the region x > d
⇒ =
R m After travelling through the electric field the components
Therefore, radial acceleration of the particle, of velocity of the particle will change as [Fig.1.181]:

v2 πqE qE
ar = = vx = vo , vy = at, a = − .
R m m
82 CHAPTER 1. ELECTRIC CHARGE AND FIELD

10. Consider an elementary segment AB, subtending a very


small angle 2θ at the center of the given ring of radius R
[Fig.1.183].
From Fig.1.183, we have AB = R (2θ) Charge on AB is

Figure 1.181

In the region x > d, the equation of the particle will be a


straight line.
y − 12 at2 at
=
x−d vo Figure 1.183

y − 12 at2 at
= Q Qθ
x−d vo dQ = (2Rθ) =
y − 12 at2 x−d 2πR π
=
at vo dQ · q Qqθ
y 1 d x d 2T sinθ = =
− = − 4π∈0 R2 4π 2 ∈0 R2
at 2 vo vo vo
Qqθ Qq
y x 1 d 2T θ = or T= 2
= − 4π 2 ∈0 R2 8π ∈0 R2
at vo 2 vo
.
−ym vo x 1 d
= −
qEd vo 2 v 11. (C) Because electric field inside the conductor is zero and
 o 
qEd d electric field lines are perpendicular to Gaussian surface.
y= −x
m v2o 2

9. (C) is correct. Given : Charge on a point mass released 1.21 Solid Angle
from the edge of a table is +q, initial velocity of the point
mass is u = 0. To find : The trajectory of the point mass The solid angle is the extension of the concept of angle from
in presence of horizontal electrical and vertical gravitational two to three dimension. So let’s start from 2D: consider a
field [Fig.1.182]. circle(Fig.1.184) and pick two rays OA and OC starting from
Components of acceleration of the point mass, m : the center O. They will divide the circumference in two parts
ABC and ADC, called arcs. The length of each arc divided
by the length of the radius will be the measure of the angle
subtended by the arc itself, i.e.,

Figure 1.182

qE
ax = , ay = g
m Figure 1.184: Plane angle
Resultant acceleration of the point mass:
arc
s  angle =
qE
2 radius
a= + g 2 = constant From Fig.1.184, we have
m

As initial velocity of the point mass was u = 0 and the arcABC ∆s


or ∆θ = = (1.111)
acceleration is constant its trajectory will be straight line. AC r
1.21. SOLID ANGLE 83

Here, ∆s is the arc length and r is the radius of the circle. The
SI unit of plane angle is “radian (rad)”.
Now, extend this idea to three dimensions. Instead of a circle
take a sphere, and instead of picking two rays pick a cone
centered in the center of the sphere [Fig.1.185]. The cone will
cross the surface of the sphere: and now to define the solid
angle measure the area (∆A) of the surface bounded by the
cone, divided by the square of the length of the radius (R2 )
(so that we have an area divided by an area).

Figure 1.187

tity ∆A cos θ/R2 is equal to the solid angle ∆Ω that the surface
Figure 1.185: A solid angle measured on a sphere. element ∆A subtends at the origin O. We also see that ∆Ω
is equal to the solid angle subtended by the area element of a
spherical surface of radius R.
∆A Note: If the line joining O to ∆A makes an angle θ with
i.e., solid angle, ∆Ω =
R2 the normal to ∆A (Fig.1.187), we should write
The unit of solid angle is steradian (sr).
∆A cos θ
Here, it is important to note that the line joining center of ∆Ω =
sphere O to ∆A is normal to ∆A. R2
Since, plane angles and the solid angles are the ratios of same A complet circle subtends an angle -
physical quantities, therefore they are dimensionless quantities.
Note that a small surface area as seen from a short distance ∆s 2πr
θ= = = 2π rad
can cover the same solid angle as a large area as seen from a r r
long distance. For example, in Fig.1.186 different areas A and
A0 (A0 > A), at positions R, R0 covers same solid angle at O. at the centre. In fact, any closed curve subtends an angle 2π
at any of the internal points. Similarly, a complete sphere
subtends a solid angle-

A 4πR2
Ω= = = 4π sr
R2 R2
at the centre. Also, any closed surface subtends a solid angle
4π sr at any internal point. So for, we have defined the solid
angles at an internal point.
Now, we find the plane and solid angles at an external point.
From Fig.1.188, it is clear that on gradually closing the curve,

Figure 1.186

Definition: The solid angle is a three dimensional angle


subtended by an object (two dimensional or three dimensional)
at a certain point in the space. It merely depends on the relative
distance of the object and its configuration with respect to the Figure 1.188
given point in the space. Solid angle subtended by a straight
line or a point is always zero. the angle subtended by AB at an external point O, finally
diminishes to zero. So, we can say that a closed curve subtends
In Fig.1.187, we see that the projection of the area element zero angle at an external point. Similarly, a closed surface also
perpendicular to the radius vector is ∆A cos θ. Thus, the quan- subtends zero solid angle at an external point.
84 CHAPTER 1. ELECTRIC CHARGE AND FIELD

Calculation of Solid Angle at the Centre of a Sphere: For a complete spherical surface, at it’s center, we have-
Mathematically, the solid angle can be defined as-
θ = 180◦ , Ω = 2π (1 − cos 180◦ ) = 4π sr
A
Ω= 2 Not only a spherical surface, all closed surfaces subtend a solid
R
The definition of a solid angle Ω. is analogous to the defini- angle of = 4π sr at their centers.
tion of a plane angle. Just as the arc length s is everywhere
perpendicular to the radius r, the area A must be everywhere 1.22 Electric Flux
perpendicular to the radius. The SI unit of solid plane angle is
"steradian (sr)". Fig.1.189 shows a sphere of radius R. Let us Latin: flux = “to flow”
Analogy With Flow of Water and Concept of Flux:
Imagine holding a ring with inside area A in a stream of
water flowing with velocity ~v , as shown in Fig.1.190. The area
~ of the ring is defined as a vector with magnitude A
vector, A,
pointing in a direction perpendicular to the plane of the ring.
Of course, for a ring, disc or other "open" surface that is not
part of a three-dimensional volume, there are two possible
directions perpendicular to the surface. Choose the direction
~ so that it makes the smallest possible angle with respect
of A
to the velocity vector. In Fig.1.190a, the area vector of the
ring is parallel to the flow velocity, and the flow velocity is
perpendicular to the plane of the ring. The product Av gives
the amount of water passing through the ring per unit time,
where v is the magnitude of the flow velocity. If the plane of
the ring is tilted with respect to the direction of the flowing
water (Fig.1.190b), the amount of water flowing through the
ring per second, is given by Av cos θ, where θ is the angle
Figure 1.189
between the area vector of the ring and the direction of the
velocity of the flowing water. The amount of water flowing
consider a a conical section AOF having vertex at O. Suppose through the ring is called the flux, Φ = Av cos θ = A ~ · ~v .
semi vertex angle of this cone is 6 AOC = θ and radius of its Since flux is a measure of volume per unit time, its units are
base is r. Now, consider a strip ABEF of radius r on this cubic meters per second m3 /s. For the case of the electric
sphere. If 6 AOB = dθ, then thickness of the strip AB = Rdθ
and radius of strip, r = R sin θ.
Curved area of this strip is given by-

dA = 2πr × (thickness AB)


∵ r = R sin θ, therefore-

dA = 2πR sin θ R dθ
θ
⇒ A = 2πR2 ∫ sin θdθ (a)
0

⇒ A = 2πR2 (1 − cos θ)
A 2πR2 (1 − cos θ)
∴ Ω= = = 2π (1 − cos θ)
R2 R2
θ
⇒ Ω = 2π (1 − cos θ) = 4π sin2 (1.112)
2
Eq.(1.112) gives the relation between semi vertex plane angle (b)
and the solid angle formed at the vertex of cone having its
vertex at the center of the sphere of radius R. Figure 1.190: Water flowing with velocity of magnitude v through
a ring of area A. (a) The area vector is parallel to the flow velocity.
At θ = 0◦ , Ω = 2π (1 − cos 0◦ ) = 0 sr (b) The area vector is at an angle θ to the flow velocity.

For a hemispherical surface, at it’s center we have- field, we define an analogous quantity and call it electric flux.
We should however note that there is no flow of a physically
θ = 90◦ , Ω = 2π (1 − cos 90◦ ) = 2π sr observable quantity unlike the case of water flow.
1.22. ELECTRIC FLUX 85

Consider a uniform electric field of magnitude E passing fluxes through each small pieces, i.e.,
through a given area A (Fig.1.191). Again, the area vector

→ − →
Z
~ with a direction normal to the surface of the area and
is A, ΦE = E · dA (1.115)
a magnitude A. The angle θ is the angle between the vector surface
electric field and the area vector, as shown in Fig.1.191. The
electric field passing through a given areau A isucalled the
electric flux and is given by- E

dA
~ ·A
ΦE = E ~ = EA cos θ (1.113)
θ
In simple terms, the electric flux is proportional to the number
of electric field lines passing through the area.
The flux through a surface of area A has a maximum value
u u
EA when the surface is perpendicular to the field (i.e. when
θ = 0◦ ), and is zero when the surfaceuis parallel to the field
(i.e. when θ = 90◦ ).
When we apply Eq.(1.113), it is often best to sketch E ~ and A
~
with their tails touching and θ between the two vectors From
θ
dA

3
Figure 1.192

Equation (1.192) is a surface integral, which means it must


be evaluated over the surface in question. Generally, the value
of ΦE depends both on the field pattern and on the surface.
u u

The Electric Flux Through a Closed Surface


Our final step, to calculate the electric flux through a closed
surface such as a box, a cylinder, or a sphere, requires noth-
ing new. We’ve already learned how to calculate the electric
flux through flat and curved surfaces, and a closed surface is
nothing more than a surface that happens to be closed.
However, the mathematical notation for the surface integral
Figure 1.191: Field lines for a uniform electric field through an over a closed surface differs slightly from what we’ve been us-
area A that is at an angle of (90◦ − θ) to the field. ing. It is customary to use a little circle on the integral sign
to indicate that the surface integral is to be performed over a
Eq.(1.113), we can also say that the electric flux ΦE is the closed surface. With this notation, the electric flux through a
magnitude of the electric field |E|~ times the magnitude of the closed surface is
component of the area vector A ~ 0 that is parallel to the electric −
→ − →
I I
ΦE = E · d A = En dA (1.116)
field (Fig.1.191):
ΦE = EA0 (1.114) where E represents the component of the electric field normal
n
0 d e
where A = A cos θ. The unit of electric flux is NC m .−1 2 to the surface.
d parallel eto2 the electric field, the Note
If you were to shine a light
still
that only the notation has changed. The electric flux is
the summation of the fluxes through a vast number of tiny
shadow cast by a surface of area A would have an area equal

→0 pieces, pieces that now cover a closed surface.
to the magnitude of A (Fig.1.191).
Direction of Area Vector: A closed surface has a distinct
A More General Expression for Electric Flux ~
In Eq.1.113 we have assumed that the electric field E ~ was con- inside and outside. The direction of the area vector dA is
chosen so that the vector points outward from the surface. If
00-203
stant over the surface. What if a surface is curved and/or
the area element is not part of a closed surface, the direction
the field varies with position? In such a case, we divide the
of the area vector is chosen so that the angle between the area
surface into many infinitesimally small pieces, each piece is so
vector and the electric field vector is less than or equal to 90°.
small that we can consider it as essentially flat and the field
is uniform over it (Fig.1.192). If dA~ is the area vector of one In case of a closed surface, the sign of the flux depends on the
angle between E ~ and dA~ as follows:
~ then
such a small piece and the electric field at this piece is E,
the flux through one such small piece is dΦE = E ~ · dA,
~ and 1. If θ < 90◦ , then E~ crosses the surface from the inside to
the flux through the entire surface comes from integrating the the outside and hence dΦE = E ~ · dA~ is positive.
86 CHAPTER 1. ELECTRIC CHARGE AND FIELD

more lines are entering than leaving, the net flux is negative.
Now we’re ready to calculate the flux through a closed surface.

EXAMPLE 61. Fig.1.194 shows a cube that has edge length l


~ that is directed along the positive
in a uniform electric field, E,
x-axis and perpendicular to the plane of one face of the cube.
What is the net electric flux passing though the cube?

S
d A3 c
l
S
S E
d A1

S
l d A2

y
x
z l x
v S
d A4

Figure 1.194
S
APPROACH To find the net flux through the cube, deter-
mine the flux passing through each surface and add them al-
gebraically.
SOLUTION 3 In Fig.1.194, the electric field lines pass through
two faces perpendicularly and are parallel to four other faces
of the cube. So, the flux through four of the faces (○, 3 ○,
4 and
S −

the unnumbered faces) is zero because E is parallel to the four


Figure 1.193: A closed surface in an electric field. The area vectors faces and therefore perpendicular to d A on these faces.
are, by convention, pointed normally outward to the surface. The net flux through faces ○ 1 and ○: 2


→ − → −
→ − →
Z Z
S
2. If θ = 90 , then E
◦ ~ grazes the surface and hence dΦE = Φ 12 = Φ1 + Φ 2 = E · d A + E · dA
1 2
E~ · dA~ is zero.
−→ −

For face (1), E is constant and directed inward but d A 1 is
◦ ◦ ~
3. If 90 < θ < 180 , then E crosses the surface from the directed outward (θ = 180◦ ). So, the flux through surface ○, 1
outside to the inside and hence dΦE = E ~ · dA
~ is negative.

→ − →
Z Z
−→ Φ1 = E · d A = E (cos 180◦ ) dA
Consider the closed surface in Fig.1.193. The vectors d A i
1 1
point in different directions for the various surface elements, Z
but for each element they are normal to the surface and point = −E dA = −EA = −El2
outward. At the element labeled ○, 1 the field lines are crossing 1

the surface from the inside to the outside and θ < 90 ; hence,
◦ −


→ − → 2 E is constant and outward and in the same di-
For face ○,
the flux dΦ1 = E · d A 1 through this element is positive. For −

rection as d A 2 (θ = 0◦ ). Therefore, flux through this face:
element ○, 2 the field lines graze the surface (perpendicular to


d A 2 ); therefore, θ = 90◦ and the flux is zero. For elements −
→ − →
Z Z
such as ○, 3 where the field lines are crossing the surface from Φ 2 = E · d A = E (cos 0◦ ) dA
2 2
outside to inside, 180◦ > θ > 90◦ and the flux is negative Z
because cos θ is negative. The net flux through the surface is = E dA = +EA = El2
2
proportional to the net number of lines leaving the surface,
where the net number means the number of lines leaving the Therefore, net flux through the cube
surface minus the number of lines entering the surface. If
more lines are leaving than entering, the net flux is positive. If ΦE = Φ12 + Φ3456 = −El2 + El2 + 0 + 0 + 0 + 0 = 0
1.22. ELECTRIC FLUX 87

1.22.1 Check Point 8


1. • A person is placed in a large, hollow, metallic sphere
that is insulated from ground. If a large charge is placed
on the sphere, will the person be harmed upon touching
the inside of the sphere?

2. • Why must hospital personnel wear special conducting


shoes while working around oxygen in an operating room?
What might happen if the personnel wore shoes with rub-
ber soles?

3. •• Consider a closed triangular box resting within a hori-


zontal electric field of magnitude E = 7.80 × 104 N/C as Figure 1.197
shown in Fig.1.195. Calculate the electric flux through
(a) the vertical rectangular surface, (b) the slanted sur-
face, and (c) the entire surface of the box.

Figure 1.198

Figure 1.195 the cone, as shown in Figure 1.198. Determine the electric
flux that enters the left-hand side of the cone.
4. •• A uniform electric field −

a î + bĵ intersects a surface of 8. •• A hemispherical surface of radius R is kept in a uni-
area A. What is the flux through this area if the surface −
→ −

form electric field E such that E is parallel to the axis
lies (a) in the yz plane? (b) in the xz plane? (c) in the of hemi-sphere [Fig.1.199]. Find the net flux linked with
xy plane? hemispherical surface-
5. •• A point charge q is located at the center of a uniform
ring having linear charge density λ and radius a, as shown
in Fig.1.196. Determine the total electric flux through a
sphere centered at the point charge and having radius R
where R < a.

Figure 1.199



9. •• An electric field is described by E = (15ı̂ + 25̂)N/C.
Find the electric flux through a surface whose area vector


is A = (0.65ı̂ + 0.35̂)m2
10. •• Calculate the electric flux through the surfaces shown
Figure 1.196 in Fig.1.200
11. ••The electric flux through the surface shown in Fig.1.201
is 25 N m2 /C. What is the electric field strength?
6. •• A pyramid with horizontal square base, 6.00 m on each
side, and a height of 4.00 m is placed in a vertical electric 12. •• A 2.0 cm×3.0 cm rectangle lies in the xy -plane. What
field of 52.0 N/C [Fig.1.197]. Calculate the total electric ~ = (100ı̂ +
is the electric flux through the rectangle if a. E
flux through the pyramid’s four slanted surfaces. 50k̂)N/C? b. E ~ = (100ı̂ + 50̂)N/C?

13. •• A 2.0 cm×3.0 cm rectangle lies in the xz -plane. What


7. ••A cone with base radius R and height h is located on a ~ = (100ı̂ +
is the electric flux through the rectangle if a. E
horizontal table. A horizontal uniform field E penetrates ~
50k̂)N/C? b. E = (100ı̂ + 50̂)N/C?
88 CHAPTER 1. ELECTRIC CHARGE AND FIELD

(a)

(a).

(b)

Figure 1.202

(b)

Figure 1.200

Figure 1.203

Figure 1.201
square surface (Figure1.204). The electric flux in SI units
associated with the surface is
14. ••A 3.0-cm-diameter circle lies in the xz -plane in a region (A) EL2 (B) EL2 /2ε0
~ = (1500ı̂+1500̂−1500k̂)N/C.
where the electric field is E (C) EL /2
2
(D) zero
What is the electric flux through the circle?

15. ••A 1.0 cm × 1.0 cm × 1.0 cm box with its edges aligned
~ = (350x +
with the xyz -axes is in the electric field E
150)ı̂N/C, where x is in meters. What is the net electric
flux through the box?

16. •• What is the net electric flux through the two cylinders
shown in Fig.1.202? Give your answer in terms of R and
E.

Multiple Choice Questions

17. •• A square surface of side L meter in the plane of Figure 1.204


the paper is placed in a uniform electric field E( volt
/m) acting along the same plane at an angle θ with the
horizontal side of the square as shown in figure1.203.
The electric flux linked to the surface, in units of volt m is- Answer Keys and Solutions
(A) EL2 (B) EL2 cos θ 1. The answer depends on whether the person is initially (a)
(C) EL sin θ
2
(D) zero uncharged or (b) charged.
18. •• A square surface of side L metres is in the plane of the (a) No. If the person is uncharged, the electric field inside


paper. A uniform electric field E ( volt /m), also in the the sphere is zero. The interior wall of the shell carries no
plane of the paper is limited only to the lower half of the charge. The person is not harmed by touching this wall.
1.22. ELECTRIC FLUX 89

(b) If the person carries a (small) charge q, the electric 6. Electric flux passing through the base is given by
field inside the sphere is no longer zero. Charge –q is [Fig1.206]-
induced on the inner wall of the sphere. The person will
get a (small) shock when touching the sphere, as all the
charge on his body jumps to the metal.
2. Conducting shoes are worn to avoid the build up of a static
charge on them as the wearer walks. Rubber-soled shoes
acquire a charge by friction with the floor and could dis-
charge with a spark, possibly causing an explosive burning
situation, where the burning is enhanced by the oxygen.
Q 1 Q
3.Ey = ke 1/2
, Ey = k 2
d (d2 + L2 ) d

which is the field of a point charge Q at a distance d along


the y axis above the charge.
Figure 1.206
3. (a) From Fig.1.205, we have
ΦE = EA cos θ
A0 = (10.0 cm)(30.0 cm)
= (52.0)(36.0) cos 180◦ = −1.87kN · m2 /C
A0 = 300 cm2 = 0.0300 m2
Note the same number of electric field lines go through the
ΦE,A0 = EA0 cos θ
base as go through the pyramid’s surface (not counting the
4
(0.0300) cos 180◦

ΦE,A0 = 7.80 × 10 base). For the slanting surfaces,
ΦE,A0 = −2.34 kN · m2 /C
ΦE = +1.87kN · m2 /C
(b) The electric flux through the inclined plane is given
.
7. APPROACH The flux entering the closed surface equals
the flux exiting the surface. The flux entering the left side
R− → − →
of the cone is ΦE = E · d A = ERh. This is the same
as the flux that exits the right side of the cone. Note that
for a uniform field only the cross sectional area matters,
not shape.
8. The hemisphere seems like a much more complicated
calcuIation; you may be tempted to break up the surface
Figure 1.205
into many small pieces and integrate using Equation

→ − →
I
by- ΦE = E .d A . This would work, but there is a much

ΦE,A = EA cos θ = 7.80 × 104 (A) cos 60.0◦


 simpler way. Imagine shining a light parallel to the
electric field so that the shadow cast by the hemisphere is

→−→
 
10.0 cm
A = (30.0 cm)(w) = (30.0 cm) a disk of radius R. now by applying equation, ΦE = E . A ,
cos 60.0◦ we can find the electric flux from the area of the shadow,
= 600 cm2 = 0.0600 m2 A.
⇒ ΦE,A = 7.80 × 104 (0.0600) cos 60.0◦
 SOLUTION Net electric flux through the hemispherical
surface-
= +2.34 kN · m2 /C

→ − →
I
(c) The bottom and the two triangular sides all lie parallel ΦE = E · dA


to E , so ΦE = 0 for each of these. Thus, −

ΦE, total = −2.34 kN·m2 /C+2.34 kN·m2 /C+0+0+0 = 0 = E(Area of surface perpendicular to E )

→ − → = E · πR2
4. (a) ΦE = E · A = (aî + bĵ) · Aî = aA
(b) ΦE = (aî + bĵ) · Aĵ = bA −
→− →
9. Electric flux, ΦE = E . A = (15î + 25ĵ).(0.65î + 0.35ĵ)
ˆ · Ak̂ = 0
(c) ΦE = (aî + bj) = (9.75 + 8.75)m2 /C = 18.5m2 /C
5. Only the charge inside radius R contributes to the total 10. In Fig.1.200(a), the electric field is uniform over the entire
flux. surface. The electric field vectors make an angle of 30◦
q
ΦE = with the planar surface. Because the normal n̂ to the
0
90 CHAPTER 1. ELECTRIC CHARGE AND FIELD

planar surface is at an angle of 90◦ with the surface, the The electric flux through the rectangle is
angle between n̂ and E~ is θ = 60◦ . −→ − →
ΦE = E · A = (100î + 50k̂)N/C · 6.0 × 10−4 m2 ĵ

The electric flux is
= 600 × 10−4 N m2 /C (î · ĵ)


→ − →
ΦE = E · A = EA cos θ
+ 300 × 10−4 N m2 /C (k̂ · ĵ) = 0.0 N m2 /C

= (200 N/C) 1.0 × 10−2 m2 cos 60◦

(b) The flux is
= 1.0 N m2 /C
~ ·A
~ = (100î + 50ĵ)N/C · 6.0 × 10−4 m2 ĵ

Φe = E
In Fig.1.200(b) the electric field vectors make an angle = 600 × 10−4 N m2 /C (î · ĵ)

of 30◦ below the surface. Because the normal n̂ to the
+ 300 × 10−4 N m2 /C ĵ · ĵ

planar surface is at an angle of 90◦ relative to the surface,
the angle between n̂ and E ~ is θ = 120◦ . = 0 N m2 /C + 3.0 × 10−2 N m2 /C


The electric flux is = 3.0 × 10−2 N m2 /C



→ − →
ΦE = E · A = EA cos θ 14. The area vector of the circle is
−2
2 ◦
= (180 N/C) 15 × 10 m cos 120 ~ = πr2 k̂ = π(0.015 m)2 ĵ = 7.07 × 10−4 m2 ĵ

A
= −2.3 N m2 /C Thus, the flux through the area of the circle is
~ ·A
ΦE = E ~
11. The electric field is uniform over the entire surface.
Visualize: Please refer to Fig.1.207. The electric field = (1500î + 1500ĵ − 1500k̂)N/C
vectors make an angle of 60◦ above the surface. Because
· 7.07 × 10−4 m2 ĵ

the normal n̂ to the planar surface is at an angle of 90◦
relative to the surface, the angle between n̂ and E ~ is
Using î · ĵ = ĵ · k̂ = 0 and ĵ · ĵ = 1, we find
θ = 30◦ . Solve: The electric flux is
ΦE = (1500 N/C) 7.07 × 10−4 m2 = 1.1 N m2 /C
~ ·A
ΦE = E ~ = EA cos θ 15. The area vector of the circle is
2
ΦE 25 N m /C ~ = πr2 k̂ = π(0.015 m)2 ĵ = 7.07 × 10−4 m2 ĵ
A

E= =
A cos θ (10 × 10−2 m) (20 × 10−2 m) cos (30◦ )
Thus, the flux through the area of the circle is
= 1.4 × 103 N/C
~ ·A
ΦE = E ~
12. The electric field is uniform over the rectangle in the xy
= (1500î + 1500ĵ − 1500k̂)N/C
plane. Solve: (a) The area vector is perpendicular to the
· 7.07 × 10−4 m2 ĵ

xy plane and points in the k̂ direction. Thus

A~ = (2.0 cm × 3.0 cm)k̂ = 6.0 × 10−4 m2 k̂


 Using î · ĵ = ĵ · k̂ = 0 and ĵ · ĵ = 1, we find
ΦE = (1500 N/C) 7.07 × 10−4 m2

The electric flux through the rectangle is
= 1.1 N m2 /C
~ ·A
ΦE = E ~
16. The electric field is uniform, and we take the area vectors
to point outward from the box. In the figure1.207, the
 
−4 2
= (100î + 50k̂) · 6.0 × 10 k̂ Nm /C
box is positioned with its edges aligned with the xyz axes,
= 3.0 × 10−2 N m2 /C and the electric field is evaluated at the input face and the
exit face. The area vectors of the six box faces are-
(b) The electric flux is ~ 1 = 1.0 × 10−2 m 2 î = 1.0 × 10−4 m2 î,
A

~ 2 = − 1.0 × 10−4 m2 î, A ~ 3 = 1.0 × 10−4 m2 ĵ,
 
  A
~ ·A
ΦE = E ~ = (100î + 50ĵ) · 6.0 × 10−4 k̂ Nm2 /C
~ 4 = − 1.0 × 10−4 m2 ĵ, A ~ 5 = 1.0 × 10−4 m2 k̂,
 
A
= 0.0 N m2 /C and A ~ 6 = − 1.0 × 10−4 m2 k̂.


6
~ is in the plane of the rectangle, which is
Assess: In (b), E X −
→ − →
ΦE = E · Ai
why the flux is zero. i=1
~ (x=0.01 −
→ −
→ −

13. The electric field over the rectangle in the xz plane is =E m) · A 1 + E (x=0.0 m) · A2
uniform. Solve: (a) The area vector is perpendicular to = (153.5 N/C) 1.0 × 10−4 m2

the xz plane and points in the ĵ direction. Thus
− (150 N/C) 1.0 × 10−4 m2


~ = (2.0 cm × 3.0 cm)ĵ = 6.0 × 10−4 m2 ĵ



A = 3.5 × 10−4 N m2 /C
1.23. GAUSS’S LAW 91

Statement of Gauss’s Law: Gauss’s Law states that the


net flux ΦE of electric field passing through any closed surface
is proportional to the net charge Qencl that is enclosed by the
same surface. The proportionally constant is 1/εo , i.e.

→ − → Qencl
I
ΦE = E .d A =
0
Proof: Let us consider a positive point charge q located at
the centre of a sphere of radius r, as shown in Fig.1.208. The
electric field created by this charge at any point the surface
of the Gaussian sphere is directed outwards and be normal to
the surface. It’s magnitude is also constant and given by-
Figure 1.207
q 1 q
E=k = (1.117)
r2 4πε0 r2
17. The electric field through the two cylinders is uniform. Therefore:
Let A = πR2 be the area of the end of the cylinder and
let the area vector point outward from the cylinder ends. E ~ = EdA cos 0◦ = EdA = 1 q dA
~ · dA
E is the electric field strength. Solve: (a) There’s no 4πε0 r2
flux through the side walls of the cylinder because E~ is
Hence, the total flux ΦE through the entire Gaussian surface is
parallel to the wall. On the right end, where E ~ points
outward, Φright = EA cos (0◦ ) = πR2 E. The field points
inward on the left, so Φleft = EA cos (180◦ ) = −πR2 E.
Altogether, the net flux is Φe = 0 N m2 /C2
(b) The only difference from part (a) is that E ~ points
outward on the left end, making Φleft = EA cos (0◦ ) =
πR2 E. Thus the net flux through the cylinder is
Φe = 2πR2 E N m2 /C2 .

Multiple Choice Questions

18. (D): Electric flux, ΦE = ~ · dA~ =


R R
E EdA cos θ =
EdA cos 90◦ = 0. (because field lines are parallel to the
R

surface.)

19. (D)
Figure 1.208

1.23 Gauss’s Law


q
I I
Gauss’s law is equivalent to Coulomb’s law for static charges, ΦE = ~ · dA
E ~= dA
4π◦ r2
although Gauss’s law will look very different. It provides q q
a different way to express the relationship between electric = 4πr2 =
4π◦ r2 ◦
charge and electric field.
The purpose of learning Gauss’s law is twofold: −
→ ~ q
I
or ΦE = E · dA = (1.118)
◦

• Gauss’s law allows the electric fields of some continuous From Eq.(1.118), it is clear that the flux through a spherical
distributions of charge to be found much more easily than surface of radius r is equal to the charge q inside the sphere
does Coulomb’s law. divided by the permittivity of free space ε0 .
Since, entire Gaussian surface of any shape forms a solid angle
• Gauss’ law is more general in that it also covers the case of magnitude of 4π sr, therefore, this flux is linked with solid
of a rapidly moving charge. For such charges the electric angle 4π sr. Now consider several closed Gaussian surfaces
lines of force become compressed in a plane at right an- surrounding the charge as shown in Fig.1.209(a). The number
gles to the direction of motion, thus losing their spherical of electric field lines passing through the spherical surface S1
symmetry. Coulomb’s law is not valid for moving charges. is the same as the number of lines passing through the non-
Thus Gauss’s law is ultimately a more fundamental state- spherical surfaces S2 and S3 . Therefore, we conclude that the
ment about electric fields than is Coulomb’s law. electric flux through any closed surface is independent of the
92 CHAPTER 1. ELECTRIC CHARGE AND FIELD

shape of the surface that encloses the charge q, and it also does we can write an equation like Eq.(1.118) for each charge and
not depend upon the particular location of q inside the surface. its corresponding field and add the results. When we do, we
The magnitude of the flux through any closed Gaussian surface obtain the general statement of Gauss’s law:
surrounding the point charge q is q/ε◦ .

→ − → Qencl
I
ΦE = E · dA = (Gauss’s law) (1.119)
The net electric flux is the 0
same through all surfaces.
The net electric flux through any closed surface is equal to the
S3 net charge inside the surface divided by the permittivity of free
S2 space ε0 ..
S1 In terms of solid angle, we can say that the complete flux linked
Q
with solid angle 4π sr is encl
ε0
1 Note: If we have a closed surface of irregular geometry, then
during the integration the value of E may be different at var-
ious locations on the surface, and the angle between E and
dA may also vary as we sum the various contributions over
the surface. But, interestingly, regardless of the shape of the
Q
surface the net flux through the closed surface is always encl ε0
(a) Key Points
The number of field lines
1. Gauss’s law is true for any closed surface, no matter what
entering the surface equals
the number leaving the its shape or size.
surface. 2. The term Qencl on the right side of Gauss’s law includes
the sum of all charges enclosed by the surface. The charges
may be located anywhere inside the surface.
3. In the situation when the surface is so chosen that there
are some charges inside and some outside the Gaussian
surface, the electric field [whose flux appears on the left
1 side of Gauss’s law] is caused partly by charges inside the
q surface and partly by charges outside. But as Fig.1.209(b)
shows, the outside charges do not contribute to the total
(net) flux through the surface.
4. The closed surface that we choose for the application of
Gauss’s law is called the Gaussian surface. It is an imag-
inary surface. There need not be any material object at
the position of the surface. You may choose any Gaus-
(b) sian surface and apply Gauss’s law. However, take care
not to let the Gaussian surface pass through any discrete
Figure 1.209: (a) Closed surfaces of various shapes surrounding a charge. This is because electric field due to a system of
positive charge. (b) A point charge located outside a closed surface. discrete charges is not well defined at the location of any
charge. (As you go close to the charge, the electric field
Now consider a point charge located outside a closed surface increases to infinity.) However, the Gaussian surface can
of arbitrary shape as shown in Fig.1.209(b). In such a case, pass through a continuous charge distribution.
any electric field line entering the surface leaves the surface at
another point. The number of electric field lines entering the 5. Gauss’s law is often useful towards a much easier calcu-
surface equals the number leaving the surface. Therefore, the lation of the electrostatic field when the system has some
net electric flux through a closed surface that surrounds no symmetry. This is facilitated by the choice of a suitable
charge is zero. Gaussian surface.
6. Finally, Gauss’s law is based on the inverse square depen-
General Integral Form of Gauss’s Law dence on distance contained in the Coulomb’s law. Any
Now, suppose the surface encloses not just one point charge q violation of Gauss’s law will indicate departure from the
but several charges q1 , q2 , q3 , . . . The total (resultant) electric inverse square law.

→ −

field E at any point is the vector sum of the E fields of the
Differential Form of Gauss’s Law
individual charges. Let Qencl be the total charge enclosed by
the surface: Qencl = q1 + q2 + q3 + · · ·. Also let E ~ be the A remarkable property of electric field expressed by the


total field at the position of the surface area element d A , then Gauss theorem suggests that this theorem be represented in
1.23. GAUSS’S LAW 93

a different form which would broaden its possibilities as an where î, ĵ, and k̂ are the unit vectors along the X-, Y -, and


instrument for analysis and calculation. Z-axes respectively. The operator ∇ itself does not have
any meaning. It becomes meaningful only in combination
In contrast to (1.119) which is called the integral form we with a scalar or vector function by which it is symbolically
shall seek the differential form of the Gauss theorem, which multiplied. For example, if we form the scalar product of
establishes the relation between the volume charge density ρ vector − →
∇ and vector E ~ = Ex î + Ey ĵ + Ez k̂, we obtain-
and the changes in the field intensity E in the vicinity of a −→ ~ ∂ ∂ ∂
given point in space. ∇ ·E = Ex + Ey + Ez
∂x ∂y ∂z
For this purpose, we first represent the charge q in the volume
It follows from (1.122) that this is just the divergence of E. ~
V enveloped by a closed surface S in the form Qencl = hρiV ,
where hρi is the volume charge density, averaged over the Thus, ~
the divergence of the field E can be written as div E ~

→ − → ~
volume V . Then we substitute this expression into Eq. (1.119) or ∇ · E (in both cases it is read as “the divergence of E”).


and divide both its sides by V , which gives So, the Gauss theorem (1.123) in terms of ∇ operator can be
written as-
1
I
E~ · dA
~ = hρi/ε0 (1.120)
V −
→ ~ ρ
∇ ·E = (1.124)
We now make the volume V tend to zero by contracting it to ε0
the point we are interested in. In this case, hρi will obviously
tend to the value of ρ at the given point of the field, and hence The Gauss theorem in the differential form is a local theo-
rem: the divergence of the field E ~ at a given point depends
the ratio on the left-hand side of Eq. (1.120) will tend to ρ/ε0 .
~ · dA
~ to V only on the electric charge density ρ at this point. This is one
The quantity which is the limit of the ratio of E
H
~ and is denoted of the remarkable properties of electric field. For example, the
as V → 0 is called the divergence of the field E field E~ of a point charge is different at different points. Gen-
by div E. Thus, by definition, erally, this refers to the spatial derivatives ∂Ex /∂x, ∂Ey /∂y,
and ∂Ez /∂z as well. However, the Gauss theorem states that
~ = lim 1
I
div E ~ A
Ed ~ (1.121) the sum of these derivatives, which determines the divergence
V →0 V
~ turns out to be equal to zero at all points of the field
of E,
The divergence of any other vector field is determined in a (outside the charge itself).
similar way. It follows from definition (1.121) that divergence ~ is positive,
At the points of the field where the divergence of E
is a scalar function of coordinates.
we have the sources of the field (positive charges), while at the
In order to obtain the expression for the divergence of the
~ we must, in accordance with (1.121), take an infinitely points where it is negative, we have sinks (negative charges).
field E,
small volume V , determine the flux of E ~ through the closed The field lines emerge from the field sources and terminate at
the sinks.
surface enveloping this volume, and find the ratio of this flux Eq.(1.124) can also be written as-
to the volume. The expression obtained for the divergence will −
→  − →
depend on the choice of the coordinate system (in different ∇ . ε0 E =ρ
systems of coordinates it turns out to be different). For
example, in Cartesian coordinates it is given by −→− →
or ∇.D = ρ (1.125)

~ = ∂Ex + ∂Ey + ∂Ez


div E (1.122) In Eq.(1.125), −
→ −

∂x ∂y ∂z D = ε0 E is called electric displacement vector.
Detailed discussion of displacement vector is out of scope of
Thus, we have found that as V → 0 in (1.120), its right-hand
~ this book.
side tends to ρ/ε0 , while the left-hand side tends to div E.
Consequently, the divergence of the field E~ is related to the
charge density at the same point through the equation EXAMPLE 62. A charge is placed at the center of a cylin-
drical surface. Find total flux passing through lateral curved
ρ
~
div E = (1.123a) surface.
ε0

∂Ex ∂Ey ∂Ez ρ


or + + = (1.123b)
∂x ∂y ∂z ε◦
Equations (1.123a) and (1.123b) represent the Gauss theorem
in the differential form. The form of many expressions and
their applications can be considerably simplified if we introduce


the vector differential operator ∇ . In Cartesian coordinates,


the operator ∇ has the form Figure 1.210

→ ∂ ∂ ∂
∇ = î + ĵ + k̂ SOLUTION Total flux linked can be divided into two parts
∂x ∂y ∂z
94 CHAPTER 1. ELECTRIC CHARGE AND FIELD

(i) Flux through longitudinal cylindrical curved surface φL


(ii) Flux through end caps φA
If end cap subtends a solid angle Ω at the center, then
q
ΦL + Φ A = (1.126)
ε◦
The solid angle corresponding to semi vertical angle θ at the
vertex of a cone [see Fig.1.211]is given by-

Figure 1.212

1.24 Applications of Gauss’s law


Selecting a Gaussian Surface: While applying Gauss’s law
we are interested in evaluating the integral
Figure 1.211
→−
− →
I I
ΦE = E .dA = EdAcosθ (1.128)
Ω = 2π (1 − cos θ) (1.127)
Since, the electric flux corresponding to solid angle 4π sr is Now note that, this integration may be complicated if product
of E and cosθ is not constant. Therefore we always select a
ε◦ , therefore, the electric flux corresponding to solid angle Ω
q

2π(1−cos θ) q (1−cos θ) q Gaussian surface in such a way that product of E and cos θ
is 4π
Ω q
ε◦ , i.e., 4π ε◦ or 2 ε◦ . remains constant. That is we have to select some symmetrical
The electric flux passing through left end cap of Fig.1.210, surfaces.
(1 − cos 45◦ ) q The types of symmetry are illustrated in Fig.1.213 and sum-
Φ1 =
2 ε marized in Table 1.3. If the object does not have any of these
  ◦ three types of symmetry, Gauss’s law, even though still true, is
1 1 q
= 1− √ unlikely to be helpful in calculating the electric field strength.
2 2 ◦ ε
Similarly, the electric flux passing through the right cap is-
  Table 1.3: Type of symmetry and the corresponding gaussian sur-
1 1 q face that should be used
Φ2 = 1− √
2 2 ◦ε
Type of Gaussian
Therefore, the electric flux passing through both end caps- Symmetry Examples
Surface
ΦA = Φ1 + Φ2 point charge,
sphere concentric
spherical charged sphere,
with point
 
1 q
= 1− √ spherical shell
2 ε ◦ line of charge,
Substituting this value in Eq.(1.126), we get- cylinder coaxial charged cylinder,
cylindrical
  with line coaxial cylindrical
q 1 q shells
ΦL = − 1− √
ε◦ 2 ε◦ charged parallel
q cylinder or box per-
=√ planar plane(s), large flat
pendicular to plane
2ε◦ object
So, the flux passing through curved cylindrical surface is √q
2ε◦

EXAMPLE 63. A cone of base radius R and height h Gauss’s Law: Problem Solving Approach


is located in a uniform electric field E parallel to its base
To apply Gauss’s law, we simply follow the following steps-
[Fig.1.212]. Find the electric flux entering the cone.
APPROACH You require to take only the projection of area 1. Sketch the charge distribution, as well as the position(s)
of cone on a plane normal to electric field and it is a triangle where the electric field is to be calculated.
of height h and base length 2R for the cone. To determine the 2. Determine the symmetry of its electric field.
required flux, just multiply this projection with electric field
E. 3. Draw the appropriate closed Gaussian surface (see Ta-
SOLUTION Area of the triangle = 12 × 2R × h = Rh. ble 1.3 ) based on the symmetry, choosing the size of the
Hence flux = ERh. surface according to where the electric field is to be de-
termined. For example, in the case of a charged sphere,
1.24. APPLICATIONS OF GAUSS’S LAW 95

i. If the electric field is everywhere tangent to a


surface, the electric flux through the surface is

→− →
ΦE = E . A = 0
ii. If the electric field is everywhere perpendicular
to a surface and has the same magnitude E at
every point, the electric flux through the surface

→− →
is ΦE = E . A = EA
(a) (c) The electric field is zero over the portion of the sur-
1 face.
Gaussian 1
surface Different portions of the Gaussian surface can satisfy dif-
1
r ferent conditions as long as every portion satisfies at least
one condition.
S
E 5. If necessary, divide the closed surface into parts. In some
l S cases, the electric field is not constant over the entire sur-
dA
face but is known over each part. In this case, divide the
Gaussian surface into different parts, ensuring that the
entire closed surface is included. For example, we earlier
1
divided the cylinder into two end caps and the curved side.
1 ~ relative to
6. For each part, determine the orientation of E
1 ~
dA and use that to find the dot product for that part.
(b)
7. Formally integrate over the Gaussian surface to compute
1 the electric flux. However, if you have a surface with uni-
1
1 form electric field strength (and constant direction relative
1 1 to the surface normal) you can take E out of the integral
1 1
1
1
1 1 1
1 and simply multiply the constant E by the surface area
1
S 1 1 A multiplied by the cos θ value.
E 1
1 8. Calculate the net charge enclosed by the Gaussian surface.
1
1
1 Remember that only the charge inside the surface needs to
1 1 be considered, and when there are positive and negative
1 1 1 S
1 1 1 E charges inside the surface it is the net charge that is used.
1 1
1 1
1
1 9. Use mathematical expression of Gauss’s law (with the ex-
1
Gaussian pression for the electric flux (from step 6) and the enclosed
surface charge (step 7)) to solve for the electric field strength-
(c)
~ = Qencl
I
ΦE = E ~ · dA

C cylindrical symmetry (b),


Figure 1.213: In spherical symmetry (a), the points on a sphere 0
have constant electric field strength. For
points at a common distance from a line have constant electric field 10. Finally you can check that your answer makes sense with
strength. In planar symmetry (c), points on a plane parallel to the limiting cases (i.e., does your relationship make sense just
charged plane have constant electric field strength. outside a surface, or at infinite distance).
EXAMPLE 64. Using Gauss’s law, find the electric field at
a Gaussian sphere is drawn concentric with the centre of a distance r from a positive point charge q, and compare it with
the object and is of radius equal to the radial distance Coulomb’s law.
where the electric field is to be computed. Sketch in the APPROACH By definition, a point is a sphere of negligible
~ and dA
directions of E ~ on your diagram. You need not radius, i.e., a sphere of zero radius, therefore we can assume
A all the charge within the Gaussian surface.
enclose a point charge as sperical and draw a spherical Gaussian
surface of radius r around it (Fig.1.214). From the symmetry
4. Be sure every part of the Gaussian surface satisfies one or
of the problem, we know that at any point, the electric field
more of the following conditions: ~ is perpendicular to the surface and directed outwards from
E
~
(a) The value of the electric field can be argued by sym- the spherical center. Thus, E//d A~ and E.d
~ A~ = EdA, with
metry to be constant over the portion of the surface. Qencl = q. Now, apply Gauss’s law-
(b) Gaussian surface is either tangent to or perpendicular
~ = Qin
I
ΦE = E ~ · dA (1.129)
to the electric field. ◦
96 CHAPTER 1. ELECTRIC CHARGE AND FIELD

and solve for E


~ · dA
SOLUTION Substituting, E ~ = EdA and Qencl = q in

L 1Q

L
L
Figure 1.215: A single point charge, +Q, is at the exact centre of
a cube. The length of each side of the cube is L.

Figure 1.214

Eq. (1.129), we get

q
I
EdA = Figure 1.216
◦
q
I
⇒ E dA =
◦
2 q q
⇒ 4πr E =
◦ a/2
1 q q
⇒ E= 2
=k 2
4π◦ r r a
Figure 1.217
which is simply Coulomb’s law. This proves that Gauss’s law
and Coulomb’s law are equivalent.
out from the cubical Gaussian surface-
EXAMPLE 65. A single positive point charge, +Q, is placed
at the exact center of a cube of side length L (see Figure 1.215). 1
φ= Qencl
(a) Is it possible to calculate the electric flux through one 0
face of the cube? If so, what is the value? 1
= q
(b) Can Gauss’s law be used to find the electric field on the ε 0
face of the cube? If so, what is the value? By symmetry all six faces have equal flux. Therefore, flux
SOLUTION (a) Yes. The enclosed charge is +Q, so by coming out from a single face q
φ=
Gauss’s law the total electric flux through all six faces of 6ε0
the cube is ε0 . By symmetry, all the faces are equal, so the
Q

electric flux through any one face is just 1/6 of this value, i.e., EXAMPLE 67. By applying Gauss’s law, show that the sta-
ble equilibrium of a charge under the effect of an electric field
6ε .
Q

(b) No! Since, we don’t know the distribution of electric field only, is impossible(Earnshaw’s Theorem 1.19).
strength across the faces, so we cannot take E out of the SOLUTION Suppose, in vacuum, we have a system of fixed
integral. point charges in equilibrium. Now, consider, one of these
charges, e.g. a charge q. Let us envelop the charge q by a
EXAMPLE 66. Find electric flux through square of side small closed surface S (Fig.1.218). For the sake of definiteness,
a, due to charge placed at distance a/2 from centre of a we assume that q > 0. For the equilibrium of this charge to
~ created by
be stable, it is necessary that the electric field E
square[Fig.1.216].
all the remaining charges of the system at all the points of the
SOLUTION Let us enclose the charge q by a cubical gaus- surface S be directed towards the charge q. Only in this case
sian surface with q at its centre[Fig.1.217]. Net flux, coming any small displacement of the charge q from the equilibrium
1.24. APPLICATIONS OF GAUSS’S LAW 97

1
1
Gaussian 1
1
surface
1
r
S
E S
E
S
l dA
Figure 1.218
1

position will give rise to a restoring force, and the equilibrium 11


1 1
state will actually be stable. But such a configuration of the 111
~ around the charge q is in contradiction to the Gauss
field E 111
theorem: the flux of E ~ through the surface S is negative, while (a) (b)
in accordance with the Gauss theorem it must be equal to zero
since it is created by charges lying outside the surface S. On Figure 1.219: (a) An infinite line of charge surrounded by a cylin-
the other hand, the fact that E is equal to zero indicates that drical gaussian surface concentric with the line.(b) (b) An end view
at some points of the surface S vector E ~ is directed inside it shows that the electric field at the cylindrical surface is constant in
magnitude and perpendicular to the surface.
and at some other points it is directed outside.
Hence it follows that in any electrostatic field a charge cannot
be in stable equilibrium. R →−
− → R − →− →
E .dA = S3 E .dA = 0
S2
−→ −→
and E || A at whole curved surface S3 , so-
R − →− →
1.24.1 Cylindrical Symmetry S3
E .dA = EdA Substituting these values and Qencl = λL in
Eq.(1.130), we get-
Field of a Line Charge
λL
Z
EXAMPLE 68. An infinitely long, straight line of uniform 0+0+ E dA =
positive charge has a charge per unit length of λ (in units of S3 ε0


charge per length). Find the electric field E at an arbitrary λL
⇒ E(2πrL) =
distance r from the line. ε0

APPROACH From each incremental charge along the line,


an electric field emanates equally in all dircctions. However, λ 2λ
⇒ E= =k (radially outward) (1.131)
by symmctry, the superposition of the ficlds from all of the 2πε0 r r
incremental charges results in a cancellation of fields parallel
to the line of charge6 . The result is a net field directed radially here, k = 4πε 1
0
Equation (1.131) gives the electric field
outward from the line. At all points at a given distance r from strength, E, at a distance r from an infinite line with charge
the line (in any direction), the field has the same magnitude. density λ. The direction of the electric field is radially away
Therefore, we match this symmetry with a Gaussian surface from the line of charge.
in the form of a cylinder of radius r and length L whose axis
is the line of charge, Fig.1.219. At every point on the cylin-

→ EXAMPLE 69. What if the line segment in this example
drical curved side S1 of the cylinder, E is parallel to the area
elements d A, and it has the same magnitude everywhere. On were not infinitely long?


the flat end caps S1 and S2 of the cylinder, E is perpendicular
−→ SOLUTION If the line charge in above example were of fi-
to d A everywhere. The net charge Qencl inside the cylinder is
nite length, the electric field would not be given by Equation
λL. Applying Gauss’s law, we obtain-
(1.131). A finite line charge does not possess sufficient sym-
metry to make use of Gauss’s law because the magnitude of

→ − → Qencl the electric field is no longer constant over the surface of the
I
ΦE = E · dA = Gaussian cylinder: the field near the ends of the line would
ε0
be different from that far from the ends. Therefore, condition

→ − → −
→ − → −
→ − → Qencl −

Z Z Z
⇒ E · dA + E · dA + E · dA = (4a) would not be satisfied in this situation. Furthermore, E
S1 S2 S3 ε0 is not perpendicular to the cylindrical surface at all points: the
(1.130) field vectors near the ends would have a component parallel to
→ −
− →
From Fig.1.220, we see that, E ⊥dA at end caps S1 and S2 , the line. Therefore, condition (4b(ii)) would not be satisfied.
therefore- For points close to a finite line charge and far from the ends,
6 We have already seen this in case of applications of Coulomb’s law Equation (1.131) gives a good approximation of the value of
for "electric field due to infinite line charge the field.
98 CHAPTER 1. ELECTRIC CHARGE AND FIELD

The Electric Field of an Infinite Plane Sheet of Charge Eq.(1.133), we conclude that the field has the same constant
value for all distances on either side of the plane of charge.
EXAMPLE 70. Use Gauss’s law to find the electric field of
an infinite plane of charge with surface charge density σ ( in
C/m2 ). EXAMPLE 71. Explain why Gauss’s law cannot be used to
calculate the electric field near an electric dipole, a charged
APPROACH From symmetry we can say that, as long as
disk, or a triangle with a point charge at each corner.
we are not near an edge, the electric field must extend per-
pendicularly away from the plane on both sides. (There is no SOLUTION The charge distributions of all these configu-
asymmetry that would cause the field lines to bend to one side rations do not have sufficient symmetry to make the use of
or the other as they extend away from the positive charges.) Gauss’s law practical. We cannot find a closed surface sur-
We match the symmetry of this field by considering a Gaussian rounding any of these distributions for which all portions of
surface in the form of a cylinder or pill box, of cross-sectional the surface satisfy one or more of conditions listed at "Gauss’s
area A, whose axis is perpendicular to the plane and whose Law:Problem solving Approach."
ends are equidistant from the plane (Fig.1.220. The net charge
enclosed by the surface is Qencl = σA. By symmetry, the field Electric Field Near a Charged Conducting Surface
emerges uniformly and perpendicularly from each end and is
tangent to the curved side of the cylinder. Now, apply Gauss’s EXAMPLE 72. Using Gauss’s law, find the electric field just

→ outside the surface of a conductor carrying a positive surface
law, and solve for E .
charge density σ.
APPROACH Near the conducting surface, the surface ap-
pears to be a flat (Fig.1.221, close-up), infinite plane, just as
the surface of the ocean seems to be flat from our perspective
standing on a beach. So, the surface charge density σ over a
small part of the conductor is uniform7 .


Notice that In the static case, no electric field E can exist
within a conductor (because it would make the conduction
charges move). So field lines extend away from the conductor,
perpendicular to the surface, Fig.1.221. From symmetry con-
siderations, we choose a cylindrical Gaussian surface as in the
previous example, but in thhis case, we can draw two types of
cylindrical Gaussian surfaces-
1. Gaussian cylinder crosses the conductor
2. One end of Gaussian cylinder is inside the conductor,
whereas the other end out side the surface of the conductor.
Figure 1.220: The Gaussian surface extends to both sides of a Apply Gauss’s law in both cases and solve for electric field − →
E.
plane of charge.
SOLUTION Case 1. Let Gaussian cylinder crosses the con-
ductor completely. 1, 2 are planer surfaces and 3 is curved
SOLUTION By, Gauss’s law, we have surface of Gaussian cylinder

→ − → Qencl Aσ
I
E · dA = = (1.132)
ε0 0
H− →− → R − →− → R − →− → R − →− →
Now, E .dA = S1 E .dA + S2 E .dA + S3 E .dA
here, S1 represents curved surface whereas S2 , S3 represent
two planer surfaces. From Fig.1.220, we have-
R − →− → → −
− →
E .dA = 0 ∵ E ⊥dA over whole surface S1 , and
RS1 −
→− → R →−
− → → −
− → −→ − →
S2
E .dA + S3 E .dA = 2EA ( E k dA, E k dA)

→ −

Note that, for S2 , both E and dA pointed towards right side,

→ −

while for S3 , both E and dA pointed towards left (Fig.1.220).
Substituting these values in Eq.(1.132), we get-
Figure 1.221

∴ 2EA = ⇒
0 Net flux linked with the Gaussian cylinder
σ I
Qencl
E= (1.133) ΦE = E.dA =
20 0
This electric field is away from the plane, left and right. 7 In general, the surface charge density σ is not constant on the surface

Because the distance from the surface does not appear in the of a conductor but depends on the shape of the conductor.
1.24. APPLICATIONS OF GAUSS’S LAW 99

→−
− → →−
− → →−
− → 2σA
Z Z Z
⇒ E .dA + E .dA + E .dA = The directions of the fields to the left, between, and right of
1 2 3 0 the sheets are shown in Fig.1.223. The resultant field depends
2σA on the values of E1 and E2 .
⇒ EA + EA + 0 = Electric field inside the conductor: Inside the conductor,
0 −
→ −

E 1 and E 2 are oppositely directed, therefore the resultant
σ
⇒ E= (1.134) electric field-
0
σ σ
Thus electric field near the conducting surface is E = εσ0 E= − =0
20 2ε0
This field is twice the value that we found in the previous
Electric field outside the conductor: Outside left or
example, and it has a constant value for all distances above −→ −

right of the conductor, E 1 and E 2 both are directed in same
the infinite plane conductor.
direction, therefore the resultant electric field-
Case 2. Let Gaussian surface does not cross the conductor i.e.,
one end is inside the conductor whereas other end is outside it σ σ σ
E= + =
[Fig.1.222] 2ε0 2ε0 ε0
Net flux linked with the Gaussian cylinder
EXAMPLE 73. A non-conducting very long cylinder of ra-
dius R has a positive uniform volume charge density ρ through-
out. Derive expressions for the electric field both (a) inside
(r < R) and (b) outside (r > R) the cylinder.

SOLUTION We have cylindrical symmetry, so the appro-


priate closed surface is a coaxial Gaussian cylinder of radius
r [Fig.1.224]. The length of the Gaussian cylinder, L, is arbi-
trary and will cancel out in the final answer.

Figure 1.222 dA

E dA
Qenclosed
I
E
ΦE = E.dA = r
0 E
dA L
→−
− → →−
− → →−
− → 2σA
Z Z Z
⇒ E .dA + E .dA + E .dA =
1 2 3 0
R
σA
⇒ EA =
0
σ Figure 1.224: The very long charged cylinder (yellow) has a uni-
⇒E=
0 form positive charge density. In grey is drawn the smaller Gaussian
cylinder for finding the electric field inside the charged cylinder.
Thus electric field near the conducting surface is E = σ0
Explanation: A conductor can be assumed as the combina-
tion of two plane sheet of charges at its surface. Let σ is the (a) Inside the cylinder (r < R) : For this part, we draw
surface charge density, then from following diagram, we have the Gaussian cylinder with radius r less than the radius, R,
of the charged cylinder (i.e., the Gaussian surface is inside
> >
the physical cylinder, but coaxial with it). The situation is
illustrated in Figure 20-24, with the Gaussian cylinder in grey.
From the symmetry of the situation, > the> positively charged
cylinder results in an electric field that points radially outward
and has constant electric field strength at any particular radial
distance.

This is a situation in which we need to divide the closed


Gaussian surface into three parts, the two end caps and the
~ points outward
curved side of the cylinder. In all cases, dA
from the closed Gaussian surface. This is pictured for the
grey Gaussian surface in Figure
> >20 − 24. For the two end
Figure 1.223 ~
caps, dA is parallel to the axis of the cylinder, pointing in
>
100 CHAPTER 1. ELECTRIC CHARGE AND FIELD

different directions at the two ends. For the curved surface of The direction of the electric field points radially outward
the cylinder, dA~ is everywhere radially outward (we only show from the axis of the cylinder.

→ −
→ (b)) Outside the charged cylinder (r > R) : In this case, we
E and d A at one point on the curved Gaussian surface, but
both are similarly radially outward for all other points on the make a Gaussian cylinder of radius larger than that of the
curved surface). charged cylinder, but otherwise the solution is similar. The
We find the electric flux by combining the contributions from electric flux process is identical to that given in part (a), with
the three parts of the surface: the same result:
Φnet = 2πrLE
ΦE net = ΦE left cap + ΦE curved surface + ΦE right cap
When computing the enclosed charge, we now use the volume

→ −→ of the charged cylinder, rather than the larger Gaussian cylin-
Everywhere along the curved surface, E and d A are par-
der:
allel to each other, while on both end caps they are per-
qenc = πR2 Lρ
pendicular to each other (see Figure 20-24). This means

→ −

that the vector dot product of E with d A yields left end We invoke Gauss’s law and solve for the electric field strength.

→ − →
cap: E · d A = EdA cos 90◦ = EdA(0) = 0 curved wall: qenc

→ −→ ΦE =
E · dA ~ = EdA cos 0◦ = EdA(1.00) = EdA right end cap: ε0

→ − →
E · d A = EdA cos 90 = EdA(0) = 0 Here E represents the

πR2 Lρ
magnitude of the electric field, and dA is the differential area 2πrLE = (1.136)
ε0
element without a vector direction. Therefore, substituting
into the relationship for electric flux, we have R2 ρ
E=
2rε0
Again, the direction of the electric field is radially outward

→ − → from the axis of the cylinder. So, within the cylinder, the field
I
ΦE left cap = E · dA = 0
left cap

→ − →
I I
ΦE curved wall = E · dA = EdA
curved wall curved wall
I
=E dA = EAcurved wall
curved wall

→ − →
I
ΦE right cap = E · dA = 0
right cap

The area of the curved wall can be determined if we imagine


it has been unwrapped into a corresponding rectangle. One
side of the rectangle is L and the other is the circumference of
the surface, 2πr : Figure 1.225: A graph of the electric field E vs. r for a uniform
volume charge density throughout an infinitely long cylinder of ra-
ΦE curved wall = EAcurved wall = EL(2πr) ~ is directed outward.
dius R. In the graph, E is positive when E
= 2πrLE
is directly proportional to the distance from the axis. Outside
Since the two end caps have zero electric flux, this is also the the cylinder, the field is the same as for the charged wire of
net electric flux: infinite length. Fig.1.225 shows a plot of E vs. r.
Φnet = 2πrLE Making sense of the result: Inside the charged cylinder,
Now we need to find the net enclosed charge. We multiply the electric field increases linearly with distance from the axis.
the volume charge density, ρ, by the volume of the Gaussian At the exact centre of the cylinder, the electric field is zero.
cylinder. The volume of a cylinder is the cross-sectional area Outside the charged cylinder, the electric field is proportional
times the length of the cylinder: to 1/r, approaching zero at infinite distance. When r = R,
we require that the results from the two parts be consistent,
qenc = πr2 Lρ which they are. Note that this solution is for the case of a
non-conducting cylinder with the charge uniformly distributed.
Finally, we invoke Gauss’s law and solve for the electric field
strength:
qenc
ΦE = 1.24.2 Spherical Symmetry
ε0
Field of a Charged Conducting Sphere
πr2 Lρ
2πrLE = (1.135) EXAMPLE 74. We place positive charge q on a solid con-
ε0
rρ ducting sphere (or spherical shell) with radius R (Fig.1.226 ).
E= Find E ~ at any point inside or outside the sphere.
2ε0
1.24. APPLICATIONS OF GAUSS’S LAW 101

APPROACH For a conducting sphere, all the charge lies on Just outside the surface of the sphere, where r = R, Eq.
its surface. So, the system has spherical symmetry. To take (1.138) takes the form-
advantage of the symmetry, we draw a spherical Gaussian sur-
face of radius r centred on the conductor. To calculate the field 1 q q
E= =k 2 (1.139)
outside the conductor, we take r to be greater than the conduc- 4πε0 R 2 R
tor’s radius R; to calculate the field inside, we take r to be less
than R. In either case, the point where we want to calculate Note: A point charge q can be considered to be the limiting
E~ lies on the Gaussian surface. Now, we use the mathematical case of a small spherical conductor whose radius tends to zero
expression of Gauss’s law (Eq.(1.137))to calculate the electric and electric field to infinity.
field E at the required position-
KEY POINT: Flux can be positive or negative:
−→ − → Qencl
I
ΦE = E · dA = (Gauss’s Law) (1.137) Note that we have chosen the charge q to be positive. If the
ε0 charge is negative, the electric field is radially inward instead
of radially outward, and the electric flux through the Gaussian
surface is negative. The electric field magnitudes outside and
at the surface of the sphere are given by the same expressions
as above, except that q denotes the magnitude (absolute
value) of the charge.

(ii) For a point inside the conducting sphere (r < R):


In this case Gaussian surface does not include any charge, i.e.
Qencl = 0, therefore Eq.(1.137) gives-
I
E dA = 0
I
⇒ E dA = 0

⇒ E(4πr2 ) = 0

⇒ E=0 (1.140)
So, the electric field inside the conductor is zero.
Making Sense Of Result:: We already knew that E ~ = 0
Figure 1.226: Calculating the electric field of a conducting sphere inside the conductor, as it must be inside any solid conductor
with positive charge q. Outside the sphere, the field is the same as when the charges are at rest. Figure 1.226 shows E as a
if all of the charge were concentrated at the centre of the sphere. function of the distance r from the centre of the sphere. Note
that in the limit as R → 0, the sphere becomes a point charge;
SOLUTION (i) For a point outside the conducting there is then only an ’outside’, and the field is everywhere
sphere (r ≥ R): In this case, the entire conductor is within given by E = q/4π0 r2 . Thus we have deduced Coulomb’s law
the Gaussian surface, so the enclosed charge Qencl = q from Gauss’s law.


By symmetry, we conclude that the field E can only be ra-
dially outward, outside the sphere. Furthermore, for a given
−→
value of r, E has the same magnitude everywhere. Therefore, EXAMPLE 75. A thin spherical shell of radius R has a total
Eq. (1.137) can be written as positive charge Q distributed uniformly over its surface. Find
the electric field inside and outside the shell.
q
I
E dA =
ε0 APPROACH Here, we use the method described in the last
problem. By symmetry, if any field exists inside the shell, it
Since, the field at each point on Gaussian surface has same
must be radial. For any point outside or on the surface of the
magnitude, so E can be taken out of integral, so-
conducting shell, it behaves like a solid conducting sphere. So,
q construct spherical Gaussian surfaces for both cases and apply
I
E dA =
ε0 Gauss’s law-

→ − → Qencl
I
2 q ΦE = E · dA = (1.141)
⇒ E(4πr ) = ε0
ε0
Solve above equation for E in each case.
1 q q
⇒ E= = k (For r > R) (1.138) SOLUTION (i) Inside the shell (r<R): Let us construct a
4πε0 r2 r2
spherical Gaussian surface of radius r < R concentric with the
This is just the inverse-square-law field for a point charge q shell (Fig.1.227). Since, there is no enclosed charge within the
concentrated at the center of the sphere. Gaussian surface, i.e., Qencl = 0, therefore, Eq.(1.141) gives-
102 CHAPTER 1. ELECTRIC CHARGE AND FIELD

+
Gaussian in each case and find corresponding Qencl . Apply Gauss’s law
Q + R +
sphere and simplify for electric field E.
SOLUTION (1) For 0 ≤ r ≤ R: When dealing with a
Spherical + + spherically symmetric charge distribution, we chose a spherical
r
shell Gaussian surface of radius r < R concentric with the charged
+ + sphere as shown in Fig.1.229.
+
Figure 1.227: Cross sectional view of shell with Gaussian surface
inside it. ρ R dA

E
I
~ · dA
~ = E 4πr2 = 0
 r
E
Gaussian
⇒ E=0 (1.142)
sphere
So, we conclude that there is no electric field inside a uniformly Figure 1.229
charged spherical shell.
(ii) Outside the shell (r ≥ R): Outside the shell, we con-
struct a spherical Gaussian surface of radius r > R concentric By symmetry, the magnitude of the electric field is constant
with the charged shell as shown in Fig.1.228 . Symmetry sug- everywhere on the spherical Gaussian surface and normal to
~ the surface at ~
any point, i.e. E//d ~ Thus:
A.
gests that
H E = constant on that surface and E is parallel to
~ i.e. E ~ · dA
~ = E 4πr2 . Since the net charge Qencl inside

dA, I I I
the Gaussian surface is equal to the total charge Q on the shell, ~ ~
E · dA = EdA = E dA = E 4πr2


the shell is equivalent to a point charge located at the center.


Therefore, Eq.(1.141) gives- It is important to notice that the volume, say V 0 , of the Gaus-
sian sphere encloses a net charge Qencl = ρV 0 ; that is:
 
0 4 3
qin = ρV = ρ πr
3

We can now use Gauss’s law to find electric field as follows:


 ρ 43 πr3

Qencl
I
~ ~
ΦE = E · dA = ⇒ E 4πr = 2
◦ ε◦
ρ
Figure 1.228: Cross sectional view of shell with Gaussian surface Then: E= r (0 ≤ r ≤ R)
3ε◦
outside it. Using the definition ρ = Q/ 34 πR3 and k = 1/(4πε◦ ), we get:


1 Q Q Q Q
E= =k 2 (r > R) (1.143) E= 3
r = k 3r (0 ≤ r ≤ R)
4πo r2 r 4π◦ R R

(iii) At the surface of the shell (r = R): In this case, (2) For r ≥ R Again, because the charge distribution is spher-
Eq.(1.143) takes the form, ically symmetric, we can construct a Gaussian sphere of ra-
dius r > R concentric with the charged sphere, as shown in
1 Q Q Fig.1.230.
(1.144) Just as when r < R, E ~ · dA
~ = E 4πr2 , but qin = Q. Thus,
H 
E= 2
=k 2
4πo r R

Electric field E vs r graph is similar to the graph obtained in Q


the case of solid conducting sphere (Fig.1.226). dA
R
E
EXAMPLE 76. A solid sphere of radius R has a uniform
volume charge density ρ and carries a total positive charge Q.
Find and sketch the electric field at any distance r away from Gaussian r
the sphere’s center. sphere

APPROACH The whole space can be divided into two


parts:(i) 0 ≤ r ≤ R and (ii) r ≥ R. Observe the symmetry Figure 1.230
1.24. APPLICATIONS OF GAUSS’S LAW 103

we can use Gauss’s law to find the electric field as follows: (ii) For point B: All the spherical shells for which point B
lies inside will make electric field zero at point B. So, electric
~ = qencl ⇒ E 4πr2 = Q
I
~ · dA field will be due to charge present from radius r to OB.

ΦE = E
◦ ◦ k 43 π OB 3 − r3 ρ −→


→ ρ [OB 3 − r3 ] −→
So, E B = OB = OB
i.e.: OB 3 30 OB 3
1 Q Q (iii) For point C: Similarly, we can say that for all the shell
E= = k 2 (r ≥ R)
4π◦ r2 r points, C lies outside the shell
k 43 π R3 − r3 ρ −→

Notice that this is identical to the result obtained for a point −
→ ρ [R3 − r3 ] −→
So, E C = OC = OC
charge. Therefore, we conclude that the electric field outside OC 3 30 OC 3
any uniformly charged sphere is equivalent to that of a point Method: II We can consider that the spherical cavity is
charge located at the center of the sphere. At r = R, the two filled with charge density ρ and also −ρ, thereby making net
cases give identical results E = kQ/R . A plot of E versus r
2 charge density zero after combining. We can consider two con-
is shown in Fig.1.231. This figure shows the continuation of E centric solid spheres: One of radius R and charge density ρ
and its maximum at r = R. and other of radius r and charge density −ρ[Fig.1.233].
Applying superposition principle:

ρ
R

Q
E k
E r2
Q
E k r
R3
0 r
R
Figure 1.231

EXAMPLE 77. A Uniformly charged solid non-conducting


sphere of uniform volume charge density ρ and radius R is hav-
ing a concentric spherical cavity of radius r. Find out electric
field intensity at following points, as shown in the Fig.1.232

Figure 1.233

−→
h −→ i

→ −
→ −
→ ρ(OA) −ρ(OA)
(i) E A = E ρ + E −ρ = + =0
3ε0 3ε0

→ −
→ −

(ii) E B = E ρ + E −ρ
−→
ρ(OB) K 43 πr3 (−ρ) −→
 
Figure 1.232
= + OB
3ε0 (OB)3
(i) Point A, (ii) Point B, (iii) Point C, (iv) Center of the sphere. 
ρ r3 ρ

−→ ρ

r3 −→

SOLUTION Method I: = − OB = 1− OB
3ε0 3ε0 (OB)3 3ε0 OB 3
(i) For point A:
We can consider the solid part of sphere to be made of large → −−→ K 43 πR3 ρ −→ K 43 πr3 (−ρ) −→
 
−→ −
number of spherical shells which have uniformly distributed (iii) EC = Eρ + E−ρ = OC 3
OC +
OC 3
OC
charge on its surface.
Now, since point A lies inside all spherical shells so electric ρ  3  −→
= 3
R − r3 OC
field intensity due to all shells will be zero. 3ε 0 (OC)
−→ −→ − → −

EA = 0 (iv) EO = E ρ + E −ρ = 0 + 0 = 0
104 CHAPTER 1. ELECTRIC CHARGE AND FIELD

EXAMPLE 78. Solve Example 77, if cavity is not concentric


and centered at point P [Fig.1.234].

Figure 1.234 Figure 1.235

SOLUTION Again assume ρ and −ρ in the cavity, (similar


to the previous example): (ii) r ≥ R

→ −→ −

(i) E A = E ρ + E −ρ
−→ −→ Z R Z R
k.4πx2 dxρ0 x ρ0 R 4

→ ρ[OA] −ρ[P A] E= dE = = r̂
⇒ EA = + r 2 4ε0 r2
3ε0 3ε0 0 0
ρ h−→ −→i ρ h−→i
= OA − P A = OP Method II:
3ε0 3ε0
Note: From above it is clear that the electric field intensity at (i) The sphere can be considered to be made of large number
point P is independent of position of point P inside the cavity. of spherical shells. Each shell has uniform charge density on
Also the electric field is along the line joining the centers of its surface. So the previous results of the spherical shell can be
the sphere and the spherical cavity. used. we can say that all the shells for which point lies inside

→ −
→ −
→ will make electric
(ii) E B = E ρ + E −ρ R rfield zero
2
at that point, so
−→ ~ (r < R) = K 0
(4πx dx)ρ 0x ρ0 r2
ρ(OB) k 43 πr3 (−ρ) −→ E = r̂
 
= + PB r2 4ε0
3ε0 (P B)3 (ii) similarly for r ≥ R, all the shells will contribute in electric

→ −→ −
→ k( 4 πR3 ρ) −→ k( 4 πr 3 (−ρ)) −→ field, therefore
(iii) E C = E ρ + E −ρ = 3OC 3  OC + 3 P C 3 PC
RR

→ −
→ −
→ K 34 πr3 (−ρ) −→ −
→ K (4πx2 dx)ρ0 x ρ0 R 4
(iv) E O = E ρ + E −ρ = 0 + PO E (r < R) = 0
= r̂
[P O]3 r2 4ε0 r2
EXAMPLE 79. A non-conducting solid sphere has volume
charge density that varies as ρ = ρ0 r, where ρ0 is a constant 1.24.3 Check Point 9
and r is distance from center. Find out electric field intensities
1. •• A point charge Q is located just above the center of the
at following positions.
flat face of a hemisphere of radius R as shown in Figure
(i) r < R (ii) r ≥ R
P19.33. What is the electric flux (a) through the curved
SOLUTION Method I: (i) For r<R: The sphere can be surface and (b) through the flat face?
considered to be made of large number of spherical shells
[Fig.1.235]. Each shell has uniform charge density. So, the
previous results of the spherical shell can be used. Consider a
shell of radius x and
 thickness dx as an element. Charge on
shell dq = 4πx2 dx ρ0 x. Q
d 0
Therefore, the electric field intensity at point P due to shell,
kdq
R
dE =
x2
Since all the shells will have electric field in same direction
Z E Z r Z R
∴ E= dE = dE + dE
0 0 r
Due to shells which lie between region r < x ≤ R, electric field Figure 1.236
at point P will be zero.
Z r Z r  r

→ kdq k.4πx2 dxρ0 x 4πkρ0 x4 2. •• Fig. 1.237 shows two Gaussian surfaces in uniform
E = + 0 = =
0 r2 0 r2 r2 4 0 electric fields. In Fig.1.237(a) the Gaussian surface is a
ρ0 r2 square box, and in Fig.1.237(b) the Gaussian surface is
= r̂ a closed cylinder. In each case, integrate ΦE = E
H
~ ·
4ε0
1.24. APPLICATIONS OF GAUSS’S LAW 105

dA~ = qencl /ε0 to find the amount of charge enclosed in


1 2q
each Gaussian surface.

1
3 2q 22q
E
2q
4 5

1 2
1
? 1 2q
3q
(a) (b)
6
Figure 1.239
(a)

5. •• Fig.1.239(b) shows three charges. Draw these charges


E
on your paper four times. Then draw two-dimensional
P,
cross sections P, P , closed surfaces
of three-dimensional P . through
L CP R
which the electric flux is (a) −q/0 , (b) q/0 , (c) 3q/0 , and
(d) 4q/0 . P, , , q

FA 5 2q/P0 FC 5 22q/P0
B.

C
(b)

C. Figure 1.237
q1 q2
3 1.238 is a closed cylin-
/C.
3. ••The Gaussian surface in Figure
der. An electric field points to the right throughout,
made up of two uniform components with magnitude FB 5 3q/P0
EL = 40.9 N/C on the left side of the cylinder and q3
u u
~ A
81.8 N/C on the right side. Integrate Φ = E·d ~=
H
ER = w
qin /0 to find the amountuof charge enclosed in the Gaus-
sian surface. Express your result in terms of the surface
3 charge density (charge per unit area)
3 σ. (Assume σ is uni- Figure 1.240
form.)

u u 6. •• Fig.1.240 shows three Gaussian surfaces and the electric


field points to the EL = 40.9 N/C ER = 81.8 N/C
u flux through each. What are the three charges q1 , q2 , and
q3 ?
ht side. Integrate
sed in the Gaussian L CP R 7. As shown in figure 1.241a closed surface intersects a spher-
3.0 cm . ical conductor. What will be the nature of the electric flux
3.0 cm coming out of the closed surface, if we place a negative
charge at point P?

3.0 cm Figure or
ng. All Rights Reserved. May not be copied, scanned, 1.238
duplicated, in whole or in part. WCN 02-2 1000 N m
3.0 cm
4. •• Fig.1.239(a) shows three charges. Draw these charges

3
on your paper four times.00-203Then draw two-dimensional
cross sections of three-dimensional closed surfaces through
which the electric flux is (a) 2q/0 , (b) q/0 , (c) 0, and (
Figure 1.241
d )5q/0 .

3
106 CHAPTER 1. ELECTRIC CHARGE AND FIELD

8. •• Find the electric field due to an infinitely long cylin-


drical charge distribution of radius R and having linear
charge density λ at a distance half of the radius from its
axis. 1 1

9. •• Five charges q1 , q2 , q3 , q4 , and q5 are fixed at their


positions as shown in figure1.242. S is Gaussian surface.
The Gauss’s law is given by E ~ ·−

ds = εq0 . Which of the 1100 nC 11 nC (inside) 2100 nC
R

following statements is correct?


~ on the LHS of the above equation will have Figure 1.244
(A) E -
Gaussian surface 12. •• The net electric flux through an octahedron is
−1000Nm2 /C. How much charge is enclosed within the
octahedron?
q1
S 13. •• 55.3 million excess electrons are inside a closed surface.
What is the net electric flux through the surface?
q2
14. •• What is the flux through each face of a cube of side a
q4 (Fig.1.245) if a point charge of magnitude q is at one of
q3 its corner?
q5

Figure 1.242

a contribution from q1 , q5 , and q3 while q on the RHS


will have a contribution from q2 and q4 only. (B) E ~
on the LHS of the above equation will have a contri-
bution from all charges while q on the RHS will have
a contribution from q2 and q4 only. (C) E ~ on the LHS Figure 1.245
of the above equation will have a contribution from all
charges while q on the RHS will have a contribution
from q1 , q3 , and q5 only. (D) Both E~ on the LHS and Multiple Choice Questions
q on the RHS will have a contribution from q2 and q4 only.
15. •• What is the flux through a cube of side a if a point
charge of q is at one of its corner?
10. •• What is the net electric flux through the torus (i.e., (A) 2q
ε0 (B) 8εq0
doughnut shape) of Fig.1.243 (C) ε0
q
(D) 2εq0 6a2

16. • A charge Q is enclosed by a Gaussian spherical surface


of radius R. If the radius is doubled, then the outward
electric flux will-
(A) increase four times (B) be reduced to half
1 (C) remain the same (D) be doubled

17. •• A hollow cylinder has a charge q coulomb within


it[Fig.1.246]. If φ is the electric flux in units of volt meter
associated with the curved surface B, the flux linked with
21 nC (inside) 1100 nC the plane surface A in units of V − m will be-
(A) 2εq0 (B) Φ3 
Figure 1.243 
(C) q
ε0 −Φ (D) 1
2
q
ε0 −Φ

18. • A charge q is located at the centre of a cube. The


11. •• What is the net electric flux through the cylinder of electric flux through any face is
Figure 1.244? 2πq
(A) 6(4πε 4πq
(B) 6(4πε
0) 0)
(C) 6(4πε0 )
πq
(D) 6(4πε
q
0)
1.24. APPLICATIONS OF GAUSS’S LAW 107

C B A

Figure 1.246

19. • A charge Q µC is placed at the centre of a cube, the


flux coming out from each face will be
(A) 6ε
Q
0
× 10−6 (B) 6εQ
0
× 10−3
(C) 24ε
Q
0
(D) 8εQ
0

20. • A charge Q is situated at the corner of a cube, the


electric flux passed through all the six faces of the cube is
(A) 6ε
Q
(B) 8ε
Q Figure 1.247
0 0
(C) ε0
Q
(D) 2ε
Q
0

21. • A point charge +q is placed at the centre of a cube (A) ρR


6ε0 + E (B) ρR
12ε0 −E
−ρR
of side l. The electric flux emerging from the cube is (C) 6ε0 + E (D) ρR
24ε0 +E
2
(A) 6ql
ε0 (B) 6l2qε0
(C) zero (D) εq0 . 27. •• For a spherically symmetrical charge distribution,
electric field at a distance r from the center of sphere


22. •• A hollow metal sphere of radius R is uniformly charged. is E = kr7 r̃, where k is a constant. What will be the
The electric field due to the sphere at a distance r from volume charge density at a distance r from the center of
the centre sphere?
(A) decreases as r increases for r < R and for r > R (A) ρ = 9kε0 rr (B) ρ = 5kε0 r3
(B) increases as r increases for r < R and for r > R (C) ρ = 3kε0 r4
(D) ρ = 9kε0 r0
(C) zero as r increases for r < R, decreases as r in-
creases for r > R 28. •• A long string with a charge of λ per unit length passes
(D) zero as r increases for r < R, increases as r in- through an imaginary cube of edge a. The maximum
creases for r > R possible flux of the electric field through the cube may
23. •• Two parallel infinite line charges with linear charge be- √

densities +λC/m and −λC/m are placed at a distance (A) λa


εo (B) 2λa
2
√εo
of 2R in free space. What is the electric field midway (C) 6λa
εo (D) ε3λa
o
between the two line charges?
(A) 2πελ0 R N/C (B) zero (A) q /ε0 (B) zero
(C) 2q /ε0 (D) 8πr2 εq0
(C) πε2λ0 R N/C (D) πελ0 R N/C

24. ••The electric field at a distance 3R 29. •• An electric dipole is placed inside a con-
2 from the centre of a
charged conducting spherical shell of radius R is E. The ducting shell. Mark the correct statement(s)
electric field at a distance R2 from the centre of the sphere (A) the flux of the electric field through the shell is
is zero
(A) zero (B) E (B) the electric field is zero at every point on the shell
(C) E2 (D) E3 (C) the electric field is not zero anywhere any where
on the shell
25. •• A hollow insulated conduction sphere is given a (D) the electric field is zero on a circle on the shell.
positive charge of 10µC. What will be the electric field at
the centre of the sphere if its radius is 2 meter?
(A) 20 µCm−2 (B) 5 µCm−2
(C) zero (D) 8 mCm−2 Answer Keys and Solutions

26. •• Figure1.247 shows a uniformly charged hemisphere of 1. (a) With δ very small, all points on the hemisphere are
radius R. It has volume charge density ρ. If the electric nearly at a distance R from the charge [Fig.1.248], so the
field at a point 2R distance above its centre is E then R2 radially out-
field everywhere on the curved surface is kQ
what is the electric field at the point which is 2R below ward (normal to the surface). Therefore, the flux is this
its center? field strength times the area of half a sphere:
108 CHAPTER 1. ELECTRIC CHARGE AND FIELD

Q 1 2 dA
d 0
dA E
R Q=0 E
Q = 90° Q

4
Figure 1.248 3 dA E
E
Q Q = 180°
Q
Copyright 2017 Cengage Learning. All Righ
Q dA Q = 90°

→ − →
Z
Φcurved = E · d A = Elocal Ahemisphere
Q = 90°
dA
  
Q 1 1 +Q
Q = k 2 4πR2 = Q(2π) = Q
R 2 4π0 20 5 Q = 90° 6
Q Q E
5
(b) The closed surface encloses zero charge so Gauss’s E dA
law gives- Copyright 2017 Cengage Learning. All Rights Reserved. May not be copied, scanned, or duplicated, in w
Figure 1.249
−Q
Φcurved + Φflat = 0 or Φflat = −Φcurved =
20

2. (a) Note: We can easily determine whether the charge is 00-203



→ − →
I
positive, negative, or zero by looking at the electric field ΦE = E · dA
lines (Fig. 1.237). Because the electric field lines pass

→ − → −
→ −

Z Z
completely through the Gaussian surfaces, the net charge = 0 + E · dA + 0 + E · dA + 0 + 0
inside each surface is zero. 2 4
By Using Gauss’s Law Although we already know −
→ − → −
→ − →
Z Z
= E · dA + E · dA
the results, we are asked to use integration in order 2 4
to practice using Gauss’s law with different Gaussian
surfaces. −
→ − → −
→ − →
Z Z
A closed box is made up of six sides. According to ⇒ ΦE = E · dA + E · dA
2 4
Gauss’s law, we must integrate over the entire closed
box, so we break up the integral into six pieces, one for
Write each dot product in terms of the magnitude of the
each side of the box. The six subscripts on the inte-
vectors and the angle between them.
grals correspond to the six sides of the box (Fig. 1.237(a)).
Z Z

→ − →
I
ΦE = E · dA ΦE = EdA cos θ + EdA cos θ
Z2 Z4

→ − → −
→ − → −
→ − →
Z Z Z
= E · dA + E · dA + E · dA ΦE = EdA cos 0 + EdA cos 180◦
1 2 3 2 4
Z Z

→ − → −
→ − → −
→ − →
Z Z Z
+ E · dA + E · dA + E · dA ΦE = EdA + −EdA
4 5 6 2 4

The electric field is uniform, so as we integrate over dA, E


To calculate the six dot products, we need to know the

→ −→ is a constant that we can pull outside the integrals.
angle θ between E and d A for each side. A separate
drawing of each side is helpful (Fig. 1.249). The area Z Z
vector dA ~ for each side is perpendicular to that side and ΦE = E dA − E dA
2 4
points outward.
The dot product is zero whenever the angle θ between The integrals are identical: Each equals the area A of one

→ −

E and d A is 90◦ , so the dot product is zero for sides square side.
1, 3, 5, and 6 . That means these four integrals are zero, ΦE = EA − EA = 0 The net electric flux is
and we are left with just two integrals. zero, and according to Gauss’s law, that means the charge
1.24. APPLICATIONS OF GAUSS’S LAW 109

enclosed by the Gaussian surface is zero. Z Z


qin ΦE = EdA cos 180◦ + EdA cos 0
ΦE = =0 L R
ε0 Z Z
⇒ qin =0 ΦE = − EdA + EdA
L R

Note: This is exactly what we predicted because the num- As for the box, the electric field is constant and the
ber of electric field lines entering the box is equal to the integrals are identical. This time, each is the area of the
number of electric field lines leaving the box. end cap.
(b) The closed cylinder is made up of three surfaces - the
left cap, the right cap, and the curved part. These are ΦE = −EA + EA = 0
labeled L, R, and CP (Fig. 1.250). Break the integral up The electric flux is zero, so the charge inside the Gaussian
into three pieces, one for each surface in Fig.1.250. surface is zero.
qin
L ΦE = = 0 qin = 0
ε0
dA E Note: We find the same result whether we use a Gaussian
θ = 180°
cylinder or a box. The process is similar, but there is
slightly less work when we use the cylinder because
the Gaussian cylinder is made up of just three surfaces
(a) instead of the six surfaces of a box. Because we are free to
choose a convenient Gaussian surface, it may be helpful
dA to choose a closed cylinder instead of a box when that is
possible.
E

θ = 90° 3. This is similar to last problem (Fig.1.237 (b)), except the


electric field is not uniform. Because more electric field
lines emerge from the Gaussian surface than enter it, we
expect the net charge inside to be positive.
CP As like last problem, the net flux associated with the
Copyright 2017 Cengage Learning. All Rights Reserved. May not be copied, scanned, or duplicated, in whole or in part. WCN 02-2
cylinder is-
R
E Z Z
θ=0 ΦE = −EL dA + ER dA
L R
dA
The electric field is constant over each end cap, so it can be
pulled outside the integrals. The magnitude of the electric
Figure 1.250 field depends on the position, and the subscripts refer to
the magnitudes in Figure 25.16.
Z Z

→ − → ΦE = −EL dA + ER dA
I

Z3
Φ =
ning. All Rights Reserved. E
E · dA
May not be copied, scanned, or duplicated, in whole or in part. WCN 02-2 L R


→ − → −
→ − → −
→ − → The integrals are identical, equaling the area A of each
Z Z
ΦE = E · dA + E · dA + E · dA
L CP R
end cap.

→ −

Figure 1.250 shows the angle θ between E and d A for ΦE = −EL A + ER A = A (ER − EL )
each surface.
According to Gauss’s law, the electric flux is proportional
The area vector points outward for all small pieces of the
to the charge inside.
curved part, while the electric field points to the right.
00-203
Therefore, the angle θ = 90◦ for the curved part and the qencl
ΦE = A (ER − EL ) =
corresponding dot product and integral are zero, leaving ε0
two integrals. The surface charge density σ is charge per unit area.

→ − → −
→ − → qencl
Z Z
ΦE = E · dA + E · dA σ= = ε0 (ER − EL )
L R A
σ = 8.85 × 10−12 C2 /N · m2 (81.8 N/C − 40.9 N/C)

Write each dot product in terms of the magnitude of the
vectors and the angle between the vectors. σ = 3.62 × 10−10 C/m2
110 CHAPTER 1. ELECTRIC CHARGE AND FIELD

Note: As expected, the charge inside the Gaussian


surface is positive.

4. Required Gaussian surfaces are shown in


Fig.1.251 For any closed surface that encloses a total

(a) (b)

(a)

(b)
(c) (d)

Figure 1.252

through the surface is ΦE = Qencl /ε0 . We can write


three equations from the three closed surfaces in the figure:
q q1 + q3 3q q1 + q2
(c) ΦA = − = ⇒ q1 + q3 = −q ΦB = = ⇒ q1 + q
0 0 0 0
−2q q2 + q3
ΦC = = ⇒ q2 + q3 = −2q
0 0
Subtracting third equation from the first gives

q1 − q2 = +q

Adding second equation to this equation,


2q1 = +4qq1 = 2q
(d)
That is, q1 = +2q, q2 = +q, and q3 = −3q.
Figure 1.251

charge Qencl , the net electric flux through the closed 7. Point charge Q induces charge on conductor as shown in
surface is Φe = Qencl /0 . figure 1.241. Net charge enclosed by closed surface is neg-
ative so flux is negative.

5. The required Gaussian surfaces are shown in Fig.1.252 8. Electric field inside a cylindrical geometry is given by-
For any closed surface that encloses a total charge 2kλr
Qencl , the net electric flux through the closed surface is E=
R2
Φe = Qencl /0 .
For r = 2,
R
we have-

6. Please refer to Figure 1.240. For any closed surface


 
2kλr 2kλ R λ
that encloses a total charge Qencl , the net electric flux E= = 2 =
R2 R 2 4π ∈0 R
1.24. APPLICATIONS OF GAUSS’S LAW 111

9. (B) In Gauss’s law, ~ ·−


→ −

ε0 , E on the LHS, the
qencl
R
E ds =
~ will have a contribution from all
electric field vector, E,
charges while qencl on the RHS will have a contribution
from enclosed charges q2 and q4 only.

G
10. Please refer to Figure 1.243. For any closed surface that
G G G
encloses a total charge Qencl , the net electric flux through
the closed surface is Φe = Qin /0 . For the closed surface
of the torus, Qin includes only the −1nC charge. Thus, Figure 1.253
the net flux through the torus is due only to this charge:

−1 × 10−9 C of side length 2a and having q at its geometric center


Φe = = −0.11kNm2 /C
8.85 × 10−12 C2 /Nm2 (Fig.1.253). This large cube is our Gaussian surface. We
This is inward flux. can apply Gauss’s law to get the electric flux through any
face of this surface and then use symmetry to get the flux
11. Please refer to Figure 1.244. For any closed surface that through any face of our original cube of side length a.
encloses a total charge Qin , the net electric flux through From Gauss’s law, we know that the flux through the
the closed surface is Φe = Qin /0 . The cylinder encloses large cube enclosing a point charge q is q/ε0 Because the
the +1nC charge only as both the +100nC and the point charge q is at the geometric center, symmetry tells
−100nC charges are outside the cylinder. Thus, us that the electric flux is the same through all six faces
of the large cube. As Figure 1.253 shows, each face of
1 × 10−9 C the large cube is composed of four squares, each of side
ΦE = = 0.11kNm2 /C
8.85 × 10−12 C2 /Nm2 length a: One face of the large cube contains four faces
This is outward flux. of our original cube. Thus, if we apply Gauss’s law and
symmetry, the flux through each of these smaller faces
12. For any closed surface enclosing a total charge Qencl , the must be-
net electric flux through the surface is 1 q q
ΦE = = . This result gives us the flux
(6)(4) ε0 240
Qencl through faces 2, 3 and 5 of our original cube. The fact
ΦE = that the electric field is parallel to faces 1, 4, and 6 of the
0
⇒ Qencl = 0 ΦE original cube tells us that the flux through faces 1, 4, and
6 is still zero.
= 8.85 × 10−12 C2 /Nm2 −1000Nm2 /C
 

= −8.85 nC
15. (B) Eight identical cubes are required so that the given
13. For any closed surface that encloses a total charge Qin , charge q appears at the center of the bigger cube. Thus,
the net electric flux through the surface is the electric flux passing through the given cube is-

Qencl
ΦE =
0
55.3 × 106 −1.60 × 10−19 C
 
=
8.85 × 10−12 C2 /Nm2
= −1.00 N m2 /C

14. APPROACH our given information is that the cube


side length is a and the cube has a point charge q at one
corner (Fig.1.245). Our task here is to use Gauss’s law,
H−→ − →
E .d A = Qεencl
0
, to solve the problem of how much elec-
tric flux passes through each face of the cube. We know
how to calculate the electric field surrounding the point Figure 1.254
charge, and we know the locations and orientations of the
faces. We also know that a cube possesses symmetries  
1 q q
that we can exploit to simplify the problem. Φ= =
SOLUTION Because the point charge q is at the 8 ε0 8ε0
corner of the cube, the cube does not enclose it and so
16. (C) : According to Gauss’s law
cannot be our Gaussian surface. However, we can enclose
the point charge q by attaching seven additional cubes, Qencl
each of side length a, to our original cube, creating a cube φE =
ε0
112 CHAPTER 1. ELECTRIC CHARGE AND FIELD

If the radius of the Gaussian surface is doubled, the


outward electric flux will remain the same. This is
because electric flux depends only on the charge enclosed
by the surface.

17. (D): Let ΦA , ΦB and φC are the electric flux linked with
G G G
surface is A, B and C. According to Gauss theorem,
q
ΦA + Φ B + Φ C = Figure 1.256
ε0 ?
Since, ΦA = ΦC , therefore 2ΦA + ΦB = εq0
or 2ΦA = εq0 − ΦB or, 2ΦA = εq0 − Φ ( Given: ΦB = Φ) 23. (D): Electric field due to an infinite line charge, [Fig.1.256]
λ
Therefore, E=

1 q
 2πε0 r
ΦA = −Φ Net electric field at mid-point O,
2 ε0
~0 = E
E ~1 + E
~2
18. (B): The total flux through the cube Φtotal = ε0 ,
q
As, E1 = E2 = λ
∴ E0 = 2E1 = λ
NC−1
therefore, the™™™
electric flux through any face 2πε0 R πε0 R

24. (A): Electric field inside the charged spherical shell is


q 4πq zero as there is no charge inside it.
Φface = =
6ε0 6 (4πε0 )
25. (C): Field inside a conducting sphere = 0.
19. (A): For complete cube, Φ = εQ0 × 10−6 .
For each face, Φ = 16 εQ0 × 10−6
26. APPROACH Apply the principle of superposition of
electric fields.
20. (B) As at a corner, 8 cubes can be placed symmetrically, (B)SOLUTION Electric field due to a uniformly

Figure 1.257

charged sphere, at any external point at distance r from


it’s center is given by-

Figure 1.255 1 Q 1 ρ(4πR3 )/3


E1 = 2
=
4πε0 r 4πε0 r2
flux linked with each cube (due to a charge Q at the cor- Since, r = 2R, therefore,
ner) will be 8εQ
0
. Now for the faces passing through the ρR
edge A, electric field E at a face will be parallel to area E1 =
12ε0
of face and so flux for these three faces will be zero. Now
as the cube has six faces and flux linked with three faces The given electric field at a distance 2R, above the
(through A ) is zero, so flux linked with remaining three hemisphere is-
faces will be 8ε
Q
0
. Hence, electric flux passed through all
the six faces of the cube is 8ε
Q
. E2 = E
0
Now, for a point at distance r = 2R, below the hemispher-
21. (D): Electric flux emerging from the cube does not depend ical section, we model the hemisphere as a super position
on size of cube. Total flux = εq0 of a sphere of charge density +σ and an upper hemisphere
of charge density −σ.
22. (C): In a uniformly charged hollow conducting sphere, So, the net electric field at lower point at distance 2R,
~ =0
(i) For r < R, E from the center of the hemisphere’s center
~ = 1 Q2 r̂; E
(ii) For r > R, E ~ decreases ρR
4πε0 |~
r |
Enet = E1 − E2 = −E
12ε0
1.25. CONDUCTORS IN ELECTROSTATIC EQUILIBRIUM 113

27. (A) By using Gauss law[Fig.1.258]- intensities of the fields of all individual charged particles of
H−→ − →
E · d S = qencl the substance, viz. electrons and nuclei. The solution of this
0
 R ρ(4πr2 dr) problem is obviously not feasible. In any case, the result
⇒ (E) 4πr2 = 0 would be so complicated that it would be impossible to use
(Note: We advise students to check dimensionally that it. Moreover, the knowledge of this field is not required for
ρ ∝ r6 ) the solution of macroscopic problems. In many cases it is
sufficient to have a simpler and rougher description which we
ρ 4πr2 dr
R 
7 2 shall be using henceforth.
 
kr 4πr =
0 Under the electric field E~ in a substance (which is called the
Z
⇒ kε0 r9 = ρr2 dr macroscopic field) we shall understand the microscopic field
averaged over space (in this case time averaging is worthless).
This averaging is performed over what is called a physically
infinitesimal volume, viz. the volume containing a large
number of atoms and having the dimensions that are many
times smaller than the distances over which the macroscopic
field noticeably changes. The averaging over such volumes
smoothens all irregular and rapidly varying fluctuations of
the microscopic field over the distances of the order of atomic
ones, but retains smooth variations of the macroscopic field
over macroscopic distances.
Figure 1.258 Thus, the field in the substance is

E = Emacro = hEmicro i . (1.145)


28. (D) The maximum
√ length of the string which can fit into
the cube is 3a, equal to√its body diagonal. The total
charge inside the cube is 3aλ, and hence the total flux The Influence of a Substance on a Field.
through the cube is- If any substance is introduced into an electric field, the
√ positive and negative charges (nuclei and electrons)8 are
Qencl 3aλ displaced, which in turn leads to a partial separation of these
Φ= = charges. In certain regions of the substance, uncompensated
ε0 εo
. charges of different signs appear. This phenomenon is called
the electrostatic induction, while the charges appearing as a
29. (C) Electric flux passes as due to +q charge of dipole result of separation are called induced charges.
+q Induced charges create an additional electric field which in
= combination with the initial (external) field forms the resul-
ε 0
tant field. Knowing the external field and the distribution
Electric flux passes due to ˘q charge of dipole of induced charges, we can forget about the presence of the
−q substance itself while calculating the resultant field, since the
= role of the substance has already been taken into account with
ε 0
the help of induced charges.
Therefore, net flux passes due to both the charges Thus, the resultant field in the presence of a substance is
q q determined simply as the superposition of the external field
= − =0 and the field of induced charges.
ε0 ε0

30. (A) Inside the surface the total charge is zero. So flux
must be zero. 1.25.2 Fields Inside and Outside a Conductor
Inside a Conductor E = 0
1.25 Conductors in Electrostatic Let us place a metallic conductor into an external electrostatic
field or impart a certain charge to it. In both cases, the
Equilibrium electric field will act on all the charges of the conductor, and
as a result all the negative charges (free electrons) will be
1.25.1 Field in a Substance
displaced in the direction against the field. This displacement
Micro- and Macroscopic Fields. (current) will continue until (this practically takes a small
The real electric field in any substance (which is called the fraction of a second) a certain charge distribution sets in, at
microscopic field) varies abruptly both in space and in time. It which the electric field at all the points inside the conductor
is different at different points of atoms and in the interstices. 8 In conductors, only free electron can move not nuclei. So there will
In order to find the intensity E ~ of a real field at a certain
be a relative displacement between nuclei and electrons due to motion of
point at a given instant, we should vectorially sum up the free electrons only.
114 CHAPTER 1. ELECTRIC CHARGE AND FIELD

vanishes. Thus, in the static case the electric field inside a - +


conductor is absent (E = 0). - +
Further, since E = 0 everywhere in the conductor, the density Before E net 0 After Eext - E net 0 + Eint
of excess (uncompensated) charges inside the conductor is - +
also equal to zero at all points (ρ = 0). This can be easily - +
explained with the help of the Gauss theorem. Indeed, since
~ ext creates an internal
inside the conductor E = 0, the flux of E through any closed Figure 1.259: An external electric field E
~ int in the conductor such that the net electric field
electric field E
surface inside the conductor is also equal to zero. And this ~ net is zero
E
means that there are no excess charges inside the conductor.
Excess charges appear only on the conductor surface with
a certain density σ which is generally different for different
points of the surface. It should be noted that the excess surface
charge is located in a very thin surface layer (whose thickness This result doesn’t depend on the size or shape of the conduc-
amounts to one or two interatomic distances). When there is tor, the magnitude or direction of the applied field, or even
no net motion of electrons within the conductor, the conduc- the nature of the material as long as it’s a conductor. This
tor is said to be in electrostatic equilibrium. In electrostatic is a macroscopic view; it considers only average fields within
equilibrium, all conductors show following four properties: the material. At the atomic and molecular level, there are
still strong electric fields near individual electrons and positive
1. The electric field is zero everywhere inside the conductor. ions. But the average field, taken over larger distances, is zero
inside a conductor in electrostatic equilibrium.
2. If an isolated conductor carries a charge, the charge resides
on its surface. Explanation of Property 2. Figure 1.260a shows a
3. The electric field just outside a charged conductor is per- cross-sectional view of a conductor with a Gaussian surface
pendicular to the surface of the conductor and has a mag- drawn just below the material surface. In equilibrium there’s
nitude σ/ε0 , where σ is the surface charge density at that no electric field inside the conductor, and thus the field is zero
point. everywhere on the Gaussian surface. Therefore, from Gauss’s
law, we have-
4. On an irregularly shaped conductor, the surface charge
density is greatest at locations where the radius of curva-
ture of the surface is smallest.
Explanation of above Properties on the basis of −
→ − → Qencl
I
Gauss’s Law: Here, we verify only the first three proper- ΦE = E · dA =
ε0
ties for the conductors. The fourth property is presented here I

→ Qencl
so that we have a complete list of properties for conductors in ⇒ 0d A =
ε0
electrostatic equilibrium, but cannot be verified until the next
Qencl
Chapter "Electric Potential". ⇒0=
ε0
Property 1. We can elaborate more about the first property
by considering the conducting slab in electrostatic equilibrium ⇒ Qencl = 0
on the left of Fig.1.259, where the free electrons are uniformly
distributed throughout the slab, i.e. E ~ int = 0. When we place
~
the slab in an external electric field Eext as in the right part of
Fig.1.259, the free electrons respond to the electric force −eE ~ Thus, net electric flux Φ, through the Gaussian surface and
by moving-in the direction, opposite the field. In Fig.1.259, the hence net charge enclosed within the Gaussian surface is also
direction of applied external electric field is towards right side, zero.
therefore electrons move to the left. In time, more negative This is true no matter where the Gaussian surface is as long
and positive charges accumulate on the left and right surfaces, as it’s inside the conductor. We can move it arbitrarily close
respectively. The resulting charge separation gives rise to an to the conductor surface and it still encloses no net charge. If
~ int within the conductor that’s oppo-
increasing electric field E there is a net charge qC (say) on the conductor, it lies outside
site to the applied field. As more charge moves, this internal the Gaussian surface, and therefore we conclude: If a conductor
field becomes stronger and after awhile, E ~ int will compensate in electrostatic equilibrium carries a net charge, that charge
~
Eext resulting in a zero net electric field inside the conduc- must reside on the conductor surface.
~ net = E
tor, i.e. E ~ ext − E
~ int = 0. The time to reach this new A Cavity Inside the Conductor: Fig.1.260(b) shows a
electrostatic equilibrium is of the order 10−6 s. cross-sectional view of same charged conductor with a cavity
Therefore, The electric field is zero inside a conductor in that is totally within the conductor. Can there be charge on
electrostatic equilibrium. this interior surface? To find out, we place a Gaussian surface
It could not be otherwise: Since a conductor contains free around the cavity, infinitesimally close but entirely within the
charges, the presence of any internal electric field would re- conductor. Because E ~ = 0 inside the conductor, there can be
sult in bulk charge motion, and we wouldn’t have equilibrium. no flux through this new Gaussian surface. Therefore, from
1.25. CONDUCTORS IN ELECTROSTATIC EQUILIBRIUM 115

sumes the entire conductor, leaving only the charges. The elec-
tric field pattern would not change at all; it would remain zero
inside the thin shell of charge and would remain unchanged for
all external points. The electric field is set up by the charges
and not by the conductor. The conductor simply provides a
pathway so that the charges can change their positions.
Now, Suppose we place a small body with a charge q inside
a cavity within a conductor (Fig.1.260c). The conductor is


uncharged and is insulated from the charge q. Again E = 0
everywhere on the Gaussian surface. If we assume that the
(a)
charge on the cavity
I wall is x, then by Gausses law. we have-

→ − → Qencl
ΦE = E · dA =
ε0

→ Qencl
I
⇒ 0 dA =
ε0
Qencl
⇒ 0=
ε0
⇒ Qencl = 0
⇒ q + x = 0 ⇒ x = −q
So according to Gauss’s law there must be a charge −q dis-
tributed on the surface of the cavity, drawn there by the charge
(b) q inside the cavity. Let charge y appears on the outer surface
of conductor, then by charge conservation principle, we have

x + y = 0 ⇒ y = −x
⇒ y = −(−q) = q

Therefore, we conclude that the charge +q must appear on the


outer surface of the conductor.
If the conductor originally had a charge qC , then charge con-
servation principle gives-

x + y = qC ⇒ y = qC − x
(c) ⇒ y = qC − (−q) = qC + q
Figure 1.260: (a) A cross sectional view of a solid conductor of
i.e., the total charge on the outer surface must be qc + q after
arbitrary shape. The broken line represents a gaussian surface that
can be as close to the surface of the conductor as we wish (b) The the charge q is inserted into the cavity.
same conductor with an internal cavity (c) An isolated charge q Explanation of Property 3. There can’t be an electric field
placed in the cavity within a conductor in electrostatic equilibrium, but there may
be a field right at the conductor surface (Fig.1.261). Such a
field must be perpendicular to the surface; otherwise, charge
Gauss’ law, we have- would move along the surface in response to the field’s parallel
I

→ − → Qencl component, and we wouldn’t have equilibrium.
ΦE = E · dA =
ε0

→ Qencl
I
⇒ 0d A =
ε0
Qencl
⇒0=
ε0
⇒ Qencl = 0

So, we conclude that there is no net charge on the cavity walls;


all the excess charge remains on the outer surface of the con-
ductor, as in Fig.1.260b.
Suppose that, by some process, the excess charges could be
"frozen" into position on the conductor surface of Fig.1.260b,
perhaps by embedding them in a thin plastic coating, and sup- Figure 1.261: The electric field at the surface of a charged con-
pose that the conductor could then be removed completely. This ductor is perpendicular to the conductor surface.
is equivalent to enlarging the cavity of Fig.1.260b until it con-
116 CHAPTER 1. ELECTRIC CHARGE AND FIELD

The Field Near a Conductor Surface


Here we will show that the electric field intensity in the
immediate vicinity of the surface of a conductor is connected
with the local charge density at the conductor surface through
a simple relation. This relation can be established with the
help of the Gauss theorem.
Suppose that the region > of the conductor surface we are Figure 1.263
Outer
surface of A −
→ − → −

E = E patch + E 0 (1.147)
charged
E# = E −

conductor
where E patch is the electric field due to charge on the patch,
E# = 0 −
→0
+++++ + and E is the electric field due all other charges. Since by
++ Newton’s third law, the patch cannot exert a force on itself,


the force on the patch must come solely from E 0 . Assuming
Gaussian
the patch to be a flat surface, from Gauss’s law, the electric
surface
E = 0 field due to the patch is
A
(


→ 20 k̂ z>0
Figure 1.262: The field just outside a charged conductor is per- E = σ (1.148)
pendicular to the surface, and its perpendicular component E⊥ is − 20 k̂ z<0
equal to σ/0 . By superposition principle, the electric field above the conduct-
ing surface is
interested in borders on a vacuum. The field lines are normal
to the conductor surface. Hence for a closed surface we shall −

 

→ σ
E above = k̂ + E 0 (1.149)
take a small cylinder and arrange it as is shown in Fig.1.262. 20
Then the flux of E ~ through this surface will be equal only
Similarly, below the conducting surface, the electric field is
to the flux through the “outer” endface of the cylinder. (the
fluxes through the lateral surface and the inner endface are −

 
−→ σ
equal to zero). Thus we obtain E ~ ·A~ = E⊥ A = σ A/ε
~ 0 , where E below = − k̂ + E 0 (1.150)
20
~
E⊥ is the projection of vector E onto the outward normal −
→0
(with respect to the conductor), A is the cross-sectional area Notice that E is continuous across the boundary. This is due
of the cylinder and σ is the local surface charge density of the to the fact that if the patch were removed, the field in the re-
conductor. Cancelling both sides of this expression by A, we maining “hole” exhibits no discontinuity. Using the two equa-
get tions above, we find

σ −
→ 1 − → −
→  − →
E⊥ = (1.146) E0 = E above + E below = E avg (1.151)
ε0 2
−→   −

If σ > 0, then E⊥ > 0, i.e. vector E ~ is directed from the But, for a conductor, E above = σ0 k̂ and E below = 0, there-
conductor surface (coincides in direction with the outward fore, above equation gives-
normal). If σ < 0, then E⊥ < 0, and vector E ~ is directed  
towards the conductor surface. −
→ 1 σ σ
E avg = k̂ + 0 = k̂ (1.152)
Relation (1.146) may lead to the erroneous conclusion that 2  0 2 0

the field E~ in the vicinity of a conductor depends only on Thus, the force acting on the patch is
the local charge density σ. This is not so. The intensity
E~ is determined by all the charges of the system under −→ −
→ σ σ2 A
F = q E avg = (σA) k̂ = k̂ (1.153)
consideration as well as the value of σ itself. 2 20
where A is the area of the patch. This is precisely the force
needed to drive the charges on the surface of a conductor to
1.25.3 Mechanical Pressure (or Surface Den- an equilibrium state where the electric field just outside the
conductor takes on the value σ / and vanishes inside. Note
sity of Force) on the Surface of a that irrespective of the sign of σ, 0the force tends to pull the
Charged Conductor patch into the field.
Let us consider the case when a charged region of the surface Using the above result, we may define electrostatic pressure on
of a conductor borders, is in a vacuum. It’s surface charge the patch as
density is σ. A small patch of charge on a conducting surface,
 2
is shown in Figure 1.263. F σ2 1 σ 1
Total electric field anywhere outside the surface as- P = = =  0 = 0 E 2 (1.154)
A 20 2 0 2
1.25. CONDUCTORS IN ELECTROSTATIC EQUILIBRIUM 117

where E is the magnitude of the field just above the patch.


The pressure is being transmitted via the electric field.
This electrostatic pressure on the patch is also called the
surface density of force on the surface of charged conductor.

EXAMPLE 80. An uncharged spherical conductor centered


at the origin has a cavity of some weird shape carved out of it
(Fig.1.264). Somewhere within the cavity is a charge q. What
is the field outside the sphere?

(a)

Figure 1.264

SOLUTION At first glance, it would appear that the answer


depends on the shape of the cavity and on the placement of
the charge. But that’s wrong: The answer is (b)


→ 1 q Figure 1.265: The electric field can be excluded from a region of
E = r̂
4π0 r2 space by surrounding it with a conducting box.

regardless. The conductor conceals from us all information


concerning the nature of the cavity, revealing only the total wire mesh provides sufficient screening for all but the most sen-
charge it contains. How can this be? The charge +q induces sitive applications. But in such case, the exterior field is now
an opposite charge −q on the wall of the cavity, which very complicated. Such shielding enclosures are called Faraday
distributes itself in such a way that its field cancels that of q, cages, after Michael Faraday. In 1816, Faraday used them to
for all points exterior to the cavity. Since the conductor carries demonstrate that no charge was present inside a conducting
no net charge, this leaves +q to distribute itself uniformly shell.
over the surface of the sphere. (It is uniform because the Applications: We can use these ideas to explain how an-
asymmetrical influence of the point charge +q is negated by
that of the induced charge −q on the inner surface.) For
points outside the sphere, then, the only thing that survives
is the field of the leftover +q uniformly distributed over the
outer surface.

1.25.4 Faraday Cage Protection


We have already seen that the electric field is zero inside any
cavity within a conductor unless there is a charge in the cavity.
This conclusion has an important practical application. For ex-
ample, suppose we need to exclude the electric field from the
region in Fig.1.265(a) enclosed within dashed lines. We can do Figure 1.266: Electrostatic shielding can protect you from a dan-
so by surrounding this region with the neutral conducting box gerous electric discharge.
of Fig.1.265(b).
This region of space is now a cavity inside a conductor, thus tistatic bags work and why your cell phone doesn’t work in a
the interior electric field is zero. The use of a conducting box to closed elevator. The antistatic bag is coated with a conductor,
exclude electric fields from a region of space is called screen- and sensitive equipment is sealed inside. If the bag comes in
ing. Solid metal walls are ideal, but in practice wire screen or contact with a charged object, the outside of the bag becomes
118 CHAPTER 1. ELECTRIC CHARGE AND FIELD

charged but the inside remains uncharged, and the electric field carry an induced charge +Q. (d) In this region, the Gaussian
inside the bag is zero. The antistatic bag is a Faraday cage. sphere S4 of Fig. 1.267 satisfies the condition r > RH2 . Because
In order for your cell phone to work, it must be able to receive qin = 2Q− Q = Q inside this surface and because E ~ 4 · dA
~=
signals that consist of changing electric and magnetic fields. If E4 4πr2 , we can use Gauss’s law to find:
your cell phone is inside a closed elevator, the elevator acts as
a Faraday cage. The electric field inside the elevator is zero, so 1 Q Q
E4 = =k 2 (r > R2 )
your cell phone cannot receive signals. The same physics tells 4π◦ r2 r
you that one of the safest places to be in a lightning storm is
Figure 1.268 shows a graphical representation of the variation
inside a car; if the car is struck by lightning, the charge tends
to remain on the metal skin of the vehicle, and little or no E
electric field is produced inside the passenger compartment.
Q 2Q
E
EXAMPLE 81. A conducting sphere of radius R carries a
net positive charge 2Q. A conducting spherical shell of inner
radius R1 (R1 > R) and outer radius R2 carries a net negative C
charge −Q. This shell is concentric with the conducting sphere.
Find the magnitude of the electric field at a distance r away −2Q
from the common center when: (a) r < R, (b) R < r < R1 ,
(c) R1 < r < R2 , and (d) r > R2 . E2 k
2Q
E r2
SOLUTION The charge distributions under consideration
Q
are characterized by being spherically symmetrical around the E4 k 2
r
common center c. This suggests that a spherical Gaussian sur- r
face of radius r is to be constructed in each case such as S1 , S2 , 0
S3 , and S4 that are displayed in Fig. 1.267. In addition, we R R1 R 2
use the fact that the electric field inside a conductor is zero
Figure 1.268
and all the excess charge will lie entirely on the outer surface
of the isolated conductor.
of the electric field E with r. In addition, the figure shows the
(a) In this region the Gaussian sphere S1 of Fig. 1.267 sat-
final distribution of the charge on the two conductors.
S4
−Q S3
EXAMPLE 82. Figure 1.269 shows a cross-sectional view of
2Q S2
R2 a thick spherical conductor. The conductor is neutral, and a
R1
S1 R small charged sphere (q = +29.5µC) hangs from an insulating
thread. The sphere is not in the center of the conductor;
C
instead, it is closer to the left side as shown.
(a) Find the charge qwall on the wall of the cavity and the

Figure 1.267

isfies the condition r < R. Because there is no charge inside


the conductor in this region, i.e. qin = 0; then, E1 = 0. (b) In
this region the Gaussian sphere S2 of Fig. 1.267 satisfies the
condition R < H r < R1 . Because 2qin = 2Q inside this surface
and because E ~ 2 · dA~ = E2 4πr , we can use Gauss’s law to
find:
1 2Q 2Q
E2 = 2
=k 2 (R < r < R1 )
4π◦ r r
(c) In this region, the Gaussian sphere S3 of Fig. 1.267 satisfies Figure 1.269: Cross-sectional view
the condition R1 < r < R2 . Because the electric field inside
an equilibrium conductor is zero, i.e. E3 = 0; then, based on charge qout on the outer surface of the conductor. Start by
Gauss’s law, the net charge qin must be zero. From this argu- sketching the electric field for all regions-inside the cavity,
ment, we find that an induced charge −2Q must be established inside the body of the conductor, and outside the conductor.
on the inner surface of the shell to cancel the charge +2Q on (b) If the radius of the conductor is R = 14.4 cm, what is the
the solid sphere. In addition, because the net charge on the magnitude of the electric field just outside the conductor?
whole shell is −Q, we conclude that its outer surface must
1.25. CONDUCTORS IN ELECTROSTATIC EQUILIBRIUM 119

(a) APPROACH The positive charge on the small sphere find the surface charge density so that we can find the electric
attracts electrons in the conductor. These electrons move close field.
to the walls of the cavity. If the positively charged sphere were SOLUTION The charge is uniformly distributed on a sphere
in the center of the cavity, the electrons would be uniformly of radius R. We divide the charge qout by the surface area of
distributed on the cavity wall. However, the electrons are more the sphere.
concentrated on the left side of the cavity because the positive
sphere is closer to the left side (Fig.1.270). The electric field qout qout 29.5 × 10−6 C
σ= = =
in the body of any conductor in electrostatic equilibrium is A 4πR2 4π (14.4 × 10−2 m)
2

zero. By choosing a Gaussian sphere that is concentric with the σ = 1.13 × 10−4 C/m2
conductor and embedded in it, we can determine the amount
of charge on the walls of the cavity. The magnitude of the electric field just outside a conductor is
SOLUTION The electric flux through the Gaussian surface given by-
σ
Uniform distribution E=
of charge on outside ε0
1.13 × 10−4 C/m2
+ E=
+ 8.85 × 10−12 C2 /N · m2
+ E
+ = 1.28 × 107 N/C
− − +
− Double Check the Result: From Figure 1.270, we see
+ −
E + − that the electric field outside the conductor is equivalent to
− the electric field produced by a charged particle located at
R = 14.4 cm
+ − the center of the objects. We can check our result by using
− + − the relationship for electric field derived from Coulomb’s law
+
− q = 29.5 mC (E = kQ/r2 ) to calculate the field at a distance of 14.4 cm
+ − − Less from such a fictitious particle with charge +29.5µC.
− + charge
− − here. kq
+ − E=
+ r2
+ E=0
8.99 × 109 N · m2 /C2 29.5 × 10−6 C
 
+ +
E= 2
More Gaussian (14.4 × 10−2 m)
chosen charge surface E = 1.28 × 107 N/C
onducting shell. here.
EXAMPLE 83. The Fig.1.271 shows a cross section of a
Figure 1.270 spherical metal shell of inner radius R. A point charge of
−5.0µC is located at a distance R/2 from the center of the
(Fig. 1.270) is zero because the electric field in the body of shell. If the shell is electrically neutral, what are the (induced)
the conductor is zero. Therefore, the net charge inside the charges on its inner and outer surfaces?
Gaussian sphere is zero, so the total charge on the inside wall
of the cavity is negative, equal in magnitude to the charge on
the small sphere inside.

→ − → qin
I
ΦE = E · dA = 0 =
ε0
qin = 0 = q + qwall
2 = −29.5µC
qwall = −q
Because the conductor is neutral, the charge on its surface
must be positive and equal to the charge on the cavity wall.
qout = −qwall
qout = −(−29.5µC) = +29.5µC
The positive charge qout on the outer surface of the conductor
is uniformly distributed, as excess charge always is on the Figure 1.271
surface of a spherical conductor. To see why this is so,
imagine that free electrons move toward the cavity wall, ANSWER Total induced charge on inner surface = 5.0µC.
leaving positively charged ions in place. It’s distribution is non-uniform. The total induce charge on
(b) APPROACH The electric field just outside any conduc- outer surface = −5.0µC. This charge will be distributed uni-
tor depends only on the surface charge density, so we need to formly.
120 CHAPTER 1. ELECTRIC CHARGE AND FIELD

1.26 Faraday’s Ice Pail Experiment


Testing Gauss’s Law Experimentally Let us consider a
historic experiment, shown in Fig.1.272. We mount a metal
a conducting container, such as pail (the “ice pail,” explaining
the name for the experiment) with a lid, on an insulating stand
so charge cannot flow to or from the ground. The container is
initially uncharged. Then we hang a charged metal ball from an
insulating thread, (Fig.1.272a), lower it into the pail, without
actually touching the sides, and put the lid on (Fig. 1.272b).
As the ball entered the pail, the needle on an electrometer (a)
attached to the outer surface of the pail was observed to deflect.
(An electrometer is a device used to measure charge.) The
needle deflected because the positively charged ball induced a
negative charge on the inner wall of the pail, which left an equal
positive charge on the outer wall. Next, we let the ball touch
the inner wall (Fig.1.272c). The surface of the ball becomes,
in effect, part of the cavity surface. The situation is now the
same as Fig. 1.260b; if Gauss’s law is correct, the net charge
on the cavity surface must be zero. Thus the ball must lose
all its charge. Finally, we pull the ball out. During these two
processes, we find that there is no change in deflection of needle (b)
of electrometer. It shows that the ball lost all its charge when
it came in contact of the pail.
This experiment was performed in the 19 th century by the
English scientist Michael Faraday, using a metal icepail with
a lid, and it is called Faraday’s icepail experiment. The result
confirms the validity of Gauss’s law and therefore of Coulomb’s
law.
EXAMPLE 84. Electric Field from Plane Plus Point
Charge: A very large (and thin) non-conducting plane slab
carries a uniform surface charge density σ (assume positively
charged). A single positive point charge, +Q, is located a dis- (c)
tance d above the plane. What is the electric field at a point
Figure 1.272: Ice pail experiment
between the point charge and the plane and at a distance b from
the plane?
APPROACH The situation is drawn in Fig.1.273. We
will use the superposition principle to solve this problem.
SOLUTION The electric field at point P due to the charged
plane is-
σ
Eσ =
2ε0
The direction of this electric field is perpendicular to the plane
and away from it (due to the positive charge), as shown in
Figure 1.273
The electric field at point P due to the point charge Q is-

~Q = k Q
E
(d − b)2 Figure 1.273: Situation of Example 20−9 with the electric field at
point P shown separately due to the non-conducting plane of charge
The direction of this electric field is away from positive point ~ σ and the point charge E
E ~Q
charge Q and directed perpendicularly towards the charged
plane.
The net electrical field at P is given by- The net electric field could be either upward or downward,
depending on the relative magnitudes of the point charge and
~ net = E
E ~σ + E~Q
the surface charge density.
Note: While the superposition technique can be powerful, you
 
σ Q
= −k k̂ have to be careful not to use it when it is not valid. If you have
2ε0 (d − b)2
1.26. FARADAY’S ICE PAIL EXPERIMENT 121

a conductor, the presence of another charged object will change ANSWER: By taking the electric field strength, E, outside
the distribution of charges on the conductor, so we could not the integral, it is implicitly assumed that the electric field
assume that the plane is still uniformly charged. strength is constant over the surface. That is not true in
this case. In conductors, the charges are free to move, and
EXAMPLE 85. Why is it normally not possible to use the more charges will be near the sharper ends. In this case, both
technique of superposition along with Gauss’s law when one of the charge distribution and the electric field strength are not
the objects is a conductor? (A) Conductors have zero interior constant.
electric fields.
(B) Charges on conductors are all on the outer surface.
(C) The electric fields on conductors (just outside) are perpen- EXAMPLE 87. Two conducting plates A and B are placed
dicular to the surface. parallel to each other. A is given a charge Q1 and B a charge
(D) Charges in conductors are free to move, which will often Q2 . Find the distribution of charges on the four surfaces.
destroy the symmetry needed for Gauss’s law.
SOLUTION Consider a Gaussian surface as shown in
ANSWER: (D) While the first three statements are true, Fig.1.275. Two faces of this closed surface lie completely
that is not why we can’t use the Gauss’s law plus superposition inside the conductor where the electric field is zero. The flux
technique. When we have two charged objects, and one of through these faces is, therefore, zero. The other parts of the
them is a conductor, the presence of the other object will closed surface which are outside the conductor are parallel to
distort the symmetry of the charge distribution and we can the electric field and hence the flux on these parts is also zero.
no longer make assumptions such as a constant-magnitude The total flux of the electric field through the closed surface
electric field on a surface. is, therefore, zero. So, from Gauss’s law, the total charge
inside this closed surface should be zero. The charge on the
inner surface of A should be equal and opposite to the inner
EXAMPLE 86. As shown in Figure1.274, we have a con- surface of B.
ducting ellipsoid of surface area A that carries a total positive The distribution should be like the one shown in figure1.276.
charge +Q. What is the electric field strength at the surface?
Find the mistake with this solution?
SOLUTION As shown in Figure 1.274, we draw a Gaussian
surface just barely outside the ellipsoid, so it has an area of
essentially A as well.
We apply Gauss’s law to solve for the electric field strength,

Figure 1.275

To find the value of q, consider the field at a point P inside the


plate A. Suppose, the surface area of each plate is A. Let σ1 ,
σ2 , are respective surface charge densities on the outer, inner
surfaces of the plate A and σ3 , σ4 are respectively the surface
charge densities on inner and outer surface of the plate B.
Now, the electric field at point P inside the plate A-

Figure 1.274: A Gaussian surface (grey) is just barely outside the


ellipsoid surface and of identical shape.

E, just outside the surface of the ellipsoid:

~ = qencl
I
E~ · dA
surface ε0
Figure 1.276
Q
I
E dA =
ε0
surface (i) due to the charge Q1 − q on the outer surface of plate A,
Q is given by-
EA =
ε0
Q σ1
E= E1 =
ε0 A 2ε0
Q1 −q
Since, σ1 = A , therefore,
Because it is positively charged, the electric field points
outward, perpendicular to the surface. Q1 − q
What error have we made in this solution? E1 = (downwards),
2Aε0
122 CHAPTER 1. ELECTRIC CHARGE AND FIELD

(ii) due to the charge +q on the inner surface of plate A,


σ2
E2 =
2ε0
Since, σ2 = A,
q
therefore,
q
E2 = (upwards),
2Aε0
Figure 1.277
(iii) Due to the charge −q on the inner surface of plate B,
σ3
E3 = EXAMPLE 88. Figure 1.278 shows three large metallic
2ε0
plates with charges −Q, 3Q and Q respectively. Determine
Since, σ3 = A,
q
therefore,
the final charges on all the surfaces.
q
E3 = (downwards),
2Aε0
(iv) due to the charge Q2 +q on the outer surface of the plate B
σ4
E4 =
2ε0
Q2 +q
Since, σ3 = A , therefore,
Q2 + q
E4 = (upward),
2Aε0
If we consider, upward direction as positive direction of electric
field, then, net electric field at point P due to all the four Figure 1.278
charged surfaces is (in the downward direction)
APPROACH In Fig.1.279 we have numbered each surface
E = −E1 + E2 − E3 + E4
area of the given plates. Now, assume that the charge on
Q1 − q q q Q2 + q surface 2 is x. Following conservation of charge, we see that
=− + − +
2A0 2A0 2A0 2A0 surfaces 1 has charge (−Q − x) i.e., −(Q + x). Since, in
Q1 2q Q2 electrostatic equilibrium, the net electric field inside a metal
=− + +
2A0 2A0 2A0 plate is always zero, therefore, the net electric field at P
Q2 − Q1 2q should be zero, i.e.,
or E= +
2A0 2A0
EP = 0
As the point P is inside the conductor, this field should be
zero. Hence, APPROACH Resultant electric field at point P inside the

Q2 − Q1 2q
E= + =0
2A0 2A0
Q1 − Q2
⇒ q= (1.155)
2
It is the electric charge on the inner surface of plate A (which
has the net electric charge Q1 ). The electric charge on the
inner surface of the plate B, is −q, i.e.,
Figure 1.279
 
Q1 − Q2
−q = −
2
left most plate -
Thus, the charge on inner surface of the plate A-
Q+x x 3Q Q
Q1 + Q2 + + + =0
Q1 − q = (1.156) 2Aε0 2Aε0 2Aε0 2Aε0
2
(Since electric fields due to all surface charges are directed in
and the charge on the outer surface of the plate B - same direction (left)

Q1 + Q2 5Q
Q2 + q = (1.157) ⇒2x + 5Q = 0 ⇒x=−
2 2
Final charge distribution is shown in Fig.1.277 Therefore, charge on surface 1
1.26. FARADAY’S ICE PAIL EXPERIMENT 123

Figure 1.280
Figure 1.282
 
5Q 3Q
= − (Q + x) = − Q − =+ SOLUTION Net electric field at P,
2 2
Since, the front surfaces of two parallel metal plates always x Q−x
acquire equal and opposite electric charges, therefore, electric − + E =0
2A0 2A0
charge on surface 3 will be −x, i.e., + 5Q
2 . 2x + 2A0 E − Q = 0
By charge conservation, the electric charge on surface 4
5Q Q Q
= 3Q − = ⇒ x= − A0 E
2 2 2
.
2 − A0 E and other side
So charge on one side is Q
Therefore, the electric charge on the opposite front surface 5
will be − 2 .
Q 
Q

Q
Again by charge conservation, the electric charge on the surface =Q−x=Q− − A0 E = + A0 E
2 2
6-  
Q 3Q
=Q− − = Remark Solve this question for Q = 0 without using the
2 2 above answer and match that answers with the answers that
Final charge distribution is shown in Fig.1.280. you will get by putting Q = 0 in the above answer.

EXAMPLE 89. An isolated conducting sheet of area A and


carrying a charge Q is placed in a uniform electric field E, 1.26.1 Check Point 10
such that electric field is perpendicular to sheet and covers all
the sheet[Fig.1.281]. Find out charges appearing on its two 1. •• The electric field strength just above one face of a cop-
surfaces. per penny is 2000 N/C. What is the surface charge density
on this face of the penny?

2. •• A spark occurs at the tip of a metal needle if the electric


field strength exceeds 3.0 × 106 N/C, the field strength at
which air breaks down. What is the minimum surface
charge density for producing a -spark?

3. •• The conducting box in Fig.1.283 has been given an


excess negative charge. The surface density of excess
electrons at the center of the top surface is 5.0 × 1010
electrons/m2 .What are the electric field strengths E1 to
E3 at points 1 to 3?
Figure 1.281
1
SOLUTION Let the electric charge on the left surface of the 2
plate is x, then by charge conservation, the electric charge on 3
the right surface of the plate will be Q − x [see Fig.1.282].
Now, consider a point P inside the metal plate. Find the net
electric field at P and equate it with zero because in electro- 2
static static equilibrium the net electric field inside the plate
.
plate should always be zero, i.e., Figure 1.283
EPE= 0 E
124 CHAPTER 1. ELECTRIC CHARGE AND FIELD

4. •• A thin, horizontal, 10-cm-diameter copper plate of E, v1 , v2 , r and η, the viscosity of oil in air.
is charged to 3.5 nC. If the electrons are uniformly Multiple Choice Questions
distributed on the surface, what are the strength and
direction of the electric field (a).0.1 mm above the center 8. •• A soap bubble has radius R, charge Q, surface tension
of the top surface of the plate? T . Find the excess pressure in it.
(b). at the plate’s center of mass? 32π 2 R2 ε0 T −q 2 64π 2 R3 ε0 T −q 2
(c).0.1 mm below the center of the bottom surface of the (A) 32π 2 R4 ε0 (B) 32π 2 R4 ε0
128π 2 R3 ε0 T −q 2
plate? (C) 32π 2 R4 ε0 (D) none of these

Answer Keys and Solutions


5. •• Figure 1.284 shows a hollow cavity within a neutral
conductor. A point charge Q is inside the cavity. What is 1. APPROACH A copper penny is a conductor. Assume
the net electric flux through the closed surface that sur- the penny to be a flat disc of radius much, much greater
rounds the conductor? than the distance from the surface at which we are mea-
suring the electric field.
SOLUTION The excess charge on a conductor resides on
the surface. The electric field at the surface of a charged
conductor is
 
~ surface = σ , perpendicular to surface
E
ε0
σ =ε0 Esurface
= 8.85 × 10−12 C2 /Nm2 (2000 N/C)


1 =17.7 × 10−9 C/m2


Q Note: Because the actual surface of a penny is not flat,
the surface charge density will not be uniform. The result
Closed surface above gives the average surface charge density, far from
Figure 1.284
the edges of the coin.
2. In electrostatic equilibrium of a conductor, the excess
charge always resides on the outer surface. The electric
6. •• Two spherical cavities, of radii a and b, are hollowed field at the surface of a charged conductor is
out from the interior of a (neutral) conducting sphere of  
radius R (Fig.1.285). At the center of each cavity a point −
→ σ
E surface = , perpendicular to surface
charge is placed-call these charges qa and qb . 0
(a) Find the surface charges σa ,*σb3.0
, andcm
σR .
Therefore,

σ = 0 Esurface
= 8.85 × 10−12 C2 /Nm2 3.0 × 106 N/C
 

= 2.7 × 10−5 C/m2

Note: It is the air molecules just above the surface that


"break down" when the E -field becomes strong enough
to accelerate stray charges to approximately 15eV between
collisions, thus causing collisional ionization. It does not
Figure 1.285 make any difference whether E points toward or away from
the surface.
(b) What is the field outside the conductor?
(c) What is the field within each cavity? 3. The excess charge on a conductor resides on the outer
(d) What is the force on qa and qb ? surface. In Fig.1.283, point 1 is at the surface of a charged
(e) Which of these answers would change if a third charge, conductor, hence
qc , were brought near the conductor?  
~ surface = σ , perpendicular to surface
E
0
7. •• Electronic charge e may be determined by Millikan’s
5.0 × 1010 1.60 × 10−19 C/m2
 
oil drop method. Oil drops of radius r acquire a terminal ⇒ Esurface =
speed v1 with downward electric field E and a speed v2 8.85 × 10−12 C2 /Nm2
with the upward electric field. Find the value of e in terms = 0.90kN/C
1.27. GAUSS’S LAW FOR GRAVITY 125

At point 2 the electric field strength is zero because this equilibrium of soab bubble, we have
point lies inside the conductor. The electric field strength
at point 3 is zero because there is no excess charge on 4T
Pgas + Pel = P0 +
the interior surface of the box. This can be quickly seen R
by considering a Gaussian surface just inside the interior 4T
⇒ Pgas − P0 = − Pel
surface of the box as shown in Figure1.283. R

4. The copper plate is a conductor. The excess charge resides Here, Pgas is the outward gas pressure , P0 is the inward
on the surface of the plate. Ignore the charge that resides atmospheric pressure and Pel is the outward electric
on the edge of the plate because the plate’s thickness is pressure on the surface of soap bubble.
much, much less than the radius. Solve: (a) One-half of The excess pressure on the soap bubble is given by,
the charge is located on the top surface and one-half on the
4T σ2
bottom surface of the copper plate, so the surface charge Pexcess = −
density is R 2ε0

q (3.5/2nC) Since, σ = Q
4πR2 therefore,-
σ= = = 2.23 × 10−7 C/m2
A π(0.10/2 m)2
4T Q2
Pexcess = −
Thus, the electric field at the surface of the plate is R 32π 2 R4 ε0

σ 2.23 × 10−7 C/m2


E= = = 2.52 × 104 N/C 1.27 Gauss’s Law for Gravity
0 8.85 × 10−12 C2 /Nm2
Because the charge on the plate is positive, the direction of We know that the acceleration due to gravity, − →g , can be
the electric field is away from the plate. Thus the electric thought of as a gravitational field analogous to the electric


~ = 25kN/C upward from the plate. (b) The
field is E field, E . The gravitational field is the gravitational force per
center of mass of the plate is in the interior of the plate, so unit mass, while the electric field is the electrical force per
E = 0.0 N/C because the electric field within a conductor unit charge. While we often use the gravitational acceleration
is zero. (c) The electric field E = 25 × 103 N/C, away at the surface of Earth, we can calculate the magnitude of ~g
from the plate, which is downward. Thus E ~ = 25kN/C at any location by using Eq.(1.158)
downward from the plate. GM
g= (1.158)
5. For any closed surface that encloses a total charge Qin , the r2
net electric flux through the closed surface is Φe = Qin /ε0 . where G is the universal gravitational constant and r is the
In the present case (Figure 1.284), the conductor is neutral radial distance from the centre of a radially symmetric large
and there is a point charge Q inside the cavity. Thus mass, M .
Qencl = Q and the flux is This can be written in terms of the unit vector r̂ that points
Q radially outward from the centre of the large mass, M :
Φe =
0 GM


g = − 2 r̂ (1.159)
qa +qb
r
6. (a) σa = − 4πa2 ; σb = − 4πb2 ; σR = 4πR2
qa qb
1 qa +qb We define a gravitational flux, Φg , in an analogous way to
(b) Eout = 4πε 0 r2 r b, where r = vector from centre of the electric flux of Equation 1.116. The integral is over a
large sphere. closed surface, and it is a vector dot product between the
(c) Ea = 4πε 1 qa
0 ra
2 ra ,
1
Eb = 4πε qb
0 rb
2 rb , where ra (rb ) is
gravitational field ~g and an outward pointing differential
the vector from center of cavity a(b) −

surface element d A :
(d) Zero
(e) σR changes (but not σa or σb ); Eoutside changes (but I
not Ea or Eb ); force on qa and qb still zero. Φg = ~
~g · dA (1.160)
surface

The gravitational flux is a scalar quantity and it can be positive


7. Equations of motion are-
or negative. Think ~g like a gravitational field here, rather than
eE + mg − 6πηrv1 = 0 (downward field)
an acceleration, although they have the same numerical value.
eE − mg − 6πηrv2 = 0 (upward field)
We call the magnitude of ~g the gravitational field strength, g.
On adding and solving for e, we get
We will now calculate the gravitational flux around Earth. For
3πη a radially symmetric planet, we have spherical symmetry, and
e= (v1 + v2 ) r our closed surface is a sphere. In Fig.1.286 we illustrate Earth
E
and a larger concentric Gaussian sphere. As long as the mass
8. (C) If T is the surface tension of the liquid of soap bubble of Earth is distributed in a radially symmetric way, the gravita-
and σ is the surface charge density, then, in electrostatic tional field must have constant magnitude (but not direction)
126 CHAPTER 1. ELECTRIC CHARGE AND FIELD

all around the closed Gaussian surface. Note that everywhere only helpful in calculating the gravitational field strength when
~ there is sufficient symmetry that we can choose a surface with
the gravitational field ~g and the differential surface vector dA
are in exactly opposite directions. The gravitational flux pass- constant field strength. The same considerations we used in the
electrical case for the type of symmetry and the corresponding
dA shape of Gaussian surface to draw apply here.
In following example, we show the application of Gauss’s law
dA for a situation that would not have been easy to find directly
dA g using integration and the law of universal gravitation.
g
g EXAMPLE 90. Gravitational Field inside and out-
side a Homogeneous Planet Differentiated planets such as
g dA Earth, with an iron core, have higher mass densities near the
core. However, in this problem, assume a spherical planet of
dA g
radius Rp and total mass Mp that has uniform mass density
r g throughout. Use Gauss’s law for gravity to find expressions for
g the acceleration due to gravity for points at distance r from the
g dA centre of the planet, for (a) r > Rp and (b) r < Rp .
dA APPROACH We have spherical symmetry. To find the ac-
dA celeration due to gravity at different radial distances, r, we
draw spherical Gaussian surfaces at distance r from the center
Figure 1.286: For the case of a Gaussian sphere around a radially of the planet. When we are outside the planet, the enclosed
>
symmetric Earth,
> the gravitational fieldg ~g and the differential area mass is simply the total mass of the planet. Inside the planet,
~ are directly opposite each other.
vectors dA we have to use the mass density to find the mass enclosed by
the spherical Gaussian surface. In both cases, the symmetry
ing through the Gaussian surface is given by- of the situation requires that the direction of the acceleration
due to gravity be radially inward.
I
Φg = ~
~g · dA
sphere
SOLUTION (a) r > Rp : Here we have a situation similar
I to that of Figure 1.287 and draw a Gaussian sphere concentric
Φg = gdA cos 180◦ with the planet, but larger:
sphere
I I
Φg = −g dA ~ = −4πGMenc
~g · dA (1.164)
sphere surface
2
Φg = −g4πr ~ are in opposite directions, ~g · dA ~ =
Since ~g and dA
here g is the scalar gravitational field strength at distance r gdA cos 180◦
= −gdA. From the symmetry of the situation,
from the centre of Earth. the gravitational field strength is constant at any particular
Now we can substitute the value of the gravitational field radial distance and can therefore be taken outside the integral:
strength, g, at distance r using Equation 1.158: I
  ~ = −4πGMp
~g · dA
GM 2
Φg = − 4πr I
r2 (1.161) − gdA = −4πGMp
Φg = −4πGM Isurface

We have only developed this for the case of a spherically −g dA = −4πGMp


symmetric mass, but it turns out we can generalize the result surface

from Equation 1.158 to obtain Gauss’s law for gravity: the − 4πr2 g = −4πGMp
gravitational flux through any closed surface is proportional
GMp
to the mass enclosed (Mencl ) by the closed surface (with an Therefore, g = − r2 , or, in vector form since the gravitational
opposite sign): field is directed radially inward,
GMp
Φg = −4πGMencl (1.162) ~g = − r̂
r2
Gauss’s law for gravity is frequently written in a form that
includes the gravitational flux relationship: (b) r < Rp : In this case, we draw a concentric spherical Gaus-
sian sphere of radius r inside the planet (see Figure 1.287).
−→
I


g · d A = −4πGMencl (1.163)
surface First we calculate the mass density of the spherical planet:

Just as we saw for the electrical case, while Gauss’s law for Mp Mp 3Mp
ρ= = 4 3 = (1.165)
gravity applies no matter the shape of the closed surface, it is V 3 πR
4πR3
1.28. EXERCISES AND PROBLEMS 127

unit vector, as
GM r
~g = − r̂
R R3
Note that The result we obtained in part (b) reduces to
the result for gravitational acceleration at the surface of a
dA
planet when we set r = R. When we are inside the surface
g r of the planet, we get the interesting result that (for this
homogeneous-density planet) the strength of the gravitational
field is directly proportional to the distance from the centre of
the planet. Right at the centre, as expected, the relationship
yields a value of zero.

Figure 1.287: To calculate the gravitational field at a point inside


the planet, we draw a concentric spherical Gaussian surface (the 1.28 Exercises and Problems
darker smaller sphere) of radius r less than the radius of the planet.
Everywhere on this Gaussian surface, ~g will have a constant mag-
~
nitude and will be> directly opposite in direction to dA.
1.28.1 Conceptual Questions
1. If you charge a pocket comb by rubbing it with a silk
scarf, how can you determine if the comb is positively or
The gravitational field ~g points radially inward and the differ- negatively charged?
ential area element dA~ radially outward:
2. Why does a shirt or blouse taken from a clothes dryer
~ = gdA cos 180◦ = −gdA
~g · dA (1.166) sometimes cling to your body?
We know from the symmetry >of the situation that the gravi- 3. Explain why fog or rain droplets tend to form around ions
tational field strength is constant
g at any particular radial dis- or electrons in the air.
>
tance. Therefore, we compute the gravitational flux using re-
sult (1.160). The integral over the differential area elements for 4. Why does a plastic ruler that has been rubbed with a cloth
the Gaussian surface simply yields the surface area of a sphere have the ability to pick up small pieces of paper? Why is
of radius r: this difficult to do on a humid day?
I 5. Three point charges, +q, +Q, and −Q, are placed at the
Φg = ~
~g · dA corners of an equilateral triangle as shown in Figure 1.288.
sphere
No other charged objects are nearby (a) What is the di-
(1.167)
I
= −g dA rection of the net force on charge +q due to the other two
sphere charges? (b) What is the total electric force on the system
Φg = −g4πr2 of three charges? Explain.

" the planet, Menc is not the


> >
For the Gaussian surface inside
entire mass but rather just the part of the mass inside the
Gaussian surface. We can calculate it using the density we
found earlier in Eq.(1.166): "

4 3 r2
Menc = πr ρ
3
4 3Mp
= πr3
3 4πR3
r3
⇒ Menc = Mp 3 (1.168)
R
Therefore, Gauss’s law for gravity in this case becomes the
following, using results (1.166) and (1.166):
Figure 1.288
Φg = −4πGMenc
r3
−g4πr2 = −4πGMp 6. You have a positively charged insulating rod and two
R3
metal spheres on insulating stands. Give step-by-step
GMP r
g= directions of how the rod, without actually touching
R3
> either sphere, can be used to give one of the spheres (a)
Since the acceleration due to gravity is radially inward, we can a negative charge, and (b) a positive charge.
write this in vector form, using the outward-pointing radial
128 CHAPTER 1. ELECTRIC CHARGE AND FIELD

7. A positively charged rod is brought close to a neutral piece 12. The balloon in Fig.1.291 was rubbed on a student’s hair.
of paper, which it attracts. Draw a diagram showing the Explain why the water drip curves instead of falling ver-
separation of charge in the paper, and explain why attrac- tically.
tion occurs.

8. A popular classroom demonstration consists of rubbing


a plastic rod with fur to give the rod charge, and then
placing the rod near an empty soda can that is on its side
(Figure 1.289). Explain why the can will roll toward the
rod.

− −
−− −
++
+
Empty
soda can

Figure 1.291

Figure 1.289 13. The form of Coulomb’s law is very similar to that
for Newton’s law of universal gravitation. What are
the differences between these two laws? Compare also
9. A metal sphere is positively charged. Is it possible for the gravitational mass and electric charge.
sphere to electrically attract another positively charged
ball? Explain your answer
14. When a charged ruler attracts small pieces of paper,
sometimes a piece jumps quickly away after touching the
ruler. Explain.
10. Contrast the net charge on a conductor to the “free
charges” in the conductor.
15. We are not normally aware of the gravitational or electric
force between two ordinary objects. What is the reason
11. A gold leaf electroscope, which is often used in physics in each case? Give an example where we are aware of
demonstrations, consists of a metal tube with a metal ball each one and why.
at the top and a sheet of extremely thin gold leaf fastened
at the other end. (See Fig.1.290) The gold leaf is attached
16. What is the physics definition of field?
in such a way that it can pivot about its upper edge. (a)
If a charged rod is brought close to (but does not touch)
the ball at the top, the gold leaf pivots outward, away 17. What are the two major reasons for introducing the field
from the tube. Why? (b) What will the gold leaf do when concept?
the charged rod is removed? Why? (c) Suppose that the
charged rod touches the metal ball for a second or so.
What will the gold leaf do when the rod is removed in 18. Explain why the test charges we use when measuring
this case? Why? electric fields must be small.

af electroscope, which 19. When determining an electric field, must we use a positive
test charge, or would a negative one do as well? Explain.
ed in physics demon-
onsists of a metal tube
al ball at the top and a 20. What is the difference between a scalar field and a vector
xtremely thin gold leaf Metal tube field? Give an example of each type.
and ball
t the other end. (See
Gold
5.) The gold leaf is 21. Assume that the two opposite charges in Fig. 1.293 are
leaf
n such a way that it 12.0 cm apart. Consider the magnitude of the electric
about its upper edge. field 2.5 cm from the positive charge. On which side of
this charge–top, bottom, left, or right–is the electric field
Figure 1.290
the strongest? The weakest? Explain.
1.28. EXERCISES AND PROBLEMS 129

22. Why do electric field lines point away from positive


charges and toward negative charges?

23. An electron initially moving horizontally near Earth’s


surface enters a uniform electric field and is deflected
upward. What can you say about the direction of the
electric field (assuming no other interaction)? What can
you say about the direction of the electric field if the
electron is deflected downward?
Figure 1.293

24. Consider the electric field at the three points indicated


30. An electron is traveling in circular motion at constant
by the letters A, B, and C in Fig. 1.292. First draw an
speed due to the effect of an electric field. Is it possible
arrow at each point indicating the direction of the net force
for the electric field to be uniform?
that a positive test charge would experience if placed at
that point, then list the letters in order of decreasing field
strength (strongest first). Explain. 31. A point charge is surrounded by a spherical Gaussian
surface of radius r. If the sphere is replaced by a cube
of side r, will ΦE be larger, smaller, or the same? Explain.

32. Eight electrons are the only charged particles inside an


isolated balloon, producing a field line flux through its
surface. (a) If eight additional electrons are set outside
the balloon to form the eight corners of a cube, how does
the field line flux through the balloon change? (b) How
does the flux change if the eight additional electrons are
all placed at one location outside the balloon instead of
being distributed at the corners of a cube?

33. Under what conditions is the electric field magnitude


Figure 1.292
nonzero inside the cavity enclosed by a hollow conducting
sphere?

25. Why can electric field lines never cross?


34. It is apparent that the electrostatic force is extremely
strong, compared to gravity. In fact, the electrostatic
26. Show, using the three rules for field lines given in Section force is the basic force governing phenomena in daily
1.15, that the electric field lines starting or ending on a life–the tension in a string, the normal forces between sur-
single point charge must be symmetrically spaced around faces, friction, chemical reactions, and so forth—except
the charge. weight. Why then did it take so long for scientists
to understand this force? Newton came up with his
gravitational law long before electricity was even crudely
27. Given two point charges, Q and 2Q, a distance ` apart, is understood.
there a point along the straight line that passes through
them where E = 0 when their signs are (a) opposite,
(b) the same? If yes, state roughly where this point will be. 35. What is the distinction between field line flux and electric
flux? Why is electric flux the preferred variable for
analyzing electric fields?
28. Consider a small positive test charge located on an electric
field line at some point, such as point P in Fig.1.293. Is 36. Objects are composed of atoms which are composed
the direction of the velocity and/or acceleration of the test of charged particles (protons and electrons); however,
charge along this line? Discuss. we rarely observe the effects of the electrostatic force.
Explain why we do not observe these effects.
29. Is it possible for a dipole that is initially moving in a
straight line to be deflected by a uniform electric field? 37. Occasionally, people who gain static charge by shuffling
their feet on the carpet will have their hair stand on end.
130 CHAPTER 1. ELECTRIC CHARGE AND FIELD

Why does this happen? 10. •• A 2.0 µC point charge and a 4.0µC point charge are
a distance L apart. Where should a third point charge
be placed so that the electric force on that third charge is
38. Why does a garment taken out of a clothes dryer some- zero?
times cling to your body when you wear it?
11. •• A −2.0µ C point charge and a 4.0µ C point charge are
a distance L apart. Where should a third point charge
be placed so that the electric force on that third charge is
1.28.2 Problems zero?
Discrete Charge Distributions
12. •• Three point charges, each of magnitude 3.00nC, are at
separate corners of a square of edge length 5.00 cm. The
1. • How many electrons are required to yield a total charge two point charges at opposite corners are positive, and
of 1.00 C? the third point charge is negative. Find the electric force
exerted by these point charges on a fourth point charge
2. • A plastic rod is rubbed against a wool shirt, thereby q4 = +3.00nC at the remaining corner.
acquiring a charge of −0.80µC. How many electrons are
transferred from the wool shirt to the plastic rod? 13. •• A point charge of 5.00µC is on the y axis at y =
3.00 cm, and a second point charge of −5.00µC is on the
3. • What is the total charge of all of the protons in 1.00 kg y axis at y = −3.00 cm. Find the electric force on a point
of carbon? charge of 2.00µC on the x axis at x = 8.00 cm.

4. •• The faraday is a unit of charge frequently encountered 14. •• A point particle that has a charge of −2.5µC is located
in electrochemical applications and named for the British at the origin. A second point particle that has a charge of
physicist and chemist Michael Faraday. It consists of ex- 6.0µC is at x = 1.0 m, y = 0.50 m.A third point particle,
actly 1 mole of elementary charges (i.e., 1 mol of elec- and electron, is at a point with coordinates (x, y). Find the
trons). Calculate the number of coulombs in 1 faraday. values of x and y such that the electron is in equilibrium.

5. •• Suppose a cube of aluminum which is 1.00 cm on a side 15. • • • Five identical point charges, each having charge Q,
accumulates a net charge of +2.50 pC. (a) What percent- are equally spaced on a semicircle of radius R as shown in
age of the electrons originally in the cube was removed? Figure 1.294. Find the force (in terms of k, Q, and R ) on
(b) By what percentage has the mass of the cube decreased a charge q located equidistant from the five other charges.
because of this removal?
y
6. •• During a process described by the photoelectric effect,
ultra-violet light can be used to charge a piece of metal.
(a) If such light is incident on a slab of conducting material
and electrons are ejected with enough energy that they
escape the surface of the metal, how long before the metal Q
has a net charge of +1.50 nC if 1.00 × 106 electrons are Q
ejected per second? (b) If 1.3 eV is needed to eject an R
electron from the surface, what is the power rating of the
light beam? (Assume this process is 100% efficient.)
Q q x
Electric Force

7. •• Two protons in an atomic nucleus are typically sepa-


rated by a distance of 2 × 10−15 m. The electric repulsive Q
force between the protons is huge, but the attractive nu- Q
clear force is even stronger and keeps the nucleus from
bursting apart. What is the magnitude of the electric Figure 1.294
force between two protons separated by 2.00× 10−15 m?

8. • A point charge q1 = 4.0µC is at the origin and a point 16. • • • The structure of the NH3 molecule is approximately
charge q2 = 6.0µC is on the x axis at x = 3.0m. (a) Find that of an equilateral tetrahedron, with three H+ ions
the electric force on charge q2 . (b) Find the electric force forming the base and an N3− ion at the apex of the tetra-
on q1 . (c) How would your answers for Parts (a) and (b) hedron. The length of each side is 1.64 × 10−10 m. Cal-
differ if q2 were −6.0µC? culate the electric force that acts on each ion.
The Electric Field
9. • Three point charges are on the x-axis: q1 = −6.0µC is at
x = −3.0 m, q2 = 4.0µC is at the origin, and q3 = −6.0µC 17. • A point charge of 4.0µC is at the origin. What are
is at x = 3.0 m. Find the electric force on q1 . the magnitude and direction of the electric field on the
1.28. EXERCISES AND PROBLEMS 131

x axis at (a)x = 6.0 m and (b) x = −10 m? (c) Sketch


the function Ex versus x for both positive and negative
values of x. (Remember that Ex is negative when E ~
points in the −x direction.)

18. •• Two point charges, each +4.0µC, are on the x axis;


one point charge is at the origin and the other is at
x = 8.0m. Find the electric field on the x axis at
(a) x = −2.0m, (b)x = 2.0m, (c)x = 6.0m, and (d) Figure 1.295
x = 10m.(e) At what point on the x axis is the electric
field zero? (f) Sketch Ex versus x for −3.0m < x < 11m.
oflength L as shown in Figure 21-38. A third point par-
ticle that has a charge equal to 2q sits at the apex ofthe
19. •• When a 2.0 nC point charge is placed at the origin, triangle. A fourth point particle that has charge q 0 is
it experiences an electric force of 8.0 × 10−4 N in the +y placed at the midpoint of the baseline making the electric
direction. (a) What is the electric field at the origin? field at the center of the triangle equal to zero. What is
(b) What would be the electric force on a −4.0-nC point the value of q 0 ?(The center is in the plane of the triangle
charge placed at the origin? (c) If this force is due to the and equidistant from all three vertices
electric field of a point charge on the y axis at y = 3.0 cm,
what is the value of that charge? 26. • • •Two equal positive point charges +q are on the y
axis; one is at y = +a and the other is at y = −a. The
20. •• The electric field near the surface of Earth points down-
electric field at the origin is zero. A test charge q0 placed
ward and has a magnitude of 150 N/C. (a) Compare the
at the origin will therefore be in equilibrium. (a) Discuss
magnitude of the upward electric force on an electron with
the stability of the equilibrium for a positive test charge
the magnitude of the gravitational force on the electron.
by considering small displacements from equilibrium along
( b ) What charge should be placed on a ping pong ball of
the x axis and small displacements along the y axis. (b)
mass 2.70 g so that the electric force balances the weight
Repeat Part (a) for a negative test charge. (c) Find the
of the ball near Earth’s surface?
magnitude and sign of a charge q0 that when placed at
21. •• Two point charges q1 and q2 both have a charge equal the origin results in a net force of zero on each ofthe three
to +6.0nC and are on the y axis at y1 = +3.0 cm and charges.
y2 = −3.0 cm respectively. (a) What is the magnitude
and direction of the electric field on the x axis at x = 27. •• Two equal positive charges q are on the y axis; one
4.0 cm? (b) What is the force exerted on a third charge point charge is at y = +a and the other is at y = −a.(a)
q0 = 2.0nC when it is placed on the x axis at x = 4.0 cm? Show that on the x axis the x component of the electric
3/2
field is given by Ex = 2kqx/ x2 + a2 .(b) Show that
22. •• A point charge of +5.0µC is located on the x axis at near the origin, where x is much smaller than a, Ex ≈
x = −3.0 cm, and a second point charge of −8.0µC is 2kqx/a3 .(c) Show that for values of x much larger than
located on the x axi at x = +4.0 cm. Where should a a, Ex ≈ 2kq/x2 . Explain why a person might expect this
third charge of +6.0µC be placed so that the electric field result even without deriving it by taking the appropriate
at the origin is zero? limit.
23. •• A −5.0µC point charge is located at x = 4.0m, y =
28. •• Two positive point charges +q are on the y axis at
−2.0m and a 12 − µC point charge is located at x =
y = +a and y = −a. A bead of mass m and charge −q
1.Om, y = 2.0m.(a) Find the magnitude and direction
slides without friction along a taut thread that runs along
ofthe electric field at x = −1.0 m, y = 0. (b) Calculate the
the x axis. Let x be the position of the bead. (a) Show
magnitude and direction ofthe electric force on an electron
that for x << a, the bead experiences a linear restor-
that is placed at x = −1.0m, y = 0.
ing force (a force that is proportional to x and directed
24. ••Two point particles, each having a charge q, sit on the toward the equilibrium position at x = 0) and therefore
base of an equilateral triangle that has sides of length L undergoes simple harmonic motion. (b) Find the period
as shown in Figure 1.295 A third point particle that has a of the motion.
charge equal to 2q sits at the apex of the triangle. Where
must a fourth point particle that has a charge equal to q 29. •• The acceleration of a particle in an electric field de-
be placed in order that the electric field at the center of pends on q/m (the charge-to-mass ratio of the particle).
the triangle be zero? (The center is in the plane of the (a) Compute q/m for an electron. (b) What is the magni-
triangle and equidistant from the three vertices.) tude and direction of the acceleration of an electron in a
uniform electric field that has a magnitude of 100 N/C? (c)
25. •• Two point particles, each having a charge equal to q, Compute the time it takes for an electron placed at rest in
sit on the base of an equilateral triangle that has sides a uniform electric field that has a magnitude of 100 N/C
132 CHAPTER 1. ELECTRIC CHARGE AND FIELD

to reach a speed of 0.01c. (When the speed of an elec- 35. •• A charged particle leaves the origin with a speed of
tron approaches the speed of light c, relativistic kinematics 3.00 × 106 m/s at an angle of 35◦ above the x axis. A uni-
must be used to calculate its motion, but at speeds of 0.01c ~ = −E0 ĵ exists throughout
form electric field given by E
or less, non-relativistic kinematics is sufficiently accurate the region. Find E0 such that the particle will cross the x
for most purposes.) (d) How far does the electron travel axis at x = 1.50 cm if the particle is (a) an electron, and
in that time? (b) a proton.

30. • The acceleration of a particle in an electric field depends 36. •• An electron starts at the position shown in Figure 1.297
on the charge-to-mass ratio of the particle. (a) Compute with an initial speed v0 = 5.00 × 106 m/s at 45◦ to the
q/m for a proton, and find its acceleration in a uniform x axis. The electric field is in the +y direction and has
electric field that has a magnitude of 100 N/C.(b) Find the a magnitude of 3.50 × 103 N/C. The black lines in the
time it takes for a proton initially at rest in such a field figure are charged metal plates. On which plate and at
to reach a speed of 0.01c (where c is the speed of light). what location will the electron strike?
(When the speed of an electron approaches the speed of Dipoles
light c, relativistic kinematics must be used to calculate
its motion, but at speeds of 0.01c or less, non-relativistic
kinematics is sufficiently accurate for most purposes.)

31. •• An electron is released from  rest in a weak electric




field E = −1.50 × 10−10 N/C ĵ. After the electron has
traveled a vertical distance of 1.0µm, what is its speed?
(Do not neglect the gravitational force on the electron.)
Figure 1.297
32. •• A 2.00 g charged particle is released from rest in a
~ = (300 N/C)î.
region that has a uniform electric field E
After traveling a distance of 0.500 m in this region, the 37. •• Two point charges, q1 = 2.0 pC and q2 = −2.0 pC,
particle has a kinetic energy of 0.120 J. Determine the are separated by 4.0 µm. (a) What is the magnitude of
charge of the particle. the dipole moment of this pair of charges? (b) Sketch the
pair and show the direction of the dipole moment.
33. •• An electron that has a kinetic energy equal to 2.00 ×
10−16 J is moving to the right along the axis of a cathode-
38. •• An electron (charge −e, mass m) and a positron (charge
ray tube as shown in Figure 1.296. An electric field

→ +e, mass m) revolve around their common center of mass
E = 2.00 × 104 N/C ĵ exists in the region between the

under the influence of their attractive coulomb force. Find
~ = 0) exists out-
deflection plates; and no electric field (E the speed v of each particle in terms of e, m, k, and their
side this region. (a) How far is the electron from the axis separation distance L.
of the tube when it exits the region between the plates?
(b) At what angle is the electron moving, with respect to 39. •• A point particle of mass m and charge q is constrained
the axis, after exiting the region between the plates? (c) to move vertically inside a narrow, friction less cylinder
At what distance from the axis will the electron strike the (Figure 1.298). At the bottom of the cylinder is a point
fluorescent screen? charge Q having the same sign as q (a) Show that the par-
ticle whose mass is m will be in equilibrium at a height
y0 = (kqQ/mg)1/2 (b) Show that if the particle is dis-
placed from its equilibrium position by a small amount
and released, it will exhibit simple harmonic motion with
1/2
angular frequency ω = (2g/y0 )

Figure 1.296

34. •• A 2.00 g charged particle is released from rest in a




region that has a uniform electric field E = (300 N/C)î.
After traveling a distance of 0.500 m in this region, the
particle has a kinetic energy of 0.120 J. Determine the Figure 1.298
charge of the particle.
1.28. EXERCISES AND PROBLEMS 133

40. ••Two neutral molecules on the x axis attract each other. approximation that the charge is a point charge on the x
Each molecule has a dipole moment p~, and these dipole axis at x = 2.5m, and compare your result with the result
moments are on the +x axis and are separated by a dis- calculated in Part (d) . (To do this you will need to assume
tance d. Derive an expression for the force of attraction that the values given in this problem statement are valid
in terms of p and d. to more than two significant figures.) Is your approximate
result greater or smaller than the exact result? Explain
41. ••Two equal positive point charges Q are on the x axis at your answer.
1 1
x = a and x = − a. (a) Obtain an expression for the
2 2 45. •• Two infinite nonconducting sheets of charge are parallel
electric field on the y axis as a function of y. (b) A bead
to each other, with sheet A in the x = −2.0 m plane and
ofmass M , which has a charge q, moves along the y axis
sheet B in the x = +2.0 m plane. Find the electric field
on a thin frictionless taut thread. Find the electric force
in the region x < −2.0 m, in the region x > +2.0 m, and
that acts on the bead as a function of y and determine
between the sheets for the following situations. (a) When
the sign of q such that this force always points away from
each sheet has a uniform surface charge density equal to
the origin. (c) The bead is initially at rest at the origin.
+3.0µC/m2 and (b) when sheet A has a uniform surface
If it is given a slight nudge in the +y direction, how fast
charge density equal to +3.0µC/m2 and sheet B has a
will the bead be traveling the instant the net force on it
uniform surface charge density equal to −3.0µC/m2 . (c)
is a maximum? (Assume any effects due to gravity are
Sketch the electric field line pattern for each case.
negligible.)
46. • A non-conducting disk of radius R lies in the z = 0
42. •• During the Millikan experiment used to determine the
plane with its center at the origin. The disk has a uni-
charge on the electron, a charged polystyrene microsphere
form surface charge density σ Find the value of z for which
is released in still air in a known vertical electric field.
EZ = σ/(4 ∈0 ) . Note that at this distance, the magni-
The charged microsphere will accelerate in the direction
tude of the electric field strength is half the electric-field
of the net force until it reaches terminal speed. The
strength at points on the x axis that are very close to the
charge on the microsphere is determined by measuring
disk.
the terminal speed. During one such experiment, the mi-
crosphere has radius of r = 5.50 × 107 m, and the field 47. • A ring that has radius a lies in the z = 0 plane with its
has a magnitudeE = 6.00 × 104 N/C. The magnitude center at the origin. The ring is uniformly charged and
ofthe drag force on the sphere is given byFD = 6πηrv, has a total charge Q. Find Ez on the z axis at (a)z =
where v is the speed ofthe sphere and η is the viscosity 0.2a, (b)z = 0.5a, (c)z = 0.7a, (d)z = a, and (e)z = 2a · (f )
of air(η = 1.8 × 10−5 N · s/m2 ) . Polystyrene has density Use your results to plot Ez versus z for both positive and
1.05 × 103 kg/m3 .(a) If the electric field is pointing down negative values of z. (Assume that these distances are
and the polystyrene microsphere is rising with a terminal exact.)
speed of 1.16×10−4 m/s, what is the charge on the sphere?
(b) How many excess electrons are on the sphere? (c) Ifthe 48. • A line charge that has a uniform linear charge density λ
direction ofthe electric field is reversed but its magnitude lies along the x axis from x = x1 to x = x2 where x1 < x2 .
remains the same, what is the new terminal speed? Show that the x component of the electric field at a point
on the y-axis is given by Ex = kλ y (cos θ2 − cos θ1 ), where
43. • • • In problem 42., there is a description of the Millikan
experiment used to determine the charge on the electron. θ1 = tan−1 (x1 /y), θ2 = tan−1 (x2 /y) and y 6= 0.
During the experiment, a switch is used to reverse the
49. •• A thin hemispherical shell of radius R has a uniform
direction of the electric field without changing its mag-
surface charge σ. Find the electric field at the center of
nitude, so that one can measure the terminal speed of
the base of the hemispherical shell.
the microsphere both as it is moving upward and as it is
moving downward. Let vu represent the terminal speed
when the particle is moving up, and vd the terminal speed Gauss’s Law
when moving down. (a) If we let u = vu + vd , show that
q = 3πηru/E, where q is the microsphere’s net charge. 50. • A single point charge (q = +2.00 µC) is fixed at the
For the purpose of determining q, what advantage does origin. An imaginary spherical surface of radius 3.00 m is
measuring both vu and vd have over measuring only one centered on the x axis at x = 5.00 m. (a) Sketch electric-
terminal speed? (b) Because charge is quantized, u can field lines for this charge (in two dimensions) assuming
only change by steps of magnitude N , where N is an in- twelve equally-spaced field lines in the xy plane leave the
teger. Using the data from Problem 42., calculate ∆u. charge location, with one of the lines in the +x direction.
Do any lines enter the spherical surface? If so, how many?
44. •• A uniform line charge that has a linear charge density λ (b) Do any lines leave the spherical surface? If so, how
equal to 3.5 nC/m is on the x axis between x = 0 and x = many? (c) Counting the lines that enter as negative and
5.0m.(a) What is its total charge? Find the electric field the ones that leave as positive, what is the net number of
on the x axis at (b) x = 6.0m, (c) x = 9.0m, and (d) x = field lines that penetrate the spherical surface? (d) What
250m.(e) Estimate the electric field at x = 250m, using the is the net electric flux through this spherical surface?
134 CHAPTER 1. ELECTRIC CHARGE AND FIELD

51. ••What is the electric flux through one side of a cube that r > a, where r is the distance from the long axis of the
has a single point charge of −3.00µC placed at its center? cylinder.

52. •• An imaginary right circular cone (Figure 1.299 ) that 57. •• Consider two infinitely long, coaxial thin cylindrical
has a base angle θ and a base radius R is in a charge free shells. The inner shell has a radius a1 and has a uniform
region that has a uniform electric field E~ (field lines are surface charge density of σ1 , and the outer shell has a
vertical and parallel to the cone’s axis). What is the ra- radius a2 and has a uniform surface charge density of σ2 .
tio of the number of field lines per unit area penetrating (a) Use Gauss’s law to find expressions for the electric
the base to the number of field lines per unit area pene- field in the three regions: 0 ≤ r < a1 , a1 < r < a2 , and
trating the conical surface of the cone? Use Gauss’s law r > a2 , where r is the distance from the axis. (b) What
in your answer. (The field lines in the figure are only a is the ratio of the surface charge densities σ2 /σ1 and their
representative sample.) relative signs if the electric field is to be zero everywhere
outside the largest cylinder? (c) For the case in Part (b),
what would be the electric field between the shells?

E Electric Charge and Field at Conductor Surfaces

58. • An uncharged penny is in a region that has a uniform


electric field of magnitude 1.60kN/C directed perpendicu-
lar to its faces. (a) Find the charge density on each face of
the penny, assuming the faces are planes. (b) If the radius
of the penny is 1.00 cm, find the total charge on one face.

θ 59. • A thin metal slab has a net charge of zero and has square
faces that have 12 -cm-long sides. It is in a region that has
a uniform electric field that is perpendicular to its faces.
The total charge induced on one of the faces is 1.2nC.
R
What is the magnitude of the electric field?
Figure 1.299 60. • • • If the magnitude of an electric field in air is as great
as 3.0 × 106 N/C, the air becomes ionized and begins to
conduct electricity. This phenomenon is called dielectric
53. •• In the atmosphere and at an altitude of 250 m, you
breakdown. A charge of 18µC is to be placed on a con-
measure the electric field to be 150 N/C directed down-
ducting sphere. What is the minimum radius of a sphere
ward and you measure the electric field to be 170 N/C
that can hold this charge without breakdown?
directed downward at an altitude of 400 m. Calculate the
volume charge density of the atmosphere in the region 61. •• A thin square conducting sheet that has 5.00 m-long
between altitudes of 250 m and 400 m, assuming it to be edges has a net charge of 80.0µC. The square is in the
uniform. (You may neglect the curvature of Earth. Why?) x = 0 plane and is centered at the origin. (Assume the
charge on each surface is uniformly distributed.) (a) Find
54. •• A non-conducting solid sphere of radius R has a volume the charge density on each side of the sheet and find the
charge density that is proportional to the distance from electric field on the x axis in the region |x|<< 5.00 m.
the center. That is, ρ = Ar for r ≤ R, where A is a (b) A thin but infinite nonconducting sheet that has a
constant. (a) Find the total charge on the sphere. ( b ) uniform charge density of 2.00µC/m2 is now placed in
Find the expressions for the electric field inside the sphere the x = −2.50 m plane. Find the electric field on the x
(r < R) and outside the sphere (r > R). (c) Sketch the axis on each side of the square sheet in the region |x|<<
magnitude of the electric field as a function of the distance 2.50 m. Find the charge density on each surface of the
r from the sphere’s center. square sheet.
55. • • •A non-conducting spherical shell of inner radius 62. • • • A large, flat, nonconducting, non-uniformly charged
R1 and outer radius R2 has a uniform volume charge surface lies in the x = 0 plane. At the origin, the surface
density ρ.(a) Find the total charge on the shell. (b) Find charge density is +3.10µC/m2 A small distance away from
expressions for the electric field everywhere. the surface on the positive x axis, the x component of the
electric field is 4.65×105 N/C. What is Ex a small distance
Gauss’s Law Applications in Cylindrical Symme- away from the surface on the negative x axis?
try Situations
63. • • • A quantum-mechanical treatment of the hydrogen
56. •• An infinitely long non-conducting solid cylinder of ra- atom shows that the electron in the atom can be treated
dius a has a uniform volume charge density of ρ0 . Show as a smeared-out distribution of negative charge of the
that the electric field is given by the following expressions: form ρ(r) = −ρ0 e−2r/a . Here r represents the distance
Ea = ρ0 a/(2ε0 ) for 0 ≤ r < a and Ea = ρ0 a2 /(2ε0 r) for from the center of the nucleus and a represents the first
1.28. EXERCISES AND PROBLEMS 135

Bohr radius which has a numerical value of 0.0529 nm.


Recall that the nucleus of a hydrogen atom consists of
just one proton and treat this proton as a positive point
charge. (a) Calculate ρ0 , using the fact that the atom is
neutral. (b) Calculate the electric field at any distance r
from the nucleus.

64. •• A thin, nonconducting, uniformly charged spherical


shell of radius R (Figure 1.300a) has a total positive charge
of Q. A small circular plug is removed from the surface.
(a) What are the magnitude and direction of the elec-
tric field at the center of the hole? (b) The plug is now
put back in the hole (Figure 1.300b). Using the result of
Part (a) , find the electric force acting on the plug. (c)
Using the magnitude of the force, calculate the "electro- Figure 1.301
static pressure" (force/unit area ) that tends to expand
the sphere.
69. ••An electric dipole that has a dipole moment of − →
p is
located at a perpendicular distance R from an infinitely
long line charge that has a uniform linear charge density λ.
Assume that the dipole moment is in the same direction
as the field of the line of charge. Determine an expression
for the electric force on the dipole.

Multiple Choice Questions


Level 1
(a) (b)
Charge & its Properties
Figure 1.300
1. The ratio of electric force (Fe ) to gravitational force
acting between two electrons will be
65. •• A uniformly charged, infinitely long line of negative (A) 1 × 1036 (B) 2 × 1039
charge has a linear charge density of −λ and is located on (C) 6 × 10 45
(D) 4 × 1042
the z axis. A small positively charged particle that has a
mass m and a charge q is in a circular orbit of radius Rin 2. Fg and Fe represent the gravitational and electrostatic
the xy plane centered on the line of charge. (a) Derive force respectively between two electrons situated at
an expression for the speed of the particle. (b) Obtain an some distance. The ratio of Fg to Fe is of the order of -
expression for the period of the particle0 sorbit. (A) 1036 (B) 101
(C) 10◦
(D) 10−43
66. •• A uniformly charged nonconducting solid sphere of ra-
dius R has its center at the origin and has a volume charge
3. One quantum of charge should be at least be equal to the
density of ρ.(a) Show that at a point within the sphere a
~ = ρ rr̂.(b) Material is re- charge in coulomb:
distance r from the center E 30 (A) 1.6 × 10−17 c. (B) 1.6 × 10−19 c.
moved from the sphere leaving a spherical cavity that has (C) 1.6 × 10−10 c (D). 4.8 × 10−10 c.
a radius b = R/2 and its center at x = b on the x axis
(Figure1.301). Calculate the electric field at points 1 and 4. The unit of charge is coulomb in SI system and esu of
2 shown in Figure 1.301. charge (or stat coulomb) in C.G.S. system 1 coulomb
Hint: Model the sphere-with-cavity as two uniform equals
spheres of equal positive and negative charge densities. (A) 3 × 109 esu (B) 1/3 × 10 9 esu


(C) 1/3 × 108 esu (D) 9 × 109 esu


67. ••Show that the electric field throughout the cavity of
~ = ρ bî
Problem 66.b is uniform and is given by E 5. The relative strengths of gravitational, electromagnetic
3ε0
and strong nuclear forces are-
68. •• The cavity in Problem 66.b is now filled with a (A) 1 : 1039 : 1036 (B) 1 : 1036 : 1039
uniformly charged nonconducting material with a total (C) 1 : 10−26
: 10−39
(D) 1 : 10−39 : 10−36
charge of Q. Calculate the new values of the electric field
at points 1 and 2 shown in Figure 1.301 6. An electron at rest has a charge of 1.6 × 10−19 C. It
starts moving with a velocity v = c/2, where c is
136 CHAPTER 1. ELECTRIC CHARGE AND FIELD

the speed of light, then the new charge on it is - will be-


(A) 1.6 × 10−19 q
coulomb (A) uniform but on the surface alone
2 (B) non uniform but on the surface alone
(B) 1.6 × 10−19 1 − 12 coulomb
q  (C) uniform inside the volume
2 2 (D) non uniform inside the volume
(C) 1.6 × 10−19 1 − 1 coulomb
−19
(D) 1.6×10
q
1 2
coulomb Coulomb’s Law
1−( 2 )
16. Two similar charge of +Q , as shown in Figure1.302
7. If a glass rod is rubbed with silk, it acquires a positive are placed at A and B. ˘q charge is placed at point
charge because- C midway between A and B. ˘q charge will oscillate if
(A) Protons are added to it. (A) It is moved towards A.
(B) Protons are removed from it. (B) It is moved towards B.
(C) Electrons are added to it. (C) It is moved upwards AB.
(D) Electrons are removed from it. (D) Distance between A and B is reduced.

8. Which one of the following is the unit of electric charge?


(A) Coulomb
(B) Newton
(C) Volt
(D) Coulomb/Volt
Figure 1.302
9. An accelerated or decelerated charge produces-
(A) Electric field only
(B) Magnetic field only 17. When the distance between two charged parti-
(C) Localized electric and magnetic fields cle is halved, the force between them becomes-
(D) Electric and magnetic fields that are radiated (A) One fourth (B) One half
(C) Double (D) Four times
10. Which one of the following statement regarding electro-
18. The force between two point charges in vacuum
statics is wrong ?
is 15N, if a brass plate is introduced between
(A) Charge is quantized
the two charges, then force between them will-
(B) Charge is conserved
(A) Becomes zero
(C) There is an electric field near an isolated charge
(B) Remains the same
at rest
(C) Becomes 30 N
(D) A stationary charge produces both electric and
(D) Becomes 60 N
magnetic fields
19. The force between an α particle and an
11. The dielectric constant for water is- electron separated by a distance of 1 Åis-
(A) 1 (B) 40 (C) 81 (D) 0.3 (A) 2.3 × 10−8 N attractive
(B) 2.3 × 10−8 N Repulsive
12. In M.K.S. System, 4π∈1
equals-
0 (C) 4.6 × 10−8 N attractive
(A) 9 × 10 N.m /C2
9 2
(D) 4.6 × 10−8 repulsive
(B) 1 N.m2 /C2
(C) 1 dyne .cm2 / stat C 2 20. Two charges are at distance (d) apart in air.
(D) (D) 9 × 109 dyne ×cm2 / stat C 2 Coulomb force between them is F . If a dielec-
tric material of dielectric constant (K) is placed
13. A stationary electric charge produces- between them, the coulomb force now becomes.
(A) Only electric fields (A) F/K (B) F K (C) F/K 2 (D) K 2 F
(B) Only magnetic field
(C) Both electric and magnetic field 21. Two point charges in air at a distance of 20 cm. from each
(D) Neither electric nor magnetic field other interact with a certain force. At what distance from
each other should these charges be placed in oil of rela-
14. Charges reside on the- tive permittivity 5 to obtain the same force of interaction –
(A) 8.9410˘2 m (B) 0.89410˘2 m
(A) Outer surface of the charged conductor (C) 89.410˘2 m (D) 8.94 × 10^2 m
(B) Inner surface of the charged conductor
(C) Inner as well as outer surface of the charged con- 22. A certain charge Q is divided at first into two
ductor parts, (q) and (Q − q). Later on the charges are
(D) None of the above placed at a certain distance. If the force of in-
teraction between the two charges is maximum then-
15. An isolated solid metallic sphere is chargedd with +Q (A) (Q/q) = (4/1) (B) (Q/q) = (2/1)
charge .The distribution of their +Q charge on the sphere (C) (Q/q) = (3/1) (D) (Q/q) = (5/1)
1.28. EXERCISES AND PROBLEMS 137

23. A unit charge is one which when placed in 32. Two small balls having equal positive charge Q (Coulomb)
vacuum one cm from an equal charge of on each are suspended by two insulating strings of equal
the same kind will repel it with a force of- length ’L’ metre, from a hook fixed to a stand. The whole
(A) 1 New- (B) 1 dyne (C) 2 dyne (D) 4 dyne set up is taken in a satellite in to space where there is
ton no gravity (state of weightlessness) Then the angle (q)
between the two strings is -
24. The permittivity 0 of vacuum is 8.86 × 10−12 (A) 0 (B) 90
C2 /N − m2 and the dielectric constant of water is (C) 180 (D) 0 < q < 180
81 . The permittivity of water in C2 /N − m2 is-
(A) 81 × 8.86 × 10−12 33. ABC is a right angle triangle AB = 3 cm,
(B) 8.86 × 10−12  BC = 4 cm charges +15, +12, −12 esu are placed
(C) 8.86 × 10−12 /81 at A, B and C respectively. The magnitude of the
(D) 81/ 8.86 × 10−12 force experienced by the charge at B in dyne is-


(A) 125 (B) 35 (C) 22 (D) 0


25. The force between two point charges placed in vac-
uum at distance 1 mm is 18 N. If a glass plate of 34. Equal charges of each 2µC are placed at a point
thickness 1 mm and dielectric constant 6, be kept be- x = 0, 2, 4, and 8 cm on the x-axis. The force ex-
tween the charges then new force between them would be- perienced by the charge at x = 2 cm is equal to -
(A) 18 N (B) 108 N (A) 5 Newton (B) 10 Newton
(C) 3 N (D) 3 × 10−6 N (C) 0 Newton (D) 15 Newton
26. Two similar and equal charges repel each 35. Three equal charges (q) are placed at corners of a
other with force of 1.6N , when placed equilateral triangle. The force on any charge is-
3 m apart. Strength of each charge is- √ 2

(A) 40µC (B) 20µC (A) Zero (B) 3 Kq


a2
2 √ 2
(C) 4µC (D) 2µC (C) √Kq3a2
(D) 3 3 Kq
a2

27. There are two charges +1 micro-coulomb and +5 36. Two identical charges of charge (q) and placed at
micro-coulomb, the ratio of force on them will be- (−a, 0) and (a, 0). Same nature charge particle is
(A) 1043 (B) 1 : 1 placed at origin. It executes S.H.M. If it is displaced-
(C) 10◦ (D) 10−43 (A) in x-direction
Superposition Principle (B) in y-direction
(C) at an angle of 45◦ from the x-axis
28. A charge Q is divided in two parts Q1 and Q2 (D) along perpendicular to the plane.
and these charges are placed at distance R. there
will be maximum repulsion between them, when- 37. Two equal negative charge (−q) are fixed at the
(A) Q2 = (Q/R), Q1 = Q − (Q/R) points (0, a) and (0, −a) on the y-axis. A posi-
(B) Q2 = (Q/3), Q1 = (2Q/3) tive charge (Q) is released from rest at the point
(C) Q2 = (Q/4), Q1 = (3Q/4) (2a, 0) on the x-axis. The charge Q will -
(D) Q1 = Q2 = Q/2 (A) execute simple harmonic motion about the origin.
(B) move to the origin and remains at rest
29. The three charges each of 5 × 10−6 coloumb are
(C) move to infinity
placed at vertex of an equilateral triangle of side
(D) execute oscillatory but not simple harmonic mo-
10 cm. The force exerted on the charge of 1µC
tion
placed at centre of triangle in newton will be-
(A) 13.5 (B) zero (C) 4.5 (D) 6.75
38. Five point charges, each of value +q coulomb, are
placed on five vertices of a regular hexagon of side L
30. A point charge q1 exerts a force F upon another charge q2 .
meter. The magnitude of the force on a point charge
If one other charge q3 be placed quite near to charge q2 ,
of value −q coulomb placed at the centre of the hexagon is-
then the froce that charge q1 exerts on the charge q2 will be 2 √ 2
(A) F (B) > F (C) (C) < F (D) zero (A) kqL 2 (B) 5 kq L2
√ 2
(C) 3 kq L2 (D) Zero
31. A mass particle (mass = m and charge = q) is
Electric Field
placed bewteen two point charges of charge q separ-
tion between these two charge is 2 L. The frequency 39. A pendulum bob of mass 80mg and carrying a
of oscillation of mass particle, if it is displaced for charge of 2 × 10−8 coul. is at rest in a hor-
a small qdistance along the line joining the charges- izontal uniform electric field of 20, 000 V m−1 .
Find the tension in the thread of pendulum -
q
(A) 2π mπε0 L3
q 1
(B) 2π mπε0 L3
q 4
q q (A) 8.8 × 10−2 N (B) 8.8 × 10−3 N
(C) 2π 4mπε0 L3
q 1
(D) 2π
q 1
16πε0 mL3 (C) 8.8 × 10 N
−4
(D) 8.8 × 10−5 N
138 CHAPTER 1. ELECTRIC CHARGE AND FIELD

40. Two charges 4q and q are placed 30 cm. apart. At what at a point on the line joining the centers of the
point the value of electric field will be zero two spheres will be zero at a distance from sphere A.
(A) 10 cm. away from q and between the charge (A) 20 cm (B) 33 cm (C) 55 cm (D) 60 cm.
(B) 20 cm. away from q and between the charge
(C) 10 cm. away from q and out side the line joining 50. Four charges +q, +q, −q and −q are placed respec-
the charge. tively at the corners A, B, C and D of a square
(D) 10 cm. away from 4q and out side the line joining of side (a), arranged in the given order. Calcu-
them. late the intensity at (O) the center

of the square
2
4 2q
(A) 4√2q
4πε0 ·a
(B) 4πε0 ·a2
41. Unit of electric field intensity is newtons/coulomb. The 2

4 2q
other unit of this can be (C) 0 ·a
πε√
4 2q
(D) πε0 ·a2
(A) Vm (B) Vm2
(C) V/m (D) V/m2 51. A ring of radius (R) carries a uniformly distributed charge
42. lf Q = 2 C coulomb and force on it is F = 100N F = 100 +Q. A point charge −q is placed on the axis of the ring
newtons Then the value of field intensity will be- at a distance 2R from the center of the ring and released
(A) 100 N/C (B) 50 N/C from rest. The particle-
(C) 200 N/C (D) 10 N/C (A) Becomes in rest condition immediately.
(B) Executes simple harmonic motion
43. Four equal but like charge are placed at four corners of (C) Motion is not SHM
a square. The electric field intensity at the center of the (D) Come at the center of ring immediately.
square due to any one charge is E then the resultant
electric field intensity at center of square will be 52. A small circular ring has a uniform charge distribution.
(A) Zero (B) 4E (C) E (D) 1/2 E On a far-off axial point distance x from the center of the
ring, the electric field is proportional to-
44. Two charges 9e and 3e are placed at a distance r. The
(A) x−1 (B) x−3/2 (C) x−2 (D) x5/4
distance of the point where the electric field intensity will
Electric flux and Gauss Laws
be zero
 is 
(A) 1+r√3 from 9e charge
  53. The tangent drawn at a point on a line of electric force
(B) √r
from 9e charge shows the-
 1+ 1/3 (A) intensity of gravity field (B) intensity of magnetic
(C) 1−r√3 from 3e charge field
  (C) intensity of electric field (D) direction of electric field
(D) √r
from 3e charge.
1+ 1/3
54. Which of the following statements concerning the electro-
45. An electric field can deflect- statics is correct-
(A) X-rays (B) Neutrons (A) electric line of force never intersect each other
(C) α-particles (D) γ - rays (B) electric lines of force start from positive charge
and end at the negative charge
46. Which one of the following relations is correct- (C) electric lines of force start or ends perpendicular
(A) 1 N/C = 108 Volt /m to the surface of a charged metal.
(B) 1 N/C = 10−6 V/m (D) all of the above
(C) 1 N/C = 1 V/m
55. When no charge is confined with in the Gauss’s surface,
(D) 1 N/C = 10−8 V/m
it implies that-
47. If mass of the electron = 9.1 × 10−31 Kg. Charge on the (A) E = 0
electron = 1.6 × 10−19 coulomb and g = 9.8 m/s2 . Then ~ and −
(B) E

ds are parallel
the intensity of the electric field required to balance the ~ and −
(C) E

ds are mutually perpendicular
weight of an electron is- ~ and ds−

(D) E are inclined at some angle
(A) 5.6 × 10−9 N/C (B) 5.6 × 10−11 N/C
(C) 5.6 × 10 N/C
−8
(D) 5.6 × 10−7 N/C 56. If electric field flux coming out of a closed surface is zero,
48. Six charges +Q each are placed at the corners of a regular the electric field at the surface will be-
(A) zero
hexagon of side (a), the electric field at the center of (B) same at all places
hexagon is- (C) dependent upon the location of points
2
(A) Zero (B) 4π∈1
0
· 6Q
a2 (D) infinites
Q2 6Q2
(C) 1
4π∈0 · a2 (D) 1
4π∈0 · √
a 2
57. If three electric di-poles are placed in some closed surface,
49. Two charged spheres A and B are charged with the then the electric flux emitting from the surface will be-
charges of +10 and +20 coulomb respectively and (A) zero (B) positive
separated by a distance of 80cm. The electric field (C) negative (D) None
1.28. EXERCISES AND PROBLEMS 139

58. For which of the following fields, Gauss’s law is valid- 65. In Fig.1.304 shown the electric lines of force emerging
(A) fields following square inverse law from a charged body. If the electric fields at A and B are
(B) uniform field EA and EB are respectively, If the distance between A
(C) all types of field and B is r then
(D) this law has no concern with the field EA > EB EA < EB
59. A charge of Q coloumb is located at the centre of a cube. EA = EB EA = (EB ) /r2
If the corner of the cube is taken as the origin, then the
flux coming out from the faces of the cube in the direction
of X - axis will be-
(A) 4πQ (B) Q/6 ∈0 (C) Q/3 ∈0 (D) Q/4 ∈0
60. A rectangular surface of 2 metre width and 4 metre
length, is placed in an electric field of intensity 20 new-
ton/C, there is an angle of 60◦ between the perpendicular
to surface and electrical field intensity. Then total flux
emitted from the surface will be- (In Volt- metre) Figure 1.304
(A) 80 (B) 40 (C) 20 (D) 160

61. A charge q is inside a closed surface and charge −q is Application of Gauss law
outside. The out going electric flux is- 66. Three charges q1 = 1µc, q2 = 2µc and q3 = −3µc and
(A) −q/0 (B) zero (C) q/0 (D) 2q/0 four surfaces S1 , S2 , S3 and S4 are shown in Fig.1.305.
62. If the electric field is uniform, then the electric lines of The flux emerging through surface S2 in N.m2 /C is -
forces are- (A) 36π × 103 (B) −36π × 103
(A) Divergent (B) Convergent (C) 36π × 10 9
(D) −36π × 109
(C) Circular (D) Parallel
63. Electric lines of forces-
(A) Exist everywhere
(B) Are imaginary
(C) Exist only in the immediate vicinity of electric
charges
(D) None of the above
64. Which one of the following diagrams Fig.1.303 shows the
correct lines of force ? Figure 1.305

67. A surface enclosed an electric dipole, the flux through the


surface is-
(A) Infinite (B) Positive
(C) Negative (D) Zero
1 1
68. Total flux coming out of some closed surface is - q
(A) q/ε0 (B) ∈0 /q (C) qε0 (D) εq0

69. A square of side 20 cm. is enclosed by a surface of


sphere of 80 cm. radius . square and sphere have the
(A) (B) same centre. four charges +2 × 10−6 c, −5 × 10−6 c,
−3 × 10−6 c, +6 × 10−6 c are located at the four corners of
a square, Then out going total flux from spherical surface
2
in N − m2 /c will be-
(A) zero (B) (16π) × 10−6
(C) (8π) × 10 −6
(D) (36π) × 10−6
70. The flux emerging out from any one face of the cube will
2 be -
(A) 6εq0 (B) 3εq0 (C) εq0 (D) 4εq0

71. The electric field inside a spherical shell of uniform surface


(C) charge density is -
(D)
(A) Zero
Figure 1.303 (B) Constant, different from zero
(C) Proportional to the distance from the centre
(D) None of the above
140 CHAPTER 1. ELECTRIC CHARGE AND FIELD

72. A cubical box of side 1 m is immersed a uniform electric Level 2


field of strength 104 N/C. The flux through the cube is-
1. 5 × 105 lines of electric flux are entering in a closed surface
(A) 104 (B) 6 × 104 (C) 2 × 104 (D) Zero
and 4 × 105 liner come out of the surface the charge
enclosed by the surface is -
73. A charge (q) is located at the centre of a cube. The (A) 00.885 × 10−6 C (B) 8.85 × 10−6 C
electric flux through any face of the cube is- (C) −8.85 × 10 C −7
(D) 8.85 × 10−8 C
(A) q0 (B) 2∈q 0 (C) 4∈q 0 (D) 6∈q 0
2. A cylinder of radius (R) and length (L) is placed in a
uniform electric al field (E) parallel to the axis of the
74. A large isolated metal sphere of radius (R) carries a fixed cyclinder the total flux for the surface of the cylinder is
charge. A small charge is placed at a distance (r) from its given by -
surface experiences a force which is - (A) 2πR2 E (B) πR2 E
(A) Proportional to R πR2 +πR2
(C) (D) zero
(B) Independent of R and E

(C) Inversely proportional to (R + r)2 3. A hemisphere (radius R) is placed in electric field as


(D) inversely proportional to r2 shown in Fig.1.306 Total outgoing flux is -
(A) πR2 E (B) 2πR2 E (C) 4πR2 E (D) πR2 E /2

75. A hollow sphere of charge does not produce an electric
field at any-
(A) Interior point
(B) Outer point
(C) Surface point
(D) None of the above
76. A spherical conductor of radius 50 cm has a surface
charge density of 8.85 × 10−6 C/m2 . The electric field
near the surface in N/C is- Figure 1.306
(A) 8.85 × 10−6 (B) 8.85 × 106
(C) 1 × 10 6
(D) Zero
4. Three identical charges each of 1¯C are kept on the
77. The electric field intensity at a point located at distance circumference of a circle of radius 1 metre forming
r(r < R) from the center of a spherical conductor (radius equilateral triangle. The electric intensity at the center
R ) charged Q will be - of the circle in N/C is -
(A) kQR/r3 (B) kQr/R3 (A) 9 × 103 (B) 13.5 × (C) 27 × 103 (D) Zero
(C) kQ/r 2
(D) zero. 103

Electric Dipole 5. The number of electrons,falling on spherical conductor


(radius = 0.1 m ) to produce. 036 N/C electric field at
the surface of conductor, is-
78. If an electric dipole is kept in a uniform electric field, then (A) 2.7 × 105 (B) 2.5 × 102
it will experience (C) 2.6 × 105
(D) 2.4 × 105
(A) a force 6. A particle of mass 6µg carrying a charge of 10−9 C, is
(B) a couple and moves placed in the electric field of strength E = 6 × 105 V/m,
(C) a couple and rotates the acceleration acquired by the particle is-
(D) a force and moves. (A) 102 m/sec2(B) 103 m/sec2(C) 105 m/sec2(D) 1020 m/sec2

79. An electric dipole consists of two opposite charges each of 7. Two small balls having equal positive charge Q on each
magnitude 1 × 10−6 C separated by a distance 2 cm. The are suspended by two insulating strings at equal length L
dipole is placed in an external field of 10 × 105 N/C. The meter, from a hook fixed to a stand. The whole set up is
maximum torque on the dipole is - taken in a satellite into space where there is no gravity.
(A) 0.2 × 10−3 N − m (B) 1.0 × 10−3 N − m Then the angle θ between two strings and tension in each
(C) 2 × 10 N − m
−3
(D) 4 × 10−3 N − m string is-
2 2
(A) 0, kq (B) π, 2kqL2
80. The ratio of the electric field due to an electric| dipole on L2 2 2

its axis and on the perpendicular bisector of the dipole is- (C) π, 4kqL2 (D) π2 , 2kqL2
(A) 1 : 2 (B) 2 : 1 (C) 1 : 4 (D) 4 : 1 8. The magnitude of the electric field strength (E) such that
an electron placed in the field would experience an elec-
trical force equal to its weight is [ assume g = 10 m/see2
81. The region surrounding a stationary electric dipole has-
(A) electric field only (A) 5.68 × 10−11 N/ C Vertically up.
(B) magnetic field only (B) 5.68 × 10−11 N/ C Vertically down.
(C) both electric and magnetic fields (C) 5.68 × 10−10 N/ C Vertically up.
(D) 5.68 × 10−10 N/ C Vertically down.
(D) neither electric non magnetic field
1.28. EXERCISES AND PROBLEMS 141

9. The electric field at the surface of a charged spherical (A) 2kλ


R (B) 2R

(C) Zero (D) None
conductor is 10KV/m. The electric field at a distance
equal to the diameter from its centre will be - 17. A thin stationary ring of radius 1 m has a positive charge
(A) 2.5 V/m (B) 2.5KV/m (C) 5.0KV/m (D) 5.0 V/m 10µC uniformly distributed over it. A particle of mass
0.9gm and having a negative charge of 1µC is placed on
10. An Electron is sitvated 3 × 10−9 m from one α - particle the axis at a distance of 1 cm from the center of the ring
and 4 × 10−9 m from another α - particle . The magni- and released then time period of oscillation of particle
tude of force on the electron , when two α - particles are will be-
5 × 10−9 m apart is - (A) 0.6sec (B) 0.2sec (C) 0.3sec (D) 0.4sec
(A) 5.64 × 10−11 N (B) 56.4 × 10−11 N
(C) 0.564 × 10−11 N (D) 564 × 10−11 N. 18. Three charges +3q, +q and Q are placed on a straight
line with equal separation. In order to make the net force
11. Two large metal plates each of area (A) carry charger +q
on q to be zero, the value of Q will be-
and −q and face each other. the plates are separated by
(A) +3q (B) +2q (C) −3q (D) −4q
a small distance (d) the electric field between the plates
would be- 19. As per diagram 1.308 point charge +q is placed at the
(A) e2q
0A
(B) EqA
0 A
(C) 0qA (D) q∈
A
0 origin O. Work done in taking another point charge −Q
from the point A [coordinates (0, a)] to another point B
12. Two parallel plates of infinite dimensions are uniformly
coordinates (a, 0)] along the straight path
 AB is- √
charged[Fig.1.307]. The surface charge density on one is
−qQ 1
σA will on the other is σB , field intensity at point C will (A) Zero (B) 4πε 2 2a
 0 a √
be-
 
(C) 4πε
qQ 1 √a (D) 4πε
qQ 1
2a
(A) Proportional to (σA − σB ) 0 a
2
2 0 a
2

(B) Proportional to (σA + σB )


(C) Zero
(D) 2σA

Figure 1.308
Figure 1.307
20. The electric field due to an electric dipole at a dis-
13. A charged particle of mass (m) is kept in equilibrium in tance r from its center in axial position is E. if the
the electric field between the plates of millikan oil drop dipole is rotated through an angle of 90◦ about its per-
experiment. If the direetion of the electric field between pendicular axis, the electric field at the same point will be-
the plate is reversed. then acceleration of the charged (A) E (B) E/4 (C) E/2 (D) 2E
particle will be-
(A) Zero (B) g/2 (C) g (D) 2 g 21. Two electric dipoles of moment P and 64P are placed in
opposite direction on a line at a distance of 25 cm. The
14. Two identical small balls, each of mass, are suspended electric field will be zero at point between the dipoles
by two light inelastic conducting threads each of length whose distance from the dipole of moment P is-
` from the same fixed point support.If the distance (d) (A) 5 cm (B) 25
9 cm (C) 10 cm (D) 13
4
cm
between two balls is very less the d is equal to-

2k(q 2
1/3  2
2/3 22. When an electric dipole P̄ is placed in a uniform electric
(A) mg (B) 2k`q
mg field Ē then at what angle between P and E the value of
 2 2/3 torque will be maximum-
k (q
(C) 2mg (D) none of these (A) 90◦ (B) 0◦ (C) 180◦ (D) 45◦

15. A metal sphere A of radius R has a charge of Q on it The 23. An electric dipole has the magnitude of its charge as q and
field at a point B outside the sphere is E. Now another its dipole moment is p. It is placed in a uniform electric
sphere of radius R having a charge −3Q is placed at point field E. If its dipole moment is along the direction of the
B. The total field at a point mid-way between A and B field, the force on it is-
due to both sphere is- (A) 2q.E
(A) 4E (B) 8E (C) 12E (D) 16E (B) q.E
(C) Zero
16. A uniformly charged rod with charge per unit length λ (D) −q.E
is bent in to the shape of a semicircle of radius R. The 24. Two opposite and equal charges 4 × 10−8 coulomb when
electric field at the centre is - placed 2 × 10−2 an away, form a dipole. If this dipole is
142 CHAPTER 1. ELECTRIC CHARGE AND FIELD

placed in an external electic field 4×103 newton/coulomb, (A) Stable when moved along the Y-axis
the value of maximum torque and the work done in rotat- (B) Stable when moved along Z-axis
ing it through 180◦ will be- (C) Stable when moved along X-axis
(A) 64 × 10−4 Nm and 64 × 10−4 J (D) Unstable in all of the above cases
(B) 32 × 10−4 Nm and 32 × 10−4 J 9. The electric field strength due to a ring of radius R at
(C) 64 × 10−4 Nm and 32 × 10−4 J a distance x from its center on the axis of ring carrying
(D) 32 × 10−4 Nm and 64 × 10−4 J charge Q is given by
1 Qx
Level 3 E=
4πε0 (R + x2 )3/2
2

1. If an electron enters into a space between the plates of At what distance from the center will the electric field be
a parallel plate capacitor at an angle α. with the plates maximum -
and leaves at an angle β to the plates. The ratio of its (A) x = R (B) x = (C) x √ = (D) x √ =
kinetic energy while entering the capacitor to that while R/2 R/ 2
10. Two point charges Q and −3Q are placed certain distance2R
leaving will be - apart. If the electric field at the location of Q be Ē, then
 2  2
sin β cos β
(A) sin (B) cos α that the location of −3Q will be-
α −
→ −

 2  2 (A) 3Ē (B) −3Ē (C) E /3 (D) − E /3
(C) cos β
cos α
(D) sin α
sin β
11. A and B are two points on the axis and the perpendicular
2. Force between two identical charges placed at a distance bisector respectively of an electric dipole. A and B are
of r in vacuum is F . Now a slab of dielectric constant far away from the dipole and at equal distances from it.
K = 4 is inserted between these two charges. The The fields at A and B are ĒA and ĒB are respectively
thickness of the slab is r/2. The force between the such that -
charges will now become - (A) ĒA = EB (B) ĒA = 2EB
(A) F/4 (B) F/2 (C) 35 F (D) 49 F (C) EA = −2EB (D) ĒA = 12 ĒB
12. A long string with a charge of λ per unit length passes
3. In a certain region of surface there exists a uniform through an imaginary cube of edge `. The maximum
electric field of 2 × 103 k̂V/m. A rectangular coil of possible flux of the electric field through the cube will be
dimensions 10 cm × 20 cm is placed in x-y plane. The -
electric flux through the coil is - √ √
(A) λ`/ε0 (B) 2λ`/ε0 (C) 6λ`2 /ε0 (D) 3λ`/ε0
(A) Zero (B) 30 V − (C) 40 V − (D) 50 V −
m m m 13. A charge Q is placed at each of two opposite corners of a
square. A charge q is placed at each of the two opposite
4. The electric flux from a cube of edge ` is φ. What will be
corners of the square. If the resultant electric field on Q
its value if edge of cube is made 2` and charge enclosed
is zero, then - √
is halved - q
(A) Q = − 2√ (B) Q = −2 2q
(A) φ/2 (B) 2φ (C) 4φ (D) φ 2 √
(C) Q = −2q (D) Q = 2 2q
5. Each of the two point charges are doubled and their 14. The electric field 5000 outside a charged long straight wire is
−1
distance is halved. Force of interaction becomes n times, given by E = − r Vm . It is radially inward. The
where n is - value of V B − V A is -
(A) 4 (B) 1 (C) 1/16 (D) 16 [ Given rB = 60 cm and rA = 30 cm]
(A) 5000 loge 2 V (B) 0 V (C) 2 V (D) 2500 V
6. Two point charges repel each other with a force of 100 N. 15. An electron moves with velocity v in x-direction. An elec-
One of the charges is increased by 10% and other is tric field acts on it in y-direction. The force on the electron
reduced by 10%. The new force of repulsion at the same acts in -
distance would be- (A) + ve direction of Y-axis
(A) 100 N (B) 121 N (B) - ve direction of Y-axis
(C) 99 N (D) None of these (C) + ve direction of Z-axis
7. A spherical charged conductor has σ as the surface (D) -ve direction of Z-axis
16. Two identical simple pendulums A and B, are suspended
density of charge. The electric field on its surface is E.
from the same point. The bobs are given positive charges,
If the radius of the sphere is doubled keeping the surface
with A having more charge than B. They diverge and reach
density of charge unchanged, what will be the electric
equilibrium, with A and B making angles θ1 and θ2 with
field on the surface of the new sphere -
the vertical respectively. Which of the following is correct
(A) E4 (B) E2 (C) E (D) 2E
-
(A) θ1 > θ2
8. Three equal and similar charges are placed at (B) θ1 < θ2
(−a, 0, 0), (0, 0, 0) and (+a, 0, 0). What is the na- (C) θ1 = θ2
ture of equilibrium of the charge at the origin- (D) The tension in A is greater than that in B
1.28. EXERCISES AND PROBLEMS 143

Statements Type Question:-


Each of the questions given below consist of Statement -
I and Statement - II. Use the following Key to choose
the appropriate answer. (A) If both Statement-I and
Statement-II are true, and Statement - II is the correct
explanation of Statement- I. (B) If both Statement −I
and Statement −II are true but Statement - II is not the
correct explanation of Statement −I. (C) If Statement
−I is true but Statement −II is false. (D) If Statement
−1 is false but Statement −II is true.
Figure 1.310
17. Statement I : Force between two charges decreases when
air separating the charges is replaced by water. Statement
II : Medium intervening the charges has no effect on force. a force F when kept apart at some distance. A third
spherical conductor having same radius as that of B but
18. Statement I : The no. of lines of force emanating from uncharged is brought in contact with B, then brought in
1µC charge in vacuum is 1.13 × 105 Statement II: This contact with C and finally removed away from both. The
follow from Gauss’s theorem in electrostatics. new force of repulsion between B and C is -[AIEEE-2004]
(A) F/4 (B) 3 F/4 (C) F/8 (D) 3 F/8
Level 4 (Previous Years Questions)
6. Four charges equal to −Q are placed at the four corners
Section A: JEE MAINS
of a square and a charge q is at its centre. If the system
is in equilibrium √ the value of q is - [AIEEE-2004]

1. When two charges are placed at a distance apart. Find the (A) − Q (1 + 2 2) (B) Q
(1 + 2
magnitude of third charge which is placed at mid point the 4 √ 4 √2)
(C) − 2 (1 + 2 2)
Q
(D) 2 (1 + 2 2)
Q
line joining the charge. So that system is in equilibrium -
[AIEEE-2002] 7. A charged oil drop is suspended in a uniform field
(A) − 4
Q
(B) − 2
Q
(C) − 3
Q
(D) −Q1 of 3 × 104 v/m so that it neither falls nor rises. The
charge on the drop will be (Take the mass of the charge
2. A charged particle q is placed at the centre O of cube of = 9.9 × 10−15 kg and g = 10 m/s2 ) - [AIEEE-2004]
length L (ABCDEFGH). Another same charge q is placed (A) 3.3 × 10−18 C (B) 3.2 × 10−18 C
at a distance L from O. Then the electric flux through (C) 1.6 × 10−18 C (D) 4.8 × 10−18 C
BCFG is[Fig.1.309]- [AIEEE-2002]
8. A charged ball B hangs from a silk thread S which makes
an angle θ with a large charged conducting sheet P, as
shown in the Figure1.311. The surface charge density σ
of the sheet is proportional to- [AIEEE 2005]
(A) cos θ (B) cot θ (C) sin θ (D) tan θ

Figure 1.309

(A) L (B) zero (C) 3L (D) q


3 πε0 L

3. Three charges ˘q1 , +q2 and ˘q3 are placed as shown in


Fig.1.310. The x-component of the force on ˘q1 is propor-
Figure 1.311
tional to – [2003]
(A) q2
b2 + q3
a2 sin θ (B) q2
b2 + q3
a2 cos θ
(C) q2
+ q3
cos θ (D) q2
− q3
cos θ
b2 a2 b2 a2
9. Two point charges +8q and −2q are located at x = 0
4. If the electric flux entering and leaving an enclosed surface and x = L respectively. The location of a point on the x
respectively is φ1 and φ2 , the electric charge inside the axis at which the net electric field due to these two point
surface will be – [2003] charges is zero is- [AIEEE-2005]
(A) (φ1 + φ2 ) /ε0 (B) (φ2 − φ1 ) /ε0 (A) 2 L (B) L/4 (C) 8 L (D) 4 L
(C) (φ1 + φ2 ) ε0 (D) (φ2 − φ1 ) ε0
5. Two spherical conductors B and C having equal radii 10. An electric dipole is placed at an angle of 30◦ to a non-
and carrying equal charges on them repel each other with uniform electric field. the dipole will experience -
144 CHAPTER 1. ELECTRIC CHARGE AND FIELD

[AIEEE 2006] Statement- 2 is false. (D) Statement-1 is false, Statement-


(A) a torque as well as a translational force 2 is true.
(B) a torque only
(C) a translational force only in the direction of the 14. Statement-1: For a charged particle moving from point P
field to point Q, the net work done by an electrostatic field on
(D) a translational force only in a directin normal to the particle is independent of the path connecting point
the direction of the field P to point Q. Statement-2: The net work done by a con-
11. If gE and gm are the accelerations due to gravity on the servative force on an object moving along a closed loop is
surfaces of the earth and the moon respectively and if zero. [AIEEE-2009]
Millikan’s oil drop experiment could be performed on the (A) Statement-1 is true, Statement-2 is true; Statement-2
two surfaces, one will find the ratio (electronic charge on is a correct explanation for Statement-1 (B) Statement-1
the moon/ electronic charge on the earth) to be- is true. Statement-2 is true; Statement-2 is not a cor-
[AIEEE 2007] rect explanation for Statement-1 (C) Statement-1 is true,
(A) 1 (B) 0 (C) gE /gM (D) gM /gE Statement-2 is false. (D) Statement- 1 is false, Statement-
2 is true.
12. A thin spherical shell of radius R has charge Q spread
uniformly over its surface. Which of the following graphs 15. Let P(r) = πR Q
4 r be the charge density distribution for a

[Fig.1.312] most closely represents the electric field E (r) solid sphere of radius R and total charge Q. For point
produced by the shell in the range 0 ≤ r < ∞, where r is ’P’ inside the sphere at distance r1 from the centre of the
the distance from the centre of the shell? [AIEEE 2008] sphere, the magnitude of electric field is - [AIEEE-2009]
Qr 2 Qr 2
(A) 0 (B) 4πQ0 r2 (C) 4π∈01R4 (D) 3π∈01R4
1
E(r) E(r)
16. A charge Q is placed at each of the opposite corners of
a square. A charge q is placed at each of the other two
corners. If the net electrical force on Q is zero, then Q/q
equals - √ [AIEEE-2009]
(A) −2 2 (B) −1 (C) 1 (D) − √12

r r 17. A thin semi-circular ring of radius r has a positive charge


O R O R q distributed uniformly over it[Fig.1.313]. The net field
(A) (B) ~ at the centre O is
E [AIEEE 2010]
E(r) E(r) (A) 4π2qε0 r2 j (B) − 4π2qε0 r2 j
(C) − 2π2qε0 r2 j (D) 2π2qε0 r2 j

r r
O R O R
(C) (D)

Figure 1.312

Figure 1.313
13. This question contains Statement-1 and Statement-2. Of
the four choices given after the statements, choose the one
18. Let there be a spherically symmetric charge distribution
that best describes the two statements.
with charge density varying as ρ(r) = ρ0 54 − Rr upto

Statement-1: For a mass M kept at the centre of a cube
r = R, and r(r) = 0 for r > R, where r is the distance
of side ’ a ’, the flux of gravitational field passing through
from the origin. The electric field at a distance r(r < R )
its sides is 4πGM.
from the origin is given by [AIEEE 2010]
Statement-2: If the direction of a field due to a point
(A) 4πρ 0r 5 r
(B) ρ0 r 5 r

− 4ε0 3 − R 
source is radial and its dependence on the distance ’ r ’ 3ε0
4ρ0 r 5
3 R
(C) 3ε0 4 − Rr (D) 3ε0 0 54 − Rr
ρ r

from the source is given as r12 , its flux through a closed
surface depends only on the strength of the source en- 19. Two identical charged spheres suspended from a common
closed by the surface and not on the size or shape of the point by two massless strings of length I are initially a
surface. [AIEEE-2008] distance d(d << 1) apart because of their mutual repul-
(A) Statement-1 is true, Statement-2 is true; Statement-2 sion. The charge begins to leak from both the spheres at
is a correct explanation for Statement-1 (B) Statement-1 a constant rate. As a result the charges approach each
is true. Statement-2 is true; Statement-2 is not a cor- other with a velocity v. Then as a function of distance x
rect explanation for Statement-1 (C) Statement-1 is true, between them, [2011]
1.28. EXERCISES AND PROBLEMS 145

2Q
(A) (B)
1 −1 e
(A) v ∝ x−1 (B) v ∝ x 2 (C) v ∝ x (D) v ∝ x 2
πa2 2πa2
2Q
(C) Q
2π(b2 −a2 ) (D) π(a2 −b2 )
20. In a uniformly charged sphere of total charge Q and radius
24. An electric dipole has a fixed dipole moment p~, which
R, the electric field E is plotted as a function of distance
makes angle θ with respect to x-axis. When subjected to
from the center. The graph which would correspond to −
→ −

the above will be[Fig.1.314]- [2012] an electric field E1 = Eı̂, it experiences a torque T1 = τ k̂.
−→
When subjected to another electric field E2 = E1ˆ it


experiences a torque T2 = −T~1 . The angle θ is: [2017]
(A) 90◦ (B) 30◦ (C) 45◦ (D) 60◦

25. Two identical conducting spheres A and B, carry equal


charge. They are separated by a distance much larger
than their diameter, and the force between them is F .
A third identical conducting sphere, C, is uncharged.
Sphere C is first touched to A, then to B, and then
(A) (B) removed. As a result, the force between A and B would
be equal to [9 Jan 2018]
/ (A) 3F
4 (B) F
2 (C) F (D) 3F
8

26. A solid ball of radius R has a charge density ρ given by


ρ = ρ0 1 − Rr for 0 ≤ r ≤ R. The electric field outside
the ball is: [15 April 2018]
ρ0 R 3 4ρ0 R3 3ρ0 R3 ρ0 R3
(A) ε0 r2 (B) 3ε0 r2 (C) 4ε0 r2 (D) 12ε 0r
2
(C) (D)
27. A charge Q is placed at a distance a/2 above the centre of
Figure 1.314 the square surface of edge a as shown in the figure1.316.
The electric flux through the square surface is:
[15 April 2018]
21. Let [ε0 ] denote the dimensional formula of the permittiv-
ity of vacuum. If M = mass, L = length, T = time and
A = electriccurrent, then: [2013] 
(A) [ε0 ] = M −1 L−3 T 2 A  (B) [ε0 ] = M −1 L−3 T 4 A2


(C) [ε0 ] = M −1 L2 T −1 A−2 (D) [ε0 ] = M −1 L2 T −1 A


22. Two charges, each equal to q, are kept at x = −a and
x = a on the x-axis. A particle of mass m and charge
q0 = 2q is placed at the origin. If charge q0 is given a
small displacement (y << a) along the y-axis, the net
Figure 1.316
force acting on the particle is proportional to: [2013]
(A) y (B) −y (C) y1 (D) − y1
(A) Q
3ε0 (B) Q
6ε0 (C) Q
2ε0 (D) Q
ε0
23. The region between two concentric spheres of radii ‘a’
and ‘b’, respectively (figure 1.315), has volume charge 28. Three charges +Q, q, +Q are placed respectively, at
density ρ = rA0 , where A is a constant and r is the distance distance, d/2 and d from the origin, on the x-axis. If the
from the centre. At the centre of the spheres is a point net force experienced by +Q, placed at x = 0, is zero,
charge Q. The value of A such that the electric field in then value of q is: [2019]
the region between the spheres will be constant, is: [2016] (A) −Q/4 (B) +Q/2 (C) +Q/4 (D) −Q/2

29. Charge is distributed within a sphere of radius R with


a volume charge density p(r) = rA2 e−2r/a where A and
a are constants. If Q is the total charge of this charge
distribution,
 the radius
 R is:  [2019]

(A) a log 1 − 2πaA
Q
(B) a
2 log 1
Q
   1− 2πaA 
(C) a log 1− 1 Q (D) a2 log 1 − 2πaA
Q
2πaA

30. Four point charges −q, +q, +q and −q are placed on


y-axis at y = −2d, y = −d, y = +d and y = +2d,
respectively. The magnitude of the electric field E at a
point on the x-axis at x = D, with D >> d, will behave
Figure 1.315 as: [9 April 2019, (II)]
146 CHAPTER 1. ELECTRIC CHARGE AND FIELD

(A) E ∝ 1
D3 (B) E ∝ 1
D (C) E ∝ 1
D4 (D) E ∝ 1
D2 36. Consider the force F on a charge ’ q ’ due to a uniformly
charged spherical shell of radius R carrying charge Q dis-
31. The bob of a simple pendulum has mass 2g and a charge tributed uniformly over it. Which one of the following
of 5.01/4 C. It is at rest in a uniform horizontal electric statements is true for F , if ’ q ’ is placed at distance r
field of intensity 2000V /m. At equilibrium, the angle from the centre of the shell? [06 Sep. 2020 (II)]
that the pendulum makes with the vertical is: ( take (A) F = 4πε0 R2 for r < R
1 Qq
g = 10 m/s2 [8 April 2019(I)]

(B) 4πε
1 Qq
2 > F > 0 for r < R
(A) tan−1 (2.0) (B) tan−1 (0.2) 0 R
(C) F = 4πε 1 Qq
2 for r > R
(C) tan−1 (5.0) (D) tan−1 (0.5) 0 R
(D) F = 4πε R2 for all r
1 Qq
32. For a uniformly charged ring of radius R, the electric 37. Two charged 0thin infinite plane sheets of uniform surface
field on its axis has the largest magnitude at a distance h charge density σ+ and σ− , where |σ+ | > |σ− |, intersect at
from its centre. Then value of h is: [9 Jan. 2019 √(I)] right angle. Which of the following best represents the
(A) √R5 (B) √R2 (C) R (D) R 2 electric field lines for this system[Fig.1.319]?
[04 Sep. 2020 (I)]
33. Shown in the Fig.1.317, is a shell made of a conductor. It
has inner radius a and outer radius b, and carries charge
Q. At its centre is a dipole p̄ as shown. In this case :
[12 April 2019,(I)]
(A) surface change density on the inner surface is uni-
Q/2
form and equal to 4πa 2

(B) electric field outside the shell is the same as that


of a point charge at the centre of the shell.
(C) surface charge density on the outer surface de-


pends on | P |
(D) surface charge density on the inner surface of the
shell is zero everywhere. (A) (B)

Figure 1.317
(C) (D)

34. An electric dipole is formed by two equal and opposite Figure 1.319
charges q with separation d. The charges have same mass
m. It is kept in a uniform electric field E. If it is slightly
rotated from its equilibrium orientation, then its angular 38. A particle of charge q and mass m is subjected to an
frequency electric field E = E0 1 − ax2 in the x-direction, where a
q ω is: q [8 April 2019,(II)]
and E0 are constants. Initially the particle was at rest at
q q
2qE
(A) qE
(B) (C) 2 qE
(D) qE
md md md 2md x = 0. Other than the initial position the kinetic energy
35. Charges Q1 and Q2 are at points A and B of a right angle of the particle becomes zero when the distance of the
triangle OAB (Fig.1.318). The resultant electric field at particle from the origin
q is: q [04 Sep 2020 q (II)]
point O is perpendicular to the hypotenuse, then Q1 /Q2 (A) a (B) 2
(C) 3
(D) 1
a a a
is proportional to: [06 Sep 2020 (I)]
x31
(A) x3 (B) x1
x2
(C) x2
x1 x2
(D) x22 39. A small point mass carrying some positive charge on it,
2 1 is released from the edge of a table[Fig.1.320]. There is
a uniform electric field in this region in the horizontal
direction. Which of the following options then correctly
describe the trajectory of the mass[Fig.1.321]? (Curves
are drawn schematically and are not to scale).
[02 Sep 2020 (II)]

Figure 1.318 40. Consider a sphere of radius R which carries a uniform


charge density ρ. If a sphere of radius R2 is carved out of
1.28. EXERCISES AND PROBLEMS 147

E
x

y
Figure 1.320
Figure 1.323

y y
√ h √  i
(A) σ
(1 + 3)ŷ − x̂2 (B) ∈σ0 1 + 23 ŷ + x̂2
 
2∈0
(A) (B) √ h √  i
(C) 2∈
σ x̂
(D) σ 3 x̂
 
x x 0
(1 + 3)ŷ + 2 2∈0 1 − 2 ŷ − 2
y y 44. A particle of mass m and charge q has an initial velocity
~v = v0 ĵ. If an electric field E~ = E0~i and magnetic field
~
B = B0 î act on the particle, its speed will double after a
(C) (D)
time: √
[7 Jan 2020, √
(II)]
x x
(A) 2mv
qE0
0
(B) 3mv0
qE0 (C) 3mv0
qE0 (D) 2mv0
qE0
Figure 1.321
45. Two identical electric point dipoles have dipole moments
P~1 = P î and P~2 = −P î and are held on the x axis at
|E~ A | distance ‘a’from each other. When released, they move
it[Fig.1.322], the ratio of magnitude of electric field along x axis with the direction of their dipole moments
|E~ B |
~ A and E
E ~ B , respectively, at points A and B due to the remaining unchanged. If the mass of each dipole is ’m’,
remaining portion is: [9 Jan. 2020 (I)] their speed when they are infinitely far apart is:
(A) 21 (B) 18
(C) 17
(D) 18 [06 Sep.
q 2020 (II)]
34 34 54 54 q
(A) P 1
(B) P 1
a
q πε0 ma a
q 2πε0 ma
(C) P
a
2
πε0 ma (D) P
a
2
2πε0 ma
46. In finding the electric field using Gauss law the formula
~
∈0 |A| is applicable. In the formula ∈0 is permittivity
qenc
|E|=
of free space, A is the area of Gaussian surface and qenc
is charge enclosed by the Gaussian surface. This equation
can be used in which of the following situation?
[8 Jan 2020, (I)]

(A) Only when the Gaussian surface is an equipoten-


Figure 1.322 tial surface.
(B) Only when the Gaussian surface is an equipoten-
~ is constant on the surface.
tial surface and |E|
(C) Only when |E|=~ constant on the surface.
41. An electric dipole of moment p~ = (î − 3ĵ + 2k̂) × 10−29
C.m is at the origin (0, 0, 0). The electric field due to this (D) For any choice of Gaussian surface.


47. An electric field E = 4xî − y 2 + 1 ĵ N/C passes through

dipole at ~r = +î + 3ĵ + 5k̂ (note that ~r · p~ = 0 ) is parallel
to: [9 Jan. 2020 (I)] the box shown in Fig.1.324. The flux of the electric field
(A) (+î − 3ĵ − 2k̂) (B) (−î + 3ĵ − 2k̂) through surfaces ABCD and BCGF are marked as φ1 and
φ11 respectively. The difference between (φ1 − φ11 ) is (in
(C) (+î + 3ĵ − 2k̂) (D) (−î − 3ĵ + 2k̂)
N m2 /C ... [9 Jan 2020 (II)]


42. A particle of mass m and charge q is released from rest in


a uniform electric field. If there is no other force on the
48. A uniformly charged disc of radius R having surface charge
particle, the dependence of its speed v on the distance x
density σ is placed in the xy plane with its center at the
travelled by it is correctly given by[Fig.1.323] (graphs are
origin. Find the electric field intensity along the z-axis at
schematic and not drawn to scale) [8 Jan. 2020 (II)]
a distance Z from
 origin:
 [27-Aug.-2021 (I)]
(A) E = 2ε0 Z 2 +R2 + Z 2
σ 1 1

43. Two infinite planes each with uniform surface charge den-
 
(B) E = 2ε 1
2σ  (Z 2 +R2 )1/2 + Z
0

sity +σ are kept in such a way that the angle between


them is 30◦ . The electric field in the region shown be- (C) E = 2εσ0 1 + (Z 2 +R Z
2 )1/2

tween them is given by: [7 Jan. 2020, (I)]


 
(D) E = 2ε0 1 − (Z 2 +R2 )1/2
σ Z
148 CHAPTER 1. ELECTRIC CHARGE AND FIELD

Figure 1.326

move through an equal distance in this uniform electric


field. Neglecting the effect of gravity, the ratio t2 /t1 is
nearly equal to [1997]
Figure 1.324 1/2 1/2
(A) 1 (B) (mp /me ) (C) (me /mp ) (D) 1836

5. A non-conducting solid sphere of radius R is uniformly


49. Figure 1.325 shows a rod AB, which is bent in a 120◦
charged. The magnitude of the electric field due to the
circular are of radius R. A charge (−Q) is uniformly
~ at sphere at a distance r from its centre [1998]
distributed over rod AB. What is the electric filed E (A) increases as r increases, for r < R
the centre

of curvature O ? [27-Aug-2021

(I)] (B) decreases as r increases, for 0 < r < ∞
(A) 8π32 ε3Q
0R
2 ( î) (B) 3 3Q
8πε0 R2 (î) (C) increases as r increases, for R < r < ∞
√ √
(C) 3 3Q
8π 2 ε0 R2 (−î) (D) 3 3Q
16π 2 ε0 R2 (î)
(D) is discontinuous at r = R
6. Three positive charges of equal value q are placed at the
vertices of an equilateral triangle. The resulting lines of
force should be sketched as in[Fig.1.327] [2001]

Figure 1.325
(A) (B)

Section B: JEE Advanced


1. Five point charges, each of value +q, are placed on five
vertices of a regular hexagon of side L. The magnitude
of the force on a point charge of value −q placed at the
centre of the hexagon is [1992]
Kq 2 Kq 2 Kq 2 Kq 2
(A) L2 (B) 4L2 (C) 2L2 (D) 8L2
(C) (D)
2. Two point charges +q and −q are held fixed at (−d, 0)
and (d, 0) respectively of a(x, y) coordinate system , then Figure 1.327
[1995]
~ at all points on the x-axis has
(A) The electric field E
the same direction 7. A point charge ‘q’ is placed at a point inside a hollow
(B) E~ at all points on the Y - axis is along ~i conducting sphere. Which of the following electric lines of
(C) Work has to be done in bringing a test charge from force pattern is correct[Fig.1.328]? [2003]
infinity to the origin
8. In Fig.1.329, charges q1 and −q1 are inside a Gaussian
(D) The dipole moment is 2qd directed along ~i
surface. Where as charge q2 is outside the surface. Electric
3. A metallic solid sphere is placed in a uniform electric
field on the Gaussian surface will be [2004]
field. The lines of force follow the path(s) shown in
Fig.1.326 as [1996] (A) only due to q2
(A) 1 (B) 2 (C) 3 (D) 4 (B) zero on the Gaussian surface
(C) uniform on the Gaussian surface
4. An electron of mass me , initially at rest, moves through (D) due to all
a certain distance in a uniform electric field in time t1 . A 9. Six charges of equal magnitude are placed at six corners
proton of mass mp , also, initially at rest, takes time t2 to of a regular hexagon[Fig.1.330]. Find arrangement the
1.28. EXERCISES AND PROBLEMS 149

11. Consider a neutral conducting sphere. A positive point


charge is placed outside the sphere. The net charge on
the sphere is then, [2007]
(A) negative and distributed uniformly over the sur-
face of the sphere
(B) negative and appears only at the point on the
(A) (B)
sphere closest to the point charge
(C) negative and distributed non-uniformly over the
entire surface of the sphere
(D) zero
12. A disk of radius a/4 having a uniformly distributed
charge 6C is placed in the x-y plane with its centre at
(−a/2 0,0 ). A rod of length ’a’ carrying a uniformly
distributed charge 8C is placed on the x-axis from
(C) (D) x = a/4 to x = 5a/4[Fig.1.332]. Two point charges −7C
and 3C are placed at (a/4, −a/4, 0) and (−3a/4, 3a/4, 0)
Figure 1.328 respectively. Consider a cubical surface formed by six
surfaces x = ±a/2, y = ±a/2, z = ±a/2. The electric flux
this cubical surface is- [2009]
(A) −2C
ε0 (B) 2C
ε0 (C) 10C
ε0 (D) 12C
ε0

Figure 1.329

charges in order PQRSTU which produce double electric


field at centre as compared to electric field produce by
single charges +q at R [2004]
(A) + + + − −− (B) + − + + −
(C) − + + + − (D) − + + − − Figure 1.332

13. Under the influence of the Coulomb field of charge +Q,


a charge −q is moving around it in an elliptical orbital.
Find out the correct statement(s) - [IIT-2009]
(A) The angular momentum of the charge −q is con-
stant
(B) The linear momentum of the charge −q is constant
(C) The angular velocity of the charge −q is constant
(D) The linear speed of the charge −q is constant
Figure 1.330
14. A few electric field lines for a system of two charges Q1 and
Q2 fixed at two different points on the x-axis are shown
in the Fig.1.333. These lines suggest that [2010]
10. Three large charged sheets having surface charge density (A) |Q1 | > |Q2 |
as shown in the Figure1.331. The sheets are placed paral- (B) |Q1 | < |Q2 |
lel to XY plane. Then electric field at point P - [2005] (C) at a finite distance to the left of Q1 the electric
field is zero
(D) at a finite distance to the right of Q2 the electric
field is zero
15. A uniformly charged thin spherical shell of radius R
carries uniform surface charge density of σ per unit area.
It is made of two hemispherical shells, held together
Figure 1.331 by pressing them with force F (see Figure1.334). F is
proportional to [IIT 2010]
(A) −4σ
0 k (B) 4σ
0 k (C) 2σ
0 k (D) − 2σ
0 k (A) ε0 σ R
1 2 2
(B) ε0 σ R
1 2 1 σ2
(C) ε0 R σ2
(D) ε10 R2
150 CHAPTER 1. ELECTRIC CHARGE AND FIELD

(A) The net electric flux crossing the plane x = +a/2


is equal to the net electric flux crossing the plane
x = −a/2
(B) The net electric flux crossing the plane y = +a/2
is more than the net electric flux crossing the plane
y = −a/2.
(C) The net electric flux crossing the entire region is
ε0 .
q

(D) The net electric flux crossing the plane z = +a/2


is equal to the net electric flux crossing the plane
x = +a/2.
Figure 1.333

Figure 1.334
Figure 1.336

19. Two non-conducting solid spheres of radii R and 2R,


16. A tiny spherical oil drop carrying a net charge q is having uniform volume charge densities ρ1 and ρ2 re-
balanced in still air with a vertical uniform electric field spectively, touch each other . The net electric field at a
−1
of strength 81π
7 × 10 Vm
5
. When the field is switched distance 2R from the centre of the smaller sphere, along
off, the drop is observed to fall with terminal velocity the line joining the centres of the spheres, is zero. The
2 × 10−3 ms−1 Given g = 9.8 ms−2 , viscosity of the air ratio (ρ1 /ρ2 ) can be [2013]
= 1.8 × 10−5 Nsm−2 and the density of oil = 900 kg m−3 , (A) −4 (B) −(32/25) (C) (32/25) (D) 4
the magnitude of q is [2010]
(A) 1.6 × 10−19 C (B) 3.2 × 10−19 C
20. Let E1 (r), E2 (r) and E3 (r) be the respective electric fields
(C) 4.8 × 10−19 C (D) 8.0 × 10−19 C
at a distance r from a point charge Q, an infinitely long
wire with constant linear charge density λ, and an infinite
17. Consider an electric field E = E0 x, where E0 is a con- plane with uniform surface charge density σ. If E1 (r0 ) =
stant. The flux through the shaded area (as shown in the E2 (r0 ) = E3 (r0 ) at a given distance r0 , then [2014]
Figure1.335) due to this field is [2011]
(A) Q = 4σπ 2
 0 (B) r0 = 2πσ
λ

(C) E1 r20 = 2E2 r0


(D) E2 2 = 4E3
r0 r0
  
2 2

21. Charges Q, 2Q and 4Q are uniformly distributed in three


dielectric solid spheres 1,2 and 3 of radii R/2, R and 2R
respectively, as shown in Figure1.337. If magnitudes of
the electric fields at point P at a distance R from the
center of spheres 1,2 and 3 are E1 , E2 and E3 respectively,
then [2014]
(A) E1 > E2 > E3 (B) E3 > E1 > E2
Figure 1.335 (C) E2 > E1 > E3 (D) E3 > E2 > E1

√ 2
22. A long cylindrical shell carries positive surface charge σ
(A) 2E0 a2 (B) 2E0 a2 (C) E0 a2 (D) E√
0a
2 in the upper half and negative surface charge −σ in the
lower half. The electric field lines around the cylinder
will look like Figure1.338 given in : (figure are schematic
18. A cubical region of side a has its center at the ori-
and not drawn to scale) [2015]
gin. It encloses three fixed point charges[Fig.1.336], −q
at (0, −a/4, 0), +3q at (0, 0, 0) and −q at (0, +a/4, 0).
Choose the correct options(s) Integer Answer Type Questions
1.28. EXERCISES AND PROBLEMS 151

25. An infinitely long solid cylinder of radius R has a uniform


volume charge density ρ. It has a spherical cavity of radius
R/2 with its center on the axis of the cylinder, as shown
in the Figure 1.339. The magnitude of the electric field at
the point P, which is at a distance 2R from the axis of the
23ρR
cylinder, is given by the expression 16k” 0
. The value of k
is [2013]

(a) sphere 3 (b) sphere 2

Figure 1.339

26. An infinitely long thin non-conducting wire is parallel to


the z-axis and carries a uniform line charge density λ. It
pierces a thin non-conducting spherical shell of radius R
in such a way that the arc PQ subtends an angle 120◦
at the center O of the spherical shell, as shown in the
(c) sphere 3
Figure1.340. The permittivity of free space is 0 . Which
Figure 1.337 of the following statements is (are) true? √ [2018]
(A) The electric flux through the shell is 3R/λε0
(B) The z-component of the electric field is zero at all
the points on the surface of the shell √
(C) The electric flux through the shell is 2R/λε0
(D) The electric field is normal to the surface of the
shell at all points

(A) (B)

(C) (D)

Figure 1.338

23. A solid sphere of radius R has a charge Q distributed in


its volume with a charge density ρ = κra , where κ and a
are constants and r is the distance from its centre. If the Figure 1.340
electric field at r = R2 is 18 times that at r = R, find the
value of a. [2009]
24. Four point charges, each of +q, are rigidly fixed at the four 27. The electric field E is measured at a point P(0, 0, d)
comers of a square planar soap film of side ’a’. The surface generated due to various charge distributions and the
tension of the soap film is y. The system of charges and dependence of E on d is found to be different for different
h 2 i1/N charge distributions. List-I contains different relations
planar film are in equilibrium, and a = k qγ , where between E and d. List-II describes different electric
’k’ is a constant. Then N is charge distributions, along with their locations. Match
152 CHAPTER 1. ELECTRIC CHARGE AND FIELD

the functions in List-I with the related charge distribu- that of the shell. Here, center of the cylinder is a point
tions in List-II. [2018] on the axis of the cylinder which is equidistant from its
top and bottom surfaces. Which of the following option(s)
List-I List-II is/are correct? [ ε0 is the permittivity of free space]
P. E is independent of d 1. A point charge Q at the [2019]
origin (A) If h > 2R and r = 3R/5 then φ = Q/5ε0
Q. E ∝ d1 2. A small dipole with point (B) If h > 2R and r > R then φ = Q/ε0
charges Q at (0, 0, 1) and (C) If h < 8R/5 and r = 3R/5 then φ = 0
−Q (D) If h > 2R and r > 4R/5 then φ = Q/5ε0
R. E ∝ d12 at (0, 0, −1). Take 2 |< d 30. Two identical non-conducting solid spheres of same mass
S. E ∝ d1x 3. An infinite line charge and charge are suspended in air from a common point
coincident with the x-axis, by two non-conducting, massless strings of same length.
with uniform linear charge At equilibrium, the angle between the strings is α. The
density λ. 4. Two infinite spheres are now immersed in a dielectric liquid of density
wires carrying uniform lin- 800kgm−3 and dielectric constant 21 . If the angle be-
ear Charge density paral- tween the strings remains the same after the immersion,
lel to the x-axis. The one then [2020]
along (y = 0, z = I) has a (A) electric force between the spheres remains un-
charge density +λ and the changed
one along (y = 0, z = −1) (B) electric force between the spheres reduces
has a charge density −λ. (C) mass density of the spheres is 840kgm−3
Take 2λ << d 5 . Infinite (D) the tension in the strings holding the spheres re-
plane charge coincident with mains unchanged
31. A circular disc of radius R carries surface charge density
the xy-plane with uniform
σ(r) = σ0 1 − Rr , where σ0 is a constant and r is the
surface charge density
distance from the center of the disc. Electric flux through
(A) P → 5; Q → 3, 4; R → 1; S → 2 a large spherical surface that encloses the charged disc
(B) P → 5; Q → 3; R → 1, 4; S → 2 completely is φ0 . Electric flux through another spherical
(C) P → 5; Q → 3; R → 1, 2; S → 4 surface of radius R4 and concentric with the disc is φ.
(D) P → 4; Q → 2, 3; R → 1; S → 5
Then the ratio φφ0 is... [2020]
28. An electric dipole with moment √ p0
2
(î + ĵ) is held fixed at
the origin O in the presence of an uniform electric field
of magnitude E0 . If the potential is constant on a circle
of radius R centered at the origin as shown in Fig.1.341, 1.29 Answer Keys and Solutions
then the correct statement(s) is/are: (ε0 is permittivity
of free space. R  dipole size) [2019] 1.29.1 Conceptual Questions
1. A plastic ruler is suspended by a thread and then rubbed
with a cloth. The ruler is negatively charged. Now bring
the charged comb close to the ruler. If the ruler is repelled
by the comb, then the comb is negatively charged. If the
ruler is attracted by the comb, then the comb is positively
charged.

2. The clothing becomes charged as a result of being tossed


about in the dryer and rubbing against the dryer sides
and other clothes. When you put on the charged object
(shirt), it causes charge separation (polarization) within
Figure 1.341 the molecules of your skin (see Fig. 16-9), which results
in attraction between the shirt and your skin.

(A) The magnitude of total electric field on any two 3. Fog or rain droplets tend to form around ions because
points of the circle will be same water is a polar molecule, with a positive region and a
(B) Total electric field at point B is E~B = 0 negative region. The charge centers on the water molecule

~
(C) Total electric field at point A is EA = 2E0 (î + ĵ) will be attracted to the ions or electrons in the air.
 1/3
(D) R = 4π∈p00 E0 4. A plastic ruler that has been rubbed with a cloth is
29. A charged shell of radius R carries a total charge Q. Given charged. When brought near small pieces of paper, it
φ as the flux of electric field through a closed cylindrical will cause separation of charge (polarization) in the bits
surface of height h, radius r and with its center same as of paper, which will cause the paper to be attracted to the
1.29. ANSWER KEYS AND SOLUTIONS 153

ruler. A small amount of charge is able to create enough


electric force to be stronger than gravity. Thus the pa-
+++++
per can be lifted. On a humid day this is more difficult +
because the water molecules in the air are polar. Those –+
–+
–+
polar water molecules will be attracted to the ruler and –+ –+
to the separated charge on the bits of paper, neutralizing Nonconductor
the charges and thus reducing


the attraction.


       Figure 1.343: A charged object brought near a nonconductor
5. The forces acting on point 

charge +q are


shown in the causes a charge separation within the nonconductor’s molecules.
Fig.1.342. The force acting on point charge +q due to
point charge −Q is along the line joining them and di-
rected toward −Q. The force acting on point charge +q roll toward the rod.
due to point charge +Q is along the line joining them and
directed away from point charge  +Q. 9. It is possible only if one of them has very large positive
(a) Because point charges
 +Q and −Q are equal in mag- charge as compared to other. Because a metal sphere is
nitude, the forces
 due

to these charges
 are equal and their a conductor, the proximity of a positively charged ball

sum (the net force on charge +q ) will be to the right . (not necessarily a conductor), will induce a redistribution
Note that the vertical components of these forces add up of charges on the metal sphere with the surface nearer
to zero. the positively charged ball becoming negatively charged.
(b) Because no other charged objects are nearby, the forces Because the negative charges on the metal sphere are
acting on this system of three point charges are internal closer to the positively charged ball than are the positive
forces and the net force acting on the system is zero . charges on the metal sphere, the net force will be attrac-
tive.
r
F+Q
10. The net charge on a conductor is the sum of all ofthe
positive and negative charges in the conductor. If a
 neutral conductor has extra electrons added to it, then
+q the net charge is negative. If a neutral conductor has
electrons removed from it, then the net charge is positive.
If a neutral conductor has the same amount of positive
 r
F−Q and negative charge, then the net charge is zero. The
free charges” in a conductor are electrons that can move

1 +Q 2 −Q about freely within the material because they are only
loosely bound to their atoms. The “free electrons” are
Figure 1.342 also referred to as “conduction electrons”. A conductor
 
may have a zero net charge but still have substantial free

charges.
6. (a) Connect the metal sphere to ground; bring the insulat-
ing rod near the metal sphere and disconnect the sphere
from ground; then remove the insulating rod. The sphere 11. (a) The charged rod pulls charge of the opposite sign
will be negatively charged. onto the metal ball. This leaves the gold leaf and metal
(b) Bring the insulating rod in contact with the metal tube with a net charge of the same sign as the rod. Since
sphere; some of the positive charge on the rod will be the gold leaf and the tube have charge of the same sign,
transferred to the metal sphere. they repel. (b) When the rod is removed the charge on
the ball spreads back over the tube and leaf and they

7. The part of the paper near the charged rod becomes become neutral again. There is no net charge and no
polarized–the negatively charged electrons in the paper electrical force so the leaf hangs vertically. (c) When
&
are attracted to the positivelyE charged rod and move to- the charged rod touches the ball it transfers charge to
ward it within their molecules[Fig.1.343]. The attraction it. This net charge spreads over the ball, tube and
occurs because the negative charges in the paper are closer leaf. When the rod is removed this net charge stays
to the positive rod than are the positive charges in the pa- on the leaf and tube and they continue to repel. The
per; therefore, the attraction between the unlike charges gold leaf continues to hang at an angle away from the tube.
is greater than the repulsion between the like charges.
8. Because the can is grounded, the presence of the nega- 12. The balloon has been charged. The excess charge on
tively charged plastic rod induces a positive charge on it. the balloon is able to polarize the water molecules in the
The positive charges induced on the can are attracted, stream of water, similar to Fig. 16–9. This polarization
via the Coulomb interaction, to the negative charges on results in a net attraction of the water toward the balloon,
the plastic rod. Unlike charges attract, so the can will so the water stream curves toward the balloon.
154 CHAPTER 1. ELECTRIC CHARGE AND FIELD

13. Coulomb’s law and Newton’s law are very similar in 21. The electric field is strongest to the right of the positive
form. When expressed in SI units, the magnitude of charge (on the line connecting the two charges), because
the constant in Newton’s law is very small, while the the individual fields from the positive charge and negative
magnitude of the constant in Coulomb’s law is quite large. charge both are in the same direction (to the right) at
Newton’s law says the gravitational force is proportional that point, so they add to make a stronger field. The
to the product of the two masses, while Coulomb’s law electric field is weakest to the left of the positive charge,
says the electrical force is proportional to the product of because the individual fields from the positive charge and
the two charges. Newton’s law produces only attractive negative charge are in opposite directions at that point,
forces, since there is only one kind of gravitational mass. so they partially cancel each other. Another indication is
Coulomb’s law produces both attractive and repulsive the spacing of the field lines. The field lines are closer to
forces, since there are two kinds of electrical charge. each other to the right of the positive charge and farther
apart to the left of the positive charge.

14. Assume that the charged plastic ruler has a negative 22. The direction of the field is defined to be the direction of
charge residing on its surface. That charge polarizes the the force on a positively charged test particle. Positive
charge in the neutral paper, producing a net attractive charges always move away from other +ve charges and
force. When the piece of paper then touches the ruler, the towards −ve charges.
paper can get charged by contact with the ruler, gaining 23. A negatively charged particle placed in a uniform electric
a net negative charge. Then, since like charges repel, the field is accelerated in the direction opposite the direction
paper is repelled by the comb. ~ and so the upward acceleration of the electron tells
of E,
you that E ~ must have a component that is directed verti-
cally downward. (There might also be a component paral-
15. For the gravitational force, we don’t notice it because the
lel to the initial motion of the electron. This would change
force is very weak, due to the very small value of G, the
the electron’s speed but not its direction.) If the electron
gravitational constant, and the relatively small value of ~ must have a component that
ordinary masses. For the electric force, we don’t notice it is accelerated downward, E
because ordinary objects are electrically neutral to a very is directed vertically upward.
high degree. We notice our weight (the force of gravity) 24. See Fig.1.344. At point A, the direction of the net force
due to the huge mass of the Earth, making a significant on a positive test charge would be down and to the left,
gravity force. We notice the electric force when objects parallel to the nearby electric field lines. At point B, the
have a net static charge (like static cling from the clothes direction of the net force on a positive test charge would
dryer), creating a detectable electric force. be up and to the right, parallel to the nearby electric
field lines. At point C, the net force on a positive test
16. A field is a set of values, each value associated with a charge would be 0. In order of decreasing field strength,
position in space surrounding one or more field sources. the points would be ordered A, B, C.

17. It is impossible to deal with the interactions of moving


charged particles without the field concept, and it is often
easier to deal with fields than with distributions of charge.

18. The test charge creates its own electric field, The mea-
sured electric field is the sum of the original electric field
plus the field of the test charge. If the test charge is small,
then the field that it causes is small. Therefore, the ac-
tual measured electric field is not much different than the
Figure 1.344
original field.

19. A negative test charge could be used. For the purposes of 25. Electric field lines show the direction of the force on a test
defining directions, the electric field might then be defined charge placed at a given location. The electric force has
as the OPPOSITE of the force on the test charge, divided a unique direction at each point. If two field lines cross,
by the test charge. Equation 16 − 3 might be changed to it would indicate that the electric force is pointing in two

→ −→
E = − F /q, q < 0 directions at once, which is not possible.

20. A scalar field, such as the temperature at any position 26. From rule 1: A test charge would be either attracted
in the space of interest, specifies a scalar value (magni- directly toward or repelled directly away from a point
tude only) at each position. A vector field, such as the charge, depending on the sign of the point charge. So
gravitational field, specifies a vector value (magnitude and the field lines must be directed either radially toward or
direction) at each position. radially away from the point charge.
1.29. ANSWER KEYS AND SOLUTIONS 155

From rule 2: The magnitude of the field due to the point not change. (b) Again the field line flux does not change
charge only depends on the distance from the point charge. because the enclosed charge does not change. (However,
Thus the density of the field lines must be the same at any in either case the field line pattern changes when the elec-
location around the point charge for a given distance from trons are placed outside the balloon.)
the point charge.
33. If there are no charged particles inside the cavity, then the
From rule 3: If the point charge is positive, then the field
electric field is zero inside the cavity, no matter what the
lines will originate from the location of the point charge.
charge distribution is outside the sphere. The electric field
If the point charge is negative, then the field lines will end
inside the cavity can be nonzero when there are charged
at the location of the point charge. Based on rules 1 and
particles inside the cavity. In either case, the electric field
2, the lines are radial and their density is constant for a
is zero within the conducting material of the sphere itself.
given distance. This is equivalent to saying that the lines
must be symmetrically spaced around the point charge. 34. One reason that it took such a long time to understand the
electrostatic force may have been because it was not ob-
27. The two charges are located as shown in Fig.1.345.
served as frequently as the gravitational force. All massive
(a) If the signs of the charges are opposite, then the point
objects are acted on by the gravitational force; however,
on the line where E = 0 will lie to the left of Q. In that
only objects with a net charge will experience an electro-
region the electric fields from the two charges will point
static force.
in opposite directions, and the point will be closer to the
smaller charge. 35. The two are proportional to each other. The field line
(b) If the two charges have the same sign, then the flux has an arbitrary value, depending on how many lines
are drawn to represent a certain amount of charge. The
H−→ −→
electric flux is uniquely defined by Φ = E .d A = Qencl
ε

36. The net charge on large objects is always very close to


zero. Hence the most obvious force is the gravitational
Figure 1.345
force.
point on the line where E = 0 will lie between the two 37. The accumulation of static charge gives the individual
charges, closer to the smaller charge. In this region, the hairs a charge. Since like charges repel and because the
electric fields from the two charges will point in opposite electrostatic force is inversely proportional to the charges
directions. separation distance squared, the hairs arrange themselves
in a manner in which they are as far away from each other
as possible. In this case that configuration is when the
28. We assume that there are no other forces (like gravity) act- hairs are standing on end.
ing on the test charge. The direction of the electric field
line shows the direction of the force on the test charge. 38. As the garment is dried it acquires a charge from tum-
The acceleration is always parallel to the force by New- bling in the dryer and rubbing against other clothing.
ton’s second law, so the acceleration lies along the field When I put the charged garment on it causes a redis-
line. If the particle is at rest initially and then is released, tribution of the charge on my skin and this causes the
the initial velocity will also point along the field line, and attractive electric force between the garment and my skin.
the particle will start to move along the field line. How-
ever, once the particle has a velocity, it will not follow the
field line unless the line is straight. The field line gives 1.29.2 Problems
the direction of the acceleration, or the direction of the
change in velocity. Discrete Charge Distributions

29. No, because the forces exerted on the two charged regions
1. APPROACH Since, total charge is given therefore, to
of the dipole are equal in magnitude but opposite in di-
determine the number of electrons, apply the principle of
rection, making the vector sum of forces zero.
quantization of charge: q = ne
30. No. The acceleration is not constant (it is centripetal and SOLUTION Here, charge of each electron is
always changing direction). Therefore the electric force e = 1.602 × 10−19 C, q = 1 C, therefore-
exerted on the electron must be nonconstant, implying
q 1C
the electric field is not uniform. n= = = 6.18 × 1018 electrons
e 1.602 × 10−19 C
31. The electric flux depends
 only on the charge enclosed by 2. The charge acquired by the plastic rod is an integral num-
the Gaussian surface Φ = ε0 , not on the shape of the
Qencl
ber of electronic charges, that is, q = n(−e). Relate the
surface. ΦE will be the same for the cube as for the sphere. charge acquired by the plastic rod to the number of elec-
trons transferred from the wool shirt:
32. (a) Because the amount of charge enclosed by the balloon q
does not change, the field line flux through its surface does q = n(−e) ⇒ n=
−e
156 CHAPTER 1. ELECTRIC CHARGE AND FIELD

Substitute numerical values and evaluate n: The number of atoms in the cube is the ratio of the mass
of the cube to the mass of an aluminum atom:
−0.80µC
n= = 5.0 × 1012
−1.602 × 10−19 C mcube ρAl Vcube
Natoms = =
mAl atom mAl atom
electrons
The mass of an aluminum atom is its molar mass divided
3. APPROACH We can find the number of coulombs of
by Avogadro’s number:
positive charge there are in 1.00 kg of carbon from Q =
6nC e, where nC is the number of atoms in 1.00 kg of car- MAl
bon and the factor of 6 is present to account for the pres- mAl atom =
NA
ence of 6 protons in each atom. We can find the number
of atoms in 1.00 kg of carbon by setting up a proportion Substituting and simplifying yields:
relating Avogadro’s number, the mass of carbon, and the
molecular mass of carbon to nC . ρAl Vcube ρAl Vcube NA
Natoms = MAl
=
SOLUTION Express the positive charge in terms of the NA
MAl
electronic charge, the number of protons per atom, and
the number of atoms in 1.00 kg of carbon: Substitute for Natoms in equation (1.169) and simplify to
Qaccumulated
Nrem e
Q = 6nc e =  
Nini Nelectrons per atom
ρAl Vcube NA
obtain: MAl
Using a proportion, relate the number of atoms in 1.00 Qaccumulated MAl
kg of carbon nC , to Avogadro’s number and the molecular =
Nelectrons ρAl Vcube eNper atom
mass M of carbon:
Substitute numerical values and evaluate NNrem :
ini
nC mC NA mc you get-
= ⇒ nc = Nrem
NA M M ≈ 1.99 × 10−15%
Nini
Substitute for nC to obtain:
(b) Express the ratio of the mass of the electrons removed
6NA mC e to the mass of the cube:
Q=
M
mrem Nrem melectron
Substitute numerical values and evaluate Q: =
mcube ρAl Vcube
6(6.022 × 1023 atoms
mo1 )(1.00 kg)(1.602 × 10
−19
C) From (a), the number of electrons removed is given by:
Q= kg
0.01201 mol
Qaccumulated
= 4.82 × 107 C Nrem =
e
4. APPROACH 1 mol of electrons = 6.02 × 1023 electrons, Substituting and simplifying yields:
i.e., NA (Avogadro’s number) electrons.
Now, apply the principle of quantization of charge q = ne, mrem
Qaccumulated
melectron
e
with n = NA = 6.02 × 102 3 electrons mcube
=
ρAl Vcube
Qaccumulated melectron
SOLUTION 1 faraday = NA e =
eρAl Vcube

= 6.02 × 1023 × (1.602 × 10−19 C) = 96470 C Substitute numerical values and evaluate mrem
:

mcube

5. APPROACH (a) The percentage of the electrons origi- mrem (2.50pC) 9.109 × 10−31 kg

nally in the cube that was removed can be found from the = g

mcube (1.602 × 10−19 C) 2.70 cm (1.00cm3 )
ratio of the number of electrons removed to the number 3

of electrons originally in the cube. (b) The percentage de- ≈ 5.26 × 10−19 %
crease in the mass of the cube can be found from the ratio
of the mass of the electrons removed to the mass of the 6. APPROACH (a) The required time is the ratio of the
cube. charge that accumulates to the rate at which it is deliv-
SOLUTION (a) Express the ratio of the electrons re- ered to the conductor. ( b ) We can use the definition of
moved to the number of electrons originally in the cube: power to find the power rating of the light beam.
SOLUTION (a) The required time is the ratio of the
Nrem
Qaccumulated
charge that accumulates to the rate at which it is deliv-
= e
(1.169) ered:
Nini N electrons
Natoms ∆q ∆q
∆t = =
per atom I dq/dt
1.29. ANSWER KEYS AND SOLUTIONS 157

Substitute numerical values and evaluate ∆t : (b) Because these are action and reaction forces, we can
1.50nC apply Newton’s 3rd law to obtain:
∆t = 
1.00 × 106 electrons −
→ −

 
−19 C
s 1.602 × 10 electron F 2,1 = − F 1,2 = −(24mN )î
1h
= 9.363 × 103 × = 2.60h (c) If q2 is −6.0µC, the force between q1 and q2 is attrac-
3600s tive and both force vectors are reversed:
(b) The power rating of the light beam is the rate at which 9 2 2

it delivers energy: 8.988× | 10 N · m /C (4.0µC)(−6.0µC)
F~1,2 = 2

∆E (3.0m)
P =
∆t
The energy delivered by the beam is the product of the = −(24mN )î
energy per electron, the electron current (that is, the num- and

→ −

ber of electrons removed per unit time), and the elapsed F 2,1 = − F 1,2 = (24mN )î
time:
∆E = Eper electron Ielectron ∆t 9. APPROACH q2 exerts an attractive electric force F~2,1
Substituting for ∆E in the expression for P and simplify- on point charge q1 and q3 exerts a repulsive
&
electric force
ing yields: F~3,1 on point charge q1 [Fig.1.347]. We can find the net
&
electric force on q1 by adding these forces (that is, by
Eper electron Ielectron ∆t using the superposition principle).
P =
∆t SOLUTION Express the net force acting on q1 :
= Eper electron Ielectron ฀ ฀
F3,1 3  2 F2,1 -1 0 1 2 3
Substitute numerical values and evaluate P: x, m
q1 6.0P C q2 4.0 PC q3 6.0PC
1.602 × 10−19 J
 
eV
P = 1.3 ×
electron eV & &
Figure 1.347 &

electrons
 
× 1.00 × 106
s −
→ →&
− −

F 1 = F 2,1 + F 3,1
= 2.1 × 10−13 W



&
Express the force that q2 exerts on q1 
7. APPROACH The force on one proton is F = kqr12q2
away from the other proton. −
→ k |q1 kq2 |
F 2,1 = & 2 î
SOLUTION The magnitude of above force, r2,1 

 1.60 × 10−19 C 2
 
8.99 × 109 N · m/C2 = 57.5 N Express the force that q3 exerts on§¨ q1 : ·
¸
2 × 10−15 m ¨ ¸
© ¹

→ k |q1 kq3 |
8. APPROACH We can find the electric forces the two F 3,1 = & 2 (−î)
charges exert on each by applying Coulomb’s law and r3,1
Newton’s 3rd law. Note that r̂1,2 = î because the vec- & Substitute and simplify §to obtain: ·
tor pointing from q1 to q2 is in the positive x direction. u ˜ P ¨¨ P  P
¸
¸ u 

© ¹
Figure1.346 shows the situation for Parts (a) and (b) −
→ k |q1 kq2 | k |q1 | |q3 |
SOLUTION (a) Use Coulomb’s law to express the force F1= 2 î − 2 î
P r2,1 P r3,1
!
|q2 | |q3 |
= k |q1 | 2 − r2 î
r2,1 3,1


Figure 1.346
Substitute numerical values and evaluate F 1 :
F~1 = 8.988 × 109 PN · m2 /CP2 (6.0µC)


that q1 exerts on q2 : 
4.0µC 6.0µC

P × − î
kq1 q2 (3.0 m)2 P(6.0 m)2
F~1,2 = 2 r̂1,2 P
= 1.5 × 10−2 N î

r1,2

Substitute numerical values and evaluate F~1,2 : 10. APPROACH The third point charge should be placed
at the location at which the forces on the third point
8.988 × 109 N · m2 /C 2 (4.0µC)(6.0µC) charge due to each of the other two point charges cancel.

F~1,2 = î = (24mN )î There can be no such place except on the line between
(3.0m)2
158 CHAPTER 1. ELECTRIC CHARGE AND FIELD

the two point charges. Denote the 2.0µC and 4.0µC point the −2.0µC and 4.0 µC point charges by the numerals 2
charges by the numerals 2 and 4, respectively, and the and 4 , respectively, and the third point charge by the
third point charge by the numeral 3 . Assume that the numeral 3 . Let the +x direction be to the right with the
2.0µC point charge is to the left of the 4.0µC point charge, origin at the position of the −2.0µC point charge and the
let the +x direction be to the right. Then the 4.0µC point 4.0µC point charge be located at x = L.
charge is located at x = L. SOLUTION Apply the condition for translational equi-
SOLUTION Apply the condition for translational equi- librium to the third point charge:
librium to the third point charge: −
→ −

F 4,3 + F 2,3 = 0
F~4,3 + F~2,3 = 0 or (1.171)
or F4,3 = F2,3 (1.172)
F4,3 = F2,3 (1.170)
Letting the distance from the third point charge to the
Letting the distance from the third point charge to the
2.0µC point charge be x, express the force that the 4.0µC
4.0µC point charge be x, express the force that the 4.0µC
point charge exerts on the third point charge:
point charge exerts on the third point charge:
kq3 q4
kq3 q4 F4,3 =
F4,3 = (L + x)2
(L − x)2

The force that the 2.0µC point charge exerts on the third The force that the −2.0µC point charge exerts on the third
charge is given by: point charge is given by:
kq3 q2
kq3 q2 F2,3 =
F2,3 = x2
x2
Substitute in equation (1.170) to obtain: Substitute for F4,3 and F2,3 in equation (1.172) to obtain:

kq3 q4 kq3 q2 kq3 q4 kq3 q2 q4 q2


= 2
= 2
⇒ 2
= 2
(L − x)2 x2 (L + x) x (L + x) x
q4 q2
⇒ = 2 Substituting q4 = 2q2 and rewriting this equation explic-
(L − x)2 x
itly as a quadratic equations yields:
Substituting q4 = 2q2 and rewriting this equation explic-
itly as a quadratic equations yields:
x2 − 2Lx − L2 = 0
x2 + 2Lx − L2 = 0 On simplifying above equation, we get-

On simplifying, we get 2L ± 4L2 + 4L2 √
x= = L ± 2L
√ 2
−2L ± 4L2 + 4L2 √
x= = −L ± 2L The root corresponding to the positive sign between the
2
terms is extraneous because it corresponds to a position
The root corresponding to the negative sign between the to the right of the 2.0µC point charge and is, therefore,
terms is extraneous because it corresponds to a position to not a physically meaningful root. Hence the third point
the left of the 2.0µC point charge and is, therefore, not a charge should be placed a distance equal to 0.41L from the
physically meaningful root. Hence the third point charge −2.0 µC charge on the side away from the 4.0 µC charge.
should be placed between the point charges and a distance
equal to 0.41L away from the 2.0 − µC charge. 12. APPROACH The configuration of the point charges
and the forces on the fourth point charge are shown in
11. APPROACH The third point charge should be placed the figure1.348 as is a coordinate system. From the figure
at the location at which the forces on the third point it is evident that the net force on the point charge q4
charge due to each of the other two point charges can- is along the diagonal of the square and directed away
cel. There can be no such place between the two point from point charge q3 . We can apply Coulomb’s law to
charges. Beyond the 4.0µC point charge, and on the express F~1,4 , F~2,4 and F~3,4 and then add them (that is,
line containing the two point charges, the force due to use the principle of superposition of forces) to find the
the 4.0µC point charge overwhelms the force due to the net electric force on point charge q4
−2.0µC point charge. Beyond the −2.0µC point charge, SOLUTION Express the net force acting on point
and on the line containing the two point charges, however, charge q4 :
we can find a place where these forces cancel because they
are equal in magnitude and oppositely directed. Denote F~4 = F~1,4 + F~2,4 + F~3,4 (1.173)
1.29. ANSWER KEYS AND SOLUTIONS 159

y, cm
Substitute numerical values in equation (1.173) and sim-
F1,4 −

plify to find F 4 :


F 4 = 3.24 × 10−5 N ĵ + 3.24 × 10−5 N î
 
q4 3.00 nC − 1.14 × 10−5 N î − 1.14 × 10−5 N ĵ
 
q2 3.00 nC
5.00
= 2.10 × 10−5 N î + 2.10 × 10−5 N ĵ
 
F2,4
This result tells us that the net force is 2.97 × 10−5 N
F3,4
along the diagonal in the direction away from the −3.0 nC
charge.

13. APPROACH The configuration of the point charges


q1 3.00 nC and the forces on point charge q3 are shown in the
x, cm Fig.1.349 as is a coordinate system. From the geometry
0 q −3.00 nC 5.00
3
of the charge distribution it is evident that the net force
Figure 1.348 on the 2.00 µC point charge is in the negative y direction.
We can apply Coulomb’s law to express F~1,3 and F~2,3 and
then add them (that is, use the principle of superposition
Express the force that point charge q1 exerts on point of forces) to find the net force on point charge q3 .
charge q4 :


→ kq1 q4
F 1,4 = 2 ĵ
r1,4

Substitute
 numerical values and
 evaluate
 F~1,4 :
2
F~1,4 = 8.988 × 10 C2 (3.00nC) (0.0500 m)2 ĵ
9 N·m 3.00nC

= 3.236 × 10−5 N ĵ


Express the force that point charge q2 exerts on point


charge q4 :


→ kq2 q4
F 2,4 = 2 î
r2,4
Figure 1.349


Substitute numerical values and evaluate F 2,4 : 
~ 9 N·m2

3.00nC SOLUTION The net force acting on point charge q3 is
F2,4 = 8.988 × 10 C2 (3.00nC) (0.0500 m)2 î = given by:
−5
F~3 = F~1,3 + F~2,3

3.236 × 10 N î
Express the force that point charge q3 exerts on point
charge q4 : The force that point charge q1 exerts on point charge q3
is:
−→ −
→ −


→ kq3 q4 F 3 = F 1,3 + F 2,3
F 3,4 = 2 r̂ 3,4 , where r̂ 3,4 is a unit
r3,4 F is given by:
vector pointing from q3 to q4 . kq1 q3
F =
r2
Express ~r3,4 in terms of ~r3,1 and ~r1,4 :

~r3,4 = ~r3,1 + ~r1,4 kq1 q3


F = 2
= (0.0500 m)î + (0.0500 m)ĵ r 2

8.988 × 109 N·m
C2 (5.00µC)(2.00µC)
=
Convert −→r 3,4 to r̂ 3,4 : (0.0300 m)2 + (0.0800 m)2
~r3,4 (0.0500 m)î + (0.0500 m)ĵ = 12.32 N
r̂3,4 = =p
|~r3,4 | (0.0500 m)2 + (0.0500 m)2
and
= 0.707î + 0.707ĵ
 

→ 3.00 cm
Substitute θ = tan−1 = 20.56◦
 numerical values2 and evaluate F 3,4 : 8.00 cm
N·m
F~3,4 = 8.988 × 109 The force that point charge q2 exerts on point charge q3
C2
  is:
3.00nC −

(−3.00nC) √ (0.707î + 0.707ĵ) F 2,3 = −F cos θî − F sin θĵ
(0.0500 2 m)2
= − 1.14 × 10−5 N î − 1.14 × 10−5 N ĵ Substitute for F~1,3 and F~2,3 and simplify to obtain:

160 CHAPTER 1. ELECTRIC CHARGE AND FIELD

Substitute for q1 and q2 and simplify to obtain:




F 3 =F cos θî − F sin θĵ − F cos θî
6 2.5
− F sin θĵ √ = 2
(r + 1.25 m) 2 r
= − 2F sin θĵ
Solving for r yields:
Substitute numerical values and evaluate F~3 : r = 2.036 m and r = −0.4386 m Because r < 0 is
unphysical, we’ll consider only the positive root.


F 3 = −2(12.32 N) sin 20.56◦ ĵ
= −(8.65 N)ĵ Use the similar triangles in the diagram to establish the
proportion involving the y coordinate of the electron:
14. APPROACH The positions of the point particles are ye 2.036 m
= ⇒ ye = 0.909 m
shown in Fig.1.350. It is apparent that the electron must 0.50 m 1.12 m
be located along the line joining the two point particles.
Use the similar triangles in the diagram to establish the
Moreover, because it is negatively charged, it must be
proportion involving the x coordinate of the electron:
closer to the particle with a charge of −2.5µC than to
the particle with a charge of 6.0µC, as is indicated in xe 2.036 m
the figure. We can find the x and y coordinates of the = ⇒ xe = 1.82n
1.0 m 1.12 m
electron’s position by equating the two electrostatic forces
acting on it and solving for its distance from the origin. The coordinates of the electron’s position are:
We can use similar triangles to express this radial dis- (xe , ye ) = (−1.8 m, −0.91 m)
tance in terms of the x and y coordinates of the electron.
SOLUTION Express the condition that must be satis-
Substitute for q1 and q2 and simplify to obtain:
6 2.5
√ = 2
(r + 1.25m) 2 r

Solving for r yields: r = 2.036m and r = −0.4386m


Because r < 0 is unphysical, we’ll consider only the pos-
itive root. Use the similar triangles in the diagram to
establish the proportion involving the y coordinate of the
electron:
ye 2.036m
= ⇒ ye = 0.909m
0.50m 1.12m
Use the similar triangles in the diagram to establish the
Figure 1.350
proportion involving the x coordinate of the electron:

fied if the electron is to be in equilibrium: xe 2.036m


= ⇒ xe = 1.82m
1.0m 1.12m
F1,e = F2,e The coordinates of the electron’s position are:

Letting r represent the distance from the origin to the elec- (xe , ye ) = (−1.8m, −0.91m)
tron, express the magnitude of the force that the particle
whose charge is q1 exerts on the electron: 15. APPROACH By considering the symmetry of the array
of charged point particles[Fig.1.351], we can see that the
kq e y component of the force on q is zero. We can apply
F1,e = √ 1
(r + 1.25 m)2 Coulomb’s law and the principle of superposition of forces
to find the net force acting on q.
Express the magnitude of the force that the particle whose SOLUTION Express the net force acting on the point
charge is q2 exerts on the electron: charge q:

→ −
→ −→
k |q2 | e F q = F Q on xaxis,q + 2 F Q at 45◦ ,q
F2,e =
r2
Express the force on point charge q due to the point charge
Substitute and simplify to obtain:
Q on the x axis:
q |q2 | −
→ kqQ
√ 1 = 2 F Q on x axis ,q = 2 î
(r + 1.25 m) 2 r R
1.29. ANSWER KEYS AND SOLUTIONS 161

Q
Q
R

Q q
x

Q
Q

Figure 1.351

Express the net force on point charge q due to the point Figure 1.352
charges at 45◦ :
−→ kqQ Noting that the magnitude of point charge q4 is three
2 F Q at 45◦ ,q = 2 2 cos 45◦ î
R times that of the other point charges and that it is
2 kqQ negative, express F~4,1 :
=√ î
2 R2
−→ F~4,1 = 3C r̂4,1
Substitute for F~Q on x axis ,q and 2 F Q at 45◦ ,q to obtain: a
 
a
  √ 
a√ 2
0− î + 0 − 2√
2 3
ĵ + 0 − 3


→ kqQ 2 kqQ = −3C r
F q = 2 î + √ î a 2
  2  √  2
R 2 R2 a
+ a√32

2 + 2√ 3
kqQ √    √ 
= 2 (1 + 2)î a a a√ 2

î + √ ĵ + k̂
R 2 2 3 3
= 3C
16. APPROACH Let the H+ ions be in the x − y
"  a  r ! #
1 1 2
plane with = 3C î + √ ĵ + k̂
 √  H1 at (0, 0, 0), H2 at (a, 0, 0), and H3 at 2 2 3 3
a a 3
2, 2 ,0 [fig.1.352]. The N−3 ion, with charge q4 in
 q  Substitute in the expression for F~1 to obtain:
our notation, is then at a2 , 2√
a
3
, a 2
3 where a = 1.64 × √ !

→ 1 3
10−10 m. To simplify our calculations we’ll set ke2 /a2 = F 1 = − C î − C î + ĵ
2 2
C = 8.56 × 10−9 N. We can apply Coulomb’s law and the
principle of superposition of forces to find the net force
"    r ! #
1 1 2
acting on each ion. + 3C î + √ ĵ + k̂
2 2 3 3
SOLUTION Express the net force acting on point √
charge q1 : =C 6k̂
−→ −
→ −
→ −

F 1 = F 2,1 + F 3,1 + F 4,1
From symmetry considerations:
Find F~2,1 : √
F~2 = F~3 = F~1 = C 6k̂

→ kq1 q2
F 2,1 = 2 r̂ 2,1 = C(−î) = −C î
r2,1 Express the condition that the molecule is in equilibrium:

Find F~3,1 : F~1 + F~2 + F~3 + F~4 = 0


→ kq3 q1 Solve for and evaluate F~4 :
F 3,1 = 2 r̂ 3,1
r3,1  
 √  F~4 = − F~1 + F~2 + F~3 = −3F~1
a 3
0 − a2 î + 0 −

2 ĵ √
=C = −3C 6k̂
a
√ !
1 3 17. APPROACH Let q represent the point charge at the
= −C î + ĵ ~ due to a point charge
2 2 origin and use Coulomb’s law for E
162 CHAPTER 1. ELECTRIC CHARGE AND FIELD



to find the electric field at x = 6.0 m and −10 m. (b) Evaluate E at x = 2.0 m :
SOLUTION (a) Express the electric field at a point P  
located a distance x from a point charge q: ~ 2
 1 1
E(2.0 m) = 36kN · m /C (î) + (−î)
(2.0 m)2 (6.0 m)2

→ kq
E (x) = 2 r̂ P,0 = (8.0kN/C)î
x


Evaluate this expression for x = 6.0m: (c) Evaluate E at x = 6.0 m :
 
9 N·m2
 

→ 8.988 × 10 C2 (4.0µC) ~ 2
 1 1
E (6.0 m) = î = (1.0kN/C)î E(6.0 m) = 36kN · m /C (î) + (−î)
(6.0 m)2 (6.0 m)2 (2.0 m)2
= (−8.0kN/C)î
~ at x = −10 m :
(b) Evaluate E


  (d) Evaluate E at x = 10 m :
9 N·m2

→ 8.988 × 10 C 2 (4.0µC)  
E (−10 m) = (−î) ~ 1 1
m) = 36kN · m2 /C

(10 m)2 E(10 (î) + (î)
(10 m)2 (2.0 m)2
= (−0.36kN/C)î
= (9.4kN/C)î
(c) The following graph[Fig.1.353] was plotted using a
(e) From symmetry considerations:
spreadsheet program:
~
E(4.0 m) = 0
(f) The following graph[Fig.1.354] was plotted using a
spreadsheet program:

Figure 1.353

18. APPROACH Let q represent the point charges of


+4.0µC and use Coulomb’s law for E ~ due to a point charge Figure 1.354
and the principle of superposition for fields to find the
electric field at the locations specified.
19. APPROACH We can find the electric field at the origin
Noting that q1 = q2 , use Coulomb’s law and the principle
from its definition and the electric force on a point charge
of superposition to express the electric field due to the
placed there using F~ = q E.~ We can apply Coulomb’s law
given charges at point P a distance x from the origin:
to find the value of the point charge placed at y = 3 cm.

→ −
→ −
→ SOLUTION (a) Apply the definition of electric field to
E (x) = E q1 (x) + E q2 (x)
obtain:
kq1 kq2
= 2 r̂ q1 ,P + r̂ q ,P
F~ (0, 0) 8.0 × 10−4 N ĵ

x (8.0 m − x)2 2 ~
E(0, 0) = =
q0 2.0nC
 
1 1
= kq1 r̂ q ,P + r̂ q ,P 5
x2 1 (8.0 m − x)2 2

= 4.0 × 10 N/C ĵ
 
 1 1
= 36kN · m /C 2
r̂q ,P + r̂ q ,P (b) The force on a point charge in an electric field is given
x2 1 (8.0 m − x)2 2 by:
F~ (0, 0) = q E(0,
~ 0)
(a) Apply this equation to the point at x = −2.0 m:
 = (−4.0nC)(400kN/C)ĵ

→ 1
E (−2.0 m) = 36kN · m2 /C

(−î) = (−1.6mN)ĵ
(2.0 m)2
(c) Apply Coulomb’s law to obtain:

1
+ (− î)
(10 m)2 kq(−4.0nC)
(−ĵ) = (−1.60mN)ĵ
= (−9.4kN/C)î (0.030 m)2
& &
1.29. ANSWER KEYS AND SOLUTIONS 163

Solving for q yields: y, cm

(1.60mN)(0.030 m)2
3.0 q1 = 6.0 nC
q=−
(8.988 × 109 N · m2 /C2 ) (4.0nC) r
= −40nC 4.0
θ x , cm
0
20. APPROACH We can compare the electric and gravi-
θ
tational forces acting on an electron by expressing their r
Eq1
ratio. Because the ping pong ball is in equilibrium under
the influence of the electric and gravitational forces acting − 3.0 q2 = 6.0 nC
on it, we can use the condition for transnational equilib-
rium to find the charge that would have to be placed on &
Figure 1.355
it in order to balance Earth’s gravitational force on it. T
SOLUTION (a) Express the magnitude of the electric
force acting on the electron: component of the electric field due to one point charge as
a function of the distance r from either point charge to
Fe = eE the point of interest:
Express the magnitude of the gravitational force acting on & −
→ kq
E x = &2 cos θî
the electron: r T
Fg = me g −

Express E x for both charges:
The ratio of these forces is:

→ kq
Fe eE E x = 2 2 cos θî
= r
Fg mg
Substitute for cos θ and r, substitute numerical values,
Substitute numerical values and evaluate Fe /Fg : and evaluate to obtain:

~ kq 0.040 m 2kq(0.040 m)
−19
 E(4.0 cm)x = 2 2 î = î
Fe 1.602 × 10 C (150 N/C) r r r3 
=
Fg (9.109 × 10−31 kg) (9.81 m/s2 ) 2 8.988 × 109 N · m2 /C2 (6.0nC)(0.040 m)
= 3/2

= 2.69 × 1012 [(0.030 m)2 + (0.040 m)2 ]
or = (34.5kN/C)î = (35kN/C)î
12

Fe = 2.69 × 10 Fg The magnitude and direction of the electric field at x =
Thus, the electric force is greater by a factor of 2.69×1012 . 4.0 cm is:
(b) Letting the upward direction be positive, apply the 35kN/Calong positive x axis.
condition for static equilibrium to the ping pong ball to −→ −

obtain: (b) Apply F = q E to find the force on a point charge q0
Fe − Fg = 0 placed on the x axis at x = 4.0 cm:
Or F~ = (2.0nC)(34.5kN/C)î
mg
− qE − mg = 0 ⇒ q = − = (69µN)î
E
Substitute numerical values and evaluate q: 22. APPROACH If the electric field at x = 0 is zero, both
2.70 × 10−3 kg 9.81 m/s2
  its x and v components must be zero. The only way this
q=− condition can be satisfied with point charges of +5.0µC
150 N/C and −8.0µC on the x axis is if the point charge +6.0µC is
= −0.177mC also on the x axis. Let the subscripts 5, −8, and 6 identify
the point charges and their fields. We can use Coulomb’s
21. APPROACH Fig.1.355 shows the locations of the point law for E~ due to a point charge and the principle of super-
charges q1 and q2 and the point on the x axis at which position for fields to determine where the +6.0µC point
we are to find E.~ From symmetry considerations we can charge should be located so that the electric field at x = 0
conclude that the y component of E ~ at any point on the x is zero.
axis is zero. We can use Coulomb’s law for the electric field SOLUTION Express the electric field at x = 0 in terms
due to point charges and the principle of superposition of the fields due to the point charges of +5.0µC, −8.0µC,
for fields to find the field at any point on the x axis and and +6.0µC:
F~ = q E~ to find the force on a point charge q0 placed on

→ −
→ −
→ −

the x axis at x = 4.0 cm E (0) = E 5µC + E −8µC + E 6µC
SOLUTION (a) Letting q = q1 = q2 , express the x =0
164 CHAPTER 1. ELECTRIC CHARGE AND FIELD

Substitute for each of the fields to obtain: ~ 2:


Substitute numerical values and evaluate E

kq5 kq6 kq−8 ~ 2 = kq2 r̂ 2,P


E 2
r2,P
r̂5 + 2 r̂6 + 2 r̂−8 = 0
r52 r6 r−8
8.988 × 109 N · m2 /C2 (12µC)

)1 =
(2.0 m)2 + (2.0 m)2
kq5 kq6 kq−8
î + 2 (−î) + 2 (−î) = 0
!
r52 r6 r−8 (−2.0 m)î + (−2.0 m)ĵ
p
(2.0 m)2 + (2.0 m)2
q5 q6 q−8 = 13.5 × 103 N/C (−0.707î − 0.707ĵ)

Divide out the unit vector î to obtain: − − 2 =0
r52 r62 r−8
= (−9.54kN/C)î + (−9.54kN/C)ĵ
Substitute numerical values to obtain:
~ 1 and E
Substitute for E ~ 2 and simplify to find E~ p:
5 6 −8 −

− 2− =0 E P = (1.44kN/C)î + (−0.575kN/C)ĵ + (−9.54kN/C)î
(3.0cm)2 r6 (4.0cm) 2

  + (−9.54kN/C)ĵ


Solving for r6 yields: = (−8.10kN/C)î + (−10.1kN/C)ĵ


 ~ P is:
The magnitude of E
 

r6 = 2.4cm p
EP = (−8.10kN/C)2 + (−10.1kN/C)2
= 13kN/C

23. APPROACHP Fig.1.356 shows the electric field vectors ~ p is:


The direction of E
P at the point of interest P due to the two point charges.
~ due to point charges and
 
We can use Coulomb’s law for E −10.1kN/C
~ P. θE = tan−1 = 230◦
the superposition principle for electric fields to find E −8.10kN/C
~ ~
We can apply F = q E to fmd the force on an electron at
(−1.0m, 0). & Note  that the angle
 returned by your calculator for
SOLUTION (a) Express the electric field at (−1.0m, & 0) tan−1 −10.1kN/C
−8.10kN/C is the reference angle and must be
& & increased by 180 to yield θE .


(b) Express and evaluate the force on an electron at point
y, m
P:
F~ = q E
~ P = −1.602 × 10−19 C [(−8.10kN/C)î

2 q2 = 12µ C
+ (−10.1kN/C)ĵ]
1
−15 −15
 
= 1.30 × 10 N î + 1.62 × 10 N ĵ
−2 −1 −

P
x, m Find the magnitude of F :
1 2 3 4
q
−1 E1 2 2
E2
F = (1.30 × 10−15 N) + (1.62 × 10−15 N)
−2 = 2.1 × 10−15 N
q1 = −5.0µ C


Find the direction of F :
& & &
Figure 1.356  1.62 × 10−15 N
 
 θF = tan−1 = 51◦
1.3 × 10−15 N

due to the point charges q1 and q2 : ~P = E


E ~1 + E
~ 2 Substi-
24. APPROACH The electric field of the 4th charged point
tute numerical values and evaluate ~
E1 particle must cancel the sum of the electric fields due
to the other three charged point particles[Fig.1.357]. By
8.988 × 109 N · m2 /C2 (−5.0µC) symmetry, the position of the 4th charged point particle

~ kq1
E1 = 2 r̂1,P = must lie on the vertical center line of the triangle. Using
r1,P (5.0 m)2 + (2.0 m)2
! trigonometry, one can show that√ the center of an equilat-
(−5.0 m)î + (2.0 m)ĵ eral triangle is a distance L/ 3 from each vertex, where
L is the length of the side of the triangle. Note that the x
p
(5.0 m)2 + (2.0 m)2
components of the fields due to the base charged particles
= −1.55 × 103 N/C (−0.928î + 0.371ĵ)

cancel each other, so we only need concern ourselves with
= (1.44kN/C)î + (−0.575kN/C)ĵ the y components of the fields due to the charged point
1.29. ANSWER KEYS AND SOLUTIONS 165

y
particles at the vertices of the triangle. Choose a coordi-
nate system in which the origin is at the midpoint of the q3 = 2 q

base of the triangle, the +x direction is to the right, and


the +y direction is upward. L 3

E4
60
E2 E1
L 3 E3 L 3
q1 60 q2
x
q4 = q '

Figure 1.358

~ 1, E
Substituting for E ~ 2, E
~ 3 , and E
~ 4 yields:

Figure 1.357 k(q) k(q) k(2q)


L 2
cos 60o ĵ + L 2 cos 60o ĵ − L 2 ĵ
( √3 ) ( √3 ) ( √3 )
SOLUTION Express the condition that must be satis- kq 0
+ ĵ = 0
fied if the electric field at the center of the triangle is to L 2
( 2√ )
3
be zero:
~i = 0
X
E 1
i=1to4
Solve for q 0 to obtain: q 0 = q
4
~ 1, E
Substituting for E ~ 2, E
~ 3 , and E
~ 4 yields:
26. APPROACH We can determine the stability of the
k(q) k(q) equilibrium in Part (a) and Part (b) by considering the
o o
 2 cos 60 ĵ +  2 cos 60 ĵ forces the equal point charges q at y = +a and y = −a
L L

3

3 exert on the test charge when it is given a small displace-
k(2q) kq ment along either the x or y axis. (c) The application of
−  2 ĵ + 2 ĵ = 0 Coulomb’s law in Part (c) will lead to the magnitude and
y
L

3
sign of the charge that must be placed at the origin in
order that a net force of zero is experienced by each of the
On simplifying for y, we get- three point charges.
L SOLUTION (a) Because Ex is in the x direction, a pos-
y = ± √ The positive solution corresponds
3 √ itive test charge that is displaced from (0, 0) in either the
to the 4th point particle being a distance L/ 3 above +x direction or the −x direction will experience a force
the base of the triangle, where it produces the same pointing away from the origin and accelerate in the direc-
strength and same direction electric field caused by the tion of the force. Consequently, the equilibrium at (0, 0)
three charges at the corners of the triangle. So
√ the charged is unstable for a small displacement along the x axis.
point particle must be placed a distance L/ 3 below the
If the positive test charge is displaced in the direction of
midpoint of the triangle.
increasing y (the +y direction), the charge at y = +a will
25. APPROACH The electric field of 4th charge must can- exert a greater force than the charge at y = −a, and the
cel the sum of the electric fields due to the other three net force is then in the −y direction; i.e., it is a restoring
charges[Fig.1.358]. Using trigonometry, one can show that √ force. Similarly, if the positive test charge is displaced in
the center of an equilateral triangle is a distance L/ 3 the direction of decreasing y (the −y direction), the charge
from each vertex, where L is the length ofthe side ofthe at y = −a will exert a greater force than the charge at
triangle. The distance from the center point of the triangle y = −a, and the net force is then in the −y direction; i.e.,
to the midpoint of the base is half this distance. Note that it is a restoring force. Consequently, the equilibrium at
the x components of the fields due to the base charges can- (0, 0) is stable for a small displacement along the y axis.
cel each other, so we only need concern ourselves with the (b) Following the same arguments as in Part (a) , one finds
y components of the fields due to the charges at the ver- that, for a negative test charge, the equilibrium is stable
tices of the triangle. Choose a coordinate system in which at (0, 0) for displacements along the x axis and unstable
the origin is at the midpoint of the base of the triangle, for displacements along the y axis.
the +x direction is to the right, and the +y direction is (c) Express the net force acting on the charge at y = +a:
upward. X kqq0 kq 2
SOLUTION Express the condition that must be satis- Fqaty=+a = 2 + = 0 Solve for q0 to obtain:
a (2a)2
fied if theXelectric field at the center of the triangle is to 1
be zero: ~i = 0
E q0 = − q
4
i=1to4
166 CHAPTER 1. ELECTRIC CHARGE AND FIELD

Remarks: In Part (c) , we could just as well have ex- 28. APPROACH From problem 27. The electric field on
pressed the net force acting on the charge at y = −a. Due the x axis, due to equal positive point charges located at
to the symmetric distribution of the charges at y = −a (0, a) and (0, −a) , is
pgiven by Ex = 2kqx(x + a )
2 2 −3/2

and y = +a, summing the forces acting on q0 at the origin We can use T = 2π m/k 0 to express the period of the
does not lead to a relationship between q0 and q. motion of the bead in terms of the restoring constant k 0 .
27. APPROACH The diagram1.359 shows the locations of (a) Express the force acting on the bead when its displace-
point charges q1 and q2 and the point on the x axis at ment from the origin is x:
which we are to find E. ~ From symmetry considerations
~ at any point 2kq 2 x
we can conclude that the y component of E Fx = −qEx = −
on the x axis is zero. We can use Coulomb’s law for the (x2 + a2 )3/2
electric field due to point charges and the principle of su-
perposition of fields to find the field at any point on the Factor a2 from the denominator to obtain:
x axis. We can establish the results called for in Parts (b) 2kq 2 x
and (c) by factoring the radicand and using the approxi- Fx = − 2
a2 ( xa2 + 1)3/2
mation 1 + α ≈ 1 whenever α << 1.
SOLUTION (a) Express the x-component of the elec- 2kq 2
For x << a: Fx = − 3 x
a
That is, the bead experiences a linear restoring force.
(b) Express the period of a simple harmonic oscillator:
r
m
T = 2π
k0

Obtain k 0 from our result in Part (a):

2kq 2
Figure 1.359 k0 =
a3
tric field due to the point charges at y = a and y = −a as Substitute for k 0 and simplify to obtain:
a function of the distance r from either charge to point P : s
s

→ kq m ma3
E x = 2 2 cos θî T = 2π 2kq2 = 2π
r 3
2kq 2
a

Substitute for cos θ and r to obtain:


29. APPROACH We can use Newton’s second law of mo-

→ kq x 2kqx 2kqx tion to find the acceleration of the electron in the uniform
E x = 2 2 î = 3 î = 3/2

r r r (x + a2 )
2 electric field and constant-acceleration equations to find
2kqx the time required for it to reach a speed of 0.01c and the
= 3/2
î distance it travels while acquiring this speed.
(x2 + a2 )
SOLUTION (a) Use data found at the back of your text


The magnitude of E x is: to compute e/m for an electron:
2kqx e 1.602 × 10−19 C
Ex = 3/2 =
(x2 + a2 ) me 9.109 × 10−31 kg
= 1.76 × 1011 C/kg
(b) For |x|<< a, x2 + a2 ≈ a2 , so:
(b) Apply Newton’s second law to relate the acceleration
2kqx 2kqx of the electron to the electric field:
Ex ≈ 3/2
= 3
(a2 ) a
Fnet eE
(c) For x >> a, the charges separated by a would appear a= =
me me
to be a single charge of magnitude 2q. Its field would be
given by Ex = 2kq
x2 . Substitute numerical values and evaluate a:
Factor the radicand to obtain:
1.60/2 × 10−19 C (100 N/C)

−3/2
a2
 
2 a=
Ex = 2kqx x 1 + 2 9.109 × 10−31 kg
x
= 1.759 × 1013 m/s2
a2
 −3/2 2kq
For a << x, 1 + x2 ≈ 1 and: Ex = 2kqx x2 = x2 = 1.76 × 1013 m/s2
1.29. ANSWER KEYS AND SOLUTIONS 167

The direction of the acceleration of an electron is opposite Substitute numerical values and evaluate ∆t:
the electric field.
(c) Using the definition of acceleration, relate the time 0.01 2.998 × 108 m/s

required for an electron to reach 0.01c to its acceleration: ∆t = = 0.3 ms
9.576 × 109 m/s2
v 0.01c 31. APPROACH Because the electric field is in the −y di-
∆t = =
a a rection, the force it exerts on the electron is in the +y
Substitute numerical values and evaluate ∆t: direction. Applying Newton’s second law to the electron
will yield an expression for the acceleration of the electron
(d) Use a constant-acceleration equation to express the
in the y direction. We can then use a constant-acceleration
distance the electron travels in a given time interval:
equation to relate its speed to its acceleration and the dis-
0.01 2.998 × 108 m/s tance it has traveled.

∆t = = 0.1704µs
1.759 × 1013 m/s2 SOLUTION Apply Fy = may to the electron to ob-
P

= 0.2µs tain:
FE − Fg = may
Substitute numerical values and evaluate ∆x : or, because FE = eE and Fg = mg,
eE − mg = may
1
∆x = vi ∆t + a(∆t)2 Solving for ay yields:
2
or, because vi = 0, eE
ay = −g
1 m
∆x = a(∆t)2
2 Use a constant-acceleration equation to relate the speed of
Substitute numerical values and evaluate ∆x: the electron to its acceleration and the distance it travels:
1 vy2 = v02 + 2ay ∆y
1.759 × 1013 m/s2 (0.1704µs)2

∆x =
2
or, because the electron starts at rest,
= 0.3 m
vy2 = 2ay ∆y ⇒ vy = 2ay ∆y
p

30. APPROACH We can use Newton’s second law of mo-


tion to find the acceleration of the proton in the uniform Substitute for ay in the expression for vy
electric field and constant-acceleration equations to find s 
the time required for it to reach a speed of 0.01c and the

eE
vy = 2 − g ∆y
distance it travels while acquiring this speed. m
SOLUTION (a) Use data found at the back of your text
to compute e/m for an electron: Substitute numerical values and evaluate vy :

e 1.602 × 10−19 C
=
mp 1.673 × 10−27 kg " 
1.602 × 10−19 C 1.50 × 10−10 N/C
 
= 9.58 × 107 C/kg vy = 2
9.109 × 10−31 kg
Apply Newton’s second law to relate the acceleration of #1/2
the electron to the electric field: 
2 −6

− 9.81 m/s 1.0 × 10 m = 5.8 mm/s
Fnet eE
a= =
mp mp
32. APPROACH We can apply the work-kinetic energy the-
Substitute numerical values and evaluate a: orem to relate the change in the object’s kinetic energy to
1.602 × 10−19 C (100 N/C) the net force acting on it. We can express the net force

a= acting on the charged body in terms of its charge and the
1.673 × 10−27 kg
electric field.
= 9.576 × 109 m/s2
SOLUTION Using the work-kinetic energy theorem, ex-
= 9.58 × 109 m/s2
press the kinetic energy of the object in terms of the net
The direction of the acceleration of a proton is in the di- force acting on it and its displacement:
rection of the electric field.
W = ∆K = Fnet ∆x
(b) Using the definition of acceleration, relate the time
required for an electron to reach 0.01c to its acceleration: Relate the net force acting on the charged particle to the
v 0.01c electric field:
∆t = = Fnet = qE
a a
168 CHAPTER 1. ELECTRIC CHARGE AND FIELD

Substitute for Fnet to obtain: Eliminate the parameter t between equations (1.174) and
(1.175) to obtain:
∆K = Kf − Ki = qE∆x
or, because Ki = 0, y(x) = −
eEy 2
x =−
eEy 2
x
Kf 2me v02 4K
Kf = qE∆x ⇒ q =
E∆x
Substitute numerical values and evaluate y(4cm):
Substitute numerical values and evaluate q:
1.602 × 10−19 C 2.00 × 104 N/C (0.0400 m)2
 
0.120 J y(0.04 m) = −
q= = 800µC 4 (2.00 × 10−16 J)
(300 N/C)(0.500 m)
= −6.40 mm
33. The trajectory of the electron while it is in the electric
field is parabolic (its acceleration is downward and con- (b) The angle at which the electron is moving, with respect
stant) and its trajectory, once it is out of the electric field to the axis, after exiting the region between the plates is
is, if we ignore the small gravitational force acting on it, given by:
linear. We can use constant-acceleration equations and  
vy
 
vy
−1 −1
Newton’s second law to express the electron’s x and y θ = tan = tan
vx v0
coordinates parametrically and then eliminate the param-
eter t to express y(x). We can find the angle with the
horizontal at which the electron leaves the electric field Using a constant-acceleration equation, express vy as a
from the x and y components of its velocity and its total function of the electron’s acceleration and its time in the
vertical deflection by summing its deflections over the first electric field:
4 cm and the final 12 cm of its flight[Fig.1.360]. vy = v0,y + ay t
(a) Using a constant-acceleration equation, express the x
or, because v0,y = 0
Fnet, ,y eEy x
vy = ay t = t=−
me m e v0

Substitute for vy to obtain:


   
eEy x eEy x
θ = tan−1 − = tan−1

me v02 2K

Substitute numerical values and evaluate θ:


" #
1.602 × 10−19 2.00 × 104 (0.0400)
 
Figure 1.360 −1
θ = tan −
2 (2.00 × 10−16 )
and y coordinates of the electron as functions of time:
= −17.8◦
x(t) = v0 t
(c) Express the total vertical displacement of the electron:
and
1 ytotal = y4 + y12
y(t) = v0,y t + ay t2 cm cm
2
Relate the horizontal and vertical distances traveled to the
Because v0,y = 0 :
screen after leaving the region between the plates to the
x(t) = v0 t horizontal and vertical components of its velocity:
and (1.174) x = v0 ∆t
1
y(t) = ay t2 and
2
y = vy ∆t
Using Newton’s second law, relate the acceleration of the
electron to the electric field: Eliminate ∆t from these equations to obtain:
Fnet −eEy  
vy
ay = = y= x = (tan θ)x
me me v0
Substituting for ay gives:
Substitute numerical values and evaluate y:
eEy 2
y(t) = − t (1.175) y = [tan (−17.7◦ )] (0.120 m) = −3.83 cm
2me
1.29. ANSWER KEYS AND SOLUTIONS 169

Substitute for y4 cm and y12 cm and evaluate ytotal : Set y = 0 and solve for E0 to obtain:
ytotal = −0.640 cm − 3.83 cm mv 2 sin 2θ
E0 =
= −4.47 cm qx
That is, the electron will strike the fluorescent screen Substitute the non-particle-specific data to obtain:
4.47 cm below the horizontal axis. 2
m 3.00 × 106 m/s sin 70◦
E0 =
q(0.0150 m)
34. APPROACH We can apply the work-kinetic energy the- m
orem to relate the change in the object’s kinetic energy to = 5.638 × 1014 m/s2
q
the net force acting on it. We can express the net force
acting on the charged body in terms of its charge and the (a) Substitute for the mass and charge of an electron and
electric field. evaluate E0 :
SOLUTION Using the work-kinetic energy theorem, ex-
 9.109 × 10−31 kg
press the kinetic energy of the object in terms of the net E0 = 5.638 × 1014 m/s2
force acting on it and its displacement: −1.602 × 10−19 C
= −3.2kN/C
W = ∆K = Fnet ∆x
(b) Substitute for the mass and charge of a proton and
Relate the net force acting on the charged particle to the evaluate E0 :
electric field:
Fnet = qE  1.673 × 10−27 kg
E0 = 5.64 × 1014 m/s2
Substitute for Fnet to obtain: 1.602 × 10−19 C
= 5.9MN/C
∆K = Kf − Ki = qE∆x
36. APPROACH We can use constant-acceleration equa-
or, because Ki = 0,
tions to express the x and y coordinates of the electron
Kf in terms of the parameter t and Newton’s second law to
Kf = qE∆x ⇒ q =
E∆x express the constant acceleration in terms of the electric
Substitute numerical values and evaluate q : field. Eliminating t will yield an equation for y as a func-
0.120J tion of x, q, and m. We can decide whether the electron
q= = 800µC will strike the upper plate by finding the maximum value
(300N/C)(0.500m)
of its y coordinate. Should we find that it does not strike
35. APPROACH We can use constant-acceleration equa- the upper plate, we can determine where it strikes the
tions to express the xand y coordinates of the particle lower plate by setting y(x) = 0[Fig.1.361] . Ignore any
in terms of the parameter t and Newton’s second law to effects of gravitational forces.
express the constant acceleration in terms of the electric
field. Eliminatingt will yield an equation for y as a func-
tion of x, q, and m that we can solve for Ey .
SOLUTION Express the x and y coordinates of the par-
ticle as functions of time:
x = (v cos θ)t
and
1 Figure 1.361
y = (v sin θ)t − ay t2
2
Apply Newton’s second law to relate the acceleration of SOLUTION Express the x and y coordinates of the elec-
the particle to the net force acting on it: tron as functions of time:
Fnet, y qE0 x = (v0 cos θ) t
ay = =
m m
and
Substitute in the y-coordinate equation to obtain: 1
y = (v0 sin θ) t − ay t2
qE0 2 2
y = (v sin θ)t − t
2m Apply Newton’s second law to relate the acceleration of
Eliminate the parameter tbetween the two equations to the electron to the net force acting on it:
obtain:
qE0 Fnet,y eEy
y = (tan θ)x − x2 ay = =
2mv 2 cos2 θ me me
170 CHAPTER 1. ELECTRIC CHARGE AND FIELD

Substitute in the y-coordinate equation to obtain: SOLUTION (a) Apply the definition of electric dipole
moment to obtain:
eEy 2 −

y = (v0 sin θ) t − t p~ = q L
2me
and
Eliminate the parameter t between the two equations to
p = (2.0pC)(4.0µm)
obtain:
= 8.0 × 10−18 C · m
eEy
y(x) = (tan θ)x − x2 (1.176)
2me v02 cos2 θ (b) If the dipole is oriented to the right[Fig.1.362], then p~
is to the right; pointing from the negative charge toward
To find ymax , set dy/dx = 0 for extrema: the positive charge.

dy eEy
= tan θ − x0
dx me v02 cos2 θ
= 0 for extrema

Solve for x0 to obtain: Figure 1.362

me v02 sin 2θ
x0 = (See remark below.) 38. APPROACH The forces the electron and the proton ex-
2eEy ert on each other constitute an action-and-reaction pair.
Because the magnitudes of their charges are equal and
Substitute x0 in y(x) and simplify to obtain ymax : their masses are the same, we find the speed of each
particle by finding the speed of either one. We’ll apply
me v02 sin2 θ Coulomb’s force law for point charges and Newton’s sec-
ymax =
2eEy ond law to relate v to e, m, k, and their separation distance
L.
Substitute numerical values and evaluate ymax : SOLUTION Apply Newton’s second law to the positron
2 to obtain:
9.109 × 10−31 kg 5.00 × 106 m/s sin2 45◦

ymax =
2 (1.602 × 10−19 C) (3.50 × 103 N/C) ke2 v2 ke2
= m 1 ⇒ = 2mv 2
= 1.02 cm L2 2L
L
Solving for v gives:
Because the plates are separated by 2 cm, the electron r
does not strike the upper plate. ke2
v=
To determine where the electron will strike the lower plate, 2mL
set y = 0 in equation (1) and solve for x to obtain:
39. APPROACH We can use Coulomb’s force law for point
me v02sin 2θ particles and the condition for translational equilibrium to
x= express the equilibrium position as a function of k, q, Q, m,
eEy
and g. In Part (b) we’ll need to show that the dis-
Substitute numerical values and evaluate x : placed point charge experiences a linear restoring force
and, hence, will exhibit simple harmonic motion.
2
9.109 × 10−31 kg 5.00 × 106 m/s sin 90◦ SOLUTION (a) Apply the condition for translational

x= equilibrium to the particle:
(1.602 × 10−19 C) (3.50 × 103 N/C)
= 4.1 cm s
kqQ kqQ
− mg = 0 = y0 =
y02 mg
Remarks: x0 is an extremum, that is, either a maximum
or a minimum. To show that it is a maximum we need (b) Express the restoring force that acts on the particle
to show that d2 y/dx2 , evaluated at x0 , is negative. A sim- when it is displaced a distance ∆y from its equilibrium
ple alternative is to use your graphing calculator to show position:
that the graph of y(x) is a maximum at x0 . Yet another
alternative is to recognize that, because equation (1.297) is kqQ kqQ
quadratic and the coefficient of x2 is negative, its graph is F = 2 − y2
(y0 + ∆y) 0
a parabola that opens downward.
or, because ∆y << y0
37. APPROACH We can use its definition to find the dipole kqQ kqQ
moment of this pair of charges. F ≈ 2 − 2
y0 + 2y0 ∆y y0
1.29. ANSWER KEYS AND SOLUTIONS 171

Simplify this expression further by writing it with a com- Substitute in equation (1.177) and differentiate with re-
mon denominator: spect to x:
2y0 ∆ykqQ 2y0 ∆ykqQ
F =− =− 
 
d 2kp1 p2 6kp1 p2
y04 + 2y03 ∆y

y 1 + 2 ∆y
4 F =− − =
0 y0 dx x3 x4
2∆ykqQ
≈−
y03 Evaluate F for p1 = p2 = p and x = d to obtain:

again, because ∆y << y0 . 6kp2


F =
From the 1 st
step of our solution: d4
kqQ
= mg
y02 41. APPROACH (a) We can use Coulomb’s law for the
electric field due to a point charge and superposition of
Substitute for kqQ
and simplify to obtain: fields to find the electric field at any point on the y
y02
axis[Fig.1.363]. (b) Using the definition of electric field will
2mg yield an expression for the electric force that acts on the
F =− ∆y bead. In Part (c) we can set dFy /dy = 0 to find the value
y0
of y that maximizes Fy and then use the work-kinetic en-
Apply Newton’s second law to the displaced particle to ergy theorem (integrating Fy from 0 to this extreme value
obtain: will yield an expression for the work done on the bead
d2 ∆y 2mg during this displacement) to find the speed of the bead
m =− ∆y
dt2 y0 when the force acting on it is a maximum.
or
d2 ∆y 2g
+ ∆y = 0
dt2 y0
the differential equation of simple harmonic motion with
p
ω = 2g/y0

40. APPROACH We can relate the force of attraction that


each molecule exerts on the other to the potential energy
function of either molecule using F = −dU/dx. We can
relate U to the electric field at either molecule due to the Figure 1.363
presence of the other through U = −pE. Finally, the
electric field at either molecule is given by E = 2kp/x3 .
SOLUTION Express the force of attraction between the
dipoles in terms of the spatial derivative of the potential SOLUTION (a) Use Coulomb’s law for the electric field
energy function of p1 : due to a point charge and superposition of fields, to ex-
press the field at point P on the y axis:
dU1
F =− (1.177)
dx −
→ −
→ −→
Ey = E1 + E2
Express the potential energy of the dipole p1 : kq1 kq2
= 2 r̂ 1,P + 2
r1,P r2,P r̂2,P
U1 = −p1 E1
where E1 is the field at p1 due to p2 . The unit vectors r̂1,p and r̂2,P are given by:
Express the electric field strength at p1 due to p2 :
r̂1,P − 1 aî + y ĵ
2kp2 r̂1,P = = 2
E1 = r1,P r1,P
x3 and
where x is the separation of the dipoles.
1
r̂ 2,P 2 aî+ y ĵ
Substitute for E1 to obtain: r̂ 2,P = =
r2,P r2,P
2kp1 p2
U1 = −
x3 Substituting for q1 , q2 , r̂1,P , r̂2,P and r1,P = r2,P =
172 CHAPTER 1. ELECTRIC CHARGE AND FIELD

h 2 i1/2 where we’ve ignored the negative value because the bead
y2 + 1
2a yields:
is given a nudge in the +y direction.
! !

→ kQ − 21 aî + y ĵ kQ 1
2 a î + y ĵ
Ey = 2 + 2 Substitute for ymax in the expression for vf to obtain:
r1,P r1,P r2,P r2,P
v
kQ

1

kQ 1
  Z a
u 4kqQ 2√2
u
= 3 − aî + y ĵ + 3 aî + y ĵ y
r1,P 2 r2,P 2 vf = u dy
t M
0
h 2 i3/2
y 2 + 12 a
kQ kQ
=h (ŷ) + h (y ĵ)
2 i3/2 2 i3/2
y2 + 1
a y2 + 1
a Evaluating the integral yields:
2 2
Z √a
2kQy 2 2 y 0.367
= 3/2 ĵ i3/2 dy ≈ a
y 2 + 14 a2 0
y 2 + 21 a
 2

(b) Relate the electric force on the bead to its charge and a

y
Z
the electric field:
2 2
Substitute for dy and simplify to
0
h
1
2 i3/2
2kqQy y2 + 2a
F~ = q E
~y = ĵ
[y 2 + 14 a2 ]3/2 obtain:
s r
where q must be positive if F~ always points away from
 
4kqQ 0.367 kqQ
vf = = 1.21
the origin. M a aM

(a) Apply Newton’s second law to the microsphere to


(c) Apply the work-kinetic energy theorem to the bead as
it moves from the origin to the location at which Fy is a y
maximum to obtain:
Wnet y = ∆K = Kf − Ki
or, because Ki = 0, Fe
1
Wdone by Fy = M vf2
2
Solving for vf yields:
r m, Ne
2Wdone by Fy
vf =
M
The work done on the bead by Fy as the bead moves from mg
the origin to ymax (the y coordinate of the point at which
Fy is a maximum) is given by:
Z ymax
2kqQy Fd
Wdone by Fy = dy
0
h 2 i3/2
y 2 + 12 a
Figure 1.364
Substituting for Wdone by Fy and simplifying yields:
v
u 4kqQ Z ymax y obtain:
vf = u dy Fe − mg − Fd = may
t
m 0
h 2 i3/2
2 1
y + 2a or, because ay = 0,
Fe − mg − Fd, terminal = 0
The condition that Fy is a maximum is:
Substitute for Fe , m, and Fd,ter min a1 to obtain:
 
dFy d  y qE − ρV g − 6πηrvt = 0
= =0

dy dy
h
1
2 i3/2  or, because q = N e,
y2 + 2 a
4
N eE − πr3 ρg − 6πηrvt = 0
Carrying out the details of the differentiation and solving 3
for the critical value ymax yields: Solve for N e to obtain:
a 4 3
ymax = √ 3 πr ρg + 6πηrvt
Ne = (1.178)
2 2 E
1.29. ANSWER KEYS AND SOLUTIONS 173

4 3 SOLUTION (a) Apply Newton’s second law to the mi-


Substitute numerical values and evaluate nr pg:
3 crosphere when the electric field is downward:

Fe − mg − Fd = may
4 3 4 3
or, because ay = 0,
πr ρg = π 5.50 × 10−7 m 1.05 × 103 kg/m3

3 3
Fe − mg − Fd,terminal = 0
9.81 m/s2 = 7.18 × 10−15 N


Substitute for Fe and Fd.terminal to obtain:


Substitute numerical values and evaluate 6πηrvt : qE − mg − 6πηrvu = 0
or, because q = N e,
6πηrvt = 6π 1.8 × 10−5 Pa · s 5.50 × 10−7 m
 
N eE − mg − 6πηrvu = 0
1.16 × 10−4 m/s = 2.16 × 10−14 N


Solve for vu to obtain:


Substitute numerical values in equation 1.178 and evaluate
Ne : N eE − mg
vu = (1.179)
7.18 × 10−15 N + 2.16 × 10−14 N 6πηr
Ne =
6.00 × 104 N/C
With the field pointing upward, the electric force is down-
= 4.8 × 10−19 C
ward and the application of Newton’s second law to the
(b) Divide the result in (a) by e to obtain: microsphere yields:

4.80 × 10−19 C Fd, terminal − Fe − mg = 0


N= =3 1
1.602 × 10−19 C
6πηrvd − N eE − mg = 0
(c) With the field pointing upward,
P the electric force is
downward and the application of Fy = may to the bead Solve for vd to obtain:
yields:
Fd, terminal − Fe − mg = 0 N eE + mg
vd = (1.180)
6πηr
O1
4 Add equations (1.179) and (1.180) and simplify to obtain:
6πηrvt − N eE − πr3 ρg = 0
3
N eE − mg N eE + mg N eE qE
Solve for vt to obtain: u = vu + vd = + = =
6πηr 6πηr 3πηr 3πηr
N eE + 43 πr3 ρg Measuring both vu and vd has the advantage that you
vt =
6πηr don’t need to know the mass of the microsphere.

Substitute numerical values and evaluate vt : (b) Letting ∆u represent the change in the terminal speed
of the microsphere due to a gain (or loss) of one electron
we have:
 
N
 ∆u = vN +1 − vN
vt = 3 1.602 × 10−19 C 6.00 × 104

C Noting that ∆v will be the same whether the microsphere
4 3

kg

m
 is moving upward or downward, express its terminal speed
−7 3

+ π 5.50 × 10 m 1.05 × 10 3 9.81 2 when it is moving upward with N electronic charges on it:
3 m s
 
N eE − mg
6π 1.8 × 10−5 Pa · s 5.50 × 10−7 m
 
vN =
6πηr
= 0.19 mm/s Express its terminal speed upward when it has N + 1 elec-
tronic charges:
42. APPROACH The free body diagram shows the forces
acting on the microsphere of mass m and having an ex- (N + 1)eE − mg
vN +1 =
cess charge of q = Ne when the electric field is downward. 6πηr
Under terminal-speed conditions the sphere is in equilib- Substitute and simplify to obtain:
rium under the influence of the electric force F~e , its weight
m~g , and the drag force F~d . We can apply Newton’s second (N + 1)eE − mg N eE − mg
∆u = −
law, under terminalspeed conditions, to relate the number 6πηr 6πηr
of excess charges N on the sphere to its mass and, using eE
Stokes’ law, to its terminal speed. =
6πηr
174 CHAPTER 1. ELECTRIC CHARGE AND FIELD

Substitute numerical values and evaluate ∆u: (e) Using the approximation that the charge is a point
charge on the x axis at x = 2.5 m, Coulomb’s law gives:
1.602 × 10−19 C 6.00 × 104 N/C
 
∆u =
6π (1.8 × 10−5 Pa · m) (5.50 × 10−7 m) kQ
Ex = 2
= 52¯m/s r1 − 12 L

Continuous Charge Distributions Substitute numerical values and evaluate Ex=250 m.




Calculating E From Coulomb’s Law 8.988 × 109 N · m2 /C2 (17.5nC)

Ex=250 m =
43. APPROACH (a) We can use the definition of λ to find
2
250 m − 12 (5.0 m)
the total charge of the line of charge[Fig.1.365]. (b), (c)
= 2.56774 mN/C = 2.6 mN/C
and (d) Equation (1.66) gives the electric field on the
axis of a finite line of charge. In Part (e) we can apply This result is about 0.01% less than the exact value ob-
Coulomb’s law for the electric field due to a point charge tained in (d). This suggests that the line of charge is too
to approximate the electric field at x = 250m. In the fol- long for its field at a distance of 250 m to be modeled
lowing diagram, L = 5.0m and P is a generic point on the exactly as that due to a point charge.
x axis.
44. APPROACH Choose a coordinate system in which the
+x direction is perpendicular to the surfaces and points
to the right. Let the charge densities on the two plates be
σ1 and σ2 and denote the three regions of interest as 1,2,
and 3 [Fig.1.366]. Choose a coordinate system in which
Figure 1.365
the positive x direction is to the right. We can apply the
equation for E near an infinite plane of charge and the
SOLUTION (a) Use the definition of linear charge den- superposition of fields to find the field in each of the three
sity to express Q in terms of λ: regions.

Q = λL = (3.5nC/m)(5.0 m) = 17.5nC
= 18nC
The electric field on the axis of a finite line charge is given
by Equation (1.66)
 
1 1
Ex = kλ −
r2 r1
(b) Substitute numerical values and evaluate Ex=6.0 m:

Figure 1.366
· m2
  
9N C
Ex=6.0 m = 8.988 × 10 3.5 × 10−9
C2 m −

  SOLUTION (a) Use the equation for E near an infinite
1 1 plane of charge to express the field in region 1 when σ1 =
− = 26 N/C
6.0 m − 5.0 m 6.0 m σ2 = +3.0µC/m2
(c) Substitute numerical values and evaluate Ex=9.0 m:

→ −
→ −

E 1 = E σ1 + E σ2
N · m2 = −2πkσ1 î − 2πkσ2 î
  
−9 C
Ex=9.0 m = 8.988 × 109 3.5 × 10
C2 m = −4πkσ î
 
1 1
− = 4.4 N/C ~ 1:
Substitute numerical values and evaluate E
9.0 m − 5.0 m 9.0 m
(d) Substitute numerical values and evaluate Ex at x = −

E 1 = −4π 8.988 × 109 N · m2 /C2 3.0µC/m2 î
 
250 m:
= − 3.4 × 105 N/C î


N · m2
  
−9 C
Ex=250 m = 8.988 × 109 3.5 × 10 Proceed as above for region 2:
C2 m
~2 = E
~σ + E
~ σ = 2πkσ1 î − 2πkσ2 î
 
1 1 E
− 1 2
250 m − 5.0 m 250 m
= 2πkσ î − 2πkσ î
= 2.56800 mN/C = 2.6 mN/C
=0
1.29. ANSWER KEYS AND SOLUTIONS 175

Proceed as above for region 3: Express the electric field on the axis of a disk charge:

→ −
→ −
→  
E 3 = E σ1 + E σ2 = 2πkσ1 î + 2πkσ2 î = 4πkσ î 1
 2
 Ez = 2πkσ 1 − q 
9N · m R2
3.0µC/m2 î

= 4π 8.988 × 10 2
1 + z2
C
5
We’re given that:

= 3.4 × 10 N/C î

→ 1 σ
(b) Use the equation for E near an infinite plane of charge Ez = σ/(20 ) =
to express and evaluate the field in region 1 when σ1 = 2 4 ∈0
+3.0µC/m2 and σ2 = −3.0µC/m2 : Equating these expressions gives:
~1 = −
E
→ −

E σ1 + E σ2 = 2πkσ1 î − 2πkσ2 î
 
σ 1
= 2πkσ î − 2πkσ î = 2πkσ 1 − q 
40 1+ R2
z2
=0

Proceed as above for region 2 : Substituting for k yields:


 
~2 = E
~σ + E~ σ = 2πkσ1 î + 2πkσ2 î = 4πkσ î
 
E σ 1 1
1 2
= 2π σ 1 − q 
 2
 40 4π0 R2
9N · m 1+
3.0µC/m2 î

= 4π 8.988 × 10 z2
C2
= 3.4 × 105 N/C î
 Solve for z to obtain:
R
Proceed as above for region 3: z=√
3
~3 = E
E ~σ + E
~ σ = 2πkσ1 î − 2πkσ2 î
46. SOLUTION The electric field at a distance z from the
1 2

= 2πkσ î − 2πkσ î center of a ring whose charge is Q and whose radius is a


=0 is given by
kQz
(c) The electric field lines for (a) and (b) are shown in Ez = 3/2
(z + a2 )
2
Fig.??:
(a) Evaluating Ez=0.2a gives

kQ(0.2a) kQ
Ez=0.2a = = 0.189 2
[(0.2a)2 + a2 ]3/2 a

(b) Evaluating Ez=0.5a gives:

kQ(0.5a) kQ
Ez=0.5a = = 0.358 2
[(0.5a)2 + a2 ]3/2 a

(c) Evaluating Ez=0.7a gives:

(a) (b) kQ(0.7a) kQ


Ez=0.7a = = 0.385 2
[(0.7a)2 + a2 ]3/2 a
Figure 1.367
(d) Evaluating Ez=a gives:

45. APPROACH The magnitude of the electric field at an kQa kQ


Ez=a = = 0.354 2
axial distance z from a disk that has a uniform charge [a2 + a2 ]3/2 a
density σ and whose radius is R is given by
(e) Evaluating Ez=2a gives:
 
1 2kQa kQ
Ez = 2πkσ 1 − q  Ez=2a = = 0.179 2
1+ R
2 [(2a)2 + a2 ]3/2 a
z2

(f) The field along the z axis is plotted in Fig.1.368. The


We can equate this expression and Ez = 1
2 σ/(20 ) and z coordinates are in units of zla and E is in units of kQla2
solve for z.
176 CHAPTER 1. ELECTRIC CHARGE AND FIELD

0.4 From the diagram we see that:


 
y x2
cos θ2 = p or θ2 = tan −1
0.2 x22 + y 2 y
and
 
y x1
cos θ1 = p or θ1 = tan−1
Ez 0.0 x21 + y 2 y

y y
Substitute for p and p to Obtain:
-0.2 x22 + y2 x21 + y2


Ex = − [− cos θ2 + cos θ1 ]
-0.4
y
-3 -2 -1 0 1 2 3 kλ
= [cos θ2 − cos θ1 ]
z/a y

Figure 1.368
48. APPROACH Consider the ring with its axis along the
z direction shown in the diagram1.370. Its radius is z =
r cos θ and its width is rdθ. We can use the equation for
the field on the axis of a ring charge and then integrate to
express the field at the center of the hemispherical shell.

r sinθ
Figure 1.369
rdθ


47. APPROACH The line charge and point (0, y) are shown r
θ
in the diagram1.369. Also shown is a line element of length
~ its charge produces at (0, y) . We can y
dx and the field dE
dE
~
find dEx from dE and then integrate from x = x1 to
x = x2 .
~ x
SOLUTION Express the x component of dE:
Figure 1.370

dEx = − 2 sin θdx
x + y2 SOLUTION Express the field on the axis of the ring of
kλ x charge:
=− 2 2
p dx
x +y x + y2
2
kzdq
dE =
kλx 2
r sin θ + r2 cos2 θ
2
3/2
=− 3/2
dx
(x2 + y2 ) kzdq
= 3
r
Integrate from x = x1 to x2 and simplify to obtain:
where z = r cos θ
x2
X
Z
Ex = −kλ 2 + y 2 )3/2
dx Express the charge dq on the ring:
x1 (x
" #x2
1 dq = σdA = σ(2π · sin θ)rdθ
= −kλ − p
x2 + y 2 x = 2πσ 2 sin θdθ
1
" #
1 1 Substitute to obtain:
= −kλ − p 2 +p 2
x2 + y 2 x1 + y 2
k(r cos θ)2πσr2 sin θdθ
dE =
" #
kλ y y r3
=− −p 2 +p 2
y x2 + y 2 x1 + y 2 = 2πkσ sin θ cos θdθ
1.29. ANSWER KEYS AND SOLUTIONS 177

Integrating dE from θ = 0 to π/2 yields: 51. APPROACH Because the cone encloses no charge, we
know, from Gauss’s law, that the net flux of the electric
π/2
field through the cone’s surface is zero[Fig.1.372]. Thus,
Z
E = 2πkσ sin θ cos θdθ
0 the number of field lines penetrating the curved surface of
 π/2 the cone must equal the number of field lines penetrating
1
= 2πkσ sin2 θ = πkσ the base and the entering flux must equal the exiting flux.
2 0

49. APPROACH We must show the twelve electric field


lines originating at q and, in the absence of other charges,
radially symmetric with respect to the location of q. While
we’re drawing twelve lines in this problem, the number of
lines that we draw is always, by agreement, in proportion
to the magnitude of q.
SOLUTION (a) The sketch of the field lines and of the

Figure 1.372

SOLUTION The flux penetrating the base of the cone


is given by:
φentering = EAbase
The flux penetrating the curved surface of the cone is given
by:

→ − →
I I
φexiting = E · d A = E cos θ dA
S S
Figure 1.371 Equating the fluxes and simplifying yields:

spherical surface is shown in the diagram1.371. Given the


I
Abase = cos θ dA = (cos θ)A curved
number of field lines drawn from q, 3 lines enter the spher- S surface
ical surface. Had we chosen to draw 24 field lines, 6 would
have entered the spherical surface. (b) Three lines leave The ratio of the density of field lines is:
the spherical surface. (c) Because the three lines that en-
ter the spherical surface also leave the spherical surface, Abase
= cos θ
the net number of field lines that pass through the surface A curved
surface
is zero. (d) Because as many field lines leave the spherical
surface as enter it, the net flux is zero. 52. APPROACH We’ll model this portion of Earth’s atmo-
50. APPROACH The flux through the cube is given by sphere as though it is a cylinder with cross-sectional area
Φnet = Qencl /ε0 where Qencl is the charge at A and height h. Because the electric flux increases with
the center of the cube. The flux through one face of altitude, we can conclude that there is charge inside the
the cube is one-sixth of the total flux through the cube. cylindrical region and use Gauss’s law to find that charge
SOLUTION The flux through one face of the cube- and hence the charge density of the atmosphere in this
region.
1 Q SOLUTION The definition of volume charge density is:
Φ1f ace = Φnet =
6 6ε0
Q
Substitute numerical values and evaluate Φ1f ace: ρ=
V
−3.00µC Express the charge inside a cylinder of base area A and
φ1 face = C2

6 8.854 × 10−12 N·m2 height h for a charge density ρ:
2
N·m
= −5.65 × 104 Q = ρAh
C
178 CHAPTER 1. ELECTRIC CHARGE AND FIELD

Taking upward to be the positive direction, apply Gauss’s Apply Gauss’s law to a spherical surface of radius r <
law to the charge in the cylinder: R that is concentric with the nonconducting sphere to
obtain:
Q = − (Eh A − E0 A) ε0 = (E0 A − Eh A) ∈0
1 Qinside
I
Er dA = Qinside ⇒ 4πr2 Er =
where we’ve taken our zero at 250 m above the surface of S  0 0
a flat Earth.
Substitute to obtain: Solve for Er and simplify to obtain:

(E0 A − Eh A) ε0 (E0 − Eh ) ε0 Qinside πAr4 Ar2


ρ= = Er<R = = =
Ah h 4πr2 0 4πr2 0 4 ∈0

Substitute numerical values and evaluate ρ : (c) Er versus r/R graph, with Er in units of A/(4ε0 ), is
plotted in Fig.1.373.
Remarks: Note that the results for (a) and (b) agree at
(150 N/C − 170 N/C) 8.854 × 10−12 C2 /N · m2

ρ= 1.0
400 m − 250 m
0.8
= −1.2 × 10−12 C/m3
Er 0.6
where we’ve been able to neglect the curvature of Earth 0.4

because the maximum height of 400 m is approximately 0.2


0.006% of the radius of Earth and because the diameter of
0.0
the cylindrical Gaussian surface can be arbitrarily small. 0.0 0.5 1.0 1.5 2.0 2.5 3.0
r/R
Gauss’s Law Applications in Spherical Symmetry
Figure 1.373
Situations

53. APPROACH We can find the total charge on the sphere r = R.


by expressing the charge dq in a spherical shell and inte-
grating this expression between r = 0 and r = R. By 54. APPROACH By symmetry, the electric fields resulting
symmetry, the electric fields must be radial. To find Er from this charge distribution must be radial. To find Er
inside the charged sphere we choose a spherical Gaussian for r < R1 we choose a spherical Gaussian surface of radius
surface of radius r < R. To find Er outside the charged r < R1 . To find Er for R1 < r < R2 we choose a spherical
sphere we choose a spherical Gaussian surface of radius Gaussian surface of radius R1 < r < R2 . To find Er for
r > R. On each of these surfaces, Er is constant. Gauss’s r > R2 we choose a spherical Gaussian surface of radius
law then relates Er to the total charge inside the surface. r > R2 . On each of these surfaces, Er is constant. Gauss’s
law then relates Er to the total charge inside the surface.
SOLUTION (a) Express the charge dq in a shell of thick- SOLUTION (a) The charge in an infinitesimal spherical
ness dr and volume 4πr2 dr: shell of radius r and thickness dr is:
dq = 4πr2 ρdr = 4πr2 (Ar)dr
dQ = ρdV = 4πρr2 dr
= 4πAr3 dr
Integrate dQ from r = R1 to R2 to find the total charge
Integrate this expression from r = 0 to R to find the total in the spherical shell in the interval R1 < r < R2 :
charge on the sphere:
R2 R2
4πCr3
Z 
R 2
Qtotal = 4πρ r dr =
Z
R
r3 dr = πAr4 0 = πAR4 3

Q = 4πA R1 R1
0
4πρ
R23 − R13

=
(b) Apply Gauss’s law to a spherical surface of radius r > 3
R that is concentric with the nonconducting sphere to
obtain: (b) Apply Gauss’s law to a spherical surface of radius r
that is concentric with the nonconducting spherical shell
1 Qencl
I
Er dA = Qencl ⇒ 4πr2 Er = to obtain:
S ε0 ε0
1 Qinside
I
Er dA = Qinside ⇒ 4πr2 Er =
Solving for Er yields: S 0 0
Qinside 1 kQinside
Er>R = 2
= Solving for Er yields:
4π0 r r2
4 4
kAπR AR Qinside 1 kQinside
= = Er = =
r2 40 r2 4π0 r2 r2
1.29. ANSWER KEYS AND SOLUTIONS 179

Q kQ
Evaluate Er<R1 : Er<R1 = 4π∈ inside 1 =
0 r2
inside = 0
r2
Substitute for Qinside to obtain:
because ρr<R1 = 0 and, therefore, Qinside = 0
2k πρ0 La2

ρ0 a2
Evaluate ER1 <r<R2 : Er>a = =
Lr 20 r
kQinside 4πkρ 3 or, because λ = ρπa 2
r − R13

ER1 <KR2 = 2
= 2
r 3r λ
ρ 3 3
 Er>a =
= 2
r − R 1 2π ∈0 r
30 r
For r > R2 : 56. APPROACH From symmetry; the field tangent to the
surfaces of the shells must vanish. We can construct a
4πρ Gaussian surface in the shape of a cylinder of radius r
R23 − R13

Qinside =
3 and length L and apply Gauss’s law to find the electric
and field as a function of the distance from the centerline of
4πkρ the infinitely long, uniformly charged cylindrical shells.
R23 − R13

Er>R2 = 2 SOLUTION (a) Apply Gauss’s law to the cylindrical
3r
ρ surface of radius r and length L that is concentric with
R23 − R13

= 2 the infinitely long, uniformly charged cylindrical shell:
30 r
Remarks: Note that E is continuous at r = R2 . I
1 Qencl
En dA = Qencl ⇒ 2πrLEn =
Gauss’s Law Applications in Cylindrical Symme- S ε0 ε0
try Situations
where we’ve neglected the end areas because there is no
55. APPROACH From symmetry, the field tangent to the flux through them.
surface of the cylinder must vanish. We can construct a Noting that, due to symmetry, En = Er , Solve for Er to
Gaussian surface in the shape of a cylinder of radius r and obtain:
length L and apply Gauss’s law to find the electric field 2kQencl
Er = (1.181)
as a function of the distance from the centerline of the Lr
infinitely long nonconducting cylinder. For r < R1 , Qencl = 0, so:
SOLUTION Apply Gauss’s law to a cylindrical surface
of radius r and length L that is concentric with the in- Er<R1 = 0
finitely long nonconducting cylinder:
Express Qencl for a1 < r < a2 :
1
I
En dA = Qinside
S  0 Qencl = σ1 A1 = 2πσ1 a1 L
or
Substitute in equation 1.181 to obtain:
Qinside
2πrLEn =
0 2k (2πσ1 a1 L) σ1 a1
Ea1 <r<a2 = =
where we’ve neglected the end areas because there is no Lr ε0 r
flux through them. Due to symmetry, En = Er . Solving (b) Set E = 0 for r > a2 to obtain:
for Er yields:
σ1 a1 + σ2 a2 σ2 a1
Qinside 2kQinside =0⇒ =−
Er = = ε0 r σ1 a2
2πrL ∈0 Lr
Express Qinside for r < a: (c) Because the electric field is determined by the enclosed
charge within the Gaussian surface, the field under these
Qinside = ρ(r)V = ρ0 πr2 L conditions would be as given above:


Substitute to obtain: σ1 a1
Ea1 <r<a2 =
ε0 r
2

2k πρ0 Lr ρ0
Er<R = = r
Lr 20 Electric Charge and Field at Conductor Surfaces
or, because λ = ρπa2 ,
57. APPROACH Because the penny is in an external elec-
λ
Er<R = r tric field, it will have charges of opposite signs induced on
2π0 a2 its faces. The induced charge σ is related to the electric
Express Qinside for r > a: field by E = σ0 . Once we know σ, we can use the defini-
tion of surface charge density to find the total charge on
Qinside = ρ(r)V = ρ0 πa2 L one face of the penny.

180 CHAPTER 1. ELECTRIC CHARGE AND FIELD

SOLUTION (a) Relate the electric field to the charge Substitute numerical values and evaluate Rmin .
density on each face of the penny: s
σ (8.988 × 109 N · m2 /C2 ) (18µC)
E= ⇒ σ = 0 E Rmin =
0 3.0 × 106 N/C
= 23 cm
Substitute numerical values and evaluate σ:
σ = 8.854 × 10−12 C2 /N · m2 (1.60kN/C)

60. APPROACH (a) One half of the total charge is on each
= 14.17nC/m2 = 14.2nC/m2 side of the conducting sheet and the electric field inside the
conducting sheet is zero. The electric field intensity just
(b) Use the definition of surface charge density to obtain: outside the surface of a conductor is given by E = |σ|/0 .
Typical field points to the left and right of the conducting
Q Q sheet are shown in diagram1.374.
σ= = 2 ⇒ Q = σπr2
A πr
Substitute numerical values and evaluate Q: y, m
2.50 m

Q = π 14.17nC/m2 (0.0100 m)2




= 4.45pC ฀ ฀ ฀ ฀
EVleft EVright EV left EVright
x, m
58. APPROACH Because the metal slab is in an external Vleft 0 Vright
electric field, it will have charges of opposite signs induced
thin square conducting sheet
on its faces. The induced charge σ is related to the electric Q = 80.0 P C
field by E = σ/ε0 .
net
2.50 m
SOLUTION Relate the magnitude of the electric field
to the charge density on the metal slab: Figure 1.374

σ (b) We can use the fact that the net charge on the
E=
0 conducting sheet is the sum of the charges Qleft and
Use its definition to express σ : Qright on its left and right surfaces to obtain an equation
relating these charges. Because the resultant electric field
Q Q is zero inside the sheet, we canV obtain
 a second equation
σ= = 2
A L in Qleft and V
Qright
P
that we can solve simultaneously with
the first equation to find Qleft and Qright . The resultant
Substitute for σ to obtain: P ฀ ฀
electric field is the superposition of three fields-the

field
Q due to the charges on the infinite nonconducting sheet
E=
L2 ε0 and the fields due to the charges on the surfaces of the
thin฀square
฀ ฀ conducting

฀ ฀ sheet.
฀ The electric
฀ ฀ field
฀ intensity
Substitute numerical values and evaluate E : due to a uniformly charged nonconducting infinite sheet
1.2nC is given by E = |σ|/20 . Typical field points for each
E= of the four regions of interest are shown in Fig.1.375.
(0.12 m)2 (8.854 × 10−12 C2 /N · m2 )
Note: The vectors in this figure are drawn consistent
= 9.4kN/C
σ 2.00 μ C/m 2
59. APPROACH From Gauss’s law we know that the elec- Q net = 80.0 μ C
tric field at the surface of the charged sphere is given by EQ right
E Qleft Einfinite
E = kQ/R2 where Q is the charge on the sphere and R is Einfinite
sheet

V V PIII
its radius. The minimum radius for dielectric breakdown
sheet

PI
corresponds to the maximum electric field at the surface PII PIV
x, m
of the sphere. E Qright −2.50 m E Q left
E Qright Einfinite
sheet
0 Einfinite E
sheet Qleft

Use Gauss’s law to express the electric field at the surface I II III IV
Qleft Qright
of the charged sphere: E Qright
E Qleft
thin square conducting sheet
kQ infinite charged nonconducting sheet
E= 2
R
Figure 1.375
Express the relationship between E and R for dielectric
breakdown:
with the charges Qleft and Qright both being positive. If
either Qleft or Qright are negative then the solution will
r
kQ kQ
Emax = 2 ⇒ Rmin = produce a negative value for either Qleft or Qright .
Rmin Emax
1.29. ANSWER KEYS AND SOLUTIONS 181

SOLUTION (a) Because the square sheet is a conduc- where A is the area of one side of the thin square conduct-
tor, half the charge on each surface is half the net charge ing sheet.
on the sheet: Because the electric field is zero inside the thin square
conducting sheet:
1 σinfinite σleft σright
2 Qnet sheet
+ − = 0 or 2.00µC/m2 +σleft −σright = 0
σ1eft = σright = 2ε0 2ε0 2ε0
A
(1.183)
Substitute numerical values and evaluate σleft and σright :
Solving equations (1.182) and (1.183) simultaneously
1 yields:
r
2 (80.0µC) µC
σleft = σright = = 1.60 2 σleft = 0.60µC/m2
(5.00 m)2 m
and
For |x|<< 5.00 m, the electric field is given by the expres-
σright = 2.60µC/m2
sion for the field just outside a conductor:
~ II :
Substitute numerical values and evaluate E
σ
E|x|<<.00 m =
0 !
~ II = 2.00 µC µC µC
m2 − 0.60 m2 − 2.60 m2
Substitute numerical values and evaluate E|x|<<.00 E î
m: C 2 
2 8.854 × 10−12 N·m 2
s
1.60µC/m2

E|x|<5.00 = kN
m = −67.8 î
8.854 × 10−12 C2 /N · m2 C
= 180.7kN/C = 181kN/C The resultant electric field in Region IV is the sum of the
fields due to the charge on the infinite nonconducting sheet
For x > 0, E|x|<<5.00 m is in the +x direction and for
and the charges on the two surfaces of the thin square
x < 0, E|x|<<.00 m is in the −x direction.
conducting sheet:

→ −
→ −
→ −→
(b) The resultant electric field in Region II is the sum of E IV = E infinite + E Qleft + E Qrght
the fields due to the infinite nonconducting sheet and the sheet
σinfinite
charge on the surfaces of the thin square conducting sheet: sheet σleft σright
= î + î + î
2ε0 2ε0 2ε0
~ II = E
E ~ infinite + E ~Q + E ~Q σinfinite + σleft + σright
!
lef ngtt
sheet
σinfinite = î
= sheet 2ε0
20
σleft σright ~ Iv :
= î − î − î Substitute numerical values and evaluate E
20 20 !
2.00 µC µC µC
 
σinfinite − σsheet − σright ~ m2 + 0.60 m2 + 2.60 m2
= î EIV = î
20 C2

2 8.854 × 10−12 N·m 2
 
Due to the presence of the infinite nonconducting sheet, kN
= 294 î
the charges on the thin square conducting sheet are redis- C
tributed on the left and right surfaces. The net charge on
the thin square conducting sheet is the sum of the charges 61. APPROACH Because the difference between the field-
on its left- and righthand surfaces: just to the right of the surface Ex,pos and the field just
to the left of the surface Ex,neg is the field due to the
Qleft + Qright = 80.0µC nonuniform surface charge, we can express Ex,neg as the
difference between Ex,pos and σ/∈0 .
where we’ve assumed that Qlett and Qright are both pos- Express the electric field just to the left of the origin in
itive. terms of Ex,pos and σ/∈0
Writing this equation in terms of the surface charge den- SOLUTION Express the electric field just to the left of
sities yields: the origin in terms of Ex, pos and σ/0 :
Qlet Qright σ
σleft + σright = + Ex,neg = Ex,pos −
A A 0
Qlett + Qright Substitute numerical values and evaluate Ex, neg :
=
A (1.182)
80.0µC 3.10µC/m2
= Ex,neg = 4.65 × 105 N/C −
(5.00 m)2 8.854 × 10−12 C2 /N · m2
= 3.20µC/m2 = 115kN/C
182 CHAPTER 1. ELECTRIC CHARGE AND FIELD

62. APPROACH Because the atom is uncharged, we know Using the integration by parts, we get:
that the integral of the electron’s charge distribution over Z r
all of space must equal its charge qe .Evaluation of this 1 h  i
x2 e−2x/a dx = e−2r/a a e−2r/a − 1 a2 − 2ar − 2r2
integral will lead to an expression for ρ0 . In (b) we can 0 4
express the resultant electric field at any point as the sum r2
 
1  r
= e−2r/a a3 e−2r/a − 1 − 2 − 2 2
of the electric fields due to the proton and the electron 4 a a
cloud. a3 
  
r r 2

SOLUTION (a) Because the atom is uncharged, the in- = 1 − e−2r/a − 2e−2r/a + 2
4 a a
tegral of the electron’s charge distribution over all of space Z r
must equal its charge e: 2 0
Substituting for r0 e−2r /a dr0 in the expression for
Z ∞ Z ∞ 0
e= ρ(r)dV = ρ(r)4πr2 dr Q(r) and simplifying yields:
0 0
r2
  
−e r
Substitute for ρ(r) and simplify to obtain: Q(r) = (1 − e−2r/a ) − 2e−2r/a + 2
4 a a
Z ∞
e=− ρ0 e−2r/a 4πr2 dr Substitute for Q(r) in equation (1.184) and simplify to
0 obtain:
Z ∞
r2 e−2r/a dr r2
  
= −4πρ0 ke ke r
0 E(r) = 2 − 2 (1 − e−2r/a ) − 2e−2r/a + 2
r 4r a a

a3 r2
Z   
ke 1   r
Integrate by parts to obtain: r2 e−2r/a dr = = 2 1− 1−e −2r/a
− 2e −2r/a
+
0 4 r 4 a a2
R∞
Substitute for 0
2 −2r/a
r e dr to obtain: Solving for ρ0 63. APPROACH See Fig.1.376. If the patch is small
yields: enough, the field at the center of the patch comes from
a3
e = −4πρ0 ( ) = −πa3 ρ0 two contributions. We can view the field in the hole as the
4 sum of the field from a uniform spherical shell of charge
Solving for ρ0 yields: Q plus the field due to a small patch with surface charge
e density equal but opposite to that of the patch cut out.
ρ0 = SOLUTION (a) Express the magnitude of the electric
πa3
(b) The field will be the sum of the field due to the proton
and that of the electron charge cloud:

E = Ep + Ecloud

Express the field due to the electron cloud:


kQ(r)
Ecloud (r) =
r2
where Q(r) is the net negative charge enclosed a distance r
(a) (b)
from the proton. Substitute for Ep and Ecloud to obtain:
Figure 1.376
ke kQ(r)
E(r) = + (1.184)
r2 r2
field at the center of the hole:
Q(r) is given by:
E = Espherical + Ehole
Z r shell
02 0 0
Q(r) = 4πr ρ (r ) dr
0
Z r Apply Gauss’s law to a spherical gaussian surface just out-
side the given sphere:
0
= 4π r02 ρ0 e−2r /a dr0
0
 Qenclosed Q
From Part (a), ρ0 = Espherical 4πr2 = =
πa3 :
−e
shell ε0 ε0
 Z r
Q(r) = 4π
−e
r02 e−2r /a dr0
0 Solve for Espherical to obtain:
πa3 0
shell
Z r
−4e 0 Q
= r02 e−2r /a dr0 Espherical =
a3 0 shell 4π ∈0 r2
1.29. ANSWER KEYS AND SOLUTIONS 183

The electric field due to the small hole (small enough so z


that we can treat it as a plane surface) is:
−σ
Ehole =
2ε0 R
Substitute for Espherical and Ehole and shell simplify to
shell m,q
obtain:
Q −σ
E=
4π ∈0 r2
+
2ε0
−λ
Q Q
= −
4π ∈0 r2 2 ∈0 (4πr2 ) O y
Q
= radially outward
8π ∈0 r2
(b) Express the force on the patch:
x
F = qE Figure 1.377
where q is the charge on the patch.
Assuming that the patch has radius a, express the propor- Substitute for E and simplify to obtain:
tion between its charge and that of the spherical shell:
r
q Q a2 2kqλ
= or q = Q v=
πa2 4πr2 4r2 m
Substitute for q and E in the expression for F to obtain: (b) The speed of the particle is equal to the circumference
 2   of its orbit divided by its period:
a Q
F = Q
4r2 8π ∈0 r2 2πR 2πR
v= ⇒T =
Q2 a2 T v
= radially outward
32π ∈0 r4 Substitute for v and simplify to obtain:
(c) The pressure is the force exerted on the patch divided r
2m
by the area of the patch: T = πR
kqλ
Q2 a2
32πε0 r 4 Q2
P = = 65. In Part (a), you can apply Gauss’s law to express E ~ as
πa2 32π 2 ε0 r4
a function of r for the uniformly charged nonconducting
64. APPROACH (a) We can apply Newton’s second law sphere with its center at the origin[Fig.1.378]. In Part (b),
to the particle to express its speed as a function of its you can use the hint to express the field at a generic point
mass m, charge q, and the radius of its path R, and the P (x, y) in the cavity as the sum of the fields due to equal
strength of the electric field due to the infinite line charge positive and negative charge densities and then evaluate
E[Fig.1.377]. (b) The period of the particle’s motion is the this expression at points 1 and 2.
ratio of the circumference of the circle in which it travels (a) The electric field at a distance r from the center of
divided by its orbital speed.
SOLUTION (a) Apply Newton’s second law to the par-
ticle to obtain:
X v2
Fradial = qE = m
R
where the inward direction is positive.
Solving for v yields:
r
qRE
v=
m
The strength of the electric field at a distance R from the
infinite line charge is given by:
2kλ
E= Figure 1.378
R
184 CHAPTER 1. ELECTRIC CHARGE AND FIELD

The vectors ~r = rr̂ and −


→ 0
the uniformly charged nonconducting sphere is given by: r = r0 r̂ 0 given by:

→ −

r = xî + y ĵ and−
→ 0
r = (x − b)î + y ĵ
E ρ = Er̂ (1.185)

where r̂ is a unit vector pointing radially outward. Apply where x and y are the coordinates of any point in the
Gauss’s law to a spherical surface of radius r centered at cavity.
the origin to obtain:
Substitute for rr̂ and r0 r̂0 in equation (1.187) and simplify
 Qinside
I
En dA = Eρ 4πr2 = to obtain:
S ε0

→ ρ ρ ρb
E = (xî + y ĵ) − [(x − b)î + y ĵ] = î
Relate Qinside to the charge density ρ: 30 30 30


ρ=
Qinside 4
⇒ Qinside = ρπr3 Because E is independent of x and y :
4 3 3
3 πr
~ 2 = ρb î
~1 = E
E
Substitute for Qinside : 30

 4ρπr3 66. APPROACH The electric field in the cavity is the sum
Eρ 4πr2 = of the electric field due to the uniform and positive charge
30
distribution of the sphere whose radius is a and the electric
Solve for Eρ to obtain: field due to any charge in the spherical cavity whose radius
ρr is b. You can use the hint given in Problem 66. to express
Eρ = the field at a generic point P (x, y) in the cavity as the
3 ∈0
sum of the fields due to equal positive and negative charge
Substitute for E in equation (1.185) to obtain: densities to show that E ~ = ρ bî.
3ε0
SOLUTION The electric field at point P (x, y) is the sum

→ ρ of the electric fields due to the two charge distributions:
Eρ = rr̂
30

→ − → −

E = E ρ + E −ρ = Eρ r̂ + E−ρ r̂ 0 (1.188)
(b) The electric field at point P (x, y) is the sum of the
electric fields due to the two charge distributions: where r̂0 is a unit vector normal to a spherical Gaussian

→ − → −
→ surface whose center is at x = b.
E = E ρ + E −ρ = Eρ r̂ + E−ρ r̂ 0 (1.186) Apply Gauss’s law to a spherical surface of radius r0 cen-
tered at x = b = R/2 to obtain:
where r̂ 0 is a unit vector normal to a spherical Gaussian
surface whose center is at x = b.  Qinside
I
En dA = E−ρ 4πr02 =
Apply Gauss’s law to a spherical surface of radius r0 cen- S 0
tered at x = b = R/2 to obtain:
Relate Qinside to the charge density −ρ : Substitute for
 Qinside Qinside to obtain:
I
En dA = E−ρ 4πr02 =
S ε0 Qinside 4
−ρ = 4 03
⇒ Qinside = − ρπr03
πr 3
Relate Qinside to the charge density −ρ : 3

Qinside 4 Solving for E−ρ yields:


−ρ = 4 03
⇒ Qinside = − ρπr03
3 πr 3 4πr03 ρ
E−ρ 4πr02 = −

30
Substitute for Qinside to obtain:
From Problem 66.:
02
 4πr03 ρ ρ 0
E−ρ 4πr =− E−ρ = − r
30 30

Solving for E−ρ yields: Substitute for Eρ and E−ρ in equation (1.188) to obtain:
ρ 0 ρ
E−ρ = − r Eρ = r (1.189)
30 3ε0

Substitute for Eρ and E−ρ in equation (1.186) to obtain: The vectors − →


r = rr̂ and −
→ 0
r = r0 r̂ 0 are given by: ~r =

→ 0
xî + y ĵ and r = (x − b)î + ĵ ĵ, where x and y are the

→ ρ ρ 0
E = rr̂ − r r̂ (1.187) coordinates of any point in the cavity.
3 ∈0 30
1.29. ANSWER KEYS AND SOLUTIONS 185

Substitute for rr̂ and r0 r̂ 0 in equation (1.189) and simplify Substitute for E in equation (1.191) to obtain:
to obtain:

E~ = ρ (xî + y ĵ) − ρ [(x − b)î + y ĵ] 


d 2kλ

2kλp
3 30 F =p =− 2
r0 dr r r
ρ
= bî
30
where the minus sign indicates that the dipole is attracted
67. APPROACH The electric field at a generic point to the line charge.
P (x, y) in the cavity is the sum of the fields due to the
positive charge density and the total charge Q.
SOLUTION The electric field at point P (x, y) is the sum
of the electric fields due to the two charge distributions:
Multiple Choice Assignments
~ =E
E ~ρ + E
~ −ρ + E
~Q (1.190)
Level 1
where r̂0 is a unit vector normal to a spherical Gaussian
surface whose center is at x = b.
From Problem 66.: Q.No. 1 2 3 4 5 6 7 8 9
Ans. D D B A B A D A D

→ −
→ ρb
E ρ + E −ρ = î Q.No. 10 11 12 13 14 15 16 17 18
3ε0 Ans. D C A A A A C D A
Q.No. 19 20 21 22 23 24 25 26 27
Substituting in equation (1.190) yields:
Ans. C A A B B A C A B

→ ρb −
→ Q.No. 28 29 30 31 32 33 34 35 36
E = î + E Q Ans. D B A A C C B B A
30
Q.No. 37 38 39 40 41 42 43 44 45
Assuming that the cavity is filled with positive charge Q: Ans. D A C A C B A B C
Q.No. 46 47 48 49 50 51 52 53 54

→ ρb Q
E = î + r0 r̂ Ans. C B A B B C C D D
30 4π ∈0 b3
Q.No. 55 56 57 58 59 60 61 62 63
Ans. C C A A C A C D B
The vectors ~r = rr̂ and −
→ 0
r = r0 r 0 are given by:
Q.No. 64 65 66 67 68 69 70 71 72
~r = xî + y ĵ and ~r0 = (x − b)î + y ĵ where x and y are the Ans. B A B D A A A A D
coordinates of any point in the cavity. Q.No. 73 74 75 76 77 78 79 80 81
Ans. D C A C D C C B A


Substitute for rr̂ and r0 r̂ 0 and simplify to obtain: E =
ρb Q
3ε0 î + 4πε0 b3 [(x − b)î + ŷ]
At point 1, x = 2b and y = 0 :
Level 2
 
~ ρb Q ρb Q
E(2b, 0) = î − [(2b − b)î] = + î
3ε0 4π0 b3 3ε0 4πε0 b2
Q.No. 1 2 3 4 5 6 7 8 9
At point 2, x = 0 and y = 0:
Ans. C D A D B B C B B
Q.No. 10 11 12 13 14 15 16 17 18
 
~ 0) = ρb î + Q [(−b)î] =
E(0,
ρb

Q

30 4πε0 b3 3ε0 4πε0 b2 Ans. A C B D A D A A A
Q.No. 19 20 21 22 23 24 25 26 27
68. APPROACH We can find the field due to the infinitely Ans. A C A A C D
long line charge from E = 2kλ/r and the force that acts
on the dipole using F = pdE/dr.
SOLUTION Express the force acting on the dipole:
Level 3
dE
F =p (1.191)
dr
Q.No. 1 2 3 4 5 6 7 8 9
The electric field at the location of the dipole is given by: Ans. B D C A D C C C C
2kλ Q.No. 10 11 12 13 14 15 16 17 18
E= Ans. C C D B A B C C A
r
186 CHAPTER 1. ELECTRIC CHARGE AND FIELD

Level 4
Section A
Q.No. 1 2 3 4 5 6 7 8 9
Ans. A B A D D B A D A
Q.No. 10 11 12 13 14 15 16 17 18
Ans. A A D A A C A C B
Q.No. 19 20 21 22 23 24 25 26 27
Ans. D C B A B D D A B
Q.No. 28 29 30 31 32 33 34 35 36
Ans. A B D D B B B C C
Q.No. 37 38 39 40 41 42 43 44 45
Ans. C C D B C B B C B
Q.No. 46 47 48 49 50
Ans. A −48 D A
Section B

Q.No. 1 2 3 4 5 6 7 8
Ans. A B D B A C B D
Q.No. 9 10 11 12 13 14 15 16
Ans. C D D A A AD A D
Q.No. 17 18 19 20 21 22 23 24
Ans. C ACD BD C C A 2 3
Q.No. 25 26 27 28 29 30 31
Ans. 6 AB B BC ABC AC 6.40
Chapter 2

Electric Potential

In last chapter we have seen that the electric field vector E~ y


provides us with a mechanism for the exertion of electric force.
In this chapter we will build on the mechanical concepts of S
work and energy, and define a scalar potential V as the work Fext
done per unit charge by the electric field E. The electrical
force of E on a charge q leads to electrical potential energy, B Uf
denoted by U . The amount of electric potential energy equals
S S
Fg 5 mg
the product of q and V
One of the advantages of using the potential energy rather s
than the force is that the potential energy is a scalar quantity, S
while force is a vector. If we know the potential energy, g
yB
we may find the force by appropriate derivatives. Similarly,
the potential is a scalar quantity, easier to handle in many A Ui
respects than the electric field, which is a vector quantity, and yA
if we know the electric potential of a charge distribution, we O
can derive the electric field due to that distribution from it.
(a)
y
2.1 Electric Potential Energy and + + + + +
S
Electrostatic Potential in Fields S Fext
E
From mechanics, we know that every conservative force field B Uf
is a potential field, i.e., the term potential energy can be as-
S S
FE 5 q0 E
sociated with every conservative force field. Like gravitational
field, the electrostatic field is also conservative force field, so s
we can associate the potential energy (U ) with it. The electric
potential energy U like the gravitational potential energy, is yB
a scalar that depends on both the source and the test object
under consideration (for example, test charge). In the case of A Ui
gravity, the important property of each object is its mass. In yA
the case of electricity, the important property is the charge. O
Electric potential energy depends on the charge of both the – – – – –
source and the object. Also, like gravity, electric potential en-
(b)
ergy depends on the configuration of the system.
Figure 2.1: The motion of a point mass m in a gravitational field
The motion of a particle with positive charge q0 in a uniform is analogous to the motion of a test charge q0 in an electric field. If
electric field is analogous to the motion of a particle of mass the speed of the particle is constant, the change in potential energy
m in the uniform gravitational field near the earth; see Fig. is related to the work done by an external agent: Wext = +∆U .
2.1.
When you move a particle of mass m, against the gravitational
field and increase it’s height from A to B (Fig.2.2a and 2.1a),
you have to do a positive work Wext . If your external force will not change and it will move with a constant velocity
on the particle, is just equal and opposite to the force due to v(say). In this case, all the external work (Wext ) increases the
the gravitational field, then the kinetic energy of the particle gravitational potential energy of the earth-particle system:

187
188 CHAPTER 2. ELECTRIC POTENTIAL

(v constant ) Wext = +∆U = Uf − Ui (2.1)

where Uf and Ui are the final and initial potential energies re-
spectively.
Similarly, when a positive charge is moved to against electric
field, its electrostatic potential energy increases.
Figure 2.2
The gravitational potential energy function near the surface of
the earth is Ug = mgy. We can obtain a function that does
not depend on m by defining the gravitational potential as the
potential energy per unit mass: Vg = Ug /m = gy. The SI unit charges tend to decrease the electrostatic potential energy.
of Vg is J/kg. The gravitational potential at a point is the Let us consider two points A and B in an electrostatic field
external work needed to lift a unit mass from the zero level of (Fig.2.2(b)), and suppose the electric force applied by electric
potential (y = 0) (say) to the given height, without a change field on a positive test charge q0 has a component f from point
in speed. A useful feature of the potential function is that B to A. So, if we move this positive test charge infinitely
it depends only the source of the field (the earth) through the slowly (without any acceleration) from A to B, we do work
value of the gravitational field strength g, and not on the value against this component of the electric field E.~ We define the
of the “test” mass, m. potential difference between B and A as the work done by
Similarly, when a charge q0 moves between two points in an external force in moving a unit positive test charge, infinitely
electrostatic field, the change in electric potential, ∆V , is de- slowly, from position A to B.
fined as the change in electrostatic potential energy per unit It is always helpful to defined the electric potential energy
charge, in terms of work done by external agent, conventionally, the
∆U potential energy is defined in terms of work done by internal
∆V = (2.2)
q0 conservative forces within the system of interacting particles,
The SI unit of electric potential is the volt (V), in honor of not in terms of work done by external agent.
Alessandro Volta, inventor of the voltaic pile (the first primitive As we have seen in mechanics, the definition of potential
electric battery). Note that energy in terms of the work done (Wc ) by the conservative
force is ∆U = Uf − Ui = −Wc . The negative sign tells us
1V = 1J/C that positive work done by the conservative force leads to
a decrease in potential energy. In an electrostatic field, the
The quantity ∆V depends only on the field set up by the source conservative force on a test charge q0 is Fc = FE = q0 E.
charges, not on the test charge. Once the potential difference Therefore, the infinitesimal change in electric potential energy
between two points is known, the external work needed to move dU = −dWE , associated with an infinitesimal displacement
a charge q0 , with no change in its speed, may be found from d~s, is-
Eq.2.1: dU = −dWE = −q0 E ~ · d~s
So, the infinitesimal change in electric potential in displace-
ment d~s, is-
(v constant ) Wext = ∆U = q0 ∆V = q0 (Vf − Vi ) (2.3) dU ~ · d~s
dV = = −E
q0
The sign of this work depends on the sign of q0 and the If the test charge q0 moves from position A to position B
relative magnitudes of Vi and Vf . If Wext > 0, the positive ~ and potential change from VA to
(Fig.2.3), in electric field E
test charge (q0 ) is moving in the direction of force applied by VB , then -
external agent, i.e., opposite to the electric field. If Wext < 0,
the positive test charge q0 is moving opposite to the force Z B
applied by external agent, i.e. the test charge q0 is moving in VB − VA = − ~ · d~s
E (2.4)
the direction of electric field. A
From Eq.(2.3), we see that only changes in potential, rather
than the specific value of Vi and Vf , are significant. One can
choose the reference point at which the potential is zero at Since the electrostatic field is conservative, the value of this
some convenient point such as infinity. In electronic circuits it line integral depends only on the end points A and B, not on
is convenient to choose the ground connection to earth as the the path taken. The sign of the integral is determined (1) by
zero of potential. If Vi = 0, we may write Vf = Wext /q0 : the signs of the components of E, and (2) by the direction of
The potential at a point is the external work needed to bring a the path taken-which is indicated by the limits.
unit positive test charge, at constant speed, from the position If ~rA and ~rB are position vectors of points A and B respec-
of zero potential to the given point. tively, then Eq.(2.4) can also be written as-
If allowed to, positive charges tend to move “downhill” in
potential, just as do ordinary masses in a gravitational field. Z B
However, negative charges tend to move “uphill” in potential. VB − VA = − ~ · d~r
E (2.5)
In an external electric field, both positive and negative A
2.1. ELECTRIC POTENTIAL ENERGY AND ELECTROSTATIC POTENTIAL IN FIELDS 189

Test charge q0 moves from A to B


along an arbitrary path.
S
S
F E

B
S
dr S
ds
q0 θ
dr

rB
r

A rA
q
(a)

Figure 2.3: The work done on charge q0 by the electric field of


charge q does not depend on the path taken, but only on the dis-
tances rA and rB .

and change in electric potential energy- (b)

Z B Figure 2.4
∆U = q0 (VB − VA ) = −q0 ~ · d~r
E (2.6)
A

You can always use these expressions without bothering about CB. Since, E is perpendicular to the displacement along BC,
no work will be done by field E ~ on a test charge along this
actual path of motion of test charge q0 .
segment. Work is done only along the segment AC parallel to
the field lines. Since only the component of the displacement
along, or against, the field lines is significant, Eq. (2.8) is often
2.1.1 Potential and Potential Energy in a written in the form
Uniform Field
In a uniform field, E~ is constant, and therefore the integral in (UniformE) ∆V = ±Ed (2.9)
Eq. (2.4) may be written as
Z Z where d is the magnitude of the component of the displacement
~ · d~s = E
E ~ · d~s = E
~ · ∆~s along, or against, the field. The positive sign applies to a
displacement opposite to the field. From Eq.(2.9) we see that
The finite change in potential ∆V associated with a finite dis- an equivalent unit for electric field is V /m: 1V /m = 1N/C
placement ∆~s takes the form Definition of 1 volt: The potential difference between two
points B and A is one volt if the work done in taking one
(Uniform E ~ ) ∆V = −E ~ · ∆~s (2.7) coulomb of positive charge from A to B is one joule.
From this definition, if a test charge of q0 coulombs is moved
Note that ∆s and ∆V depend only on the initial and final through a potential difference (p.d.) of V volt, then the work
positions, not on the path taken. Figure 2.4 shows a uniform done W in joules is given by
~ = E î. Let us find the change in potential in going from
field E
point A to point B, which are separated by a distance d along
the lines. Since the electric field has only an x component, W = q0 V (2.10)
Eq. (2.7) reduces to ∆V = −Ex ∆x. If we write Ex = E and
∆x = +x, we have
Note: Electric field lines always point in the direction
V (x) − V (0) = −Ex (2.8) of decreasing electric potential, as shown in Figure 2.5a
F Electric potential energy or self energy of a system is the
The potential decreases linearly along the x axis, as depicted work done by external force against the system conservative
in the graph of Fig. 2.4b. Notice that the field lines point force in assembling the charges from infinite separation to
from high potential to low potential. Suppose now that the present configuration without change in kinetic energy of
actual path in Fig. 2.4 is replaced by the two steps AC and any particle.
190 CHAPTER 2. ELECTRIC POTENTIAL

position, therefore, the work done by external forces in moving


the charge q0 from A to B is
B

→ −

Z
WAB = (− F ext ).(−dr)
A
B
→ −
− →
Z
= F ext .dr
A

[Taking increasing direction of − →


r as +ve] Since,
the direction of electric force is just opposite to the direction
of applied external force, therefore, we can also write-

B
→ −
− →
Z
WAB = − F E .dr (2.11a)
A

Figure 2.5: (a) When the electric field E ~ is directed downward, −


→ −→
point B is at a lower electric potential than point A. When a
as, F E = q0 E , therefore-
positive test charge moves from point A to point B, the charge-field
B
→−
− →
Z
system loses electric potential energy. (b) When an object of mass
WAB = −q0 E .dr (2.11b)
m moves downward in the direction of the gravitational field ~g , the A
object-field system loses gravitational potential energy.
This work is done against electrostatic repulsive force and get
stored as electrostatic potential energy in the two charge sys-
2.1.2 Electrostatic Potential Energy and Po- tem.
tential Difference due to a Point Charge Note that, at every point in electric field, a particle with charge
~ due to a point charge q placed at q0 possesses a certain electrostatic potential energy, and this
Let us consider the field E
work done increases its potential energy by an amount equal
the origin. Now, imagine that we bring a test charge q0 from a
to potential energy difference between points A and B.
point A to a point B against the repulsive force on it due to the
Thus, potential energy difference,
charge q. With reference to Fig. 2.6, this will happen if q and q0
are both positive or both negative. For definiteness, let us take B
→−
− →
Z
q, q0 > 0. Two remarks may be made here. First, we assume ∆U = UB − UA = WAB = −q0 E .dr (2.12)
A

The integral in this expression is a line integral whose value is


independent of the path of integration between points A and
B-that is precisely why Eq.2.12 involves only a difference in
the value of the potential energy at the two end points.
Therefore, we can define electric potential energy difference
between two points as the work required to be done by an ex-
ternal force, against conservative electrostatic system force, in
Figure 2.6 moving (without accelerating) the charge q0 from one point to
another for electric field of any arbitrary charge configuration.
that the test charge q0 is so small that it does not disturb the In terms of the system electrostatic force, the potential en-
original configuration, namely the charge q at the origin (or ergy difference can be defined as the negative of the work
else, we keep q fixed at the origin by some unspecified force). done by the system’s net electrostatic force in moving the
Second, in bringing the charge q0 from A to B, we apply an test charge q0 from one point to another very slowly with
external force Fext just enough to counter the repulsive electric constant velocity within the system’s electric field.
force FE (i.e,Fext = −FE ). This means there is no net force
on or acceleration of the charge q0 when it is brought from In Eq. 2.12, electric field E ~ and ~r are directed in same
A to B, i.e., it is brought with infinitesimally slow constant direction, so, ~
E.d~
r = Edr, therefore Eq. 2.12 can also be
speed. In this situation, work done by the external force is written as-
the negative of the work done by the electric force, and it gets Z B
fully stored in the form of electrostatic potential energy of the
∆U = UB − UA = WAB = −q0 Edr (2.13)
charge q and q0 system. If the external force is removed on A
reaching B, the electric force will take the charge away from q
- the stored energy (potential energy) for position B, is used From this result, we see that if q0 is positive, then ∆U is neg-
to provide kinetic energy to the test charge q0 in such a way ative. We conclude that a system consisting of a positive
that the sum of the kinetic and potential energies is conserved. charge and an electric field loses electric potential
Since, electrostatic force applied by q on q0 is the function of energy when the charge moves in the direction of
2.1. ELECTRIC POTENTIAL ENERGY AND ELECTROSTATIC POTENTIAL IN FIELDS 191

the field. This means that an electric field does work on is identical to Eq.2.14. For segment 2, which follows a
a positive charge when the charge moves in the direction of circumference at a distance rB from the origin, the integral
the electric field. (This is analogous to the work done by the is zero because the electric force F~E is perpendicular to the
gravitational field on a falling object, as shown in Figure 2.5b.) path segment d~s everywhere. So, the result for the change in
If a positive test charge is released from rest in this electric potential energy is still given by Eq.2.14.
~ in the direction of E
field, it experiences an electric force q0 E ~ Note: Although we have shown the result for circular segment
(downward in Fig. 2.5a). Therefore, it accelerates downward, 2, the result holds for paths of any shape(Fig.2.3).
gaining kinetic energy. As the charged particle gains Equation (2.14) shows that the change in electric potential
kinetic energy, the charge-field system loses an equal energy is given by the difference of two functions, U (rB ) and
amount of potential energy. This is accordance with the U (rA ). We can therefore choose the zero of the potential
conservation of energy in an isolated system. energy function to be at whatever value of r we like. It is
If q0 is negative, then ∆U in Equation (2.13) is positive and convenient and natural to choose zero potential energy to be
the situation is reversed: A system consisting of a nega- at infinity. We can do this if we let rA → ∞ and let rB take
tive charge and an electric field gains electric potential on a general value r in Eq. (2.14):
energy when the charge moves in the direction of the
qq0
field. If a negative charge is released from rest in an electric ∆U = U (r) − U (rA )|rA →∞ = k
field, it accelerates in a direction opposite the direction of the r
field. In order for the negative charge to move in the direction We then say that the potential energy of a system having
of the field, an external agent must apply a force and do charge q0 a distance r from charge q is the difference in poten-
positive work on the charge. tial energy between that point and infinity. When we reverse
Now, since, the magnitude of electric field at P , due to point the roles of q and q0 , the potential energy of q at a distance
charge q at O is given by- r from q0 is again kqq0 /r. We can then say that the electric
q 1 potential energy U (r) for a system of two point charges q and
E = k 2, with k =
r 4πε0 q0 separated by a distance r is-
Therefore, Eq.2.13, gives-
Z rB Z rB
q dr qq0
∆U = −q0 k 2 dr = −kqq0 U (r) = k (2.15)
rA r r r2 r
 rB  A 
−1 1 1
= −kqq0 = kqq0 − It is indeed true that U (r) = 0 in the limit r → ∞. Thus
r rA rB rA the system has no potential energy when the two charges are
 rB
infinitely far apart. Note that the potential energy of the two
  
−1 1 1
⇒ ∆U = −kqq0 = kqq0 − (2.14)
r rA rB rA charges depends only on the distance r between them and on
the magnitudes and signs of the charges. Two important com-
here, ~rA and ~rB are the position vectors of points A and B,
respectively, with respect to charge q at origin O. ments may be made at this stage:
Since, electric force is conservative, therefore, the work done 1. The right side of Eq. (2.12) depends only on the initial and
final positions of the charge. It means that the work done
by an electrostatic field in moving a charge from one point
to another depends only on the initial and the final points
and is independent of the path taken to go from one point
to the other. This is the fundamental characteristic of
a conservative force. The concept of the potential energy
would not be meaningful if the work depended on the path.
The path-independence of work done by an electrostatic
field can be proved using the Coulomb’s law.

2. Equation (2.12) defines potential energy difference in


terms of the physically meaningful quantity work.
Clearly, potential energy so defined is undetermined to
within an additive constant. What this means is that the
Figure 2.7: The two points are not along the same radius. The actual value of potential energy is not physically signifi-
path is here taken to run radially outward to the radius of point B,
cant; it is only the difference of potential energy that is
and then follow the circumference at that radius.
significant. We can always add an arbitrary constant α to
potential energy at every point, since this will not change
by this force is path independent. So, electric potential energy the potential energy difference:
difference between two points A and B (Fig.2.7) is still given
by Eq.2.14, whether q0 follows dashed path or directly moves (UB + α) − (UA + α) = UB − UA (2.16)
from point A to B.
Explanation: For segment 1 (Fig.2.7), which runs outward Put it differently, there is a freedom in choosing the point
radially from A to a distance rB from the origin, the result where potential energy is zero. A convenient choice is to
192 CHAPTER 2. ELECTRIC POTENTIAL

have electrostatic potential energy zero at infinity. With with d~r = dxî + dy ĵ + dz k̂.
this choice, if we take the point A at infinity, we get from Now, substitute the given values and integrate for given limits.
Eq.(2.12) SOLUTION Given that E ~ = 10xî − 30y 2 ĵ, therefore-
W∞B = UB − U∞ = UB (2.17) Z (3,4)
∆V = − ~ r
E.d~
Since the point B is arbitrary, Eq. (2.17) gives the definition of (0,0)
electrostatic potential energy of the system containing source Z (3,4)
(or sources) of electric field and test charge q0 at any general =− (10xî − 30y 2 ĵ).(dxî + dy ĵ + dz k̂)
point. (0,0)
Z 3 Z 4
Electric potential energy of a system, containing the source
=− 10xdx + 30y 2 dy
(or sources) of electric field and the test charge q0 at a given 0 0
point with in the electric field, is the work done by the ex-  2 3  3 4
1 x y
ternal force in bringing the test charge q0 infinitely slowly = −10 + 30
2 3
from infinity to that point.  2 0  3 0
From Eq.2.12, we can also express the potential difference 3 4
= −10 + 30
between points A and B, as- 2 3
   
9 64
Z B = −10 + 30 = −45 + 640 = 595V
WAB →−
− → 2 3
VB − VA = =− E .dr (2.18)
q0 A
EXAMPLE 93. How much work is done by the electrical


In this definition, the infinitesimal displacement d r is inter- force when a point charge is brought from infinity to rest at a
preted as the displacement between two points in space rather distance r from a fixed charge of the opposite sign? What is
than the displacement of a point charge as in Eq.2.12. the meaning of the sign of your result?
Rr
EXAMPLE 91. E ~ = (100 V/m)î − (50 V/m)ĵ. Calculate SOLUTION The work done is given by W = ∞ F~ ·d~s, where
potential difference between (0, 0) and (3, 4). F~ is the electrical force between the charges. The quantity is
the negative of the corresponding change of electric potential
APPROACH From eq. (2.18), electric potential difference , energy, as in
between two points A and B, in terms of electric field, is given
by-
2.1.3 Check Point 1
Z B −

∆V = VB − VA = − ~ r
E.d~ 1. •• The electric field in a region is given by E = (Ax + B)î,
A where E is in NC−1 and x is in metres. The values of con-
with d~r = dxî + dy ĵ + dz k̂. stants are A = 20 SI unit and B = 10 SI unit. If the poten-
Now, substitute the given values and integrate for given limits. tial at x = 1 is V1 and that at x = −5 is V2 , then V1 −V2 is:
~
SOLUTION Given that E = (100V/m)î − (50V/m)ĵ, (A) 320 V (B) −48 V (C) 180 V (D) −520 V
therefore-
~
2. •• From the electric field E(r) = kQs /r2 r̂ produced by


a positively charged particle, find the (usual) expression for


Z (3,4)
∆V = − ~ r
E.d~ its electric potential.
(0,0)
Z (3,4)
3. •• One of these is an impossible electrostatic field. Which
=− (100î − 50ĵ).(dxî + dy ĵ + dz k̂)
(0,0) one?
Z 3 Z 4 (a) E~ = K[xy î + 2yz ĵ + 3xz k̂];
h i
=− 100dx + 50dy (b) E~ = K y 2 î + 2xy + z 2 ĵ + 2yz k̂ .

0 0
= −100[x]30 + 50[y]40 Here K is a constant with the appropriate units. For the
possible one, find the potential, using the origin as your
= −100[3 − 0] + 50[4 − 0] = −100 V reference point.
 
~ =
EXAMPLE 92. E 4. •• In some region of space, the electric field is given by
10xî − 30y 2 ĵ . Calculate potential
~ = Axı̂ + By 2 ̂. Find the electric potential difference
E
difference between (0, 0) and (3, 4).
between points whose positions are (xi , yi ) = (a, 0) and
APPROACH From eq. (2.18), electric potential difference , (xj , yf ) = (0, b). The constants A, B, a, and b have the
between two points A and B, in terms of electric field, is given appropriate SI units.
by-
5. •• On moving a charge of 20C by 2 cm, 2 J of work is done,
then the potential difference between the points is-
Z B
∆V = VB − VA = − ~ r
E.d~ (A) 0.1 V (B) 8 V (C) 2 V (D) 0.5 V
A
2.1. ELECTRIC POTENTIAL ENERGY AND ELECTROSTATIC POTENTIAL IN FIELDS 193

rf rf  
Q
Z Z
Answer Key and Solutions ∆V = − Edr = − k 2 dr
ri r r
1. APPROACH From eq. (2.18), electric potential differ- rf   i
1
Z
ence , between two points A and B, in terms of electric V = −kQ dr
field, is given by- r r2
 i  rf   rf
1 1
Z B = − kQ − = kQ
∆V = VB − VA = − ~ r
E.d~ (1) r ri r ri
A
Evaluate between limits and reduce.
with d~r = dxî + dy ĵ + dz k̂.
Now, substitute the given values and integrate within the kQ kQ
∆V = −
given limits. rf ri
SOLUTION Given that- E ~ = (Ax+B)î, A = 20, B = 10,
By convention, we can take the electrostatic potential zero
therefore,-
at infinity, therefore, the expression for the electric potential
~ r = (Ax + B)î.(dxî + dy ĵ + dz k̂) = (Ax + B)dx at a distance r from the source will become-
E.d~
potential at x = 1 is V1 , potential at x = −5 is V2 , all the ∆V = V (r) − V (∞) = V (r) − 0 = V
values are in SI units. kQ
Therefore, equation 1, gives- V =
r
Z x=−5
V2 − V1 = − ( Ax + B)dx 3. APPROACH Electric field is a conservative force field,
x=1
~ we have
therefore, for correct expression of electric field E,
Put values of A and B. −
→ ~
∇ ×E =0
20  2 x=−5 x=−5 The potential difference between any two points in an elec-
V2 − V1 = − x x=1 − 10[x]x=1
2 tric field is given by Eq.(2.18).
= −10(24) − 10(−6) = −240 + 60
Z B
= −180 V →−
− →
VB − VA = − E .dr
So, V1 − V2 = 180 V A

2. APPROACH Sketch the electric field and include a path here, d~r = dxî + dy ĵ + dz k̂
from ri to rf (Fig. 2.8). You must select a specific path to integrate along. It doesn’t
matter what path you choose, since the answer is path-
independent, but you simply cannot integrate unless you
have a definite path in mind.
SOLUTION

î ĵ k̂
rf −
→ ~
(a) ∇ × E


1 = K ∂x
∂ ∂
∂y ∂z
xy 2yz 3zx
dr
r̂ = K[î(0 − 2y) + ĵ(0 − 3z) + k̂(0 − x)] 6= 0
ri + ~
So, E1 is an impossible electrostatic field.

î ĵ k̂
E −
→ ~
(b) ∇ × E2 = K ∂x
∂ ∂ ∂
∂y ∂z
y 2 2xy + z 2 2yz
= K[î(2z − 2z) + ĵ(0 − 0) + k̂(2y − 2y)] = 0
~ 2 is a possible electrostatic field.
so E

Figure 2.8 Let’s go by the indicated path:


~ · d~l = y 2 dx + 2xy + z 2 dy + 2yzdz K
 

→ E
For the particle, electric field E and d~r (a small portion of
the path from ri to rf ) both point in the r̂ direction, so Step I: y = z = 0; dy = dz = 0.E ~ · d~l = ky 2 dx = 0.
their dot product is Edr. Now, to get potential difference, Step II: x = x0 , y :0 → y0 , z = 0.dx = dz = 0.
apply Eq. 2.18, from limit ri to rf . ~ · d~l = k 2xy + z 2 dy = 2kx0 ydy.
E
SOLUTION From Eq.2.18, we can write- R
~ · d~l = 2kx0 y0 ydy = kx0 y 2 .
R
II
E 0 0
Z rf
~ · d~r Step III: x = x0 , y = y0 , z : 0 → z0 ; dx = dy = 0.
∆V = − E (i) ~ · d~l = 2kyzdz = 2ky0 zdz.
ri
E
194 CHAPTER 2. ELECTRIC POTENTIAL

z q1

(x0 , y0 , z0 ) q2

III ✲y
I✚✚ r1

✚✚
❂ ✲ r2 q3
x ❂
✚ II

Figure 2.9 r3
A
q0
R z0
~ · d~l = 2y0 k ky0 z02 .
R
E zdz =
III
R0(x0 ,y0 ,z0 ) Figure 2.10: The potential energy associated with a charge q0 at
~ · d~l = −k x0 y 2 + y0 z 2

V (x0 , y0 , z0 ) = − 0 E 0 0 point a depends on the other charges q1 , q2 , and q3 and on their
or V (x, y, z) = −k xy + yz 2
2

distances r1 , r2 , and r3 from point A.

4. Aa2 /2 − Bb3 /3
 
that for every electric field due to a static charge distribution,
the force exerted by that field is conservative.
5. APPROACH From Eq.(2.18), the potential difference be-
Equations 2.15 and 2.19 define U to be zero when distances
tween two points A and B and the work done by external
r1 , r2 , . . . are infinite that is, when the test charge q0 is very far
force WAB , in moving a point charge q0 , are related as-
away from all the charges that produce the field. As with any
WAB
Z B
→−
− → potential-energy function, the point where U = 0 is arbitrary;
VB − VA = =− E .dr we can always add a constant to make U equal zero at any point
q0 A
we choose. In electrostatics problems it’s usually simplest to
SOLUTION (A) Put the given values of WAB and q and choose this point to be at infinity. When we analyze electric
solve for VB − VA circuits, other choices will be more convenient.
SOLUTION Given that, WAB = 2J, q0 = 20C Equation 2.19 gives the potential energy associated with
−→
So, 2
VB − VA = 20 = 0.1 V the presence of the test charge q0 in the E field produced
by q1 , q2 , q3 , . . . But there is also potential energy involved in
assembling these charges. If we start with charges q1 , q2 , q3 , . . .
all separated from each other by infinite distances and then
2.2 Electric Potential Energy with bring them together so that the distance between qi and qj is
Several Point Charges rij , the total potential energy U is the sum of the potential
energies of interaction for each pair of charges. We can write

→ this as
Suppose the electric field E in which charge q0 moves is caused
by several point charges q1 , q2 , q3 , . . . at distances r1 , r2 , r3 , . . .
from q0 , as in Fig.2.19. The total electric field at each point 1 X qi qj
U= (2.20)
is the vector sum of the fields due to the individual charges, 4π0 i<j rij
and the total work done on q0 during any displacement is the
sum of the contributions from the individual charges. From This sum extends over all pairs of charges; we don’t let
Eq.2.15, we conclude that the potential energy associated with i = j (because that would be an interaction of a charge with
the test charge q0 at point A in Fig. 2.19 is the algebraic sum itself), and we include only terms with i < j to make sure
(not a vector sum): that we count each pair only once. Thus, to account for the
  interaction between q3 and q4 , we include a term with i = 3
q0 q1 q2 q3 q0 X qi
U= + + + ··· = (2.19) and j = 4 but not a term with i = 4 and j = 3
4π0 r1 r2 r3 4π0 i ri
Methods of calculation: We can find the electric poten-
When q0 is at a different point B, the potential energy is given tial energy of system of particles by any of the following two
by the same expression, but r1 , r2 , . . . are the distances from methods-
q1 , q2 , . . . to point B. The work done by electric field on test Method 1: Keep all charges at ∞ separation from each other
charge q0 , when it moves from A to B along any path is equal and then bring them one by one in present configuration and
to the difference UB − UA between the potential energies when calculate the work done.
q0 is at B and at A.
We can represent any charge distribution as a collection of PEsys = ΣWi
point charges, so Eq.2.15 shows that we can always find a
potential-energy function for any static electric field. It follows Method 2: Find PE of each charge due to electric field of
2.2. ELECTRIC POTENTIAL ENERGY WITH SEVERAL POINT CHARGES 195

the other charges. So, we can say that the net potential energy of the system is
equal to the sum of potential energy of each pair of charges.
P E1 + P E2 + P E3 + . . . II In this approach, first of all, calculate the
P Esys = (2.21) APPROACH
2 net electric potential at each vertex due to the charges placed
where PE1 = PE12 + PE13 + . . . at other vertices. Now, multiply the potential values of each
PE2 = P21 + PE23 vertex with corresponding corresponding charge. Finally the
In the numerator of RHS of Eq.(2.21), each pair of charge 2 × of the sum of all these values gives net electric potential
1

comes twice, that is why we divided it by 2. Eq.(2.21) is useful energy of the system.
for symmetric charge arrangements. SOLUTION Electric Potential Energy of the System-
U = 12 [q1 (VA/B + VA/C ) + q2 (VB/A + VB/C ) + q3 (VC/A + VC/B )]

EXAMPLE 94. Find the potential energy of the system kq1 q2 kq1 q3 kq2 q3
⇒ U= + +
shown in Fig.2.11. Each edge is of length a. a a a
3 3
" #!
1X X
A q1 ⇒ U= qi × Vi/a
2 i=1 r=1

Similarly, for n charges, we have-


a a " n #!
n
1X X
⇒ U= qi × Vi/a
2 i=1 r=1
B a C
q2 q3 EXAMPLE 95. Determine the interaction energy of the
point charges of the following setup
Figure 2.11

APPROACH I In this approach, we find the work done


by external agent in bringing each charge, one by one, from
infinity to shown positions. The work is done against field
establised by charges already reached to their position.
SOLUTION Work done in bringing first charge q1 from
infinity to position A.-

W1 = 0
(in this case there was no electric field)
Now, this charge establishes an electric field in it’s surround-
ings. So, work done in bringing second charge q2 from infinity
to position B against the electric field established by the Figure 2.12
charge q1
APPROACH There are 4 C2 = 6 pairs of charges. Apply
kq1 q2 Eq.(2.20) for all pairs and simplify for U .
W2 =
a SOLUTION Net electric potential energy of the arrangement
Now, at position C, both q1 and q2 produce their electric is given by-
fields. Therefore, in bringing, a third charge q3 from infinity
to position C, we have to do work against the electric field U = UAB + UAC + UAD + UBC + UBD + UCD
established by q1 and q2 . This work done is given by- kq 2 kq 2 kq 2 kq 2 kq 2 kq 2
=− 2 + √ − 2 − 2 + √ − 2
a ( 2a)2 a a ( 2a)2 a
kq1 q3 kq2 q3
W3 = + kq 2
a a = −3 2 ( Here k = 1/4πε0 )
Net work done in forming the system ABC is stored in the a
form of the electric potential energy of the system. Some EXAMPLE 96. Eight point charges are placed at the corners
times we call it the self energy of the system. of a cube of edge a as shown in the figure. Find the work done
So, self energy of the system ABC, in disassembling this system of charges.
U = W1 + W2 + W3 APPROACH If Ui is the initial electric potential energy of
kq1 q2 kq1 q3 kq2 q3 the system and Uf is the final electric potential energy of the
=0+ + + system, then work done by external agent is defined as-
a a a
kq1 q2 kq1 q3 kq2 q3
⇒ U=
a
+
a
+
a W = change in potential energy = Uf − Ui (1)
196 CHAPTER 2. ELECTRIC POTENTIAL
 
+q −q 1 (2q)(8q) (2q)(q) (q)(8q)
U= + +
4π0 9 x 9−x
−q +q q2

8 1 4

= + + .
2π0 9 x 9−x

−q +q The value of x for which potential energy is minimum is given


by dU/dx = 0 i.e.,
+q −q dU q2

1 4

= − 2+ = 0.
Figure 2.13 dx 2π0 x (9 − x)2

On simplifying, we get- x = 3, x = −9. The position x = −9,


is not acceptable, so the acceptable value of x is x = 3.
Disassembling the system means putting the charges at Now, the electric field at the position of q due to the other
infinite separation. So, final electrostatic potential energy of two charges is
the system Uf = U∞ = 0
Now, calculate the initial potential energy ((Ui )) of the system  
E~ = 1 2q

8q
ı̂
by applying Eq.(2.19) and substitute these values in Eq.(??) 4π0 x2 (9 − x)2
and then solve for W .  
q 2 8 −

SOLUTION In the given system, there are 8 C2 = 28 pairs = − ı̂ = 0 .
of charges. The charge pairs on cube edges (twelve edges 4π0 (3)2 (9 − 3)2
of length a each) have unlike charges. The charge pairs
on face-diagonals (six √faces, two diagonals per face, twelve Note: Since, the force on the charge q is given by F~ =
~ = −dU/dxı̂. So, corresponding to minimum potential en-
qE
face-diagonal of length 2a each) have like charges.
The charge pairs ergy of the system, the charge q will be in the state of a stable
√ on main-diagonals (four main-diagonal each equilibrium.
with length 3a ) have unlike charges. Thus, the initial
potential energy of the system is
EXAMPLE 98. Two fixed, equal, positive charges, each of
magnitude q = 5 × 10−5 C are located at points A and B sepa-
q2 q2 q2 5.824 q 2
 
1 rated by a distance 6 m. An equal and opposite charge moves
Ui = −12 + 12 √ − 4 √ =−
4π0 a 2a 3a 4π0 a towards them along the line COD, the perpendicular bisector of
the line AB. The moving charge, when reaches the point C at a
On substituting these values of Ui and Uf in Eq. 1, we get- distance of 4 m from O, has a kinetic energy of 4 J. Calculate
5.824 q 2 the distance of the farthest point D which the negative charge
W = 0 − Ui =
4π0 a will reach before returning towards C.

EXAMPLE 97. Three point charges q, 2q and 8q are to be A


placed on a 9 cm/ long straight line. Find the positions where • +q
the charges should be placed such that the potential energy of −q
this system is minimum. In this situation, what is the elec- D • •C
tric field at the position of the charge q due to the other two O
charges?
• +q
B
APPROACH To obtain the minimum potential energy of the
system, the charges, of larger magnitude, should be placed at Figure 2.15
the extreme positions. If the charge q is at distance x from 2q,
then corresponding to minimum potential energy, dU/dx = 0 APPROACH At the farthest point, negative charge changes
and d2 U/dx2 = −ve. By applying this concept, determine the its direction of motion, so, the velocity of negative charge
value of x and then calculate the value of electric field at the at this point will be zero. Since, there are only electrostatic
position of q. forces on the charged particle, therefore, by applying the law
SOLUTION In Fig. 2.14, the total potential energy of the of conservation of mechanical energy, we can find the position
given system is of farthest point.
SOLUTION If D is the farthest point at a distance rOD
2q q 8q from O, then the velocity of negative charge at D is vd = 0.
• • • From Fig.2.16, we have-
x 9−x
p
rAC = rBC = 32 + 42 = 5 m
Figure 2.14 The electrostatic potential energies of the system for negative
2.3. ELECTRIC POTENTIAL 197

A Work done by external force in rotating electric dipole anti-


• +q clock wise by small angle dθ against electric force
3m −q
•C
dW = τ dθ
D •
rOD O 4m θ2 θ2
3m
Z Z
Wext = τ dθ = pEsinθ dθ = −pE [cosθ2 − cosθ1 ]
• +q θ1 θ1
B
In this case the work done by electric field Wel =
Figure 2.16 pE [cosθ2 − cosθ1 ]
If θ1 = 0, θ2 = θ, then

charge placed at C and D- Wext = −pE [cosθ − 1]

q2 Wel = pE [cosθ − 1]
 
1 1
UC = − +
4π0 rAC rBC The work done by external force is stored in the form of electro-
2
9
5 × 10−5 static potential energy of the system. If we take zero potential

2 9 × 10
=− = −9 J energy at θ1 = 90◦ , then for θ2 = θ, we have
5
Change in potential energy, ∆U = Wext = −pEcosθ


q2 i.e., ∆U = −−→
 
1 1 45 p .E
UD = − + =− J
4π0 rAD rBD rAD When θ = 0◦ , the dipole moment is in the direction of the
The kinetic energy of the negative charge at C is KC = 4 J field and the dipole is in stable equilibrium. If it is slightly
and at D is KD = 21 mvd2 = 0. displaced, it performs oscillations.
By conservation of mechanical energy, we have- When θ = 180◦ , the dipole moment is opposite to the direction
of the field and the dipole is in unstable equilibrium.
KC + UC = KD + UD
On substituting the values kf KC , UC , KD and UD in above 2.3 Electric Potential
expression and simplifying for rAD , we get
A point charge q is the source of an electric field E~ that exists
rAD = 45/5 = 9 m in the surrounding space. The electric field affects any charge
q0 introduced into that space through a force F~ on q0 given by
Now, applying Pythagoras theorem in triangle AOD, we get- F~ = q0 E.
~ We have seen in Section 2.1.2 that the introduction
of a charge q0 at a distance r from q gives rise to the potential

energy U (r) of Eq.2.15. If we write U (r) = q0 V (r), we can
q p
rOD = rAD 2 − r2 92 − 32 = 72 = 8.48 m.
AO =
make a statement analogous to the statement about the electric
field: The charge q is the source of an electric potential (or just
2.2.1 Potential Energy of a Dipole in a Uni- potential) V (r), within which any charge q0 a distance r from
form Electric Field q will have potential energy U (r) = q0 V (r). Strictly speaking
q0 should be a small test charge, so that its presence does not
Let us consider an electric dipole in a uniform external electric
disturb charge q or any other charge distribution that gives
field Fig.2.17.
rise to the electric potential. This description tells us how to
define the electric potential due to a charge distribution:

U (~r)
V (~r) ≡ lim (2.22)
q0 →0 q0

where U (~r) is the potential energy of the test charge q0 in


the presence of the charge distribution. The potential V (~r)
is independent of the test charge q0 , just as the electric field,
defined by E~ = F~ /q0 , is independent of the test charge.

Figure 2.17: (a) An electric dipole has an electric dipole moment 2.3.1 The Electric Potential of a Point
p
~ in an external uniform electric field E.~ The angle between p ~
~
Charge
and E is θ. The line connecting the two charges represents their
rigid connection and their center of mass is assumed to be midway Let’s calculate the electric potential of the simplest possible
between them. (b) Representing the electric dipole by a vector p ~ in system: one point charge. Consider a test charge q0 separated
~ and showing the direction of the torque
the external electric field E by a distance r from a single point charge q. As Eq. (24-5)
~τ into the page by the symbol ⊗ shows, the potential energy of the system is U (r) = q0 q/4πε0 r,
198 CHAPTER 2. ELECTRIC POTENTIAL

and hence U (r)/q0 = q/4πε0 r. We have found the electric potential energy decreases in that way. The charge speeds up
potential of a point charge q at a distance r from the charge: as it moves to lower potentials.
If, as before, we choose the potential to be zero at infinity,
q then Eq. (2.25) implies:
V (r) = (2.23)
4πε0 r Work done by an external force in bringing a unit positive
charge from infinity to a point = electrostatic potential (V )
In Eq.2.23, we have assumed that zero potential energy is at at that point.
infinity and, as a consequence, we have taken the electric po-
In other words, the electrostatic potential (V ) at any point in
tential due to a charge q to be zero at infinity. To emphasize
a region with electrostatic field is the work done, by an external
this point, we might say that Eq.2.23 is the potential of a single
agent, in bringing a unit positive charge (without acceleration)
charge with respect to infinity.
from infinity to that point.
As for potential energy, the only physically relevant feature
To obtain the work done per unit test charge, we should take
of the potential is how it differs between two points. The elec-
a test charge q0 , obtain the work done W in bringing it from
tric potential difference due to the charge q between the points
infinity to the point and determine the ratio W/q0 . Also, the
A and B at locations ~rA and ~rB is given by (Fig. 24-3):
external force at every point of the path is to be equal and op-
UB − UA q

1 1
 posite to the electrostatic force on the test charge at that point.
∆V = VB − VA = = − (2.24)
q0 4πε0 rB rA
In mathematical form
Here, we have abbreviated V as a function of rA , or V (rA ), Z r

→ − 1 q
as VA , and so forth. We can obtain another formulation of V = − E .d →r =
∞ 4πε0 r
the electric potential difference by using Eqs.2.13 and 2.24 and
substituting F~ = q0 E:
~ It is a scalar quantity and it’s SI unit is "joule/coulomb" or
Z r "volt".
UB − UA
∆V = =− ~ · d~r
E (2.25) Note that the direction of electric field is always from high
q0 rA voltage to low voltage. It is always much easier to work with
−→
Here, the electric potential difference is expressed as a path- V (scalar) than with vector E (vector).
independent integral over an electric field. The electric field in EXAMPLE 99. A conducting bubble of radius a and thick-
Eq. 2.25 is not necessarily the electric field of a point charge. ness t(t  a) has potential V . Now the bubble collapses into a
Equation 2.13 is the potential energy change when a test charge droplet. Find the potential of the droplet.
q0 moves from point A to point B in the field of any charge
distribution. Thus Eq. 2.25 is a general expression for the SOLUTION The volume of liquid in a bubble of radius a
electric potential difference between two points. Any charge and thickness t is given by
distribution produces an electric field, and an electric potential  3 !
is associated with any charge distribution. 4 3 4 3 4 3 t
Vb = π(a + t) − πa = πa 1+ −1
Recall (Section 2.1.2) that the change in the potential energy 3 3 3 a
of a system is equal to the negative of the work done by the 4 3

t

system in moving an object from point A to point B. Equiva- = πa 1 + 3 − 1 (since t  a)
3 a
lently, UB − UA is the work done by an external agent to move
the object. By the parallel relation between force and field we ≈ 4πa2 t
can then interpret Eq. 2.25 to mean that The volume of liquid in the droplet of radius r formed by
The electrical potential difference VB −VA is the work collapsing the bubble, is-
per unit charge that must be done by an external agent
to move a test charge from point A to point B without 4 3
Vd = πr
changing its kinetic energy. 3
If there is no external agent, then a change in potential, By conservation of volume, we have
which corresponds to a change in potential energy of the test
charge, must be accompanied by a corresponding change in the Vb = Vd
kinetic energy of the test charge. 4 3
πr = 4πa2 t
With knowledge of the electric potential V (~r) due to a charge 3
distribution and the magnitude of a test charge q0 we immedi- ⇒ r = 3a2 t
1/3
ately have the potential energy U (~r) of the system composed
of the distribution and the test charge q0 placed at the position The potential on a spherical shell of radius a and charge q is-
~r :
U (~r) = q0 V (~r) (2.26) V = q/(4π0 a)
In the absence of other forces, this equation tells us that a Therefore, the charge on bubble having potential V is-
positive test charge q0 in the presence of an electric potential
will move toward lower values of the potential because the q = 4π0 aV
2.4. POTENTIAL GRADIENT 199

So, by conservation of charge, the charge on the droplet will SOLUTION (C) The uniform electric field in the region is
also be q = 4π0 aV . ~ = Eı̂. Let d~rx = dxı̂ and d~ry = dy̂ be the small displace-
E
The electric potential on the droplet is given by ment vectors along x and y-axes.
 a 1/3 The potentials at the point B and C relative to the point A
q 4π0 aV
Vd = = 1/3
= V are given by
4π0 r (3a2 t) 3t
Z Z 1
EXAMPLE 100. Two equal point charges are fixed at x = VB = VA − ~ · d~rx = VA −
E E dx = VA − Ex
−a and x = +a on the x-axis. Another point charge Q is placed 0
Z  
at the origin. Find the change in electrical potential energy of VC = VA − ~ · d~ry = VA
E ~ ⊥ d~ry
∵E
Q, when it is displaced by a small distance x along the x-axis.
~ but does not change
Note that the potential decreases along E
~
in a direction perpendicular to E.
q Q q
• • •
(−a, 0) O O′ (a, 0)
x 2.4 Potential Gradient
Figure 2.18 We can quantify the relation between potential and field by
considering the potential difference dV between two nearby
SOLUTION Let O be the origin and O be a point to the points. Suppose they’re separated by a small displacement dr
0

right of O at a distance x. in the 3-D space. Then, we have-


The potentials at O and O due to charges at (−a, 0) and (a, 0)
0
~ r .d~r
dV = −E (2.27)
are
q q q
VO = + = ~ and d~r are always directed in the same
4π0 a 4π0 a 2π0 a In above Eq.(2.27), E
q q direction, therefore, we can also write above equation as-
VO0 = +
4π0 (a + x) 4π0 (a − x)
q

a
 dV = −Er dr
=
2π0 a2 − x2 On dividing both sides of above equation by dr, we get-
The potential energy of charge Q placed in a potential V is
QV . Thus, the change in potential energy of charge Q when it dV
or Er = − (2.28)
is displaced by a small distance x is dr
 
qQ a 1 Eq.(2.28) gives an the expression for radial component of elec-
∆U = QVO0 − QVO = −
2π0 a2 − x2 a tric field.
qQ x 2
qQ x 2
As an example, we have shown that the potential at a radial
= ≈ . ( for x  a)
2 2
2π0 a (a − x ) 2π0 a3 distance r from a point charge q is

EXAMPLE 101. A uniform electric field pointing in positive 1 q


V =
x direction exists in a region. Let A be the origin, B be the 4π0 r
point on the x-axis at x = +1 cm, and C be the point on the  
y-axis at y = +1 cm. Then the potentials at the points A, B d 1 q
so, Er = − 1 q
= 4π 0 r
2
and C satisfy dr 4π0 r
(A) VA < VB (B) VA > VB (C) VA < VC (D) VA > VC which is in agreement with Coulomb’s law.
Note that, if position vector r = xî + y ĵ + z k̂, then
d~r = dxî + dy ĵ + dz k̂
y(cm)
Similarly, if points A and B are separated by small dis-
~
E placement dx in the x-direction. Then, we have-
1 C
dV = −Ex dx (2.29)
B where we handled the dot product by considering only the
A x(cm) ~ along the displacement.
1 component of E
On dividing both sides of Eq.2.69 by dx, we get-
Figure 2.19
dV
Rf Ex = − (2.30)
APPROACH Apply the relation, VB − VA = − ~ d~r
E. dx
i
alonf x and y directions. here, Ex is the electric-field component in the x-direction.
200 CHAPTER 2. ELECTRIC POTENTIAL

Similarly, we can write the expressions for y - and z- APPROACH Electric field in terms of electric potential is
components as given below- defined as-  
~ = − ∂V î + ∂V ĵ + ∂V k̂
E (2.35)
dV dV ∂x ∂y ∂z
Ey = − , Ez = − (2.31)
dy dz SOLUTION Given that, V = 3x2 y + y 2 + yz therefore-
When a function depends on more than one variable, as the ∂V ∂ ∂ d
3x2 y + y 2 + yz = 3x2 y = 3y x2 = 6xy
  
potential generally does, we write derivatives with the par- =
∂x ∂x ∂x dx
tial derivative symbol ∂ instead of d to indicate the rate ∂V ∂
3x2 y + y 2 + yz = 3x2 + 2y + z

of change with respect to only one variable. Thus we have =
∂y ∂y
Ex = −∂V /∂x, Ey = −∂V /∂y, and Ez = −∂V /∂z. ∂V ∂ ∂
3x2 y + y 2 + yz =

Now, the entire electric-field vector can be written as- = yz = y
∂z ∂z ∂z
~ = Ex î + Ey ĵ + Ez k̂
E (2.32) Substituting these values in Eq.(2.35), we get-
~ = 6xy î + (3x2 + 2y + z)ĵ + y k̂
E
On substituting the values of Ex , Ey and Ez in above equation, EXAMPLE 103. V = − 100 then calculate intensity of elec-
r
we get- tric field at (3, 4).
  APPROACH Since, electric potential is given  in terms of
∂V ∂V ∂V
~ =−
E î + ĵ + k̂ (2.33) position r in 2-D, therefore, we use Eq.(2.28) E = − dV
to
∂x ∂y ∂z p dr
get an expression for E. Here, ~r = xî + y ĵ and r = x + y 2
2
100
Equation (2.33) confirms that the electric field is strong SOLUTION E = − dV d
dr = − dr − r = 100
√ r2
where the potential changes rapidly. The minus sign here says Here, r = 32 + 42 = 5, therefore
100
that if we move in the direction of increasing potential, then E = (5)2 = 4 V /m
we’re moving against the electric field. EXAMPLE 104. V = 3x + 4y + 5z. Find electric field in-
We can also write Eq.(2.33) as- tensity.
  APPROACH Since, electric potential is given in terms of
~ =− ∂ ∂ ∂ −→
E î + ĵ + k̂ V = − ∇ V (2.34) cartesian coordinates, therefore, apply Eq. (2.35).
∂x ∂y ∂z SOLUTION Given that- V = 3x + 4y + 5z, therefore-
∂x = 3, ∂y = 4, and ∂z = 5
∂V ∂V ∂V
−→ −→
or E = − ∇ V = − grad V Substituting these values in Eq. (2.35), we get-

→ ∂ ∂ ∂
Here, ∇ = î + ĵ + k̂ is called the gradient operator.  
∂x ∂y ∂z
E~ = − ∂V î + ∂V ĵ + ∂V k̂
The maximum rate of change of potential at right angles ∂x ∂y ∂z
to an equipotential surface in an electric field is defined as
 
= − 3î + 4ĵ + 5k̂ V /m
potential gradient.

→ Magnitude of electric field is given by-
The quantity, ∇ V is called the potential gradient.
At any point in an electric field, the component of electric ~ = −(3î + 4ĵ + 5k̂)
E
intensity in any direction is equal to the negative of the √
So, |E| ~ = 5 2 V /m
potential gradient in that direction.
Clearly, the unit of potential gradient is volts per meter, EXAMPLE 105. V = x2 y + y 2 z + z 2 x. Find electric field
while electric intensity or force per unit charge is expressed in intensity at (1, 2, 3).
newtons per coulomb. However,
APPROACH Again, electric potential is given in terms of
volt joule/coul N·m N cartesian coordinates, therefore, apply Eq. (2.35).
= = = SOLUTION Given that- V = x2 y + y 2 z + z 2 x, therefore-
m m coul · m coul
∂x = 2xy + z , ∂y = x + 2zy, and ∂z = y + 2zx
∂V 2 ∂V 2 ∂V 2

so that the volt/meter and the newton/coulomb are equivalent Substituting these values in Eq. (2.35), we get-
units.  
~ ∂V ∂V ∂V
E=− î + ĵ + k̂
Note: Potential is a scalar quantity but the gradient of ∂x ∂y ∂z
potential is a vector quantity In cartesian co-ordinates, the
 
= − (2xy + z 2 )î + (x2 + 2zy)ĵ + (y 2 + 2zx)k̂ V /m
potential gradient is defined by Eq.(2.34).
As electric potential is a scalar, it’s often easier to calculate Therefore, at point (1, 2, 3)
the potential and then use Equation (2.33) to get the field.  
~ = − (2(1)(2) + 32 )î + (12 + 2(3)(2))ĵ + (32 + 2(3)(1))k̂ V /m
E
EXAMPLE 102. If V = 3x2 y + y 2 + yz, then find an expres-  
sion for electric field E~ = − 13 î + 13 ĵ + 10 k̂ V /m
2.4. POTENTIAL GRADIENT 201
p √
~
and |E|= 132 + 132 + 102 = 438 V /m charge is moved from (−a, 0, 0) to (0, a, 0) is
(A) positive
(B) negative
2.4.1 Electric Potential For a System of (C) zero
Charges (D) depends on the path connecting the initial and final
For a total of N point charges, the potential V at any point P positions.
can be derived from the principle of superposition. APPROACH Work done by electric field, when a point
Recall that potential due to q1 at point P : charge Q moves from potential VA to potential VB , is given
by-
1 q1
V1 = W = −Q(VB − VB )
4πε0 r1
SOLUTION (C) The charge configuration is shown √ in the
Therefore, the total potential at point P due to all N point figure 2.21. The point A(−a, 0, 0) is at a distance rA = 5a/2
from both the√ charges. Also, the point B(0, a, 0) is at a dis-
tance rB = 5a/2 from both the charges.

y
(0, a, 0) • B
−q•
(0, 0, − a2 )
(−a, 0, 0)
• x
A
q
• (0, 0, a )
2
z
Figure 2.20 Figure 2.21

charges: The potentials at the point A and B are given by


V = V1 + V2 + V3 + . . . VN (principle of superposition)
1 q 1 q
  VA = − =0
1 q1 q2 q3 qN 4π0 rA 4π0 rA
V = + + + ... +
4πε0 r1 r2 r3 rN 1 q 1 q
VB = − =0
4π0 rB 4π0 rB
N
X qi
N
1 X qi Since VA = VB , the work done in taking a point charge from
⇒ V =k = (2.36) A to B is W = Q(VB − VA ) = 0.
ri 4πε0 i ri
i Note: Since, the electrostatic forces are conservative therefore,
the work done by them do not depend on the path.
When we have a continuous distribution of charge along a line,
over a surface, or through a volume, we divide the charge into EXAMPLE 107. A charge +q is fixed at each of the
elements dq, and the sum in Eq. (2.36) becomes an integral: points x = x0 , x = 3x0 , x = 5x0 , · · · , ∞ on the x-axis and
a charge −q is fixed at each of the points x = 2x0 , x = 4x0 ,
Integral over charge distribution x = 6x0 , · · · , ∞. Here x0 is a positive constant. Take the
electric potential at a point due to a charge Q at a distance r

4pP0 L r Distance from charge element


1 dq Charge element
V = from it to be Q/(4π0 r) . Then, the potential at the origin due
to the above system of charges is
Electric constant to where potential is measured (A) zero (B) 8π0 xq0 ln 2
(2.37) q ln 2
(C) infinite (D) 4π 0 x0

Note: Remember that there doesn’t have to be a charge at a APPROACH Due to several charge, the electric potential
given point for a potential V to exist at that point. (In the at any point is given by Eq. 2.36, so apply it and solve for V
same way, an electric field can exist at a given point even if at origin-
there’s no charge there to respond to it.)
SOLUTION (D) Using Eq.2.36, the electric potential at the
origin due to the given system of charges is
EXAMPLE 106. Positive and negative point charges of equal
magnitude are kept at 0, 0, a2 and 0, 0, − a2 , respectively.
  
q 1 1 1 1 q
The work done by the electric field when another positive point V = − + − + . . . = ln 2
4π0 x0 1 2 3 4 4π0 x0
202 CHAPTER 2. ELECTRIC POTENTIAL

EXAMPLE 108. Six point charges are kept at the vertices the particle. Since, electric force is conservative, therefore, we
of a regular hexagon of side L and centre O, as shown in the can always apply the law of conservation of mechanical energy.
figure. Given that K = 4π 1 q
0 L
2 , which of the following state- If electric potential at any point on the x axis is V , then the
ment(s) is (are) correct? position of potential barrier i.e., maximum potential, can be
obtained by applying the relation ∂V ∂x = 0.
L Once you get the value of x0 , put x = x0 in the expression for
F E
+q −q V and find net potential energy of the system for this position
P of moving charged particle. By conservation of energy, this
energy will be equal to net initial kinetic energy of the particle
A S T D for x = ∞.
+2q O −2q To get the kinetic energy of the particle at origin, again apply
the conservation of mechanical energy principle and solve for
R KE.
B C
+q −q SOLUTION Given q = 1µC = 10−6 C, Q = 8µC = 8 ×
10−6 C, q0 = 0.1µC = 10−7 C, m = 6 × 10−4 kg and a =
3/2 m. Consider a point P at a distance x from the ori-
p
Figure 2.22
gin.
(A) The electric field at O is 6K along OD. The potential at P due to given charge distribution is
(B) The potential at O is zero.  
(C) The potential at all points on the line PR is same. 1 2Q 2q
V (x) = √ −√
(D) The potential at all points on the line ST is same. 4π0 x2 + 9a2 x2 + a2

APPROACH Apply the principle of superposition of electric The potential varies with x and attains its maximum at x0
field as given in last chapter and principle of super position of (Fig. ?? b ). For maximum value of V (x), we have-
electric potentials as given by Eq. 2.36.
" #
dV (x) 2x Q q
SOLUTION (A) (B) (C) The electric field at O due to the =− −
charges at A and D is 4K along OD, due to the charges at B
dx 4π0 (x2 + 9a2 )3/2 (x2 + a2 )
3/2
(2.38)
and E is 2K along OE and due to the charges at C and F is = 0.
2K along OC. For the given geometry, resultant of these fields
is 6K along OD. On substituting, p Q = 8q inpequation (2.38) and solving for
The potential at O is x0 , we get x0 = 5/3a = 5/2 m. The potential at x0 is
V0 = V (x0 ) = 2.7 × 104 V.
X 1 qi 1 X Now, by conservation of mechanical energy, we can write-
VO = = qi = 0
4π0 L 4π0 L 1 p
mv02 = q0 V0 , =⇒ v0 = 2q0 V0 /m = 3 m/s
For any point on PR, we have pairs of equal and opposite 2
charges at the same distance making the potential at any point The potential energy of the system for q0 at the origin is
on PR zero. It may be seen that potential at points on OS is  
1 2Qq0 2qq0
positive and that on OT is negative. We encourage you to U= − = 2.4 × 10−3 J
show that the potential on ST (at a distance x from O, taken 4π 0 3a a
positive towards the right) is If K is the kinetic energy of q0 at the origin, then the conser-
 vation of mechanical energy, 12 mv02 = K + U , gives
q 2 2
V (x) = √ −√ 1
4π0 2 2
L + x + xL L + x2 − xL
2
K = mv02 − U = 3 × 10−4 J
4x
 2
− 2 EXAMPLE 110. Two fixed charges −2Q and Q are located
L − x2
at the points with coordinates (−3a, 0) and (+3a, 0) respec-
EXAMPLE 109. Four point charges +8µC, p −1µC,p−1µC, tively in the x-y plane.
p +8µC arc pfixed at thc points − 27/2m, − 3/2m,
and
+ 3/2mand + 27/2m respectively on thc y-axis. A parti-
(a) Show that all points in the x − y plane where the electric
cle of mass 6 × 10−4 kg and charge +0.1µC moves along the
potential due to the two charges is zero, lie on a circle.
x direction. Its speed at x = +∞ is v0 . Find the least value
Find its radius and the location of its centre.
of v0 for which the particle will cross the origin. Also find the
kinetic energy of  Assume that space (b) Give the expression V (x) at a general point on the x-axis
 the particle at the origin.
1 9 2 2 and sketch the function V (x) on the whole x-axis.
is gravity free. = 9 × 10 Nm /C
4πε0
(c) If a particle of charge +q starts from rest at the centre of
APPROACH To cross the origin, the initial kinetic energy the circle, show by a short quantitative argument that the
of the particle should be enough to cross the potential barrier particle eventually crosses the circle. Find its speed when
applied by given four charges for any position x = x0 (say) of it does so.
2.4. POTENTIAL GRADIENT 203

y y, V

3a • +Q

−2Q Q
x
a • −q −3a a 3a 5a 9a
P q0
• • x
v0 m
−a • −q
Figure 2.24

−3a • +Q
(a) Eq. (2.39) as-
V   
 Q
4π0
2
+ 1
3a−x  , if x ≤ −3a;
 3a+x



V (x) = Q
4π0
−2
+ 1
3a−x  , if − 3a < x ≤ 3a;
V0   x+3a
Q −2 1
if x > 3a.

+ ,


4π0 x+3a x−3a
x (2.41)
x0
From Eq.(2.41), it is clear that- V → −∞ as x → −3a and
(b)
V → ∞ as x → 3a. The potential is zero at x = a and at
Figure 2.23 x = 9a (see Fig.2.24).
The potential at the centre of circle (x = 5a) is
 
Q −2 1 Q
APPROACH For part (a), find the electric potential at any V = + =
4π0 8a 2a 16π0 a
point P (x, y) due to the charge −2Q located at (−3a, 0) and
the charge Q located at (3a, 0). For zero potential points, which has a positive value. The potential at the circumference
equate it to zero and find a relation between the coordinates of the circle is zero. (c) A positive charge moves from a higher
x and y. The relation between x and y will give you the equa- potential to a lower potential. By conservation of energy,
tion of path of zero potential points. For part (b), just find the decrease in the potential energy is equal to increase in kinetic
electric potential at (x, 0, 0) due to both the charges and apply energy i.e.,
the principle of superposition. To get speed of particle at the 1 q
mv 2
= qQ
, which gives v = Qq
center of circle, apply the principle of conservation of energy 2 16π0 a 8π0 ma

SOLUTION (a) The net electric potential at the point


P (x, y) due to the charge −2Q located at (−3a, 0) and the
charge Q located at (3a, 0) is given by 2.4.2 Check Point 2
" # 1. •• In Figure 2.25, two points A and B are located within
Q 1 2 a region in which there is an electric field. The potential
V = −p
difference ∆V = VB − VA is-
p
4π0 (x − 3a)2 + y 2 (x + 3a)2 + y 2

For zero potential points: V = 0


1 2
⇒ p =p
2
(x − 3a) + y 2 (x + 3a)2 + y 2
⇒ (x − 5a) + y = (4a)2 .
2 2

which is an equation of circle of radius 4a and centre (5a, 0).


(b) The potential on the x axis is given by
 
Q −2 1
V (x) = + (2.39)
4π0 |x + 3a| |x − 3a|

By definition of modulus function, we have-

if x ≥ 0;

x,
|x|= (2.40)
−x if x < 0

Using definition of modulus function (Eq.(2.40)), we can write Figure 2.25


204 CHAPTER 2. ELECTRIC POTENTIAL

(A) +ve (B) −ve (C) 0 (D) insufficient data 1 cm


0V 50 V 100 V
2. •• As per the diagram a point charge +q is placed at 2
1 cm
the origin O. Work done in taking another point charge
−Q from the point A [ coordinates (0, a)] to another
point B [coordinates (a, 0) ] along the straight
 path
 √AB is- 0V
(A) zero (B) 4πεqQ 1
2 · 2a 3
 √  0a 
50 V

−qQ 1
(C) 4πε 0 a
2 · 2a (D) 4πεqQ 1
0 a
2 · √a2 1
100 V
y
Figure 2.28: Equipotential lines

A
8. •• The electric potential at a point (x, y, z) is given by
V = −x2 y − xz 3 + 4. The electric field atthat point is
~ = î2xy + ĵ x2 + y 2 + k̂ 3xz − y 2
(A) E
~ = îz 3 + ĵxyz + k̂z 2
(B) E
~ = î 2xy − z 3 + ĵxy 2 + k̂3z 2 x
(C) E

O x
~ = î 2xy + z 3 + ĵx2 + k̂3xz 2
(D) E

B
Figure 2.26 9. •• If potential (in volts) in a region is ex-
pressed as V (x, y, z) = 6xy − y + 2yz, the
√ electric field (in N/C) at point (1, 1, 0) is
3. •• If V = −5x + 3y + 15z then find magnitude of electric (A) −(2î + 3ĵ + k̂) (B) −(6î + 9ĵ + k̂)
field at point (x, y, z). (C) −(3î + 5ĵ + 3k̂) (D) −(6î + 5ĵ + 2k̂)
4. Fig. 2.27 is a graph of the electric potential in a region of 10. •• In a region, the potential is represented by V (x, y, z) =
space where E ~ is parallel to the x-axis. Draw a graph of 6x − 8xy − 8y + 6yz, where V is in volts and x, y, z
Ex versus x. are in metres. The electric force experienced by
a charge
√ of 2 coulomb situated at point (1, 1, 1) is
V (V) (A) 6 5 N (B) 30
√N
(C) 24 N (D) 4 35 N(2014)
10
11. •• The electric potential V at any point (x, y, z) (all in
metre) in space is given by V = 4x2 volt. Find the electric
0 x (cm)
2 4 6 8 field at the point (1 m, 0 m, 2 m).
12. •• A metal sphere with radius ra is supported on an
210
insulating stand at the center of a hollow, metal, spherical
Figure 2.27 shell with radius rb . There is charge +q on the inner
sphere and charge −q on the outer spherical shell. (a)
Calculate the potential V (r) for (i) r < ra ; (ii) ra < r < rb ;
5. •• In Fig.2.28, a 1 cm × 1 cm grid is superimposed on a (iii) r > rb . (Hint: The net potential is the sum of the
contour map of the potential. Estimate the strength and potentials due to the individual spheres.) Take V to be
direction of the electric field at points 1,2 , and 3 . Show zero when r is infinite. (b) Show that the potential of the
your results graphically by drawing the electric field vectors inner sphere with respect to the outer is
on the contour map.  
q 1 1
Vab = −
6. •• In a certain region of space, the electric potential is 4π0 ra rb
V (x, y, z) = Axy − Bx2 + Cy, where A, B, and C are posi- (c) Use Eq. E = − ∂V ∂r and the result from part (a) to show
tive constants. (a) Calculate the x−, y−, and z-components that the electric field at any point between the spheres has
of the electric field. (b) At which points is the electric field magnitude
equal to zero? Vab 1
E(r) =
(1/ra − 1/rb ) r2
7. •• In a certain region of space the electric potential is given
by V = +Ax2 y − Bxy 2 , where A = 5.00 V/m3 and B = (d) Use Eq. E = − ∂V ∂r and the result from part (a) to find
8.00 V/m3 . Calculate the magnitude and direction of the the electric field at a point outside the larger sphere at a
electric field at the point in the region that has coordinates distance r from the center, where r > rb . (e) Suppose the
x = 2.00 m, y = 0.400 m, and z = 0 charge on the outer sphere is not −q but a negative charge
2.4. POTENTIAL GRADIENT 205

of different magnitude, say −Q. Show that the answers for


A (0, a)
parts (b) and (c) are the same as before but the answer for
part (d) is different.

~ is zero throughout a certain region of space, is the


13. •• If E
potential necessarily also zero in this region? Why or why
not? If not, what can be said about the potential? +q
O B (a, 0)
14. •• An alpha particle of energy 5MeV is scattered
through 180◦ by a fixed uranium nucleus. The Figure 2.30
distance of closest approach is of the order of
(A) 1 (B) 10−10 cm
(C) 10−12 cm (D) 10−15 cm There are three regions of different slope:

∆V /∆x = (20 V)/(0.020 m) = 1000 V/m
0 < x < 2 cm
+Ze Ex = −1000 V/m
P 
∆V /∆x = 0 V/m
O 2 < x < 4 cm
Ex = 0 V/m
d

∆V /∆x = (−20 V)/(0.040 m) = −500 V/m
Figure 2.29 4 < x < 8 cm
Ex = 500 V/m
The results are shown in Fig.2.31.
15. •• A dipole of dipole moment −→p is kept along an electric

→ −
→ −
→ Ex (V/m)
field E such that E and p are in the same direction.
Find the work done in rotating the dipole by an angle π. 1000
(A) W = 2Ep (B) W = − − 2Ep
(C) W = Ep (D) W = − − Ep 500
16. An electric dipole, made up of a positive and a negative
0 x (cm)
charge, each of magnitude 1 µC and placed at a distance 2 4 6 8
2 cm apart, is placed in an electric field 105 N/C. Compute
the maximum torque which the field can exert on the dipole, 2500 The value of Ex is the negative
of the slope of the potential graph.
and the work that must be done to turn the dipole from a
position θ = 0◦ to = 180◦ 21000
(A) 6 × 10−3 N.m and 4 × 106 joule
(B) 3 × 10−3 N.m and 4 × 109 joule Figure 2.31
(C) 4 × 10−3 N.m and 4 × 10+6 joule
(D) 2 × 10−3 N.m and 4 × 103 joule
5. APPROACH The electric field is perpendicular to the
equipotential lines, points "downhill," and depends on the
Answer Key and Solutions slope of the potential hill.
1. (B). When moving straight from A to B, E~ and d~s both The potential is highest on the bottom and the right. An
point toward the right. Thus, the dot product E~ · d~s in elevation graph of the potential would look like the lower-
~
Equation ∆V = VB − VA = −E · d~s is positive and ∆V is right quarter of a bowl or a football stadium.
negative. SOLUTION Some distant but unseen source charges have
created an electric field and potential. We do not need to
2. (A) Work done is equal to zero because the potential of A
see the source charges to relate the field to the potential.
and B are the same = 4πε 1 q
0 a Because E ≈ −∆V /∆s, the electric field is stronger where
No work is done if a particle does not change its potential
the equipotential lines are closer together and weaker where
energy. i.e. initial potential energy = final potential energy.
they are farther apart. If Figure 2.28 were a topographic
map, you would interpret the closely spaced contour lines
at the bottom of the figure as a steep slope.
 

→ ∂V ∂V ∂V
3. E = − ĩ + j̃ + k̃
∂x ∂y√ ∂z Fig.2.32 shows how measurements of ∆s from the grid are
= −(−5î + 3ĵ + 15k̂) combined with values of ∆V to determine E. ~ Point 3 re-

→ √ √
⇒| E |= 25 + 9 + 15 = 49 = 7 unit quires an estimate of the spacing between the 0 V and the
100 V surfaces. Notice that we’re using the 0 V and 100 V
4. The electric field is the negative of the slope of the potential ~ at a point on the 50 V
equipotential surfaces to determine E
graph. equipotential.
206 CHAPTER 2. ELECTRIC POTENTIAL

9. (D): The electric field ~


h E and potential i V in a region are
~
related as E = − ∂x î + ∂y ĵ + ∂z k̂ Here, V (x, y, z) =
∂V ∂V ∂V

6xy − y + 2yz

∴E ~ = − ∂ (6xy − y + 2yz)î + ∂ (6xy − y + 2yz)ĵ
∂x ∂y
= −[(6y)î + (6x − 1 + 2z)ĵ + (2y)k̂]

At point (1, 1, 0),

~ = −[(6(1))î+(6(1)−1+2(0))ĵ+(2(1))k̂] = −(6î+5ĵ+2k̂)
E

10. (D): Here, V (x, y, z) = 6x − 8xy − 8y + 6yz The x, y and z


components of electric field are
Figure 2.32 ∂V ∂
Ex = − = − (6x − 8xy − 8y + 6yz)
∂x ∂x
Note: The directions of E ~ are found by drawing downhill = −(6 − 8y) = −6 + 8y
vectors perpendicular to the equipotentials. The distances ∂V ∂
Ey = − = − (6x − 8xy − 8y + 6yz)
between the equipotential surfaces are needed to determine ∂y ∂y
the field strengths. = −(−8x − 8 + 6z) = 8x + 8 − 6z
~ are given by- ∂V ∂
6. APPROACH Components of E, Ez = − = − (6x − 8xy − 8y + 6yz) = −6y
∂z ∂z
Ex = − ∂x , Ey = − ∂y and Ez = − ∂V
∂V ∂V
∂z
SOLUTION V = Axy − Bx2 + Cy ~ = Ex î + Ey ĵ + Ez k̂
E
(a) Ex = − ∂V
∂x = −Ay + 2Bx = (−6 + 8y)î + (8x + 8 − 6z)ĵ − 6y k̂
∂V At point (1, 1, 1)
Ey = − = −Ax − C
∂y
~ = (−6 + 8)î + (8 + 8 − 6)ĵ − 6k̂ = 2î + 10ĵ − 6k̂
E
∂V
Ez = =0
∂z ~ is
The magnitude of electric field E
(b) E = 0 requires that Ex = Ey = Ez = 0. Ez = 0 q
~ = Ex2 + Ey2 + Ez2 = (2)2 + (10)2 + (−6)2
p
everywhere. Ey = 0 at x = −C/A. And Ex is also E
equal to zero for this x, any value of z and y = 2Bx/A = √ √
= 140 = 2 35 NC−1
(2B/A)(−C/A) = −2BC/A2 . EVALUATE: V doesn’t de-
pend on z so Ez = 0 everywhere. Electric force experienced by the charge
√ √
7. APPROACH Components of E, ~ are given by- F = qE = 2C × 2 35 NC−1 = 4 35 N
Ex = − ∂V
∂x , E y = − ∂V
∂y and E z = − ∂V
∂z
SOLUTION says E ~ = 1 |q|2 r̂ is the electric field due to 11. The electric field is given by
4π0 r
a point charge q.
~ = − ∂V ı̂ − ∂V ̂ − ∂V k̂ = −8xı̂
 
E
(a) Ex = − ∂x = − ∂x
∂V ∂ √ 2
kQ
2 2
∂x ∂y ∂z
x +y +z

r3 .
kQx kQx
= (x2 +y 2 +z 2 )3/2
= ~ (1m,
Therefore, E 0m, 2m) = −8ı̂.
Similarly, Ey = r3 and Ez= kQz
kQy
r3 .  12. APPROACH For a solid metal sphere or for a spherical
(b) From part (a), E = r2 r + yrĵ
kQ xî z k̂ kQ
shell, V = kqr outside the sphere and V = R at all points
+ r = r 2 r̂
kq

~ is a vector and has components.


Note: V is a scalar. E inside the sphere, where R is the radius of the sphere. When
the electric field is radial, E = − ∂V
∂r
8. (D): The electric potential at a point, SOLUTION (a) (i) r < ra: This  region is inside both
spheres. V = ra − rb = kq ra − rb . (ii) ra < r < rb :
kq kq 1 1

V = −x2 y − xz 3 + 4 This region is outside the  innershell and inside the outer
  shell. V = r − rb = kq 1r − r1b . (iii) r > rb : This region
kq kq
~ = −∇V
The field E ~ =− ∂V
î + ∂V
ĵ + ∂V

∂x ∂y ∂z is outside both spheres and V = 0 since outside a sphere the
potential is the same as for a point charge. Therefore the
~ = î 2xy + z 3 + ĵx2 + k̂ 3xz 2 potential is the same as for two oppositely charged point
 
∴ E
2.4. POTENTIAL GRADIENT 207

charges at the same  location.  These potentials cancel. 15. (A) W = U = U2 − U1


(b) Va = 4π0 ra − rb
1 q q
and Vb = 0, so Vab = Now U2 = (Epcos) = Ep
 
4π0 q ra − rb .
1 1 1
U1 = (Epcos0◦ ) = Ep
 
(c) Between the spheres ra < r < rb and V = kq 1r − r1b . Therefore, W = 2Ep.
16. (D) The torque exerted by an electric field E on a dipole
 
∂V q ∂ 1 1 1 q
E=− =− − =+ 2 of moment p is given by
∂r 4π0 ∂r r rb 4π0 r
Vab 1
=  2 τ = pE sin θ
1
− 1 r
ra rb
where θ is the angle which the dipole is making with the
electric-field.
(d) From Eq. E = − ∂V ∂r , E = 0, since V is constant (zero) Corresponding to maximum torque, θ = 90◦ , therefore-
outside the spheres. (e) If the outer charge is different, then
outside the outer sphere the potential is no longer zero but
1 (q−Q)
τmax = pE
is V = 4π1 q
0 r
− 4π1 Q
0 r
= 4π r . All potentials inside
0
Here, p = q (2l) = 1 × 10−6 × 0.02 C/m and E = 105 N/C
the outer shell are just shifted by an amount V = 4π 1 Q
.
0 rb Therefore, τmax = 1 × 10−6 × 0.02 × 105 = 2 × 10−3 N.m
Therefore relative potentials within the shells are not
The work done in rotating the dipole from an angle θ = 0◦
affected. Thus (b) and (c) do not change. However, now
to 180◦ is given by-
that the potential does vary outside the spheres, there is Rθ
an electric field there: W = θo pE sinθ dθ = pE (cos θ0 − cos θ)
Here, θ0 = 0◦ and θ = 180◦
Therefore, W = pE (cos 0◦ − cos 180◦ ) = 2pE = 4 ×
 
∂V ∂ kq −kQ
E=− =− + 10−3 joule
∂r ∂r r r
 
kq Q k
= 2 1− = 2 (q − Q) 2.4.3 Electric Potential at a Point P(r, θ) Due
r q r
to an Electric Dipole
Note: In part (a) the potential is greater than zero for all
r < rb . Let an electric dipole AB, of length 2l, is placed along x-axis.
The center of the dipole is at origin O and charges −q and +q
13. Since, E = −∂V /∂r, therefore, E~ = 0 everywhere implies a
are situated at points A and B respectively (Fig.2.33). P (r, θ)
constant potential and not necessarily zero potential. is a point where electric potential due to this electric dipole is
required. If r >> l, then we can assume AP ||OP ||BP .
14. APPROACH If we assume the space as gravity free, then
In this case, 6 P AB = 6 P OB = θ.
the force acting on the alpha particle is electrostatic in na-
Now, draw perpendiculars OC and BD from points O and B
ture, which is conservative. So, we can apply the principle
on the lines AP and BP respectively. From Fig. 2.33, we have
of conservation of mechanical energy. According to this
principle,
Ki + Ui = Kf + Uf
Here letters have their usual meanings.
SOLUTION (C) Initially, kinetic energy of the alpha par-
ticle is Ki = 5MeV and its potential energy is Ui = 0 (be-
cause it is far away from the nucleus). The charge of the
uranium nucleus is Q = Ze = 92e and charge on the alpha
particle is q = 2e. Let O be the centre of the uranium nu-
cleus. The alpha particle starts moving towards O and is
scattered by an angle 180◦ at P .
The distance of the closest approach is OP = d. The ki-
netic energy of the alpha particle at P is Kf = 0 (since its
Figure 2.33
velocity is zero) and its potential energy is

Uf = 2Ze2 /(4π0 d) . AC = OD = l cos θ and CP ≈ OP , BP ≈ DP .


Therefore, AP = AC + CP ≈ AC + OP = l cos θ + r
Apply conservation of energy, Ki + Ui = Kf + Uf , to get or r2 ≈ r + l cos θ and BP ≈ DP = OP − OD
or r1 ≈ r − l cos θ
2
9 × 109 (2)(92) 1.6 × 10−19 Electric potential at point P (r, θ), due to charge −q at A is-

2Ze2
d= =
4π0 Ki (5 × 106 ) (1.6 × 10−19 ) kq −kq
= 5.3 × 10−14 m. V(−) = ≈ [∵ r2 ≈ r + l cos θ]
r2 r + l cos θ
208 CHAPTER 2. ELECTRIC POTENTIAL

And, the electric potential at point P (r, θ), due to charge +q For π/2 < θ < π, we have cos θ < 0. So-
at B is-
kq kq kp cos θ
V(+) = ≈ [∵ rl ≈ r − l cos θ] V ≈ <0
r1 r − l cos θ r2

Therefore, net electric potential at P due to both the charges Figure 2.35 shows equipotential surfaces for the dipole in Fig-
of the electric dipole, ure 2.34. Most of the equipotential surfaces look like flattened
spherical shells. Solid lines indicate positive electric poten-
tials, and dashed lines indicate negative electric potentials.
 
1 1
V = V− + V+ = kq − The solid black line in the center of the figure represents the
r − l cos θ r + l cos θ
  zero-potential surface, a flat plane. For a dipole, the electric
r + l cos θ − r + l cos θ kp cos θ potential is zero both at infinity and at any point on the plane
= kq = 2
r2 − l2 cos2 θ [r − l2 cos2 θ] shown in black. Notice that the sharpest change in the poten-
For r  l tial is between the two charges, near the plane.
kp cos θ
V ≈ Equipotential surfaces
r2
here angle θ is measured from the dipole moment p~ to the
position vector −

r that extends from the center of the dipole
to the point of interest. The sign of the electric potential
depends on the angle θ (Fig. 2.34). Special Cases −

+
(i) End on Position: For End on positions, θ = 0 or θ = π
and then cos θ = ±1, therefore-

kp cos θ kp
V ≈ 2
=± 2
r r
Figure 2.35: Equipotential surfaces for an electric dipole
(ii) Broadside on Position: For broadside on positions, θ =
π/2 and then cos θ = 01, therefore

kp cos θ 2.4.4 Calculation of Electric Field due to a


V ≈ =0
r2 Dipole from Potential Formula
(iii) For 0 < θ < π/2, we have cos θ > 0. So- There are two components of field one is in radial and one in
perpendicular direction
C B A
V<0 V=0 V>0

r
r r

Figure 2.36
u
− Radial component of electric field is given by-
+

p
∂V
Er = −
Figure 2.34 ∂r
transverse component of electric field-
kp cos θ 1 ∂V
V ≈ >0 Eθ = −
r2 r ∂θ
2.4. POTENTIAL GRADIENT 209

On substituting the values of V in above expressions, ve get- ring is given by-


1 dq
Z
2kp cos θ
Er = V=
r3 4π0 (a + x2 )1/2
2

1 −kp sin θ

kp sin θ Figure 2.37 shows√that the distance from each charge element
Eθ = − 2
= dq √to P is r = x2 + a2 . Therefore, on taking the factor
r r r3
1/ x2 + a2 outside the integral in above equation, we get-
These components are shown in Fig.2.36.
Net electric fields at point P is given by- 1 1
Z
V = dq
q 4π0 (a2 + x2 )1/2
2 2
E= (Er ) + (Eθ ) 1 Q
or V = p (2.42)
kp p 4πε0 (a2 + x2 )
E = 3 1 + 3 cos2 θ
r Note: When x is much larger than a, our expression for V
Angle of Field from Radial Direction is given by- becomes approximately V = Q/4πε0 x, which is the potential
at a distance x from a point charge Q. Very far from a charged
Eθ 1 ring, its electric potential looks like that of a point charge.
tan φ = = tan θ
Er 2
EXAMPLE 111. A circular ring of radius R with uniform
1 positive charge density λ per unit length is located in the y − z
tan φ = tan θ
2 plane with its centre at the origin O. A particle of √ mass m
and positive charge q is projected from the point P (R 3, 0, 0)
2.4.5 Electric Potential due to a Charged on the positive x-axis directly towards O, with an initial speed
Ring v. Find the smallest (non-zero) value of the speed v such that
the particle does not return to P .
In Fig.2.37, a ring of radius a is shown. A charge Q is
uniformly distributed over the circumference of of this ring.
y
P is a point on the axis of the ring at a distance x from the
center of the ring, where electric potential is to be calculated.
Notice that no vector considerations are necessary here
because electric potential is a scalar.
APPROACH Because the ring consists of a continuous v
distribution of charge rather than a set of discrete charges, • x
O P
we must use the integration technique to calculate electric
potential. z
We divide the ring into infinitesimal segments and use
Eq.(2.37) to find V . All parts of the ring (and therefore all
(a)
elements of the charge distribution) are at the same distance
from P . V
The electric potential at P due to the charge element dq of
1 dq 1 dq
the ring is given by dV = =
4πε0 r 4πε0 (a2 + x2 )1/2
• x
dq O P
(b)

Figure 2.38
a2 x2
a
APPROACH From Eq.(2.42), the electric potential at any
point P (x, 0, 0) on the axis of charged ring, is given by-
P
x 1 Q
Q x or V = p (i)
4π0 (a2 + x2 )

The potential decreases monotonically from center O(0, 0, 0)


to P (x, 0, 0) and it is maximum for x = 0, i.e., at the center O
Figure 2.37 of the ring. So, the particle cannot come back to P if it just
crosses O. Since, the force acting on the particle is only elec-
Hence, the electric potential at P due to the uniformly charged trostatic force, which is conservative in nature, therefore, the
210 CHAPTER 2. ELECTRIC POTENTIAL

total mechanical energy of the ring-particle system will remain here UA is the electric potential energy of the system when
conserved. Now, apply conservation of mechanical energy to the charged particle was at any point on ring 1 and UB is the
get the velocity of particle at point P . potential energy of the system for the position of the charged
SOLUTION The charge on the ring is Q = 2πRλ.√Therefore, particle on ring 2.
from Eq. (i), the electric potential at a point P (R 3, 0, 0)- SOLUTION (B) The potential at A due to the charge Q1 on
the ring 1 is given as:
1 Q
VP = (ii)
4πε0 2R Q1
VA1 = k
R
The electric potentials at the point O(0, 0, 0)-
The potential at A due to the charge Q2 on the ring 2 is
1 Q given as:
VO = , (iii)
4πε0 R Q2 Q2
VA2 = k √ =k √
2
R +R 2 R 2
Net √
mechanical energy of the particle-ring system at point
P (R 3, 0, 0)- Total potential at A is

1
EP = KP + Ue l = mv 2 + qVP (iv)
2
here, v is the speed of the particle at point P .
If the charged particle just crosses the center O of the ring,
then it’s velocity and hence it’s kinetic energy at O will be
zero. So, net mechanical energy of the particle ring system at
point O(0, 0, 0)-
EO = qVO (v)
On applying, the principle of conservation of mechanical energy
for points P and O, we get- Figure 2.39

EP = EO
 
1 Q2
Substituting the values from Eq.(v) and (iv) in above, we get- VA = VA1 + VA2 =k Q1 + √
R 2
1
mv 2 + qVP = qVO The potential energy of charge q at A is
2 (vi)
1
or mv 2 = q(VO − VP )
 
q Q2
2 UA = qV A = k Q1 + √
R 2
On substituting the values of VP and VO , from Eq. (ii) and
(iii) in Eq.(vi), we get Similarly, the potential energy of charge q at B is
The energy required by a particle of charge q to reach the  
q Q1
point O from the point P is q (VO − VP ). Thus, UB = qV B = k Q2 + √
R 2
1 1 qQ qλ
mv 2 = = The work done in moving a charge q from point A to B is:
2 4π0 2R 40
On simplifying, we get W = ∆U = UB − UA
 
q Q1 Q2
Q2 + √ − Q1 − √
p
v= qλ/(2ε0 m) =k
R 2 2

EXAMPLE 112. Two identical thin rings, each of radius R, 1 2−1
= (Q2 − Q1 ) √
are co-axially placed a distance R apart. If Q1 and Q2 are 4π0 R 2
respectively the charges uniformly spread on the two rings, the
work done in moving a charge q from the center of one ring 2.4.6 Electric Potential Due to a Charged
that of the other is: √
2−1
Disc at a Point on it’s Geometric Axis
(A) zero (B) q (Q1 − Q2 ) √2πε R
√ 1 +Q2
√ 0
A non-conducting disc of radius R has a uniform surface charge
(C) q 2 Q4πR (D) q (Q1 /Q2 ) √2πε2+1
0R density σ C/m2 . Let us calculate the potential at a point on the
APPROACH To find the work done in moving a charge q axis of the disc at a distance x from its centre. The symmetry
from the center of one ring to that of the other, apply relation- of the disc tells us that the appropriate choice of element is a
ring of radius x and thickness
√ dx. All points on this ring are at
W = ∆U = UB − UA the same distance z = x2 + r2 , from the point P. The charge
2.4. POTENTIAL GRADIENT 211

Both forces are conservative in nature, therefore, total me-


chanical energy of the earth-disc-particle system, will always
remain conserved.
To find the height H, from where the particle is released, we
R apply the principle of conservation of mechanical energy.
r 2 x2
r Electric potential due to disc of radius R at distance x from
it’s center is given by Eq. (2.43)-
P σ
x x V = [(R2 + x2 )1/2 − x] (i)
2ε0
In this problem, radius, R = a and x = H, therefore V at
height H-
dA 5 2pr dr σ
VP = [(a2 + H 2 )1/2 − H]
dr 2ε0
Considering the position of disc as reference level, the gravi-
Figure 2.40 tational potential energy of the earth-particle system for the
particle at height H is-

on the ring is dq = σdA = σ(2πrdr) and so the potential due [Ugrav ]P = mgH
to the ring is
1 dq 1 (σ2πrdr) The electric potential energy of the disc particle system for
dV = = √ the position of particle at height H-
4πε0 z 4πε0 r2 + x2
Since potential is scalar, the potential due to the whole disc is qσ
given by [Uel ]P = qVP = [(a2 + H 2 )1/2 − H]
Z R 2ε0
σ rdr
V= √ Since, the particle is released from rest, therefore it’s kinetic
2ε0 0 r 2 + x2 energy at height H-
σ σ
= [(r + x2 )1/2 ]R
2
0 = [(R2 + x2 )1/2 − x]
2ε0 2ε0
KP = 0
σ
or V = [(R2 + x2 )1/2 − x] (2.43) So, net mechanical energy of the disc-particle-earth system for
2ε0
the position of particle at height H, is given by-
Let us see this expression at large distance when x  R.
EP = [Ugrav ]i + [Uel ]i + Ki
1 Q qσ
V = = mgH + [(a2 + H 2 )1/2 − H] + 0 (iii)
4πε0 x 2ε0

where Q = πR2 σ is the total charge on the disc. = mgH + [(a2 + H 2 )1/2 − H]
2ε0
Thus, we conclude that at large distance, the potential due to
Electric potential at the center O of disc is-
the disc is the same as that of a point charge Q.
At the centre of the disc x = 0, therefore σa
VO =
V = 2εσ0 R 2ε0

EXAMPLE 113. A non-conducting disc of radius a and uni- Therefore, electrostatic potential energy of the disc-particle
form positive surface charge density σ is placed on the ground system for particle at O is
with its axis vertical. A particle of mass m and positive charge σa
[Uel ]O = qVO = q
q is dropped, along the axis of the disc from a height H with 2ε0
zero initial velocity. The particle has q/m = 40 g/σ
Since, disc surface is selected as reference level for gravitational
(a) Find the value of H if the particle just reaches the disc. potential energy, therefore at O-

(b) Sketch the potential energy of the particle as a function of [Ugrav ]O = 0


its height and find its equilibrium position. Since, the particle just reaches the disc, so its kinetic energy
at O will be zero.
(a) APPROACH There are two forces on the charged
KO = 0
particle:
So, net mechanical energy of the earth-disc-particle system for
position of particle at O, is given by-
1. Downward gravitational force
EO = [Ugrav ]f + [Uel ]O + [K]O
2. Upward electrostatic repulsive force due to electric field σa σa
produced by the disc. =q +0+0=q
2ε0 2ε0
212 CHAPTER 2. ELECTRIC POTENTIAL

Now, apply conservation of mechanical energy and solve for H SOLUTION Total potential energy (U ) of the particle at
SOLUTION By conservation of mechanical energy, we have point P is the sum of its gravitational and electrostatic po-
tential energies i.e.,
EP = EO (v) qσ hp 2 i
U = mgh + a + h2 − h
Substituting the values of EP and EO in Eq.(v), we get 2ε
h p 0 i
= mg 2 a2 + h2 − h

SOLUTION At equilibrium position, we have


 
dU 2H
= mg √ −1 =0
dH a2 + H 2
√ √
which gives Hmin = a/ 3 and Umin = 3mga. The Fig.
2.41 shows the variation of U with height H. Note that the
equilibrium is stable (i.e., d2 U/dhH 2 > 0 ).

2.4.7 Electric Potential Due to a Shell


(a) Suppose, charge Q is uniformly distributed over the surface of
U a conducting shell of radius R. We will calculate the electric
potential at a point-
2mga
(a) outside the shell; (r > R)
(b) on the surface of the shell (r = R)
(c) inside the shell (r < R).

3mga (a) At points outside a uniform spherical distribution, the
√ h electric field is
a/ 3 −
→ 1 Q
E = r̂
4πε0 r2
(b)
−→
since E is radially outward, therefore-
Figure 2.41
→−
− →
E .dr = Edr
qσ σa since V (∞) = 0 , therefore, we have
mgH + [(a2 + H 2 )1/2 − H] = q
2ε0 2ε0

→ →
Z
mgH = q (VO − VQ ) V(∞) − V(r) = − E · d− r
qσ h p i Z ∞
= a + H − a2 + H 2 Q
2ε0 0−V =− dr
qσ h p i r 4πε0 r2
⇒ H= a + H − a2 + H 2
2ε0 mg 1 Q
⇒ V = (2.44)
Substitute q/m = 4ε0 g/σ to get 4πε0 r
From Eq.(2.44), we see that the potential due to a uniformly
h p i
H = 2 a + H − a2 + H 2
charged shell is the same as that due to a point charge Q at
the center of the shell.
p
⇒ a2 + H 2 = a + H/2
(b) For a point on the surface of the shell, r = R, therefore
⇒ H = 4a/3
Eq.(2.44), we have-
(b) APPROACH Net force on the particle, is equal to -ve
of potential energy gradient, i.e., 1 Q
V = (2.45)
4π0 R
∂U
F =− (vi)
∂H (c) At points inside the shell, E = 0 . So, the work done in
bringing a unit positive charge from a point on the surface to
At equilibrium position, F = 0, so from Eq.(vi), we have- any point inside the shell is zero. Thus, the potential has a
∂U fixed value at all points within the spherical shell and is equal
=0 to the potential at the surface.
∂H
Therefore, the potential energy attains extremum at the equi- 1 Q
V =
librium position. 4π0 R
2.4. POTENTIAL GRADIENT 213

++
++ + Charged conductor (c) Electric Potential at an internal Point of Uniformly
+ + Charged Sphere (r < R): To find the potential at some
+ +
+ point r inside the sphere, we use Eq. (2.18),
+ +R
+ +
+ + B
++
Z
+
Graph of potential VB − VA = − Edr
A

V 1 Q along a radial path that begins at some point where the


V = potential is known. We choose a point on the surface of the
4pP0 R
sphere, and integrate the electric field from that point (at
1 Q
V = r = R ) to a point inside where we want to find the potential
4pP0 r (at r = r). The electric field at any point inside the non
conducting sphere of radius R at r = r, is given by Er = k Qr
O r R3
and electric potential at r = R is VR = k Q
R , therefore-
Figure 2.42: Electric potential V at points inside and outside a Z r
Q r
Z
positively charged spherical shell/spherical conductor Vr − VR = − Er dr = −k 3 rdr
R R R
r=r
Q r2 Q
= k 3 R2 − r 2

= −k 3
Variation of electric potential with the distance from the centre R 2 r=R 2R
(r)
Note that, all above results also hold for a conducting sphere Q
R2 − r 2 (2.49)

⇒ Vr − VR = k
whose charge lies entirely on the outer surface. 2R3
Using VR = kQ/R in the last result we reach to the following
2.4.8 Electric Potential due to a Non- relation:
conducting Charged Sphere
r2
 
Q
Suppose a charge Q is uniformly distributed throughout a Vr = k 3− 2 (0 ≤ r ≤ R) (2.50)
2R R
non-conducting spherical volume of radius R. Let the volume
charge density of this nonconducting sphere is ρ. We want to
find expressions for the potential at- At r = 0, we have V0 = 3kQ/2R, and at r = R, we get
(a) an external point (r > R); VR = kQ/R as expected. Figure 22.21 sketches the electric
(b) on the surface (r = R) potential in the two regions 0 ≤ r ≤ R and r ≥ R.
(c) internal point (r < R)
where r is the distance of the point from the centre of the The variation of electric potential with distance from the
sphere. center is shown in Fig.-
(a) Electric Potential at an External Point of Uni- Note: A point charge q can be considered to be the limiting
formly Charged Sphere: Let O be the centre of a non-
conducting sphere of radius R, have a charge Q distributed R
uniformly over its entire volume.
For r ≥ R, the electric field E is radial and has a magnitude: r

Q
Er = k (r ≥ R) (2.46) V Q r2
r2 Vr = k 2R 3 −
3k Q R2
V0 =
This is the same as the electric field due to a point charge, 2R Q
and hence the electric potential at any point of radius r in this Vr = k r
2V
region is given by: 3 0

Q 0 r
Vr = k (r ≥ R) (2.47) R
r
Figure 2.43: A sketch of the electric potential V (r) as a function
(b) Electric Potential at a Point on the Surface of of r in the two regions 0 ≤ r ≤ R and r ≥ R. The curve for the
the Uniformly Charged Non-conducting Sphere: For region 0 ≤ r ≤ R is parabolic and joins smoothly with the curve for
a point on the surface of the nonconducting sphere, r = R, the region r ≥ R, which is hyperbola
therefore Eq.(2.47) gives-
case of a small spherical conductor whose radius tends to zero
Q and potential to infinity.
VR = k (2.48)
R
214 CHAPTER 2. ELECTRIC POTENTIAL

2.4.9 Potential on the edge of a Uniformly electric potential at that point.


Charged Disc: SOLUTION 1. Sketch the x axis and place the two charges
on it. Let r1 be the distance from q1 to an arbitrary field point
To calculate the potential at point O on the circumference of P at position x on the x axis, that is, r1 = |x|. Let r2 be the
the disc, let us divide the disc in large number of rings with O as distance from q2 to P, that is, r2 = |x − a|(Fig.2.45).
center. ABD and EFG are the arcs of two such concentric rings 2. Write the potential as a function of the distances to the two
having common center O. The potential due to one segment charges:
between r and r + dr is given by-
kq1 kq2
1 dq V = +
dV = . r1 r2
4π0 r kq1 kq2
= + (x 6= 0, x 6= a)
Here, dq = σ (Area of ring) = σ (arc ABD × dr) = |x| |x − a|
σ (r × 2θ × dr) = σ (2rθ) dr Note that V → ∞ both as x → 0 and as x → a, and V → 0
1 σ (2rθ) dr σ
∴ dV = = θ dr V(x)
4π0 r 2π0
Further, r = OH cos θ = 2R cos θ

0 a x

Figure 2.46

Figure 2.44 both as x → ∞ and as x → ∞ as one would expect.


Figure 2.46 shows V versus x on the x axis for q1 = q2 > 0.

∴ dr = −2Rsinθ dθ
2.5 Potential Energy in an External
Hence, dV = − 2πε
σ
2Rθ sin θ dθ
0 Field
0 π/2
σR
Z Z
∴ V = dV = θ sin θdθ 2.5.1 Potential energy of a single charge
π/2 πε0 0 −

In this case the external field E is not produced by the given
On solving, we get- charge(s) whose potential energy is to be calculated. E ~ is pro-
σR duced by sources external to the given charge(s). In such sit-
V = −

πε0 uations only the external electric field E or the electrostatic
EXAMPLE 114. A point charge q1 is at the origin, and a potential V , due to external source(s), is given. We assume
second point charge q2 is on the x axis at x = a. Find an that the test charge q0 does not significantly affect the sources
expression for the electric potential everywhere on the x axis producing the external field. This is true if q0 is very small, or
as a function of x. the external sources are held fixed by other unspecified forces.
Even if q0 is finite, its influence on the external sources may
still be ignored in the situation when very strong sources far
y −

away at infinity produce a finite field E in the region of in-
q1 q2 terest. Note again that we are interested in determining the
P
x
potential energy of a given charge q0 (and later, a system of
a r2 = | x – a|
charges) in the external field; we are not interested in the po-
tential energy of the sources producing the external electric
r1 = |x| field.
The external electric field E~ and the corresponding external
x potential V may vary from point to point. By definition, V at
a point P is the work done in bringing a unit positive charge
Figure 2.45 from infinity to the point P . (We continue to take potential
at infinity to be zero.) Thus, work done in bringing a charge
APPROACH Find electric potentials due to each charge sep- q0 from infinity to the point P in the external field is q0 V .
arately at a point on the x-axis and add them to find the net This work is stored in the form of potential energy of q0 . If
2.5. POTENTIAL ENERGY IN AN EXTERNAL FIELD 215

the point P has position vector ~r relative to some origin, we


S S
can write:
Z Z
W = q dV = −q ~ · d~l
E
P P
Potential energy of q0 at r in an external field T S
Z Z
= −q ~ · d~l − q
E ~ · d~l
E
P T
U = q0 V (r) (2.51) Z 0 Z 0
= −q (Eı̂) · (−dy̂) − q (Eı̂) · (−dxı̂)
where V (r) is the external potential at the point r. Z
b
0
a

Thus, if an electron with charge q = e = 1.6×10−19 C is = qE dx = −qEa


accelerated by a potential difference of ∆V = 1 volt, it would a
gain energy of q∆V = 1.6×10−19 J. Note that the potentials at the point S and P are related by
This unit of energy is defined as 1 electron volt or 1 eV, i.e., VP = VS − Ea.
1eV = 1.6×10−19 J. The units based on eV are most commonly Aliter: The electric force on the point charge is F~ = qEı̂
used in atomic, nuclear and particle physics, and its displacement is ~s = ~rS − ~rP = −aı̂ − b̂. Thus, W =
(1keV = 103 eV = 1.6 × 10−16 J, 1MeV = 106 eV = 1.6 × F~ · ~s = (qEı̂) · (−aı̂ − b̂) = −qEa.
10 −13
J, 1GeV = 109 eV = 1.6×10−10 J and 1TeV = 1012 eV =
1.6 × 10−7 J) 2.5.2 Potential Energy of a System of Two
Charges in an External Field
EXAMPLE 115. A point charge q moves from point P to
point S along the path PQRS (Fig.2.47) in a uniform electric Now we calculate the potential energy of a system of two
~ pointing parallel to the positive direction of the x-axis.
field E charges q1 and q2 located at r1 and r2 , respectively, in an
The coordinates of points P, Q, R and S are (a, b, 0), (2a, 0, 0), external field? First, we calculate the work done in bringing
(a, −b, 0), (0, 0, 0) respectively. Find the work done by the field the charge q1 from infinity to r1 . Work done in this step is
in the above process. q1 V (r1 ). Next, we consider the work done in bringing q2 to
r2 . In this step, work is done not only against the external
y field E but also against the field due to q1 .

Work done on q2 against the external field


P ~
E
= q2 V (r2 ) (2.52)
S x
Q Work done on q2 against the field due to q1
q1 q2
R = (2.53)
4πε0 r12
Figure 2.47
where r12 is the distance between q1 and q2 . By the superposi-
tion principle for fields, we add up the work done on q2 against
the two fields (E and that due to q1 ):
Work done in bringing q2 to r2
SOLUTION The work done by the conservative forces
(electrostatic, gravitational, etc.) is independent of the path q1 q2
= q2 V (r2 ) + (2.54)
i.e., it depends only on the initial and final points. Thus, the 4πε 0 r12
work done by the field along path P → Q → R → S is same Thus, Potential energy of the system = the total work done in
as the work done along the path P → T → S. The work done assembling the configuration
q1 q2
y = q1 V (r1 ) + q2 V (r2 ) + (2.55)
~
E 4πε0 r12
EXAMPLE 116. (a) Determine the electrostatic potential
P
energy of a system consisting of two charges 7µC and −2µC
b (and with no external field) placed at (−9 cm, 0, 0) and (9 cm,
S a x 0, 0) respectively. (b) How much work is required to separate
T Q
the two charges infinitely away from each other? (c) Suppose
that the same system of charges is now placed in an external
R electric field E = A 1/r2 ; A = 9 × 105 Cm−2 . What would


the electrostatic energy of the configuration be?


Figure 2.48
−12
SOLUTION (a) U = k q1rq2 = 9×109 × 7×(−2)×10
0.18 = −0.7J
by the field in moving a charge q along the path P → T → S (b) W = U2 − U1 = 0 − U = 0 − (−0.7) = 0.7 J.
is given by (c) The mutual interaction energy of the two charges remains
216 CHAPTER 2. ELECTRIC POTENTIAL

unchanged. In addition, there is the energy of interaction of


the two charges with the external electric field. We find,

7µC −2µC
q1 V (~r1 ) + q2 V (−

r2 ) = A +A
0.09m 0.09m
and the net electrostatic energy is

q1 q2 7µC −2µC
q1 V (~r1 ) + q2 V (−

r2 ) + =A +A − 0.7J
4πε0 r12 0.09m 0.09m
= 70 − 20 − 0.7 = 49.3J

2.6 Equipotential Surfaces and Field Figure 2.49: Equipotential lines and electric field lines for a pair
of charges of equal magnitude but opposite sign.
Lines
An equipotential surface is a threedimensional surface on which
the electric potential V is the same at every point. If a test
charge q0 is moved from point to point on such a surface, the
electric potential energy q0 V remains constant. In a region
where an electric field is present, we can construct an equipo-
tential surface through any point. In diagrams we usually show
only a few representative equipotentials, often with equal po-
tential differences between adjacent surfaces. No point can be
at two different potentials, so equipotential surfaces for differ-
ent potentials can never touch or intersect.

2.6.1 Different Equipotential Surfaces


Because potential energy does not change as a test charge Figure 2.50: Two equal positive charges
moves over an equipotential surface, the electric field can do


no work on such a charge. It follows that E must be perpen- 1. The Equipotential Surface For a Point Charge
dicular to the surface at every point so that the electric force
−→ The electric potential V at any position r from a point charge
q0 E is always perpendicular to the displacement of a charge
moving on the surface. Field lines and equipotential surfaces q, due to it, is given by-
are always mutually perpendicular. In general, field lines are q q
curves, and equipotentials are curved surfaces. For the special V (x, y, z) = k = kp = V0 = constant
r2 (x + y 2 + z 2 )
2
case of a uniform field, in which the field lines are straight, par-
allel, and equally spaced, the equipotentials are parallel planes k2 q2
⇒ x2 + y 2 + z 2 = = constant
perpendicular to the field lines. V02
5. Equipotential Surfaces Due to an Electric Dipole
Thus, the surfaces are concentric spheres with the origin (the
In Fig.2.49, electric field lines and equipotential lines are plot-
location of the charge) as the centre and radius given by
ted for a set of two charges of equal magnitude, one positive
and one negative. The field lines are closest together near the kq
charges, indicating that the electric field is strongest there. R=
V0
Very near each charge, the effect of that charge dominates on
the equipotential plots, which are not strongly different from 2. The Equipotential Surface of an Infinite Line
circles, but farther away the superposition of the two electric Charge Carrying a Positive Charge Density λ
potentials produces more noticeable effects. Note that elec-
tric field lines always cross equipotential lines perpendicularly. . Let the line charge be along the z-axis. The potential due to
Equipotential Surfaces due to two Identical Positive a line charge at a point P is given by
Charges: λ
Fig.2.50 shows equipotential surfaces and electric field lines due V (r) = − lnr
2π0
to two identical positive point charges. It may appear that two
equipotential surfaces intersect at the center of Fig.2.50, in vi- where r is the distance of the point P frompthe line charge.
olation of the rule that this can never happen. In fact this is Since the line charge is along the z-axis, r = x2 + y 2
a single figure- 8 -shaped equipotential surface. so that V (r) = − 4π
λ
0
ln x2 + y 2
2.6. EQUIPOTENTIAL SURFACES AND FIELD LINES 217

electric field line

1 q

Figure 2.53

If the positive charges in above diagrams, are replaced by neg-


equipotential
ative charges, and vice versa, the equipotential surfaces will be
Figure 2.51 the same but the sign of the potential will get reversed. For
example, the surfaces with potentials potential V = +30 V
and V = −50 V (say) will have potential V = −30 V and
Now, V = +50 V, respectively.
V = constant = V0
4π0 V0
⇒ ln x2 + y 2 = − 2.6.2 Properties of Equipotential surface

λ
⇒ x2 + y 2 = e−4πε0 V0 /λ (i) Potential difference between two points in an equipoten-
tial surface always remains zero.
which represents cylinders with axis along the z-axis with radii (ii) If a test charge q0 moves from one point to the other on
r = e−2π0 V0 /λ . As V0 increases, radius becomes smaller. Thus such a surface, the electric potential energy q0 V remains
constant.
(iii) No work is done by the electric force, when the test charge
moves along this surface.
(iv) Two equipotential surfaces can never intersect each other
because the point of intersection will have two potentials
which is of course not acceptable.
(v) Field lines and equipotential surfaces are always mutually
perpendicular.

EXAMPLE 117. Figure 2.54 shows lines of constant poten-


tial in a region in which an electric field is present. The values
of the potential of each line is also shown. Of the points A, B
and C, which one has the maximum magnitude of electric field
Figure 2.52 ~
E

the cylinders are packed closer around the axis, showing that
the field is stronger near the axis.

3. Uniformly Charged Plane Surface (infinite)


σ A
V = V0 − |z|
20
(V0 is the potential at z = 0) B
Equipotential surface means
50 V
V = constant 40 V C
σ 30 V
⇒ V0 − |z| = C
20 20 V
⇒ |z| = constant 10 V
Figure 2.54
Note that, the shapes of the equipotential surfaces in above
diagrams will not change if the sign of each charge is reversed.
218 CHAPTER 2. ELECTRIC POTENTIAL

APPROACH From relation E = −∂V /∂x., we can say that,


the magnitude of E ~ will be greater for greater value ∂V /∂x.
SOLUTION The potential difference between all the succes-
sive lines of constant potential is ∆V = 10 V. The perpendic-
ular distances between successives lines at the point A and at
the point C are almost equal but it is smaller at the point
B i.e., ∆xA = ∆xC > ∆xB . Hence, |EB | = ∆V /∆xB >
∆V /∆xA = |EA | = |EC | .
(a) Electric field at the surface of
conductor.
2.7 Equipotentials and Conductors Cross section of equipotential
surface through P
Statement: When all charges are at rest, the surface of a
conductor is always an equipotential surface. Gaussian surface
(in cross section)
Since the electric field E~ is always perpendicular to an
B
Surface
– P
of cavity
A

– Conductor
– (b) Cavity in a conductor.
+ + ++
+ + –
++ + + Figure 2.56: (a) At all points on a conductor’s surface, the electric

field must be perpendicular to the surface. If E had a tangential
– component, a net amount of work would be done on a test charge by
moving it around a loop as shown-which is impossible because the
– electric force is conservative. (b) If the cavity contains no charge,
every point in the cavity is at the same potential, the electric field
S – is zero everywhere in the cavity, and there is no charge anywhere
E on the surface of the cavity.

Cross sections of equipotential surfaces


Electric field lines closed path integral of work done by an electric field should
always be zero.
Figure 2.55: When charges are at rest, a conducting surface is In simple words if you take a charge and move it around in
always an equipotential surface. Field lines are perpendicular to a
space in any manner and at the end come back at the same
conducting surface.
point where you started then you have done net zero work.
equipotential surface, we can prove this statement by proving Here, we have considered a rectangular path for our conve-
that when all charges are at rest, the electric field just outside nience. However, you can choose any path and the formula
a conductor must be perpendicular to the surface at every will hold good.
point ( Fig.2.55). We know that E ~ = 0 everywhere inside the
Let us transport a test charge q0 around the loop abcda
conductor; otherwise, charges would move. In particular, at in Fig. ??. The segments bc and da (and the associated
any point just inside the surface the component of E ~ tangent
work) may be made arbitrarily small (because the length is
to the surface is zero. It follows that the tangential component
~ is also zero just outside the surface. If it were not, a very small. Think F~ .~s i.e., work), and no work is done in
of E
the segment cd because, as already shown, the field is zero
charge could move around a rectangular path partly inside
everywhere inside the conductor. If the field just outside the
and partly outside (Fig. ??) and return to its starting point
conductor has a component E|| , parallel to the surface, this
with a net amount of work having been done on it. This
component does work equal to q0 E|| that is the parallel com-
would violate the conservative nature of electrostatic fields, so
~ just outside the surface must ponent of E~ vector at the surface (if any) does work on the
the tangential component of E
be zero at every point on the surface. test charge q0 equal to q0 times l(length of ab) times parallel
Explanation: Electric potential difference between any two component of E ~ at surface E| |.
points in electric field is given by- So, the net work done in transporting the test charge q0
Z B along a closed path abcda,
VB − VA =
WAB
=− E~ ·−

dl
q0 Wabcda = Wab + Wbc + Wcd + Wda 6==
A

Since, the electric field is conservative in naturte, therefore the i.e., the net work done is non zero. It is impossible, because, it
2.7. EQUIPOTENTIALS AND CONDUCTORS 219

shows that the electric force field is not conservative. To avoid (A) −2V (B) 2V (C) 4 V (D) V
this contradiction, we must conclude that there cannot be a
component of E ~ parallel to the surface, and that E ~ is therefore
perpendicular to the surface.
Thus, E~ is perpendicular to the surface at each point, prov-
ing our statement.
It also follows that when all charges are at rest, the entire
solid volume of a conductor is at the same potential. Equation
RB − →
(2.18) [VB −VA = WqAB =− A E ~ · dr] states that the potential
0
difference between two points A and B within the conductor’s
RV
solid volume, VB − VA , is equal to the line integral A E ~ · d~l of
~
the electric field from A to B. Since E = 0 everywhere inside
the conductor, the integral is guaranteed to be zero for any
Figure 2.57
two such points A and B. Hence the potential is the same for
any two points within the solid volume of the conductor. We
describe this by saying that the solid volume of the conductor
is an equipotential volume. APPROACH First indicate the charge distribution and then
Theorem 1: In an electrostatic situation, if a conductor con- find the potential difference by applying the formula of poten-
tains a cavity and if no charge is present inside the cavity, tial at any point due to charged sphere/spherical shell.
then there can be no net charge anywhere on the surface of the SOLUTION (D) Since, the uncharged shell initially encloses
cavity. charge Q, therefore, due to induction, the charge on inner sur-
Proof: To prove this theorem, we first prove that every face of the shell will be −Q (Fig.2.57). By charge conservation,
point in the cavity is at the same potential. In Fig. ??, the the charge on the outer surface of the shell will be +Q.
conducting surface A of the cavity is an equipotential surface, The electric potential at any point A on the inner surface of
as we have just proved. Suppose point P in the cavity is at a shell is given by-
different potential; then we can construct a different equipo- kQ k(−Q) kQ
tential surface B including point P . VA = + + (2.56)
a b b
Now consider a Gaussian surface, shown in Fig. ??, between
the two equipotential surfaces. Because of the relationship where, k = 4πε
1
= 9.0 × 109 N.m2 /C 2
between E ~ and the equipotentials, we know that the field at 0
Electric potential on surface of outer shell is
every point between the equipotentials is from A toward B, or
else at every point it is from B toward A, depending on which kQ k(−Q) kQ
VB = + + (2.57)
equipotential surface is at higher potential. b b b
In either case the flux through this Gaussian surface is cer-
Therefore, the potential difference is
tainly not zero. But then Gauss’s law says that the charge
enclosed by the Gaussian surface cannot be zero. This con- 
1 1

tradicts our initial assumption that there is no charge in the ∆VAB = VA − VB = kQ −
a b
cavity. So the potential at P cannot be different from that at
the cavity wall. Given, that ∆VAB = V , so-
The entire region of the cavity must therefore be at the same
potential. But for this to be true, the electric field inside the
 
1 1
kQ − =V (2.58)
cavity must be zero everywhere. Finally, Gauss’s law shows a b
that the electric field at any point on the surface of a conductor
Now, if the shell is given an extra charge −4Q, it
is proportional to the surface charge density σ at that point.
will uniformly get spread on the outer surface of the
We conclude that the surface charge density on the wall of the
shell. In this case, the new potential difference will be
cavity is zero at every point.
∆VAB = VA − VB
Note: Don’t confuse equipotential surfaces with the Gaus-    
sian surfaces. Gaussian surfaces have relevance only when we kQ k(−4Q) kQ k(−4Q)
= + − +
are using Gauss’s law, and we can choose any Gaussian surface a b b b
that’s convenient. We cannot choose equipotential surfaces;
 
1 1
the shape is determined by the charge distribution. = kQ − = V [ from Eq.((2.58))]
a b
It is a very important result which shows that the potential
EXAMPLE 118. A solid conducting sphere, having a charge difference between the outer surface of the spherical shell and
Q, is surrounded by an uncharged conducting hollow spherical inner sphere is independent on the charge of the shell.
shell. Let the potential difference between the surface of
the solid sphere and that of the outer surface of the hollow EXAMPLE 119. Two conducting spheres 1 and 2, having
shell be V . If the shell is now given a charge of −4Q, radii a and b charged to q1 and q2 respectively. Find the po-
the new potential difference between the same two surfaces is tential difference between 1 and 2.
220 CHAPTER 2. ELECTRIC POTENTIAL

EXAMPLE 121. A metal sphere of radius R, carrying charge


q, is surrounded by a thick concentric metal shell (inner radius
a, outer radius b, as in Fig.2.60). The shell carries no net
charge.
(a) Find the surface charge density σ at R, at a, and at b.
(b) Find the potential at the center, using infinity as the refer-
ence point.
(c) Now the outer surface is touched to a grounding wire, which
lowers its potential to zero (same as at infinity). How do your
Figure 2.58 answers to (a) and (b) change?

SOLUTION The potential on the surface of the sphere 1 is


given by -
1 q1 1 q2
V1 = + (2.59)
4πε0 a 4πε0 b
The potential on the surface of the sphere 2 is given by,
1 q1 1 q2
V2 = + (2.60)
4πε0 b 4πε0 b
1 q1 1 q1
⇒ V1 − V2 = −
4πε0 a 4πε0 b Figure 2.60
 
q1 1 1
= −
4πε0 a b SOLUTION (a) σR = 4πR q
2 ; σa = −q2 ; σb = 4πb
−q
2
R0 −→− → R b  1 q 4πa Ra
EXAMPLE 120. A point charge q is kept at a distance l from (b) V (0) = − ∞ E . dl = − ∞ 4πε0 r2 dr − b (0)dr −
a charged conducting sphere of radius R, having charge Q over R R  1 q  R0 1 q q q

4πε 2 dr − (0)dr = 4πε + −
its surface. Find electric potential due to induced charges at a 0 r R 0 b R a
Ra
any point P inside the sphere. (c) σb → 0(the charge "drains off"); V (0) = − ∞ (0)dr −
RR 1 q
R0 1 q q

SOLUTION Potential due to induced charges is zero as cen- a 4πε0 r2 − R (0)dr = 4πε0 R − a
tre C is equidistant from all induced charges. Since, every
EXAMPLE 122. A total charge q is spread uniformly over
point inside this conducting sphere will be at equal potential,
the inner surface of a non-conducting hemispherical cup of in-
therefore-
ner radius a. Calculate (a) the electric field and (b) the electric
kq kQ potential at the center of the hemisphere (Consider the cup as
VC = VP = + a stack of rings).
l R
SOLUTION Consider a circular strip symmetric about z -axis
of radius r and width adθFig. 2.61 ). The charge on the strip
is
2πr adθ qr dθ
dq = q 2
= = qsinθ dθ
2πa a
(a) At the centre of the hemisphere, the x -component of the

Figure 2.59

Also at P
VP = Vdue to q + Vdue to induced charges + Vdue to Q

kq kQ kq kQ
⇒ + = + + Vdue to induced charge Figure 2.61
l R x R
kq kq
Vinduced charge = − field will be cancelled for reasons of symmetry. The entire field
l x will be contributed by the z -component alone.

→ −
→ −

E external + E conductor + E induced = 0
qsinθ dθcosθ
Resultant electric field inside material is zero dE = dEz =
4πε0 a2
2.7. EQUIPOTENTIALS AND CONDUCTORS 221

π/2
q q
Z Z
smooth horizontal plane and the other is projected along the
∴ E= dEz = sinθ cosθ dθ =
4πε0 a2 0 8πε0 a2 plane directly towards the first from a distance d with speed u.
R π/2 Find the closest distance of approach.
(b) dV = qsinθ
4πε0 a ; V =
dθ q q
R
dV = 4πε0 a 0
sinθ dθ = 4πε0 a
SOLUTION As the mass 2m is not fixed, it will also move
EXAMPLE 123. An alpha particle (two protons, two neu-
away from m due to repulsion. The distance between the par-
trons) moves into a stationary gold atom (79 protons, 118 neu-
ticles is minimum when their relative velocity is zero i.e., when
trons), passing through the electron region that surrounds the
they have equal velocities.
gold nucleus like a shell and headed directly toward the nucleus
Hence at closest approach, v1 = v2
(Fig.). The alpha particle slows until it momentarily stops
when its center is at radial distance r = 9.23 fm from the
nuclear center. Then it moves back along its incoming path.
(Because the gold nucleus is much more massive than the al-
pha particle, we can assume the gold nucleus does not move.)
What was the kinetic energy Ki of the alpha particle when it
(a)
was initially far away (hence external to the gold atom)? As-
sume that the only force acting between the alpha particle and
the gold nucleus is the (electrostatic) Coulomb force and treat
each as a single charged particle.

(b)
Gold
nucleus
v 50 Figure 2.63
S
v
By conservation of linear momentum, we have-
Alpha
Particle
r mu = mv1 + 2mv2
v2 = v1 = u/3
Figure 2.62
By conservation of mechanical energy, we have-
APPROACH During the entire process, the mechanical en- Loss in KE = gain in PE
ergy of the alpha particle + gold atom system is conserved.
q2
   
When the alpha particle is outside the atom, the system’s ini- 1 2 1 2 1 2 1 1
mu − mv + 2mv2 = −
tial electric potential energy Ui is zero because the atom has 2 2 1 2 4πε0 x d
an equal number of electrons and protons, which produce a net 1 u2 q2
 
1 1 1
mu2 − m (1 + 2) = −
electric field of zero. However, once the alpha particle passes 2 2 9 4πε0 x d
through the electron region surrounding the nucleus on its way 1 q2

1 1

2
to the nucleus, the electric field due to the electrons goes to mu = −
3 4πε0 x d
zero. The reason is that the electrons act like a closed spheri-
cal shell of uniform negative charge and such a shell produces 1 1 4πε0 mu2
= +
zero electric field in the space it encloses. The alpha particle x d 3q 2
still experiences the electric field of the protons in the nucleus, 3q 2 d
x= 2
which produces a repulsive force on the protons within the 3q + 4πε0 mu2 d
alpha particle.
As the incoming alpha particle is slowed by this repulsive
2.7.1 Check Point 3
force, its kinetic energy is transferred to electric potential en-
ergy of the system. The transfer is complete when the alpha 1. •• The labeled points in Figure 2.64 are on a series of
particle momentarily stops and the kinetic energy is Kf = 0. equipotential surfaces associated with an electric field.
SOLUTION According to principle of conservation of Rank (from greatest to least) the work done by the elec-
mechanical energy, we have- tric field on a positively charged particle that moves from
A to B; from B to C; from C to D; from D to E.

Ki + Ui = Kf + Uf 2. •• Equipotential surfaces associated with an electric field


which is increasing in magnitude along the X-direction are-
Here, Ui = 0 and Kf = 0, therefore-
(A) planes parallel to Y Z-plane
(2e) (79e) (B) planes parallel to XY -plane
−12
Ki = k = 3.94 × 10 J = 24.6M eV (C) planes parallel to XZ-plane
9.23 × 10−15
(D) coaxial cylinders of increasing radii around the x-
EXAMPLE 124. Two particles of mass m and 2m carry a axis
charge q each. Initially the heavier particle is at rest on a
222 CHAPTER 2. ELECTRIC POTENTIAL

10 V. The potential at the centre of the sphere is


B (A) zero
9V A (B) 10 V
(C) same as at a point 5 cm away from the surface
8V E (D) same as at a point 25 cm away from the surface.
D
C
7V Answer Key and Solutions
1. B → C, C → D, A → B, D → E Moving from B to C
6V decreases the electric potential by 2 V, so the electric field
performs 2 J of work on each coulomb of positive charge
Figure 2.64 that moves. Moving from C to D decreases the electric
potential by 1 V, so 1 J of work is done by the field. It
takes no work to move the charge from A to B because the
3. •• For the equipotential surfaces in Figure 2.64, what electric potential does not change. Moving from D to E
is the approximate direction of the electric field? increases the electric potential by 1 V, and thus the field
(A) Out of the page
does −1 J of work per unit of positive charge that moves.
(B) Into the page
(C) Toward the top of the page 2. (A) Any surface which has same electrostatic potential at
(D) Toward the bottom of the page. every point is called an equipotential surface. Electric field
is always perpendicular to an equipotential surface. There-
4. •• A metallic solid sphere is placed in a uni-
fore, X-direction is perpendicular only to Y Z-plane.
form electric field. The lines of force fol-
low the path(s) shown in the figure as 3. (D) The electric field points in the direction of decreasing
(A) 1 (B) 2 (C) 3 (D) 4 electric potential.

4. (D) In electrostatics (i.e., when charges are stationary and


1 charge density does not vary with the time), electric field
inside a conductor remains zero and the electric field lines
2 remain normal to the surface and never enter inside the
3 conductor.
4 5. APPROACH Electric potential at a distance r from the
Figure 2.65 center of a uniformly charged non-conducting sphere of
radius R is given by-

5. •• A uniformly charged non-conducting solid sphere of ra- Q


Vr = k (r ≥ R) [see Eq.(2.47)] (i)
dius R has potential V0 (measured with respect to ∞ ) on r
its surface. For this sphere, the equipotential surfaces with
potentials 3V2 0 , 5V4 0 , 3V4 0 and V40 have radius R1 , R2 , R3 , and Q
VR = k [see Eq.(2.48)] (ii)
R4 respectively. Then, R
(A) R1 = 0 and R2 > (R4 − R3 )
(B) R1 6= 0 and (R2 − R1 ) > (R4 − R3 )
(C) R1 = 0 and R2 < (R4 − R3 ) r2
 
Q
(D) 2R < R4 Vr = k 3− 2 (0 ≤ r ≤ R) [see Eq.(2.40)] (iii)
2R R

Substitute the given values of V in above appropriate equa-


Q tions and simplify for r.
SOLUTION (C, D) Given that the electric potential of
the sphere with respect to ∞, is V0 , therefore, from Eq.(ii)-

kQ kQ
V0 = ⇒ = V0 (iv)
R R R
At the center of sphere, r = 0, therefore, from Eq.(iii), we
have-
Figure 2.66 Q

02

3kQ 3
VC = k 3− 2 = = V0
2R R 2R 2
6. •• A hollow metal sphere of radius 5 cm is 3
charged such that the potential on its surface is or VC = V0 (v)
2
2.8. CONNECTED CONDUCTING SPHERES 223

Calculation of distances from the center of the E1


E2
sphere for given potentials:
(i ) For potential VR1 = 3V2 0 q1 q2
From Eq.(v), it is clear that- R2
R1
VR1 = VC ⇒ R1 = 0

(ii ) For potential VR2 = 5V4 0 Figure 2.67


As potential decreases for outside points. Thus, according
to the given problem, we can write
Potential at any point on the surface of charged sphere having
5V0 kQ charge q1 and radius R1 is-
3R2 − R2 2

VR2 = =
4 2R3 q1
5V0 V0 V1 =
or = 3R2 − R22
 4πε0 R1
4 2R 2
 2 Potential at any point on the surface of charged sphere having
5 R2 charge q2 and radius R2 is-
⇒ =3−
2 R
q2
 2
R2 5 1 R V2 =
⇒ =3− = ⇒ R2 =√ 4πε0 R2
R 2 2 2 Therefore, in electrostatic equilibrium:
Similarly, (iii ) For potential VR3 = 3V0
4 q1 q2 q1 R1
V1 = V2 ⇒ = =⇒ =
R1 R2 q2 R2
3V0 kQ 3 kQ
V R3 = ⇒ = × Surface charge density of the sphere of radius R1 , is-
4 R3 4 R
q1
4 σ1 =
or R3 = R 4πR12
3
(iv ) For potential V = V40 Surface charge density of the sphere of radius R2 , is
kQ V0 kQ 1 kQ q2
Now, VR4 = = ⇒ = × σ2 =
R4 4 R4 4 R 4πR22
or R4 = 4R

σ1 q1 R 2 R1 R22 R2
∴ = . 22 = . = (2.61)
6. (B) Note that the potential inside a hollow conducting σ2 q2 R1 R2 R12 R1
sphere is always constant and its value is equal to the po-
tential at it’s surface. So, the potential at the center will So, if R1 < R2 , then σ1 > σ2 and the surface electric field
be 10 V. Note that the electric field inside the hollow con- E1 > E2 .
ducting sphere is zero. Note: Charge distribution on a conductor does not have to be
uniform.

2.8 Connected Conducting Spheres 2.9 Earthing


Now consider two charged conducting spheres with different Earth is very big conducting sphere. We assume potential of
radii connected with a long conducting wire, as shown in Fig- earth is zero because its size is very large.
ure 2.67. We assume that the spheres are far enough apart When a conducting body is joined to earth through a con-
that the charge distribution on one does not directly affect ducting wire. Potential of body also becomes zero.
the other. We already know that the electric field is stronger In Fig.2.68a, charge +q is given to inner sphere of radius
where the electric field lines are closer together and weaker a placed inside a larger concentric sphere of radius b(> a).
where the electric field lines are widely separated. Electric By using Gausses’ law, we can show that the induced charge
charge is placed on one of the spheres. Some of that charge produced at inner surface of bigger sphere is −q. Let the charge
will then flow through the conducting wire so both spheres on the outer surface of larger sphere is x.
are charged. In electric equilibrium, the charge must be dis- Since, the outer sphere is earthed, therefore, the net electric
tributed so the electric potential is the same on the two spheres potential at any point on the surface of outer sphere must be
(since otherwise charges would flow in the conducting wire and zero, i.e.,-
the system would not be in electrical equilibrium as assumed). Vouter = 0
Two conductors connected can be seen as a single conductor. q (−q + x)
Therefore, electric potential will be identical everywhere on the ⇒ k +k =0
b b
connected spheres. ⇒ x=0
224 CHAPTER 2. ELECTRIC POTENTIAL

(a) Outer sphere earthed

Figure 2.69

2.10.1 Electric Potential Energy for Contin-


uous Charge System
This energy is also known as self-energy2 .
(b) Inner sphere earthed
( i ) Potential Energy (Self Energy) of a Uniformly
Figure 2.68
Charged Conducting Sphere or Spherical Shell
To calculate it, lets use method 1: Take an uncharged shell.
Now bring charges one by one from infinity to the surface of
So, there will not be any charge on the outer surface of the the shell. The work required in this process will be stored as
larger sphere, only −q charge will be induced on the inner potential Energy. Suppose, at any instant the given charge
surface of the outer sphere.
In Fig. 2.68b the inner sphere is earthed and the net given
charge on the outer sphere is q, then-

Vinner = 0
kq 0 kq
⇒ + =0
a b
a
⇒ q 0 = −q
b Figure 2.70

on the conducting spherical shell is q and now we are giving


extra charge dq to it.
The work required to bring dq charge from infinity to the shell
2.10 Corona Discharge is-

Let us consider an arbitrary shaped charged conductor. Now, if dW = (dq)(Vf − Vi )


we fit circles at different positions on the surface of the conduc- 
Kq

Kq
tor, then the radii of these circles will be the radii of curvatures ⇒ dW = dq −0 = dq
R R
at these positions respectively. Since, the radius of curvature
is smallest at sharpest end of the conductor, therefore, from Therefore, net work done in charging the shell up to charge
Eq.2.61, it is clear that, the electric field is much greater in Q, is given by-
sharply pointed regions, it may become large enough for the Q
Kq KQ2
Z
air or other gas to break down and conduct a current because W = dq =
we have ionized material near those points. Corona discharge 0 R 2R
is electric discharge due to ionization of material in a fluid This work will be stored in the form of electrostatic potential
(such as air) surrounding a conductor. This discharge happens energy (self energy). So, the electrostatic potential energy of
near pointed regions on conductors. The minimum electric field the charged spherical shell-
needed for breakdown is called the breakdown field and is typ-
ically 3MV/m for fairly dry air. Corona discharge is used in KQ2
U=
ozone generators, in electrical precipitators that remove partic- 2R
2 For a system of stationary or moving discrete charges, the net elec-
ulates in smokestacks, and in forced-air heating systems. It is trostatic potential energy can be calculated as discussed above but for a
also important in many other applications, including lightning continuous charge distribution, we must also consider the self-energy of
protection rods. each body in the system.
2.10. CORONA DISCHARGE 225

(ii) Self Energy of Uniformly Charged Solid Sphere


To make a non conducting charged solid sphere, we have to
assemble charged particles (each of infinitely small step sized
charge dq) one over the other until a sphere of required charge
and radius is formed. For it, we bring the charged particles Figure 2.72
one-by-one from infinity to the sphere so that the size of the
sphere increases. Suppose we have given charge q to the sphere,
APPROACH To find the total energy of the system, add the
self energies of both spheres and the potential energy due to
mutual interaction.
SOLUTION Utotal = Uself + Uinteraction
Q21 Q22 Q1 Q2
= + +
8πε0 R1 8π0 R2 4πε0 r
Figure 2.71 EXAMPLE 127. Two uniformly charged concentric spheri-
cal shells of radii R1 and R2 (R2 > R1 ) have charges Q1 and
and its radius becomes ‘x’. Now we are giving extra charge dq Q2 respectively. Find out total energy of the system.
to it, which will increase its radius by dx.
∴ Work required to bring charge dq from infinity to the sphere
 
kq kqdq
dW = dq (Vf − Vi ) = dq −0 =
x x

here, q = ρ 34 πx3 ⇒ dq = ρ(4πx2 dx) ∴ Total work required




to give charge Q to the sphere Figure 2.73


Z R 4 3 2

ρ πx ρ(4πx dx)
W = k 3 APPROACH To find the total energy of the system, add the
x
0 self energies of both spherical shells and the potential energy
2 due to mutual interaction.
On simplifying it, we get: W = 5 R = Uself (for a non
3 kQ
SOLUTION Utotal = Uself1 + Uself2 + Uinteraction
conducting solid sphere) Q2 Q2 Q1 Q2
= 8πε01R1 + 8πε02R2 + 4πε0 R2

♦ The total electrical energy of a system of objects is always EXAMPLE 128. Give reasons-
equal to the sum of self energies of each object and the electrical
potential energy due to mutual interaction between the objects. 1. A comb run through one’s dry hair attracts small bits of
If the objects are charged particles, then the self energy term paper. Why? What happens if the hair is wet or if it
will be zero. is a rainy day? (Remember, a paper does not conduct
electricity.)
EXAMPLE 125. A uniformly charged spherical shell of ra-
dius R and charge q, is expanded to a radius 2R. Find the 2. Ordinary rubber is an insulator. But special rubber tyres
work performed by the external agent against electric forces of aircraft are made slightly conducting. Why is this nec-
and work done by electric forces, in this process. essary?

APPROACH The work done by external agent against elec- 3. Vehicles carrying inflammable materials usually have
tric forces is always equal to the change in electric potential metallic ropes touching the ground during motion. Why?
energy of the system, i.e., 4. A bird perches on a bare high power line, and nothing hap-
Wext = Uf − Ui pens to the bird. A man standing on the ground touches
the same line and gets a fatal shock. Why?
Here, Ui and Uf are initial and final electric potential energies
SOLUTION
of the spherical shell.
Work done by electric forces Wel = −Wext 1. This is because the comb gets charged by friction. The
SOLUTION Wext = Uf − Ui = 16πε q2 q2
− 8πε q2
= − 16πε molecules in the paper gets polarised by the charged comb,
0R 0R 0R
resulting in a net force of attraction. If the hair is wet,
q2 or if it is rainy day, friction between hair and the comb
Wel = −Wext =
16πε0 R reduces. The comb does not get charged and thus it will
not attract small bits of paper.
EXAMPLE 126. Two non-conducting hollow uniformly
charged spheres of radii R1 and R2 with charge Q1 and Q2 2. To enable them to conduct charge (produced by friction)
respectively are placed at a distance r. Find out total energy of to the ground; as too much of static electricity accumu-
the system. lated may result in spark and result in fire.
226 CHAPTER 2. ELECTRIC POTENTIAL

3. Reason similar to 2.
4. Current passes only when there is difference in potential.

2.11 The Van de Graaff Generator


This is a machine that can build up high voltages of the order
of a few million volts. The resulting large electric fields are
used to accelerate charged particles (electrons, protons, ions)
to high energies needed for experiments to probe the small scale
structure of matter. The principle underlying the machine is
as follows.
Suppose, we have a large spherical conducting shell of radius
R, on which we place a charge Q. This charge spreads itself
uniformly all over the sphere. The field outside the sphere is
just that of a point charge Q at the center; while the field
Figure 2.74
inside the sphere vanishes. So the potential outside is that of
a point charge; and inside it is constant, namely the value at
the radius R. We thus have:

Potential inside conducting spherical shell of radius R carrying


charge Q = constant
1 Q
= (2.62)
4πε0 R
Now, as shown in Fig. 2.74, let us suppose that in some way
we introduce a small sphere of radius r, carrying some charge
q, into the large one, and place it at the centre. The potential
due to this new charge clearly has the following values at the
radii indicated:
Potential due to small sphere of radius r carrying charge q
1 q
= at surface of small sphere
4πε0 r
1 q
= at large shell of radius R (2.63)
4πε0 R
Figure 2.75
Taking both charges q and Q into account we have for the total
potential V and the potential difference the values
into the larger one, we can in this way keep piling up larger
and larger amount of charge on the latter. The potential (Eq.
 
1 Q q
V (R) = + 1) at the outer sphere would also keep rising, at least until we
4πε0 R R
reach the breakdown field of air.
This is the principle of the van de Graaff generator. It is
 
1 Q q
V (r) = +
4πε0 R r a machine capable of building up potential difference of a few
  million volts, and fields close to the breakdown field of air
q 1 1
V (r) − V (R) = − (2.64) which is about 3 × 106 V/m. A schematic diagram of the
4πε0 r R van de Graaff generator is given in Fig.2.75. A large spherical
Assume now that q is positive. We see that, independent conducting shell (of few meters radius) is supported at a height
of the amount of charge Q that may have accumulated on the several meters above the ground on an insulating column. A
larger sphere and even if it is positive, the inner sphere is always long narrow endless belt insulating material, like rubber or silk,
at a higher potential: the difference V (r) − V (R) is positive. is wound around two pulleys – one at ground level, one at the
The potential due to Q is constant upto radius R and so cancels center of the shell. This belt is kept continuously moving by a
out in the difference! This means that if we now connect the motor driving the lower pulley. It continuously carries positive
smaller and larger sphere by a wire, the charge q on the former charge, sprayed on to it by a brush at ground level, to the top.
will immediately flow onto the matter, even though the charge There it transfers its positive charge to another conducting
Q may be quite large. The natural tendency is for positive brush connected to the large shell. Thus positive charge is
charge to move from higher to lower potential. Thus, provided transferred to the shell, where it spreads out uniformly on the
we are somehow able to introduce the small charged sphere outer surface. In this way, voltage differences of as much as 6
2.12. THE MILLIKAN OIL-DROP EXPERIMENT 227

or 8 million volts (with respect to ground) can be built up. At terminal speed of the drop, the viscous drag force is given
Note: It is possible to increase the potential of the dome by-
until electrical ionization occurs in the air. Since the ion-
ization breakdown of air occurs at an electric field of about FD = 6πrηvT
3 × 106 V /m, a sphere of 1m  can be raised to maximum of
Vmax = ER = 3 × 106 V /m (1m) = 3 × 106 V . The dome’s where r is the radius of the oil drop, η is the viscosity of the
electric potential can be increased further by placing the dome air, and vT is the terminal velocity of the falling drop.
in vacuum and by increasing the radius of the sphere.

2.12 The Millikan Oil-Drop Experi-


ment
Robert Millikan conducted a series of experiments during
1909-1913 in which he measured e, the magnitude of the
elementary charge on an electron, and demonstrated the
quantized nature of this charge. His apparatus, diagrammed
in Figure 2.76, contains two parallel metallic plates separated
by distance d. Oil droplets from an atomizer are allowed to
pass through a small hole in the upper plate. Millikan used x
-rays to ionize the air in the chamber so that freed electrons (b) When the electric field is
turned on, the droplet moves up-
would adhere to the oil drops, giving them a negative charge.
(a) With the electric field off, the ward at terminal velocity − →v 0T
A horizontally directed light beam is used to illuminate the droplet falls at terminal velocity under the influence of the elec-
oil droplets, which are viewed through a telescope whose long −
→v T under the influence of the tric, gravitational, and drag
axis is perpendicular to the light beam. When viewed in this gravitational and drag forces. forces.
manner, the droplets appear as shining stars against a dark
Figure 2.77: The forces acting on a negatively charged oil droplet
background and the rate at which individual drops fall can be
in the Millikan experiment.
determined.
Let’s assume a single drop having a mass m and carrying
From above two equations, we can write-

mg = 6πrηvT

mg
⇒ r= (2.65)
6πηvT
Now suppose a battery connected to the plates sets up an
electric field between the plates such that the upper plate is at
−→
the higher electric potential. In this case, a third force q E acts
−→
on the charged drop. Since, q is negative and E is directed
downward, this electric force is directed upward as shown in
Figure 2.77(b). If this upward force is strong enough, the

→0
Figure 2.76 drop moves upward and the drag force F D acts downward.


When the upward electric force q E balances the sum of the
a charge q is being viewed and its charge is negative. If no gravitational force and the downward drag force − →0
F D , the
electric field is present between the plates (Figure 2.77(a)), drop reaches a new terminal speed vT 0 in the upward direction.
the two forces acting on the charge are-
1. Gravitational force m− →
g (acting downwards) qE = mg + FD


2. Viscous drag force F D (acting upwards)
Note that there is also a buoyant force on the oil drop due to ⇒ qE = mg + 6πrηvT0 (2.66)
the surrounding air. This force can be taken as a correction On substituting the value of r from Eq. (2.65) to Eq. (2.66),
term in the gravitational force m~g on the drop. For now, we we get-
will not consider it in our analysis.
The drag force is proportional to the drop’s speed. When the mg 0
qE = mg + v
drop reaches its terminal speed vT the two forces balance each vT T
other, i.e.
vT0
 
mg
∴ q= 1+ (2.67)
mg = FD E vT
228 CHAPTER 2. ELECTRIC POTENTIAL

Since the electrodes are parallel plates, the magnitude of the • If two copper spheres of the same radii, one hollow and
electric field can be readily calculated from the potential dif- the other solid, are charged to same potential, then both
ference (V ) between the plates and the plate separation (d). contain same charge because charge resides on outer sur-
Thus, by measuring the terminal velocities with the field off face.
and the field on, it is possible to calculate the charge on a
drop. • If one charged body is enclosed by a conducting body and
Millikan made the necessary measurements on a number of connected to it, the charge will be transferred to the outer
drops and calculated the charge on each drop. He showed that surface of the conductor.
all the drops had charges that are integer multiples of a fun-
damental unit of charge (e), i.e., • Induced charge q 0 = q 1 − 1

K

q = ne n = 0, −1, −2, −3, . . . For any metal (less or more conducting K = ∞

In his initial results, published in 1910 , he reported a value of ∴ If two metals (one less and other more conducting) are
1.63 × 10−19 C for the fundamental charge. After modifying his placed in the same electric field then induced charge on
equipment and measuring a larger number of drops, he pub- both of them will be same and it is: q 0 = q
lished a revised value of 1.59 × 10−19 C in 1913 . The currently
accepted value for the fundamental (or elementary) charge, e, • Photon can never has charge because charge cannot exist
is approximately 1.602 × 10−19 C. without rest mass.

• Quarks (charge q = ± 32 e, ± 13 e) have not been found to


2.13 Important Points exist in free state

Note: Attraction b/w two Similar Charged body: • As in ucm there is an acceleration therefore in ucm a
Two similar point charges can never attract each other how- charged particle can radiate energy.
ever great be the difference b/w the charges but two extended
charged body may attract with great difference of charges b/w 2x2
them
Above we see when the first charge is very very large compared
to 2nd we find the +ve charges are displaced at the opposite 2.13.1 Check Point 4
end and negative charges are concentrated on the end nearer
to the 1st charge. 1. •• The arc AB with the center C and the infinitely long
As –ve charges come nearer due to electrostatic induction there wire having linear charge density λ are lying in the same
occurs attraction b/w the charges. plane. The minimum amount of work to be expended
• Frankline (i.e., esu of charge) is the smallest unit of charge to move a point charge q0 from point A to B through a
while faraday is largest. circular path AB of radius a is equal to:
(A) 2πε
q0
ln 23 (B) 2πε
q0 λ
ln 32
1 0 0
1 coulomb = 3 × 109 esu of charge = emu of charge (C) 2πε
q0 λ
ln 23 (D) √q2πε

10 0 0

Practical units of charge are


amp × hr (= 3600 coulomb) and f araday (96500 coulomb)
The esu of charge is also called static coulomb (stat. coulomb)
or frankline (fr) and is related to emu of charge through the
relation
emu of charge
= 3 × 1010 = c
esu of charge
Figure 2.78
Difference b/w Charge and Mass
No doubt both charge and mass are the basic scalar properties
of matter, charge differs from mass in the following respects:
1. In SI units, charge is a derived physical quantity (q = It) 2. • • • Consider a system of three charges 3q , 3q and − 2q 3
while mass fundamental. placed at points A, B and C, respectively, as shown in the
2. Charge cannot exist without mass but mass can exist with- figure. Take O to be the centre of the circle of radius R
out charge. and angle CAB = 60◦ .
3. Charge is of two types, viz., positive and negative, but mass (A) (A) The electric field at point O is 8πq0 R2 directed
is only of one type (positive) along the negative x-axis
4. Charge is conserved but mass does not. (B) (B) The potential energy of the system is zero.
5. Accelerated charge radiates energy but mass does not. (C) (C) The magnitude of the force between the charge
q2
7. Though both charge and mass are quantized, the quantum C and B is 54π 0R
2

of charge is e while that of mass is not yet clear. (D) The potential at point O is 12π q
0R
.
2.14. IMAGE METHOD 229

y (or the charged body), induces an opposite charge on


the surface of the conductor which gets distributed non
B uniformly over it’s surface. Calculation of induced charge
• density at any point on the surface of the conductor is
very difficult. As a result of which, calculating electric
potential or field at any point in space over the conductor,
is also very difficult because we don’t know the exact
C• x induced charge distribution over the surface of conductor.
O In such cases, we use image method. Let us consider this
60◦ method by using a simple example of a point charge q
near an infinite plane conductor(Fig.2.80a).
The idea of this method lies in that we must find another

A
Figure 2.79

Answer Key and Solutions


1. (B) E = λ
2πε0 x
(a) (b) (c)
VB 2a
λ dx
Z Z
dV = −E dx = − Figure 2.80
V _A 2πε0 3a x

λ 3 problem which can be easily solved and whose solution


⇒ VB − VA = ln or a part of it can be used in our problem. In our case
2πε0 2
such a simple problem is the problem about two charges:
Therefore, the work done by external agent = q0 λ 3
2πε0 ln 2 . q and −q.
The field of this system is well known. It’s equipotential
2. (C) The charges at A, B, and C are qA = q/3, qB = q/3,
surfaces are shown by dashed lines and field lines are
and qC = −2q/3. The electric fields at O due to qA and qB
shown by solid lines in Fig.2.80b.
are equal in magnitude but opposite in direction. Thus,
Let us make the conducting plane coincide with the mid-
the resultant electric field at O is only due to charge qC
dle equipotential surface (its potential V = 0 ) and remove
and is given by
~O = − q the charge −q. In this case, the field and potential in
E ı̂ the upper half-space will remain unchanged3 (Fig.2.80c).
6π0 R2
Electric potential V = 0 on the conducting plane and
The triangle ABC is right-angled with 6 A = 60 ◦ 6
√ , C = everywhere at infinity.
90 , and rAB = 2R. Thus, rAC = R and rBC = 3R. The

It should be noted that we can arrive at this conclusion
potential energy for the given charge distribution is
proceeding from the properties of a closed conducting
shell, since both half-spaces separated by the conducting
 
1 qA qB qA qC qB qC
U= + + plane are electrically independent of one another, and the
4π0 rAB rAC rBC
 2 2 removal of the charge −q will not affect the field in the
2q 2

1 q 2q
= − − √ 6= 0 upper half-space.
4π0 18R 9R 9 3R Thus, in the case under consideration the field differs
The magnitude of force between qC and qB is from zero only in the upper half-space. In order to
calculate this field, it is sufficient to introduce a fictitious
1 qB qC q2 image charge q 0 = −q, opposite in sign to the charge q,
FBC = 2 = by placing it on the other side of the conducting plane at
4π0 rBC 54π0 R2
the same distance as the distance from q to the plane.
The potential at O is The fictitious charge q 0 creates in the upper half-space
the same field as that of the charges induced on the
1 plane. This is precisely what is meant when we say that
V = (qA /R + qB /R + qC /R) = 0
4π0 the fictitious charge produces the same “effect” as all
the induced charges. We must only bear in mind that
2.14 Image Method the “effect” of the fictitious charge extends only to the
half-space where the real charge q is located. In another
Sometimes we have problems in which a point charge (or half-space the field is absent.
a charged body), is placed at a certain distance from a 3 It can be shown by uniqueness theorem which is a topic of graduation

plane conductor of infinite dimensions. This point charge physics and out of the scope of this book
230 CHAPTER 2. ELECTRIC POTENTIAL

Summing up, we can say that the image method is EXAMPLE 130. A point charge q is at a distance l from
essentially based on the idea to find another problem an infinite conducting plane. Find the density of surface
(configuration of charges) in which the field configuration charges induced on the plane as a function of the distance r
in the region of space, we are interested in, is the same. from the base of the perpendicular dropped from the charge
q onto the plane.

Procedure

To calculate electric field or potential at any point in


the space above a given conducting surface, first find
the image position of the given charge by considering
conducting surface as a reflecting mirror. Now introduce
a fictitious image charge q 0 = −q, opposite in sign to the
charge q, at this image position. Apply the superposition
principle and find the net field or potential due to both
the charges– source charge and it’s image charge. Figure 2.82
Note that, under the combined effect of both the charges
(source and it’s image charge) the potential at every APPROACH The relation between surface charge
point on the surface of the conductor will always be zero. density σ and electric field near the surface of an infinite
So, the necessary condition for this method is that the conductor (in vacuum) is given by-
surface of the conductor must be equipotential and its σ
E=
potential must be zero. Every surface of infinite or very ε0
Therefore, σ = ε0 E
large dimensions always satisfies this condition. Let us
Now, find net E at a point P which is at a distance r
consider some examples based on this method-
from the point O(Fig.2.82), by applying image method
and then calculate σ by using the above relation σ = ε0 E.
EXAMPLE 129. A point charge q is placed between two SOLUTION The image of charge q at distance l above
mutually perpendicular half-planes (Fig. 2.81a). Find the the surface is −q at the perpendicular distance l from the
location of fictitious point charges whose action on the plane conductor(Fig.2.82). Both the point charges are
charge q is equivalent to the action of all charges induced equidistant ‘x’ from point P therefore, the magnitudes of
on these half planes. electric fields at P due to the charges q and −q will be
same and it is Eq = 4πεq0 x2 . Therefore, the net electric
field at P -
q l
E = 2Eq cos α = 2 2 x
4πε
 0 x 
l l
∵ cos α = = √
x l2 + r 2
ql
Hence, σ = ε0 E = − 3/2
2π (l + r2 )
2
here the minus sign indicates that the induced charge is
(a) (b)
opposite to sign to the point charge q.

Figure 2.81
EXAMPLE 131. A thin conducting ring of radius R,
having a charge q, is arranged so that it is parallel to an
SOLUTION In this case, we find all image positions infinite conducting plane at a distance l from it. Find- (a)
of point charge q in two planes inclined at right angles. the surface charge density at a point of the plane, which
Figure2.81b shows all three image positions. Images 1 is symmetric with respect to the ring. (b) the electric field
and 2 are the direct images of the charge q, therefore potential at the centre of the ring.
each of them have opposite charge −q. The image 3 is
the superposed image of images 1 and 2 therefore it has
charge opposite to that of images 1 and 2 i.e., +q. Note APPROACH It can be easily seen that in accordance
that the net potential due to all the charges (sources with the image method, a fictitious charge −q must be
and images) at every point on the surfaces of both the located on a similar ring but symmetrically on the other
conductor planes will be zero. So, these three fictitious side of the conducting plane (Fig.2.83). Indeed, only in
charges create just the same field within the “right angle” this case the potential of the midplane between these
as the field of the charges induced on the conducting rings is equal to zero, i.e. it coincides with the potential
planes. of the conducting plane. Let us now use the formulas we
already know.
2.14. IMAGE METHOD 231

Figure 2.84

help of the image method. Integrating this equation over


Figure 2.83 x = l to x = ∞, we find Z

q2 dx q2
W =− 2
=−
In order to find σ at the point O, we must, according 16πε0 l x 16πε0 l
Remark. An attempt to solve this problem in a different
to Enet = σ/ε0 , find the field E at this point (Fig.2.83).
way (through potential) leads to an inaccurate result
From Eq.(1.79), the electric field E on the axis of a
which differs from what was obtained by us by a factor of
charged ring of radius R at distance, x = l, is-
kql ql two. This is because the relation W = q (V1 − V2 ) is valid
E= 2 = only for conservative fields. However, in the reference
(l + R2 )3/2 4πε0 (l2 + R2 )3/2
Since, the image ring has charge −q, therefore both rings system fixed to the conducting plane, the electric field of
will produce electric field in the same direction. So net induced charges is not a conservative field: a displacement
electric Enet field at O (Fig.2.83) will be double of the of the charge q leads to a redistribution of the induced
field produced by a single ring. charges, and their field turns out to be time-dependent.
The net electric field potential at the centre of the ring
will be equal to the sum of potentials, at the centre of the EXAMPLE 133. Three unlike point charges are
ring, produced by given ring and the image ring. arranged as shown in Fig.2.85a, where AOB is the
From Eq.(2.42), the electric potential at any axial position right angle formed by two conducting half-planes. The
x = l of a charged ring of radius a = R having charge q, magnitude of each of the charges is |q| and the distances
is given by - between them are shown in the figure. Find (a) the total
charge induced on the conducting half-planes and (b) the
1 q force acting on the charge −q.
or V = p (2.68)
4πε0 (R + l2 )
2

The center of given ring is at distance, 2l from the center


of image ring. The net potential at the center of the
source ring will be equal to scalar sum of potentials due
to source and it’s image ring.
ql
SOLUTION Enet = 2E =
2πε0 (l + R2 )3/2
2
Therefore, surface charge density at point O-
ql
σ = Enet ε0 = 2π(l2 +R 2 )3/2

(b) The potential at the center of the ring is equal to the


algebraic sum of the potentials at this point created by
the charges q and −q: 
1 q q (a) (b)
V = −√
4πε0 R R2 + 4l2
Figure 2.85
EXAMPLE 132. A point charge q is at a distance l
from an infinite conducting plane [Fig.2.84]. Find the APPROACH The half-planes forming the angle AOB
work of the electric force acting on the charge q done go to infinity, and hence their potential V = 0. It can be
upon its slow removal to a very large distance from the easily seen that a system having equipotential surfaces
plane. with V = 0 coinciding with the conducting half-planes
has the form shown in Fig.2.85b. So, we extrapolate both
the planes to get all the images of given charges q, −q
SOLUTION By definition, the work of this force done and q at points 1, 2 and 3 respectively. All the images
upon an elementary displacement dx (Fig.2.84) is given formed at same position 4 in the lower left corner of the
by dashed square. Each charge q at 1 and 3 produces their
q2 image of charge −q at point 4 whereas −q at point 2
dW = Fx dx = − dx,
4πε0 (2x)2 produces q at point 4. Therefore, net image charge-
where the expression for the force is obtained with the Σqimage = −q − q + q = −q
232 CHAPTER 2. ELECTRIC POTENTIAL

Hence the action of the charges induced on the conducting 12. State clearly the difference (a) between electric potential
half-planes is equivalent to the action of the fictitious and electric field, (b) between electric potential and electric
charge −q placed in the lower left corner of the dashed potential energy.
square.
SOLUTION (a) The net induced charge is −q. 13. Figure 2.86 shows a point particle that has a positive charge
(b) By reducing the system to four point charges, we can +Q and a metal sphere that has a charge −Q. Sketch the
easily find the required force electric field lines and equipotential surfaces for this system
√(see Fig.2.85b) of charges.
2 2 − 1 q2
F = F2 − F1 =
4πε0 2a2

2.15 Questions and Exercises


2.15.1 Conceptual Questions
1. Distinguish between electric potential and electric potential
energy. Figure 2.86

2. A negative charge moves in the direction of a uniform elec-


tric field. Does the potential energy of the charge–field 14. Can a particle ever move from a region of low electric po-
system increase or decrease? Does the charge move to a tential to one of high potential and yet have its electric
position of higher or lower potential? potential energy decrease? Explain.

3. A proton is moved to the left in a uniform electric field 15. Figure 2.87 shows a point particle and metal sphere. Both
that points to the right. Is the proton moving in the di- have equal charge +Q. Sketch the electric field lines and
rection of increasing or decreasing electric potential? Is the equipotential surfaces for this system of charges.
electrostatic potential energy of the proton increasing or
decreasing?

4. An electron is moved to the left in a uniform electric field


that points to the right. Is the electron moving in the di-
rection of increasing or decreasing electric potential? Is the
electrostatic potential energy of the electron increasing or
decreasing?
Figure 2.87
5. If the electric potential is uniform throughout a region of
space, what can be said about the electric field in that re-
gion? 16. Two equal positive point charges are separated by a finite
distance. Sketch the electric field lines and the equipotential
6. What would happen to you if you were on an insulated surfaces for this system.
stand and your potential was increased by 10 kV with re-
spect to the Earth? 17. Two point charges are fixed on the x-axis. (a) Each has
a positive charge q. One is at x = −a and the other is
7. Why is the electron-volt often a more convenient unit of at x = +a. At the origin, which of the following is true?

→ −

energy than the joule? (A) E = 0 and V = 0, (B) E = 0 and V = 2kq/a, (C)

→ −

E = 2kq/a2 î and V = 0, (D) E = 2kq/a2 î and
 
8. Can there be a potential difference between two conductors V = 2kq/a,
that carry like charges of the same magnitude? (b) One point charge has a positive charge +q and the other
9. If V is known at only a single point in space, can E be ~ has a negative charge −q. The positive point charge is at
found at that point? Explain your answer. x = −a and the negative point charge is at x = +a. At


the origin, which of the following is true? (A) E = 0 and
10. If two points are at the same potential, does this mean that −→ −→
V = 0, (B) E = 0 and V = 2kq/a, (C) E = 2kq/a2 î

no work is done in moving a test charge from one point to −

and V = 0, (D) E = 2kq/a2 î and V = 2kq/a,

the other? Does this imply that no force must be exerted?
Explain. 18. A uniform electric field is parallel to the x axis. In what
direction can a charge be displaced in this field without any
11. If a negative charge is initially at rest in an electric field, external work being done on the charge?
will it move toward a region of higher potential or lower
potential? What about a positive charge? How does the 19. The electrostatic potential (in volts) is given by
potential energy of the charge change in each instance? V (x, y, z) = 4.00|x|+V0 , where V0 is a constant, and
2.15. QUESTIONS AND EXERCISES 233

x is in meters. (a) Sketch the electric field for this po- 28. Consider a metal conductor in the shape of a football. If it
tential. (b) Which of the following charge distributions is carries a total charge Q, where would you expect the charge
most likely responsible for this potential: (A) A negatively density σ to be greatest, at the ends or along the flatter
charged flat sheet in the x = 0 plane, sides? Explain. [Hint: Near the surface of a conductor,
(B) a point charge at the origin, E = σ/0 .]
(C) a positively charged flat sheet in the x = 0 plane, −

(D) a uniformly charged sphere centered at the origin? 29. If you know V at a point in space, can you calculate E at


Explain your answer. that point? If you know E at a point can you calculate
V at that point? If not, what else must be known in each
case?
20. The electric potential is the same everywhere on the
surface of a conductor. Does this mean that the surface 30. A conducting sphere carries a charge Q and a second iden-
charge density is also the same everywhere on the surface? tical conducting sphere is neutral. The two are initially
Explain your answer. isolated, but then they are placed in contact. (a) What
can you say about the potential of each when they are in
contact? (b) Will charge flow from one to the other? If so,

→ −

21. If V = 0 at a point in space, must E = 0? If E = 0 at some how much? (c) If the spheres do not have the same radius,
point, must V = 0 at that point? Explain. Give examples how are your answers to parts (a) and (b) altered?
for each.
31. At a particular location, the electric field points due north.
22. When dealing with practical devices, we often take the In what direction(s) will the rate of change of potential be
ground (the Earth) to be 0 V. (a) If instead we said the (a) greatest, (b) least, and (c) zero?
ground was −10 V, how would this affect V and E at other 32. Why is it important, when soldering connectors onto a piece
points? (b) Does the fact that the Earth carries a net charge of electronic circuitry, to leave no pointy protrusions from
affect the choice of V at its surface? the solder joints?
23. Explain why electric field lines are always perpendicular to 33. Equipotential lines are spaced 1.00 V apart. Does the dis-
equipotential surfaces. tance between the lines in different regions of space tell you
−→
anything about the relative strengths of E in those regions?
24. Can two equipotential lines cross? Explain.
If so, what?
25. What can you say about the electric field in a region of 34. If the electric field − →
E is uniform in a region, what can you
space that has the same potential throughout? infer about the electric potential V ? If V is uniform in a


26. A satellite orbits the Earth along a gravitational equipo- region of space, what can you infer about E ?
tential line. What shape must the orbit be? 35. Is the electric potential energy of two unlike charges positive
or negative? What about two like charges? What is the
27. Suppose the charged ring of Example 8 of "Electric Po-
significance of the sign of the potential energy in each case?
tential" was not uniformly charged, so that the density of
charge was twice as great near the top as near the bottom. 36. Fig.2.89 shows the x-component of E ~ as a function of x.
Assuming the total charge Q is unchanged, would this affect Draw a graph of V versus x in this same region of space.
the potential at point P on the axis (Fig. 2.88)? Would it Let V = 0 V at x = 0 m and include an appropriate vertical


affect the value of E at that point? Is there a discrepancy scale.
here? Explain.

dq

(x 2
+ R2 1
R )2
P
x

Figure 2.89

Figure 2.88: Calculating the potential at point P, a distance x


from the center of a uniform ring of charge. 37. Fig. shows the electric potential as a function of x. Draw
a graph of Ex versus x in this same region of space.
234 CHAPTER 2. ELECTRIC POTENTIAL

than, or equal to the electric potential at point b ? Ex-


plain.
b. Rank in order, from largest to smallest, the magnitudes
of the potential differences ∆Vab , ∆Vcd , and ∆Vef .
c. Is surface 1 an equipotential surface? What about sur-
face 2 ? Explain why or why not.

Figure 2.90

~1
38. For each contour map in 2.91, estimate the electric fields E
~ ~
and E2 at points 1 and 2 . Don’t forget that E is a vector.

Figure 2.93

41. Fig.2.94 shows a negatively charged electroscope. The gold


leaf stands away from the rigid metal post. Is the electric
potential of the leaf higher than, lower than, or equal to the
(a)
potential of the post? Explain.

(b)

Figure 2.91

Figure 2.94
39. An electron is released from rest at x = 2 m in the potential
shown in Fig.2.92. Does it move? If so, to the left or to the
right? Explain. 42. The two metal spheres in Fig.2.95 are connected by a metal
wire with a switch in the middle. Initially the switch is
open. Sphere 1, with the larger radius, is given a positive
charge. Sphere 2, with the smaller radius, is neutral. Then
the switch is closed. Afterward, sphere 1 has charge Q1 , is
at potential V1 , and the electric field strength at its surface
is E1 . The values for sphere 2 are Q2 , V2 , and E2
a. Is V1 larger than, smaller than, or equal to V2 ? Explain.
b. Is Q1 larger than, smaller than, or equal to Q2 ? Explain.
c. Is E1 larger than, smaller than, or equal to E2 ? Explain.

43. Fig.2.96 shows a graph of Ex vs x. What is the potential


Figure 2.92 difference between xi = 1.0 m and xf = 3.0 m ?

44. Fig.2.97 is a graph of Ex vs x. The potential at the origin


is −50 V. What is the potential at x = 3.0 m?
40. Fig.2.93 shows an electric field diagram. Dashed lines 1 and
2 are two surfaces in space, not physical objects. 45. What are the magnitude and direction of the electric field
a. Is the electric potential at point a higher than, lower at the dot in Fig.2.98?
2.15. QUESTIONS AND EXERCISES 235

1 cm

1 cm

200 V
458 0V x
Figure 2.95
2200 V
Ex (V/m) Figure 2.99
200

100

0 x (m)
1 2 3

2100
(a)
Figure 2.96
V (V)

50

0 x (cm)
1 2 3
250

(b)

Figure 2.100

Figure 2.97 48. The electric potential in a region of uniform electric field
is −1000 V at x = −1.0 m and +1000 V at x = +1.0 m.
What is Ex ?

49. The electric potential along the x-axis is V = 100x2 V,


where x is in meters. What is Ex at (a) x = 0 m and (b)
x = 1 m?

50. The electric potential along the x-axis is V = 100e−2x V,


where x is in meters. What is Ex at (a) x = 1.0 m and (b)
x = 2.0 m?
Figure 2.98
2.15.2 Problems
46. What are the magnitude and direction of the electric field In all the problems in this section, assume that the
at the dot in Fig.2.99? electric potential is zero at distances far from all
charges unless otherwise stated.
47. Fig.2.100 show V versusx graph. Draw the corresponding
graph of Ex versus x.
Potential Differences in a Uniform Electric Field
236 CHAPTER 2. ELECTRIC POTENTIAL

1. •• A uniform electric field of magnitude 250 V/m is directed 7. ••The difference in potential between the accelerating
in the positive x direction. A +12.0 − µC charge moves plates in the electron gun of a TV picture tube is about
from the origin to the point (x, y) = (20.0 cm, 50.0 cm). (a) 25000 V. If the distance between these plates is 1.50 cm,
What is the change in the potential energy of the charge- what is the magnitude of the uniform electric field in this
field system? (b) Through what potential difference does region?
the charge move?
8. •• Suppose an electron is released from rest in a uni-
2. • An infinite nonconducting sheet has a surface charge den- form electric field whose magnitude is 5.90 × 103 V/m. (a)
sity σ = 0.10µC/m2 on one side. How far apart are equipo- Through what potential difference will it have passed after
tential surfaces whose potentials differ by 50 V? moving 1.00 cm ? (b) How fast will the electron be moving
after it has traveled 1.00 cm?
3. •• When an electron moves from A to B along an electric
field line in Fig. 2.101, the electric field does 3.94×10−19 J 9. •• A uniform electric field of magnitude 325 V/m is directed
of work on it. What are the electric potential differences in the negative y direction in Figure 2.103. The coordinates
(a) VB − VA of point A are (−0.200, −0.300)m, and those of point B are
(b) VC − VA , and (0.400, 0.500)m. Calculate the potential difference VB −VA ,
(c) VC − VB ? using the blue path.

Figure 2.101

4. •• The electric field in a region of space has the components


Ey = Ez = 0 and Ex = (4.00 N/C)x. Point A is on the
y axis at y = 3.00 m, and point B is on the x axis at
x = 4.00 m. What is the potential difference VB − VA ?
Figure 2.103
5. •• A graph of the x component of the electric field as a
function of x in a region of space is shown in Fig. 2.102.
The scale of the vertical axis is set by Exs = 20.0 N/C. 10. ••Starting with the definition of work, prove that at ev-
The y and z components of the electric field are zero in this ery point on an equipotential surface the surface must be
region. If the electric potential at the origin is 10 V, (a) perpendicular to the electric field there.
what is the electric potential at x = 2.0 m, (b) what is the
11. ••A block having mass m and charge +Q is connected to
greatest positive value of the electric potential for points on
a spring having constant k. The block lies on a frictionless
the x axis for which 0 ≤ x ≤ 6.0 m, and (c) for what value
horizontal track, and the system is immersed in a uniform
of x is the electric potential zero?
electric field of magnitude E, directed as shown in Figure
2.104. If the block is released from rest when the spring is
unstretched (at x = 0 ), (a) by what maximum amount does
the spring expand? (b) What is the equilibrium position
of the block? (c) Show that the block’s motion is simple
harmonic, and determine its period. (d) What If? Repeat
part (a) if the coefficient of kinetic friction between block
and surface is µk .
Figure 2.102
12. ••An insulating rod having linear charge density λ =
40.0µC/m and linear mass density µ = 0.100 kg/m is re-
6. •• What are (a) the charge and (b) the charge density on leased from rest in a uniform electric field E = 100 V/m
the surface of a conducting sphere of radius 0.15 m whose directed perpendicular to the rod (Fig. 2.105). (a) Deter-
potential is 200 V (with V = 0 at infinity)? mine the speed of the rod after it has traveled 2.00 m. (b)
2.15. QUESTIONS AND EXERCISES 237

14. •(a) Find the potential at a distance of 1.00 cm from a


proton. (b) What is the potential difference between two
points that are 1.00 cm and 2.00 cm from a proton? (c)
What If? Repeat parts (a) and (b) for an electron.

15. •• Consider a particle with charge q = 1.0µC, point A at


distance d1 = 2.0 m from q, and point B at distance d2 =
1.0 m. (a) If A and B are diametrically opposite each other,
Figure 2.104 as in Fig. 2.107a, what is the electric potential difference
VA − VB ? (b) What is that electric potential difference if A
and B are located as in Fig. 2.107b?

(a)

(b)

Figure 2.107
Figure 2.105

16. •• A spherical drop of water carrying a charge of 30pC has


What If? How does your answer to part (a) change if the a potential of 500 V at its surface (with V = 0 at infinity).
electric field is not perpendicular to the rod? Explain. (a) What is the radius of the drop? (b) If two such drops
of the same charge and radius combine to form a single
13. •• A particle having charge q = +2.00µC and mass m =
spherical drop, what is the potential at the surface of the
0.0100 kg is connected to a string that is L = 1.50 m long
new drop?
and is tied to the pivot point P in Figure P25.14. The
particle, string and pivot point all lie on a frictionless hor- 17. •• Figure 2.108 shows a rectangular array of charged par-
izontal table. The particle is released from rest when the ticles fixed in place, with distance a = 39.0 cm and the
string makes an angle θ = 60.0◦ with a uniform electric charges shown as integer multiples of q1 = 3.40pC and
field of magnitude E = 300 V/m. Determine the speed of q2 = 6.00pC. With V = 0 atinfinity, what is the net electric
the particle when the string is parallel to the electric field potential at the rectangle’s center? (Hint: Thoughtful ex-
(point a in Fig. 2.106). amination of the arrangement can reduce the calculation.)

Figure 2.106: Top View


Figure 2.108

Electric Potential and Potential Energy Due to 18. •• In Fig.2.109, what is the net electric potential at point
Point Charges P due to the four particles if V = 0 at infinity, q = 5.00fC,
and d = 4.00 cm?
238 CHAPTER 2. ELECTRIC POTENTIAL

23. •• A charge +q is at the origin. A charge −2q is at x =


2.00 m on the x axis. For what finite value(s) of x is (a)
the electric field zero? (b) the electric potential zero?

24. •• The three charges in Figure 2.112 are at the vertices of


an isosceles triangle. Calculate the electric potential at the
midpoint of the base, taking q = 7.00µC.

Figure 2.109

19. •• In Fig. 2.110, particles with the charges q1 = +5e


and q2 = −15e are fixed in place with a separation of
d = 24.0 cm. With electric potential defined to be V = 0
at infinity, what are the finite (a) positive and (b) negative
values of x at which the net electric potential on the x axis
is zero? Figure 2.112

25. ••Two point charges, Q1 = +5.00nC and Q2 = −3.00nC,


are separated by 35.0 cm. (a) What is the potential energy
of the pair? What is the significance of the algebraic sign of
your answer? (b) What is the electric potential at a point
midway between the charges?
Figure 2.110 26. ••Show that the amount of work required to assemble four
identical point charges of magnitude Q at the corners of a
square of side s is 5.41ke Q2 /s
20. •• Two particles, of charges q1 and q2 , are separated by
distance d in Fig. 2.110. The net electric field due to the 27. • • • Two insulating spheres have radii 0.300 cm and
particles is zero at x = d/4. With V = 0 at infinity, locate 0.500 cm, masses 0.100 kg and 0.700 kg, and uniformly
(in terms of d ) any point on the x axis (other than at infin- distributed charges of −2.00µC and 3.00µC. They are
ity) at which the electric potential due to the two particles released from rest when their centers are separated by
is zero. 1.00 m. (a) How fast will each be moving when they
collide? (Suggestion: consider conservation of energy and
21. •• Given two 2.00 − µC charges, as shown in Figure P25.16, of linear momentum.) (b) What If? If the spheres were
and a positive test charge q = 1.28 × 10−18 C at the origin, conductors, would the speeds be greater or less than those
(a) what is the net force exerted by the two 2.00 − µC calculated in part (a)? Explain.
charges on the test charge q? (b) What is the electric field
at the origin due to the two 2.00 − µC charges? (c) What is
28. • • • Review problem. Two insulating spheres have radii
the electric potential at the origin due to the two 2.00 − µC
r1 and r2 , masses m1 and m2 , and uniformly distributed
charges?
charges −q1 and q2 . They are released from rest when their
centers are separated by a distance d. (a) How fast is each
moving when they collide? (Suggestion: consider conser-
vation of energy and conservation of linear momentum.)
(b) What If? If the spheres were conductors, would their
speeds be greater or less than those calculated in part (a)?
Figure 2.111 Explain.

22. ••At a certain distance from a point charge, the magnitude 29. •• Two particles, with charges of 20.0nC and −20.0nC,
of the electric field is 500 V/m and the electric potential are placed at the points with coordinates (0, 4.00 cm) and
is −3.00kV. (a) What is the distance to the charge? (b) (0, −4.00 cm), as shown in Figure P25.28. A particle
What is the magnitude of the charge? with charge 10.0nC is located at the origin. (a) Find
the electric potential energy of the configuration of the
2.15. QUESTIONS AND EXERCISES 239

three fixed charges. (b) A fourth particle, with a mass of


2.00 × 10−13 kg and a charge of 40.0nC, is released from
rest at the point (3.00 cm, 0). Find its speed after it has
moved freely to a very large distance away.

Figure 2.114

of 2.00 × 107 m/s directly toward the nucleus (charge +79e


). How close does the alpha particle get to the nucleus
before turning around? Assume the gold nucleus remains
Figure 2.113
stationary.

34. •• An electron starts from rest 3.00 cm from the center of a


30. •• A light unstressed spring has length d. Two identical
uniformly charged insulating sphere of radius 2.00 cm and
particles, each with charge q, are connected to the oppo-
total charge 1.00nC. What is the speed of the electron when
site ends of the spring. The particles are held stationary a
it reaches the surface of the sphere?
distance d apart and then released at the same time. The
system then oscillates on a horizontal frictionless table. The 35. •• Calculate the energy required to assemble the array of
spring has a bit of internal kinetic friction, so the oscilla- charges shown in Figure 2.115, where a = 0.200 m, b =
tion is damped. The particles eventually stop vibrating 0.400 m, and q = 6.00µC.
when the distance between them is 3d. Find the increase
in internal energy that appears in the spring during the os-
cillations. Assume that the system of the spring and two
charges is isolated.

31. •• Two point charges of equal magnitude are located along


the y axis equal distances above and below the x axis, as
shown in Figure P25.30. (a) Plot a graph of the potential
at points along the x axis over the interval −3a < x < 3a.
You should plot the potential in units of ke Q/a. (b) Let
the charge located at −a be negative and plot the potential
along the y axis over the interval −4a < y < 4a

32. •• A small spherical object carries a charge of 8.00nC. At Figure 2.115


what distance from the center of the object is the poten-
tial equal to 100 V? 50.0 V? 25.0 V? Is the spacing of the 36. •• Four identical particles each have charge q and mass m.
equipotentials proportional to the change in potential? They are released from rest at the vertices of a square of
side L. How fast is each charge moving when their distance
33. ••• In 1911 Ernest Rutherford and his assistants Geiger and
from the center of the square doubles?
Marsden conducted an experiment in which they scattered
alpha particles from thin sheets of gold. An alpha particle, 37. •• How much work is required to assemble eight identical
having charge +2e and mass 6.64 × 10−27 kg, is a product point charges, each of magnitude q, at the corners of a cube
of certain radioactive decays. The results of the experiment of side s ?
led Rutherford to the idea that most of the mass of an atom
is in a very small nucleus, with electrons in orbit around it- 38. •• A positive point charge +Q is located on the x axis
his planetary model of the atom. Assume an alpha particle, at x = −a. (a) How much work is required to bring an
initially very far from a gold nucleus, is fired with a velocity identical point charge from infinity to the point on the x
240 CHAPTER 2. ELECTRIC POTENTIAL

axis at x = +a ? (b) With the two identical point charges Determine (a) the potential at x = 0, 3.00 m, and 6.00 m,
in place at x = −a and x = +a, how much work is required and (b) the magnitude and direction of the electric field at
to bring a third point charge −Q from infinity to the origin? x = 0, 3.00 m, and 6.00 m
(c) How much work is required to move the charge −Q from
the origin to the point on the x axis at x = 2a along the 46. •• The electric potential inside a charged spherical conduc-
semicircular path shown (Figure 2.116)? tor of radius R is given by V = ke Q/R, and the potential
outside is given by V = ke Q/r. Using Er = −dV /dr, de-
rive the electric field (a) inside and (b) outside this charge
distribution.

47. •• Over a certain region of space, the electric potential is


V = 5x−3x2 y+2yz 2 . Find the expressions for the x, y and z
components of the electric field over this region. What is the
magnitude of the field at the point P that has coordinates
(1, 0, −2)m?

48. •• Figure 2.117 shows several equipotential lines each la-


Figure 2.116 beled by its potential in volts. The distance between the
lines of the square grid represents 1.00 cm. (a) Is the mag-
nitude of the field larger at A or at B ? Why? (b) What is
39. •• A charge of +2.00nC is uniformly distributed on a ring of
E at B? (c) Represent what the field looks like by drawing
radius 10.0 cm that lies in the x = 0 plane and is centered
at least eight field lines.
at the origin. A point charge of +1.00nC is initially located
on the x axis at x = 50.0 cm. Find the work required to
move the point charge to the origin.

40. •• A dipole of moment 0.50e · nm is placed in a uniform


electric field that has a magnitude of 4.0 × 104 N/C. What
is the magnitude of the torque on the dipole when (a) the
dipole is aligned with the electric field, (b) the dipole is
transverse to (perpendicular to) the electric field, and (c)
the direction of dipole makes an angle of 30◦ with the di-
rection of electric field? (d) Defining the potential energy
to be zero when the dipole is transverse to the electric field,
find the potential energy of the dipole for the orientations
specified in Parts (a) and (c).

Obtaining the Value of the Electric Field from the


Electric Potential

41. •• How much work is done (by a battery, generator, or some Figure 2.117
other source of potential difference) in moving Avogadro’s
number of electrons from an initial point where the electric
potential is 9.00 V to a point where the potential is - 5.00 V
Electric Potential Due to Continuous Charge Dis-
? (The potential in each case is measured relative to a
tributions
common reference point.)

42. •• An ion accelerated through a potential difference of 49. •• Consider a ring of radius R with the total charge Q
115 V experiences an increase in kinetic energy of 7.37 × spread uniformly over its perimeter. What is the potential
10 −17
J. Calculate the charge on the ion. difference between the point at the center of the ring and a
point on its axis a distance 2R from the center?
43. ••(a) Calculate the speed of a proton that is accelerated
from rest through a potential difference of 120 V. (b) Cal- 50. •• In Fig. 2.118, a plastic rod having a uniformly dis-
culate the speed of an electron that is accelerated through tributed charge Q = −25.6pC has been bent into a circular
the same potential difference. arc of radius R = 3.71 cm and central angle φ = 120◦ . With
V = 0 at infinity, what is the electric potential at P , the
44. ••What potential difference is needed to stop an electron center of curvature of the rod?
having an initial speed of 4.20 × 105 m/s ?
51. •• A plastic rod has been bent into a circle of radius
45. •• The potential in a region between x = 0 and x = 6.00 m R = 8.20 cm. It has a charge Q1 = +4.20pC uniformly dis-
is V = a + bx, where a = 10.0 V and b = −7.00 V/m. tributed along one-quarter of its circumference and a charge
2.15. QUESTIONS AND EXERCISES 241

53. •• A rod of length L (Fig.2.121) lies along the x axis with


its left end at the origin. It has a nonuniform charge density
λ = αx, where α is a positive constant. (a) What are the
units of α ? (b) Calculate the electric potential at A.

Figure 2.118

Q2 = −6Q1 uniformly distributed along the rest of the cir- Figure 2.121
cumference (Fig. 2.119). With V = 0 at infinity, what is
the electric potential at (a) the center C of the circle and
(b) point P , on the central axis of the circle at distance 54. •• For the arrangement described in the previous problem,
D = 6.71 cm from the center? calculate the electric potential at point B, which lies on the
perpendicular bisector of the rod a distance b above the x
axis.
P
55. Calculate the electric potential at point P on the axis of the
annulus shown in Figure 2.122, which has a uniform charge
D density σ.
Q2

R C
b
Q1 a
Figure 2.119 P
x
52. •• In Fig. 2.120, three thin plastic rods form quarter-
circles with a common center of curvature at the origin.
The uniform charges on the three rods are Q1 = +30nC,
Q2 = +3.0Q1 , and Q3 = −8.0Q1 What is the net elec-
Figure 2.122
tric potential at the origin due to the rods?

56. •• A wire having a uniform linear charge density λ is bent


into the shape shown in Figure 2.123. Find the electric
potential at point O

R
2R 2R
O

Figure 2.120 Figure 2.123


242 CHAPTER 2. ELECTRIC POTENTIAL

Electric Potential Due to a Charged Conductor charge leaks off in sparks, by producing dielectric break-
57. •• How many electrons should be removed from an initially down of the surrounding dry air, as shown in Figure 2.124.
uncharged spherical conductor of radius 0.300 m to produce Determine (a) the charge on the dome and (b) the radius
a potential of 7.50kV at the surface? of the dome.

58. •• A spherical conductor has a radius of 14.0 cm and charge 62. •• The Bohr model of the hydrogen atom states that
of 26.0µ. Calculate the electric field and the electric poten- the single electron can exist only in certain allowed orbits
tial (a) r = 10.0 cm, (b) r = 20.0 cm, and (c) r = 14.0 cm around the proton. The radius of each Bohr orbit is r =
from the center. n2 (0.0529 nm) where n = 1, 2, 3, . . . . Calculate the electric
potential energy of a hydrogen atom when the electron (a)
59. ••• Electric charge can accumulate on an airplane in flight. is in the first allowed orbit, with n = 1, ( b) is in the second
You may have observed needle-shaped metal extensions on allowed orbit, n = 2, and (c) has escaped from the atom,
the wing tips and tail of an airplane. Their purpose is with r = ∞. Express your answers in electron volts.
to allow charge to leak off before much of it accumulates.
The electric field around the needle is much larger than 63. •• An electron is released from rest on the axis of a uniform
the field around the body of the airplane, and can become positively charged ring, 0.100 m from the ring’s center. If
large enough to produce dielectric breakdown of the air, the linear charge density of the ring is +0.100µC/m and
discharging the airplane. To model this process, assume the radius of the ring is 0.200 m, how fast will the electron
that two charged spherical conductors are connected by a be moving when it reaches the center of the ring?
long conducting wire, and a charge of 1.20µC is placed on
the combination. One sphere, representing the body of the 64. •• Four balls, each with mass m, are connected by four non-
airplane, has a radius of 6.00 cm, and the other, represent- conducting strings to form a square with side a, as shown
ing the tip of the needle, has a radius of 2.00 cm. (a) What in Figure 2.125. The assembly is placed on a horizontal
is the electric potential of each sphere? (b) What is the nonconducting frictionless surface. Balls 1 and 2 each have
electric field at the surface of each sphere? charge q, and balls 3 and 4 are uncharged. Find the maxi-
mum speed of balls 1 and 2 after the string connecting them
Applications of Electrostatics is cut.

60. • • • Lightning can be studied with a Van de Graaff gen-


erator, essentially consisting of a spherical dome on which
charge is continuously deposited by a moving belt. Charge
can be added until the electric field at the surface of the
dome becomes equal to the dielectric strength of air. Any
more charge leaks off in sparks, as shown in Figure 2.124.
Assume the dome has a diameter of 30.0 cm and is sur-
rounded by dry air with dielectric strength 3.00 × 106 V/m.
(a) What is the maximum potential of the dome? (b) What
is the maximum charge on the dome?

Figure 2.125

Figure 2.124
65. •• A disk of radius R (Fig.2.126) has a nonuniform sur-
face charge density σ = Cr, where C is a constant and r
61. •• The spherical dome of a Van de Graaff generator can be is measured from the center of the disk. Find (by direct
raised to a maximum potential of 600kV; then additional integration) the potential at P .
2.15. QUESTIONS AND EXERCISES 243

R O
P

x q q
Figure 2.127

5. 1 e.s.u. of potential is equal to-


Figure 2.126
(A) 1/300 volt (B) 8 × 1010 volt
(C) 300 volt (D) 3 volt
66. •• A gold nucleus is 100fm 1fm = 10−15 m from a proton, 6. The earth’s surface is considered to be at -


which initially is at rest. When the proton is released, it (A) Zero potential (B) Negative Potential
speeds away because of the repulsion that it experiences (C) Infinite Potential (D) Positive Potential
due to the charge on the gold nucleus. What is the proton’s 7. Electric potential is a -
speed a large distance (assume to be infinity) from the gold (A) Vector quantity (B) Scalar quantity
nucleus? (Assume the gold nucleus remains stationary.) (C) Neither vector Nor scalar (D) Fictious quantity
8. The electric potential V at any point (x, y, z) in space is
67. ••During a famous experiment in 1919, Ernest Rutherford given by V = 4x2 volt. The electric field E in V/m at the
shot doubly ionized helium nuclei (also known as alpha par- point (1, 0, 2) is
ticles) at a gold foil. He discovered that virtually all of (A) +8 in x direction (B) 8 in −x direction
the mass of an atom resides in an extremely compact nu- (C) 16 in +x direction (D) 16 in −x direction
cleus. Suppose that during such an experiment, an alpha
9. ABC is equilateral triangle of side 1m. Charges are placed
particle far from the foil has an initial kinetic energy of 5.0
at its corners as shown in fig. O is the mid- point of side
MeV.|Ifthe alpha particle is aimed directly at the gold nu-
BC the potential at point (O) is-
cleus, and the only force acting on it is the electric force of
repulsion exerted on it by the gold nucleus, how close will
it approach the gold nucleus before turning back? That
A
is, what is the minimum center-to-center separation of the
alpha particle and the gold nucleus?

Multiple Choice Questions


Level 1
Potential and potential difference B C
O
1. When charge of 3 coulomb is placed in a Uniform electric Figure 2.128
field, it experiences a force of 3000 newton, within this
field, potential difference between two points separated (A) 2.7 × 103 V (B) 1.52 × 105 V
by a distance of 1 cm is- (C) 1.3 × 10 V
3
(D) −1.52 × 105 V
(A) 10 V (B) 90 V (C) 1000 V (D) 3000 V 10. In a region where E = 0, the potential (V ) varies with
distance r as-
2. A uniform electric field having a magnitude E0 and (A) V α 1r (B) ∨α. r
direction along positive x-axis exists. If the electric po- (C) Vα r12 (D) V = const. independent of (r)
tential (V) is zero at x = 0 then its value at x = +x will be-
11. Charges of + 3 × 10−9 are placed at each of the four
10

(A) Vx = xE0 (B) Vx = −x · E0
(C) Vx = x2 E0 (D) Vx = x2 E0 corners of a square of side 8 cm. The potential at the
intersection
√ of the diagonals is
3. The dimensions of potential difference are - √
(A) ML2 T−2 Q−1 (B) MLT−2 Q−1 (A) 150√2 V (B) 1500 2 V
(C) MT Q −2 −2
(D) M L2 T −1 Q−1 (C) 900 2 V (D) 900 V
4. Three equal charges are placed at the three cor- 12. An equipotential surface is that surface-
ners of an isosceles triangle as shown in the figure. (A) On which each and every point has the same po-
The statement which is true for electric potential V tential
and the field intensity E at the centre of the triangle is- (B) Which has negative potential
(A) V = 0, E = 0 (B) V = 0, E 6= 0 (C) Which has positive potential
(C) V 6= 0, E = 0 (D) ∨ =6 0, E 6= 0 (D) Which has zero potential
244 CHAPTER 2. ELECTRIC POTENTIAL

13. The surface of a conductor - 20. If a unit charge is taken from one point to another over
(A) is a non-equipotential surface an equipotential surface then-
(B) has all the points at the same potential (A) Work is done on the charge
(C) has different points at different potential (B) Work is done by the charge
(D) has at least two points at the same potential (C) Work on the charge is constant
14. The electron potential (V) as a function of distance (x) (D) No work is done
[in meters] is given by V = 5x2 + 10x − 9 Volt. The 21. In an electric field the work done in moving a unit
value of electric field at x = 1 m would be- positive charge between two points is the measures of-
(A) 20Volt/m (B) 6Volt/m (A) Resistance
(C) 11Volt/m (D) −23Volt/m (B) Potential difference
15. Some equipotential lines are as shown is Fig.2.129 E1 , E2 (C) Intensity of electric field
and E3 are the electric fields at points 1,2 and 3 then - (D) Capacitance
(A) E1 = E2 = E3 (B) E1 > E2 > E3 22. State which one of the following is correct?
(A) Joule = Coulomb × Volt
(C) E1 > E2 , E2 < E3 (D) E1 < E2 < E3
(B) Joule = Coulomb / Volt
(C) Joule = Volt / Ampere
(D) Joule = Volt × Ampere
23. One electron volt (eV ) of energy is equal to -
(A) 1.6 × 10−12 ergs (B) 4.8 × 10−10 ergs.
(C) 9 × 10 ergs.
11
(D) 3 × 109 ergs.
24. The K.E. in electron Volt gained by an α− particle when
it moves from rest at point where its potential is 70 to a
point where potential is 50 volts, is -
(A) 20 eV (B) 20 MeV.
(C) 40 eV (D) 40 MeV.
25. An α - particle moves towards a nucleus at rest, if
kinetic energy of α-particle is 10MeV and atomic number
Figure 2.129 of nucleus is 50 . The distance of closest approach will be -
(A) 1.44 × 10−14 m (B) 2.88 × 10−14 m
16. Three charges 2q, −q, −q are located at the vertices of an (C) 1.44 × 10 −10
m (D) 2.88 × 10−10 m
equilateral triangle. At the circum center of the triangle. 26. A point charge (q) moves form point (P ) to point
(A) The field is zero but potential is not zero. (S) along the path PQRS as shown in Fig. 2.131 in
a uniform electric field E, ~ pointing co-parallel to the
(B) The field is non-zero but the potential is zero.
(C) Both, field and potential are zero. positive direction of the x-axis. The coordinates of
(D) Both, field and potential are non- zero the points P, Q, R and S are (a, b, 0), (2a, 0, 0), (a, −b, 0)
Electric potential energy and work done and (0, 0, 0) respectively. The work done by the
field in the above process is given by the expression
17. A point positive charge of Q ’ units is moved round (A) qEq (B) −qE a
√ i
(C) q E a 2 (D) qE [(2a)2 + b2
p
another point positive charge of Q units in circular path.
If the radius of the circle r is the work done on the charge
Q ’ in making one complete revolution i -
QQ0
(A) 4π∈Q
0r
(B) 4π∈ 0r
0
(C) 4π∈
Q
0r
(D) 0
18. A proton is projected with velocity 7.45 × 105 m/s to-
wards an another proton which is at rest. The minimum
approach is-
(A) 10−12 m (B) 10−14 m
(C) 10 −10
m (D) 10−8 m
19. Three charges are placed as shown in fig if the elec- Figure 2.131
tric potential energy of system is zero, then Q : q -
(A) Q −2
(B) Q 2 27. Two identical thin rings, each of radius R metres, are
q = 1 q = 1
coaxially placed at a distance (R) metres apart. If Q1
(C) q = 2
Q −1
(D) q = · 14
Q
coul and Q2 coul are respectively The charges unifor-
maly spread on the two rings. The work done in moving
a charge (q) from the centre of one ring to that of other

is -
(A) zero (B) q(Q1√−Q 2 )( 2−1)

√ ( 2·4ε0 πR
√ )
(C) q (4ε
2(Q1 +Q2 )
(D) q(Q1 +Q2 )( 2+1)

0 πR) ( 2·4ε0 πR)
Figure 2.130
Applications of Gauss’s Law
2.15. QUESTIONS AND EXERCISES 245

28. A solid conducting sphere having a charge Q is surrounded


by an uncharged concentric conducting hollow spherical
shell Let the potential difference between the surface of 1 2
the solid sphere and that of the outer surface of the hollow
1 2
shell be V . If the shell is now given a charge of 3Q the B
1 2
new potential difference between the same two surfaces is
1 2
(A) V (B) 2 V
(C) 4 V (D) ˘2 V 1 2
1 2
29. The dependence of electric potential V on the distance ’r’ 1 A 2
from the center of a charged spherical shell is shown by. 1 2
1 2
1 2
1 2
1 2

(A) (B) Figure 2.133

potential gradient at this point will be- (in M.K.S.)


(A) 5.71 × (B) 3.71 × (C) 18.81 × (D) 1.881 ×
105 105 105 105
(C) (D) 35. The electric flux coming out of the equi-potential surface
is-
Figure 2.132 (A) perpendicular to the surface
(B) parallel to the surface
Electric dipole (C) in all directions
(D) zero
30. The electric potential at a point due to an electric dipole
will be- 36. A charge Q is distributed over two concentric hollow
(A) k p~r·~
r
3 (B) k p~r·~
r
2
spheres of radii (r) and (R) > (r) such the surface
(C) k(~pr×~r) (D) k(~pr×~2
r) densities are equal. Find the potential at the common
31. An electric dipole of dipole moment p is aligned parallel centre- 2
Q(R2 +r )
to a uniform electric field E. The energy required to (A) 4πεQ
× (r+R)
2 (B)
(R+r) 4πε0 (r+R)
rotate the dipole by 90◦ is:
0
Q(r+R)
(C) (D) none of these
(A) p2 E (B) pE (C) ∞ (D) pE 2 4πε0 (R2 +r 2 )

37. The earth had a net charge equivalent to 1 electron /m2


32. An electric dipole of moment p is placed in an electric
of surface area of radius 6.4 × 106 m. Its potential would
field of intensity E. The dipole acquires a position
be-
such that the axis of the dipole makes an angle θ with
(A) +0.12 V (B) −0.12 V (C) +1.2 V (D) −1.2 V
direction of the field. Assuming that the potential
energy of the dipole to be zero when θ = 90◦ , the torque
38. The electric potential at the surface of an atomic nucleus
and the potential energy of the dipole will respectively be:
(Z = 50) of radius 9 × 10−15 m is -
(A) pE sin θ, −pE cos θ (B) pE sin θ, −2pE cos θ
(A) 80 V (B) 8 × 106 V
(C) pE sin θ, 2pE cos θ (D) pE cos θ, −pE sin θ
Miscellaneous (C) 8 × 10 V
4
(D) 8 × 102 V
39. A hollow metal sphere of radius 5 cm is charged such that
33. A proton is first placed at A and then at B between the the potential on its surface is 10 V. The potential at the
two plates of a parallel plate capacitor charged to a poten- centre of the sphere is -
tial difference of V volt as shown in Fig.2.133. The force (A) 0 V
on proton at A is- (B) 10 V
(A) more than at B (C) Same as at point 5 cm away from the surface
(B) less than at B (D) Same as at point 25 cm away from the surface
(C) equal to that at B Level 2
(D) nothing can be said
1. Electric potential in an electric field is given as V = K/r,
34. In electric field, a 6.75µC charge experiences 2.5 N force, (K being constant), if position vector − →r = 2î + 3ĵ + 6k̂
when placed at distance of 5 m from the origin. Then then electric field will be
246 CHAPTER 2. ELECTRIC POTENTIAL

(A) (2î+3ĵ+6k̂)K
(B) (2î+3ĵ+6k̂)K 13. An electric dipole is placed along the x-axis at the origin
243 343
(C) (3î+2ĵ+6k̂)K
(D) (6î+2ĵ+3k̂)K O. A point P is at a distance of 20 cm from th is origin
243
2. At any point (x,
343
0, 0) the electric potential V is such that OP makes an angle π3 with the x-axis. If the
1000 1500 500
volt, then electric field at x = 1 m electric field at P makes an angle θ with the x-axis. the

x + x2 + x3
value of θ would be
(A) 5500(ĵ + k̂)V/m (B) 5500îV/m √ 
(A) π3 (B) π3 + tan−1 23
(C) 5500
√ (ĵ + k̂)V/m
2
(D) 5500
√ (î + k̂)V/m
2
√ 
3. Potential difference between centre and the surface of a (C) 2π3 (D) tan −1
2
3

sphere of radius R with uniform charge density σ with in 14. An electric dipole of moment p̄ is placed normal to the
it will be2 2
lines of force of electric intensity Ē, then the work done
(A) σR
6ε0 (B) σR4ε0 in deflecting it through an angle of 180◦ is
2
(C) zero (D) σR2∈0
(A) pE (B) +2pE (C) −2pE (D) Zero
4. Two conducting spheres of radii r1 and r2 are equally
15. An electric dipole of moment P placed in a uniform
charged. The ratio of their potential is-
electric field Ē has minimum potential energy when the
(A) r12 /r22 (B) r22 /r12
angle between p̄ and Ē is-
(C) r1 /r2 (D) r2 /r1
(A) Zero (B) π2 (C) π (D) 3π
5. Two similar rings P and Q ( radius = 0.1mt ) are placed 2
co-axially at a distance 0.5 m apart . The charge on
16. An electric dipole has the magnitude of its charge as q
P and Q is 2µC and 4µC respectively. Work done in
and its dipole moment is p. It is placed in a uniform
moving a 5µC charge from center of P to the center of Q is-
electric field E. If its dipole moment is along the direction
(A) 1.28 J (B) 0.72 J (C) 0.144 J (D) 1.44 J
of the field, the force on it and its potential energy are
6. Three point charge −q, +q and −q are placed along a respectively-
straight line at equl distances( say r meter) Electric (A) 2q.E and minimum
potential energy of this system of charges will be if +q (B) q.E and p.E
charge is in the middle- (C) Zero and minimum
−3q 2
(A) 4π −8q 2
(B) 3π (D) q.E and maximum
0r 0r
2 2
−3q −q
(C) 8π 0r
(D) 8π 0r
Level 3
7. Four equal charges of charge q are placed at corner of 1. A conducting sphere of radius R is charged to a potential
a square of side a. Potential energy of the whole system is- of V volt. Then the electric field at a distance r(> R)
from the centre of the sphere would be -
 
4kq2 4kq2
(A) a (B) a 1 + 2 2 √1
2

1 kq
2
kq 2
  (A) RV (B) RrV
(C) Vr (D) Rr3V
(C) 2√ (D) r2
1 2

2 a a 4 + √
2 2
8. The potential of a charged drop is v. This is divided into 2. The variation of electric potential with dis-
n smaller drops, then each drop will have the potential as ; tance from a fixed point is shown in Fig.2.134.
(A) n v
−1
(B) n v
2/3 What is the value of electric field atx = 2m -
(C) n3/2 v (D) n−2/3 v (A) 0 (B) 6/2 (C) 6/1 (D) 6/3
9. 8 small droplets of water of same size and same charge
form a large spherical drop. The potential of the large
drop, in comparison to potential of a small drop will be -
(A) 2 times (B) 4 times
(C) 8 times (D) same
10. In Millikan’s oil drop experiment an oil drop carrying
a charge Q is held stationary by a potential differ-
ence 2400 V between the plates. To keep a drop of
half the radius stationary the potential difference had
to be made 600 V. What is the charge on the second drop-
(A) Q
4 (B) Q2 (C) Q (D) 3Q
2
Figure 2.134
11. There is an electric field E in X-direction. If the work
done on moving a charge 0.2C through a distance of 2 m 3. A positive point charge q is carried from a point B to a
along a line making an angle 60◦ with the X-axis is 4.0, point A in the electric field of a point charge +Q at origin
what √
is the value of E O. If the permittivity of free space is ε0 , the work done
(A) 3 N/C (B) 4 N/C in the process is given by (where a = OA and b = OB  )
(C) 5 N/C (D) None of these (A) 4πε
qQ 1 1
(B) qQ 1 1

a + b  4πε0 a − b 
12. A ball of mass 1 g and charge 10−3 C moves from a point qQ
(C) 4πε
0
1
− 1
(D) qQ 1
+ 1
2 2 2 b2
A. where potential is 800 volt to the point B where 0 a b 4πε0 a
4. Two conducting spheres each of radius R carry charge q.
potential is zero. Velocity of the ball at the point B is They are placed at a distance r from each other, where
20 cm/s. The velocity of the ball at the point A will be- r > 2R. The neutral point lies at a distance r/2 from
(A) 22.8 cm/s (B) 228 cm/s either sphere. If the electric field at the neutral point due
(C) 16.8 cm/s (D) 168 m/s
2.15. QUESTIONS AND EXERCISES 247

to either sphere be E, then the total electric potential at (A) −6î+8ĵ (B) 6î − 8ĵ (C) î + ĵ (D) Zero
that point will be -
(A) rE/2 (B) rE (C) RE/2 (D) RE 13. A hollow conducting sphere of radius R has charge (+Q)
on its surface. The electric potential within the sphere at
5. A ring of radius R carries a charge +q. √ A test charge
a distance r = R3 from the centre is -
−q0 is released on its axis at a distance 3R from its
center. How much kinetic energy will be acquired by the (A) 0 (B) 4πε
1 Q
0 r
(C) 4πε
1 Q
0 R
(D) 4πε
1 Q
0 r
2

test charge when it reaches the center of the ring -


14. A particle has a mass 400 times than that of the electron
(A) 4πε
1 qq0
(B) 4πε 1 qq0
0 R 0 2R and charge is double than that of a electron. It is
(C) 4πε
1 √ qq0
(D) 1 qq0
0 3R 4πε 0 3R accelerated by 5 V of potential difference. Initially the
6. Two spheres of radii r1 and r2 are at the same potentials.
particle was at rest. Then its final kinetic energy will be -
If their surface densities of charges be σ1 and σ2 respec-
(A) 5 eV (B) 10 eV (C) 100 eV (D) 2000 eV
tively, then σ1 /σ2 −
2 2
(A) r1 /r2 (B) r2 /r1 (C) (r1 /r2 ) (D) (r2 /r1 ) 15. Two equal positive charges are kept at points A and B.
7. A proton and an electron are released infinite distance The electric potential at the points between A and B
apart and the attracted towards each other. Which of the (excluding these points) is studied while moving from A
following statement about their kinetic energy is true - to B. The potential-
(A) Kinetic energy of electron is more than that of (A) Continuously increases
proton (B) Continuously decreases
(B) Kinetic energy of electron is less than that of pro- (C) Increases then decreases
ton (D) Decreases then increases
(C) Kinetic energy of electron = kinetic energy of pro- Passage Type Question :- In the diagram (given
ton below) the broken lines represent the paths followed
(D) None of the above is true as it depending on the by particles W, X, Y and Z respectively through the
distance between the particles constant field E. The numbers below the field represents
8. Two conducting spheres of radii r1 and r2 are charged meters.
such that they have the same electric field on their sur-
faces.pThe ratio of the electric potential at their centers is -
(A) r1 /r2 (B) r1 /r2
(C) r12 /r22 (D) None of the above
9. A charge +Q at A(See figure) produces electric field
E and electric potential V at D. If we now put
charges −2Q and +Q at B and C respectively,
then the electric field and potential at D will be -
(A) √
E and 0 (B) 0 and V
(C) 2E and √V2 (D) √E2 and √V2

Figure 2.136
Figure 2.135

10. If a positive charge is shifted from a low-potential region


to a high-potential region, the electric potential energy - 16. If the particles begin and end at rest, and all are positively
(A) increases charged, the same amount of work was done on which
(B) decreases particles?
(C) remains the same (A) W and Z (B) W, Y and Z
(D) may increase or decrease (C) Y and Z (D) W, X, Y and Z
11. A particle of mass 0.002 kg and a charge 1µC is held at
17. If the particles started from rest and all are positively
rest on a frictionless horizontal surface at a distance of 1 m
charge which particles must have been acted upon by a
from a fixed charge of 1mC. If the particle is released, it
force other than that produced by the electric field.
will be repelled. The speed of the particle when it is at a
(A) W and Y (B) X and Z
distance of 10 m from the fixed charge is -
(C) X, Y and Z (D) W, X, Y and Z
(A) 60 ms−1 (B) 75 ms−1 (C) 90 ms−1 (D) 100 ms−1
12. Electric potential is given by: V = 6x − 8xy 2 − 8y + 6yz − 18. If the particles are positively charged, which particles in-
4z 2 Electric field at the origin is - creased their electrical potential energy -
248 CHAPTER 2. ELECTRIC POTENTIAL

(A) X and Z (A) 4πε2Q 2q


− 4πε 2Q
(B) 4πε + 4πεq0 R
0R 0R 0R
(B) Y and Z (C) (q+Q) 2 2Q
(D) 4πε
4πε0 R 0R
(C) W, X, Y and Z 3. A charged particle ’ q ’ is shot towards another charged
(D) Since the electric field is constant none of the par- particle ’ Q ’, which is fixed, with a speed v. It approaches
ticles increased their electrical potential energy. Q upto a closest distance r and then returns. If q were
given a speed of ’ 2v 0 , the closest distances of approach
19. Five equal and similar charges are placed at the corners of
would be - [AIEEE-2004]
a regular hexagon as shown in the Fig.2.137. What is the (A) r (B) 2r (C) r/2 (D) r/4
electric field and potential at the center of the hexagon -
(A) 4πε
5 q
, 5 q2 (B) 4πε
1 q
, 5 q2 4. Two thin wire rings each having a radius R are placed at
0 ` 4πε0 ` 0 ` 4πε0 t
(C) 4πε0 `2 , 4πε0 `
1 q 5 q
(D) 4πε0 ` , 4πε
1 q 1 q
2
a distance d apart with their axes coinciding. The charges
0 `
on the two rings are +q and −q. The potential differ-
ence between the centres of the two rings is- [AIEEE-2005]
h i
(A) QR/4π ∈0 d2 (B) 2π∈Q
0 hR
1
− √R21+d2
i
(C) zero (D) 4π∈Q
0
1
R − √ 1
2
R +d 2

5. Two insulating plates are both uniformly charged in


such a way that the potential difference between them is
V2 − V1 = 20 V. (i.e. plate 2 is at a higher potential).
Figure 2.137
The plates are separated by d = 0.1 m and can be treated
as infinitely large. An electron is relaeased from rest on
Statements Type Question:-
the inner surface of plate 1 . What is its speed when it hits
Each of the questions given below consist of Statement -
plate 2? e = 1.6 × 10−19 C, me = 9.11 × 10−31 kg −
I and Statement - II. Use the following Key to choose the
[AIEEE 2006]
appropriate answer.
(A) 1.87 × 106 m/s (B) 32 × 10−19 m/s
(A) If both Statement-I and Statement-II are true,
(C) 2.65 × 106 m/s (D) 7.02 × 1012 m/s
and Statement - II is the correct explanation of
Statement- I.
(B) If both Statement −I and Statement −II are true
but Statement - II is not the correct explanation
of Statement −I.
(C) If Statement −I is true but Statement −II is false.
(D) If Statement −1 is false but Statement −II is true.
20. Statement I: If a point charge q is placed in front of an
infinite grounded conducting plane surface, the point
charge will experience a force.
Statement II : The force is due to the induced charge on Figure 2.138
the conducting surface which is at zero potential.
6. Two spherical conductors A and B of radii 1 mm and
21. Statement I: Electrons move away from a region of lower 2 mm are separated by a distance of 5 cm and are
potential to a region of higher potential. uniformly charged. If the sphere are connected by a
Statement II: Since an e− has negative charge. conducting wire then in equilibrium condition, the ratio
of the magnitude of the electric fields at the surfaces of
22. Statement I: Work done in moving a charge between any spheres A and B is - [AIEEE 2007]
two points in an electric field is independent of the path (A) 2 : 1 (B) 1 : 4 (C) 4 : 1 (D) 1 : 2
followed by the charge, between these points.
Statement II: Electrostatic forces are non conservative. 7. An electric charge 10−3 µC is placed at the origin (0, 0)
of X − Y co-ordinate
√ √ system. Two points A and B are
situated at ( 2, 2) and (2, 0) respectively. The potential
Level 4 (Previous Years Questions)
difference between the points A and B will be- [AIEEE
Section A: JEE MAINS 2006]
(A) 9 V (B) 0 V (C) 2 V (D) 4.5 V
1. On moving a charge of 20 coulombs by 2 cm, 2J of work
8. Charges are placed on the vertices of a square as shown.
is done, then the potential difference between the points ~ be the electric field and V the potential at the cen-
Let E
is – [AIEEE-2002]
tre. If the charges on A and B are interchanged with those
(A) 0.1 V (B) 8 V (C) 2 V (D) 0.5 V
on D and C respectively, then [AIEEE 2007]
2. A thin spherical conducting shell of radius R has a (A) E~ remains unchanged, V changes
charge q. Another charge Q is placed at the centre (B) Both E ~ and V change
of the shell. The electrostatic potential at a point P a ~
(C) E and V remain unchanged
distance R/2 from the centre of the shell is - [AIEEE 2003] ~ changes, V remains unchanged
(D) E
2.15. QUESTIONS AND EXERCISES 249

Figure 2.140

potential at the origin and VA the potential at x = 2 m


is: [JEE MAIN 2014]
Figure 2.139
(A) −80 J (B) 80 J (C) 120 J (D) −120 J
15. Three concentric metal shells A, B and C of respective
9. The potential at a point x (measured in µm ) due to some radii a, b and c (a < b < c) have surface charge densities
changes situated on the x-axis is given by V (x) = 20 +σ, −σ and +σ respectively. The potential of shell B is
/ x2 − 4 volts. The electric field E at x = 4µm is given [JEE
by [AIEEE 2007] h 2 2 i h MAIN 2018] i
σ a2 −b2
(A) σ0 a −b + c (B) + c
(A) 5/3 Volt /µm and in the -ve x direction a
h 2 2 i 0 h b i
(B) 5/3 Volt /µm and in the +ve x direction σ b2 −c2
(C) σ0 b −c
b + a (D) 0 c + a
(C) 10/9 Volt µm and in the -ve x direction
(D) 10/9 Volt µm and in the +ve x direction 16. A charge Q is distributed over two concentric conducting
10. Two points P and Q are maintained at the potentials of thin spherical shells radii r and R(R > r). If the surface
10 V and −4 V, respectively. The work done in moving charge densities on the two shells are equal, the electric
100 electrons from P to Q is - [AIEEE-2009] potential at the common centre is: [JEE MAIN 2019]
(A) −9.60 × 10−17 J (B) 9.60 × 10−17 J 1 2(R+r)
(A) 4πε0 (R2 +r2 ) Q (B) 4πε1 (R+r)
2 2 Q
0 2(R +r )
(C) −2.24 × 10−16 J (D) 2.24 × 10−16 J (R+2r)Q (R+r)
(C) 4πε0 2(R2 +r2 )
1
(D) 4πε0 2(R2 +r2 ) Q
1
11. The electrostatic potential inside a charged spherical ball
is given by φ = αρ + b where r is the distance from the
2

centre; a, b are constants. Then the charge density inside


ball is [AIEEE 2011]
(A) −6aε0 r (B) −24πaε0 r (C) −6aε0 (D) −24πaε0 r
12. This question has statement 1 and statement 2. Of the
four choices given after the statements, choose the one
that best describes the two statements.
An insulating solid sphere of radius R has a uniformly
positive charge density ρ. As a result of this uniform
charge distribution there is a finite value of electric
potential at the centre of the sphere, at the surface of
the sphere and also at a point outside the sphere. The
electric potential at infinity is zero. [JEE MAIN 2012]
Statement 1: When a charge q is taken from the centre Figure 2.141
to the surface of the sphere, its potential energy changes
by 3εqρ
0 17. A solid conducting sphere, having a charge Q, is sur-
Statement 2: The electric field at a distance r(r < R)
rounded by an uncharged conducting hollow spherical
from the centre of the sphere is 3ε ρr
0 shell. Let the potential difference between the surface
(A) Statement 1 is true, Statement 2 is true, Statement 2
of the solid sphere and that of the outer surface of the
is not the correct explanation for statement 1.
hollow shell be V. If the shell is now given a charge of
(B) Statement 1 is true, Statement 2 is false
−4Q, the new potential difference between the same two
(C) Statement 1 is false, Statement 2 is true
surfaces is: [JEE MAIN 2019]
(D) Statement 1 is true, Statement 2 is the correct
(A) −2V (B) 2V (C) 4V (D) V
explanation for statement 1
18. A positive point charge is released from rest at a distance
ro from a positive line charge with uniform density.
13. A charge Q is uniformly distributed over a long rod AB
The speed (v) of the point charge, as a function of
of length L as shown in the figure. The electric potential
instantaneous distance r from line charge, is proportional
at the point O lying at distance L from the end A is:
to: [JEE r MAIN 2019]
[JEE MAIN 2013]  
3Q
(A) 8πεQ0 L (B) 4πε 0L
(A) v ∝ e +r/ro
(B) v ∝ ln r
ro
Q ln 2
(C) 4πε0 L ln 2
Q
(D) 4πε0 L    
(C) v ∝ ln rro (D) v ∝ rro
14. Assume that an electric field E ~ = 30x2 ı̂ exists in space. 19. The electric field in a region is given by − →
E = (Ax + B)î,
Then the potential difference VA − V0 , where V0 is the where E is in N C −1 and x is in metres. The values of
250 CHAPTER 2. ELECTRIC POTENTIAL

R
  X
A B
Figure 2.144


(A) √R
2+1
(B) √ 2R
√2+1
(C) √R
2−1
(D) √ 2R
2−1

Figure 2.142 25. Four equal point charges Q each are placed in the xy
plane at (0, 2), (4, 2), (4, −2) and (0, −2). The work
required to put a fifth charge Q at the origin of the
constants are A = 20 SI unit and B = 10 SI unit. If the coordinatesystem will be: [JEE MAIN 2019]
potential at x = 1 is V1 and that at x = −5 is V2 , then Q2 Q2

(A) 4πε 1 + √1 (B) 1 + √1
V1 − V2 is: [JEE MAIN 2019] 0
2
3 4πε0 5
Q2
(A) 320V (B) −48V (C) 180V (D) −520V (C) √Q
2 2πε0
(D) 4πε0

20. A system of three charges are placed as shown in the 26. Three charges Q, +q and +q are placed at the vertices
Fig.2.143. If D  d, the potential energy of the system is of a right-angle isosceles triangle as shown below. The
best given by: [JEE MAIN 2019] net electrostatic energy of the configuration is zero, if the
value of Q is: [JEE MAIN 2019]
(A) 1+−q

2
(B) +q


√ 2q
D (C) −2q (D) 2+1
+q  O 
–q

Q
d

Figure 2.143

h 2
i h 2 i
2qQd
(A) 1
4πεo − qd − qQd
2D 2 i (B) 1
4πεo −q + D2 i
h 2
h d2
(C) 1
4πεo + qd + qQd
D2 (D) 1
4πεo − qd − qQd
D2
21. In free space, a particle A of charge 1µC is held fixed at a
point P. Another particle B of the same charge and mass Figure 2.145
4µg is kept at a distance of 1mm from P. If B is released,
then its velocity at a distance of 9mm from P is: [JEE 27. The given graph shows variation (with distance r from
MAIN 2019]  centre) of: [JEE MAIN 2019]
( Take 4πε1
o
= 9 × 10 9
N m2 −2
C (A) Electric field of a uniformly charged sphere
(B) Potential of a uniformly charged spherical shell
(A) 1.0m/s (B) 3.0 × 104 m/s (C) Potential of a uniformly charged sphere
(C) 2.0 × 103 m/s (D) 1.5 × 102 m/s (D) Electric field of a uniformly charged spherical shell
22. A point dipole p~ = −p0 x̂ is kept at the origin. The
28. An electric field of 1000V /m is applied to an electric
potential and electric field due to this dipole on the y-axis
dipole at angle of 45◦ . The value of electric dipole
at a distance d are, respectively: (Take V = 0 at infinity)
moment is 10−29 C − m. What is the potential energy of
[JEE MAIN 2019]
the electric dipole? [JEE MAIN 2019]
(A) 4πε|~p0|d2 , 4πεp~0 d3 (B) 0, 4πε −~p
0d
3
(A) −20 × 10−18 J (B) −7 × 10−27 J
(C) 0, 4πεp~0 d3 (D) 4πε|~p0|d2 , 4πε
−~p
0d
3 (C) −10 × 10−29 J (D) −9 × 10−20 J
23. A charge Q is distributed over three concentric spher-
29. Determine the electric dipole moment of the system of
ical shells of radii a, b, c(a < b < c) such that their
three charges, placed on the vertices of an equilateral
surface charge densities are equal to one another.
triangle, as shown in the figure: [JEE MAIN 2019]
The total potential at a point at distance r from their √ ĵ−î
common centre, where r < a, would be: [JEE MAIN 2019] (A) 3ql 2 √ (B) (ql) î+
√ ĵ
√ 2
Q(a2 +b2 +c2 ) (C) 2qlĵ (D) − 3qlĵ
(A) 12πε
Q ab+bc+ca
(B)
0 abc 4πε0 (a3 +b3 +c3 )
Q(a+b+c) 30. There is a uniform spherically symmetric surface charge
(C) Q
4πε0 (a+b+c) (D) 4πε0 (a2 +b2 +c2 ) density at a distance R0 from the origin. The charge dis-
24. Two electric dipoles, A, B with respective dipole moments tribution is initially at rest and starts expanding because
d~A = −4qaî and d~B = −2qaî are placed on the x-axis of mutual repulsion. The figure that represents best the
with a separation R, as shown in the figure The distance speed v(R(t)) of the distribution as a function of its in-
from A at which both of them produce the same potential stantaneous radius R(t) is: [JEE MAIN 2019]
is: [JEE MAIN 2019]
2.15. QUESTIONS AND EXERCISES 251

32. Two isolated conducting spheres S1 and S2 of radius 23 R


and 13 R have 12µC and −3µC charges, respectively, and
are at a large distance from each other. They are now
connected by a conducting wire. A long time after this is
done the charges on S1 and S2 are respectively:
[JEE MAIN 2020]
(A) 6µC and 3µC (B) 4.5µC on both
(C) +4.5µC and −4.5µC (D) 3µC and 6µC
33. Concentric metallic hollow spheres of radii R and 4R
hold charges Q1 and Q2 respectively. Given that surface
charge densities of the concentric spheres are equal, the
potential difference V (9R) − V (4R) is: [JEE MAIN 2020]
3Q2
(A) 4πε
Q2
0R
(B) 4πε 0R
3Q1 3Q1
(C) 16πε0 R (D) 4πε 0R

34. A two point charges 4q and −q are fixed on the x-axis at


x = −d 2 and x = 2 , respectively. If the third point charge
d

’ q ’ is taken from the origin to x = d along the semicircle


as shown in the figure, the energy of the charge will:
Figure 2.146 [JEE MAIN 2020]
2q 2 2q 2
(A) Increase by 3πε0 d (B) Increase by 4πε 0d
q2 4q 2
(C) decrease by 4πε0 d (D) decrease by 3πε0 d

y
>

>
4q –q x

Figure 2.149

35. A particle of charge q and mass m is subjected to an


Figure 2.147 electric field E = E0 1 − ax2 in the x-direction, where a
and E0 are constants. Initially the particle was at rest at
x = 0. Other than the initial position the kinetic energy
(A) u(R[t]) (B) u(R[t])
of the particle becomes zero when the distance of the
particle from the origin
q is: q [JEE MAIN q 2020]
(A) a (B) 1
a (C) 3
a (D) 2
a

R0 R(t) 36. A solid sphere of radius R carries a charge Q+q distributed


R0 R(t)
uniformly over its volume. A very small point like piece
of it of mass m gets detached from the bottom of the
(C) u(R[t]) (D) u(R[t])
sphere and falls down vertically under gravity. This piece
V0
carries charge q. If it acquires a speed v when it has fallen
through a vertical height y (Fig.2.150), then (assume the
remaining portion
h to be spherical) i [JEE MAIN 2020]
Ru R(t) R0 R(t) (A) v = 2y 4πε0 (R+y)3 m + g
2 QqR
h i
(B) v 2 = y 4πε0qQ
R 2 ym + g
h i
Figure 2.148 (C) v 2 = y 4πε0 R(R+y)m
qQ
+g
h i
(D) v 2 = 2y 4πε0 R(R+y)m
qQ
+g
31. An electric field of 1000V /m is applied to an electric 37. Ten charges are placed on the circumference of a circle
dipole at angle of 45◦ . The value of electric dipole of radius R with constant angular separation between
moment is 10−29 C · m. What is the potential energy of successive charges. Alternate charges 1,3, 5, 7, 9 have
the electric dipole? [11 Jan 2019, (II)] charge (+q) each, while 2, 4, 6, 8, 10 have charge (−q)
(A) −20 × 10−18 J (B) −10 × 10−29 J each. The potential V and the electric field E at the
(C) −7 × 10−27 J (D) −9 × 10−20 J centre of the circle are respectively: [Take V = 0 at
252 CHAPTER 2. ELECTRIC POTENTIAL

being center of the ring) in volts is [1994]


(A) +2 (B) −1 (C) −2 (D) 0
4. A charge +q is fixed at each of the point x = x0 , x = 3x0 ,
x = 5x0 , . . . ad inf. on the x-axis, and charges −q is fixed
at each of the point x = 2x0 , x = 4x0 ,x = 6x0 , . . . ad inf.
Here x0 is a positive constant. Take the electric potential
at a point due to a charge Q at a distance r from it to be
Q/(4πε0 r). Then the potential at the origin due to the
above system of charges is [1998]
Figure 2.150
(A) -1 (B) 8πε0 xq 0 /n2
q ln 2
(C) ∞ (D) 4πε 0 x0
5. An ellipsoidal cavity is carved within a perfect conductor.
infinity] [JEE MAIN 2020] A positive charge q is placed at the centre of the cavity
(A) V = 0; E = 0 10q
(B) V = 4πε ;E = 0 The points A and B are on the cavity surface as shown in
0R
10q
(C) V = 4πε ;E = 10q
(D) V = 0, E = 4πε10q the figure. Then [1999]
0R 4πε0 R2 0R
2
(A) Electric field near A in the cavity = electric field
38. Two identical electrical point dipoles have dipole moments near B in the cavity
p~1 = pî and p~2 = −pî and held on the x-axis at distance (B) Charge density at A = charge density at B
’ a ’ from each other. When released, they move along (C) Potential at A 6= potential at B
the x-axis with the direction of their dipole moments (D) Total electric field flux through the surface of the
remaining unchanged. If the mass of each dipole is ’ m0 , cavity is q/ε0 .
their speed when they are infinitely far apart is:
q [JEE
q MAIN 2020]
(A) P 3
(B) P 1
a
q 2πε0 ma a
q 2πε0 ma
(C) P
a
1
πε0 ma (D) P
a
2
πε0 ma

39. Consider two charged metallic spheres S1 and S2 of radii


R1 and R2 , respectively. The electric fields E1 (on S1 )
and E2 ( on S2 ) on their surfaces are such that EE2 = R 2 .
1 R1

V1 ( on S1 )
Then the ratio V2 ( on S2 ) of the electrostatic potentials
on each sphere is: [JEE MAIN 2020]
 3    2 Figure 2.151
(A) R2R1
(B) R1R2
(C) R2
R1
(D) R 1
R2

40. The two thin coaxial rings, each of radius ’ a ’ and 6. Three charges Q, +q and +q are placed at the vertices of
having charges +Q and −Q respectively are separated by a right- angled isosceles triangle as shown in fig. The net
a distance of ’s’. The potential difference between the electrostatics energy of the configuration is zero if Q is
centres of hthe two ring iis: [JEE hMAIN 2021]i equal to [2000]
(A) 1+−q
√ (B) −2q
√ (C) −2q (D) + q
(A) 4πε0 a − √s2 +a2
Q 1 1
(B) 2πεQ 1
0 ha
− √s21+a2 2 2+ 2
h i i
(C) 4πε
Q
0
1
a + √ 1
2
s +a2
(D) Q
2πε0 a
1
+ √ 1
2
s +a 2

SECTION - B (JEE Advanced)


1. Two identical thin rings, each of radius R, are coaxially
placed a distance R apart. If Q1 and Q2 are respectively
the charges uniformly spread on the two rings, the work
done in moving a charge q from the centre of one ring to
that of the other is √
[1992]
(A) zero (B) q(Q1√−Q 2 )( 2−1)
24πε R
√ √
0

(C) q 2(Q 1 +Q2 )


4πε0 R (D) q(Q1√/Q 2 )( 2+1)
24πε0 R
2. The electric potential V at any point x, y, z in space is Figure 2.152
given by V = 4x2 volt / meter 2 . The electric field at the
point (1 m, 0, 2 m) is [1992] 7. A uniform electric field pointing in positive x - direction
(A) 8 V/m (B) 4 V/m (C) 16 V/m (D) 4/3 V/m exists in a region. Let A be the origin, B be the point
3. A nonconducting ring of radius 0.5 m carries a total on the x-axis at x = +1 cm and C be the point on the
charge of 1.11 × 10−10 C distributed non-uniformly on its y axis at y = +1 cm. Then the potentials at the points
curcumference producing an electric field E everywhere in A, B, and C satisfy. [2001]
~ ·−
→ (A) ∨A < VB (B) ∨A > VB
R 1=0
space. The value of the line integral 1=∞ −E dl (I = 0
(C) VA < VC (D) VA > VC
2.15. QUESTIONS AND EXERCISES 253

8. Two equal point charges are fixed at x = −a and x = +a charges given to the shells, Q1 : Q2 : Q3 is- [2009]
on the x-axis. Another point charge Q is placed at the (A) 1 : 2 : 3 (B) 1 : 3 : 5 (C) 1 : 4 : 9 (D) 1 : 8 :
origin. The change in the electrical potential energy of 18
Q, when it is displaced by a small distance x along the 13. A spherical metal shell A of radius RA and a solid metal
x-axis, is approximately proportional to - [2002] sphere B of radius RB (< RA ) are kept far apart and
(A) x (B) x2 (C) x3 (D) 1/x each is given charge 0 + Q0 . Now they are connected by a
thin metal wire. Then [2011]
9. Positive and negative point charges of equal magnitude (A) EA inside
=0 (B) Qa >| Qb
are kept at 0, 0, a2 and 0, 0, −a 2 , respectively. The (C) σσab = R Rb
(D) EA
onsuface onsufface
< EB
work done by the electric field when another positive point a

charge is moved from (−a, 0, 0) to (0, a, 0) is [2007] Paragraph 14.-15.


(A) positive A dense collection of equal number of electrons and
(B) negative positive ions is called neutral plasma. Certain solids
(C) zero containing fixed positive ions surrounded by free electrons
(D) depends on the path connecting the initial and can be treated as neutral plasma. Let ’ N0 be the number
final positions density of free electrons, each of mass ’ m ’. When
10. Consider a system of three charges 3q , 3q and − 2q the electrons are subjected to an electric field, they are
3 placed
at point A, B and C, respectively, as shown in the figure. displaced relatively away from the heavy positive ions.
Take O to be the centre of the circle of radius R and angle If the electric field becomes zero, the electrons begin to
CAB = 60◦ . [2008] oscillate about the positive ions with a natural angular
(A) The electric field at point O is 8πεq0 R2 direction frequency ’ ωp ’ which is called the plasma frequency.
along the negative x-axis To sustain the oscillations, a time varying electric field
(B) The potential energy of the system is zero needs to be applied that has an angular frequency ω,
(C) The magnitude of the force between the charges where a part of the energy is absorbed and a part of it is
q2 reflected. As ω approaches ωp all the free electrons are set
at C and B is 54πε 0R
2
to resonance together and all the energy is reflected. This
(D) The potential at point O is 12πεq
0R is the explanation of high reflectivity of metals. [2011]

14. Taking the electronic charge as ‘e’ and the permittivity


as ‘ε0 ’. Use dimensional analysis to determine the correct
expression
q for ωp .
(A) me Ne
(B) me
p 0
Ne
q e
N e2
(C) mε0 (D) N e2
p mε0
15. Estimate the wavelength at which plasma reflection will
occur for a metal having the density of electrons N ≈
4 × 1027 m−3 . Taking e0 = 10−11 and m ≈ 10−30 , where
these quantities are in proper Sl units.
(A) 800 nm (B) 600 nm (C) 300 nm (D) 200 nm
Figure 2.153 16. Two large vertical and parallel metal plates having a
separation of 1 cm are connected to a DC voltage source
of potential difference X. A proton is released at rest
11. STATEMENT 1: For practical purposes the earth is
midway between the two plates. It is found to move at
used as a reference at zero potential in electrical circuits.
45◦ to the vertical just after release. Then X is nearly
STATEMENT 2: The electrical potential of a sphere
[2012]
of radius R with charge Q uniformly distributed on the
(A) 1 × 10−5 V (B) 1 × 10−7 V
surface is given by 4πεQ0 R
(C) 1 × 10 V
−9
(D) 1 × 10−10 V
(A) STATEMENT-1 is True, STATEMENT-2 is True; 17. Consider a thin spherical shell of radius R with centre at
STATEMENT-2 is a correct explanation for the origin, carrying uniform positive surface charge den-
STATEMENT-1 sity. The variation of the magnitude of the electric field
(B) STATEMENT-1 is True, STATEMENT-2 is True; ~
|E(r)| and the electric potential V (r) with the distance
STATEMENT-2 is NOT a correct explanation for r from the centre, is best represented by which graph?
STATEMENT-1 [2012]
(C) STATEMENT-1 is True, STATEMENT-2 is False
(D) STATEMENT-1 is False, STATEMENT-2 is True
12. Three concentric metallic spherical shells of radii 18. Two non-conducting spheres of radii R1 and R2 and
R, 2R, 3R are given charges Q1 , Q2 , Q3 respectively. It carrying uniform volume charge densities +ρ and −ρ,
is found that the surface charge densities on the outer respectively, are placed such that they partially overlap,
surfaces of the shells are equal. Then, the ratio of the as shown in the figure. At all points in the overlapping
254 CHAPTER 2. ELECTRIC POTENTIAL

~
|E| V ~
|E| V

0 R r 0 R r
(A) (B)
~
|E| V ~
|E| V
Figure 2.156

the figure. Charged oil drops of density 900kgm−3 are


0 R r 0 R r released through a tiny hole at the center of the top
(C) (D) disc. Once some oil drops achieve terminal velocity,
the switch is closed to apply a voltage of 200V across
Figure 2.154 the discs. As a result, an oil drop of radius 8 × 10−7 m
stops moving vertically and floats between the discs. The
number of electrons present in this oil drop is ... (neglect
region, [2013]
the buoyancy force, take acceleration due to gravity
= 10ms−2 and charge on an electron (e) = 1.6 × 10−19 C
[2020]

ρ
−ρ

R2
R1

Figure 2.155

(A) The electrostatic field is zero Figure 2.157


(B) The electrostatic potential is constant
(C) The electrostatic field is constant in magnitude
22. A point charge q of mass m is suspended vertically by a
(D) The electrostatic field has same direction
string of length l. A point dipole of dipole moment p~ is
Integer Answer Type now brought towards q from infinity so that the charge
19. A point charge +Q is placed just outside an imaginary
moves away. The final equilibrium position of the system
hemispherical surface of radius R as shown in the figure.
including the direction of the dipole, the angles and dis-
Which of the following statements is/are correct? [2017]
tances is shown in the figure below. If the work done in
(A) The circumference of the flat surface is an equipo-
bringing the dipole to this position is N × (mgh), where
tential
g is the acceleration due to gravity, then the value of N is
(B) The electric flux passing through the curved sur-
 (Note that for three coplanar forces keeping a point mass
face of the hemisphere is − 2Q0 1 − √12 in equilibrium, sin θ is the same for all forces, where F is
F

(C) Total flux through the curved and the flat surfaces any one of the forces and θ is the angle between the other
is Q0 two forces) [2020]
(D) The component of the electric field normal to the
flat surface is constant over the surface. √
Paragraph [23-24] Two point charges −Q and +Q/ 3
20. A particle, of mass 10−3 kg and charge 1.0C, is initially at
are placed in the xy-plane at the origin (0, 0) and a point
rest. At time t = 0, the particle comes under the influence
−→ (2, 0), respectively, as shown in the figure. This results in
of an electric field E (t) = E0 sin ωtî, where E0 = 1.0 an equipotential circle of radius R and potential V = 0
N C −1 and ω = 103 rads−1 . Consider the effect of only in the xy-plane with its center at (b, 0). All lengths are
the electrical force on the particle. Then the maximum measured in meters. [2021]
speed, in ms−1 , attained by the particle at subsequent
times is ... [2018]
23. The value of R is ... meter.
21. Two large circular discs separated by a distance of 0.01m
are connected to a battery via a switch as shown in 24. The value of b is ... meter.
2.16. ANSWER KEYS AND SOLUTIONS 255

Electron energy levels are conveniently quoted in electron-


volts and then nuclear energy levels in MeV show a clear
difference in terms of scale.

Then using eV/c2 with the appropriate prefix as a unit of


mass also becomes convenient; e.g. the mass of the electron
as 500keV/c2 and that of the proton as 1GeV/c2 . It is not
an SI unit.

8. Yes.
Figure 2.158 ~ cannot be determined without knowing V at a con-
9. No. E
tinuum of points.
2.16 Answer Keys and Solutions 10. Not necessarily. If two points are at the same potential,
then no net work is done in moving a charge from one point
2.16.1 Conceptual Questions
to the other, but work (both positive and negative) could
1. When one object B with electric charge is immersed in the be done at different parts of the path. No. It is possible
electric field of another charge or charges A, the system that positive work was done over one part of the path, and
possesses electric potential energy. The energy can be mea- negative work done over another part of the path, so that
sured by seeing how much work the field does on the charge these two contributions to the net work sum to zero. In this
B as it moves to a reference location. We choose not to case, a non-zero force would have to be exerted over both
visualize A0 s effect on B as an action-at-a-distance, but parts of the path.
as the result of a two-step process: Charge A creates elec-
tric potential throughout the surrounding space. Then the 11. The negative charge will move toward a region of higher
potential acts on B to inject the system with energy. potential and the positive charge will move toward a region
of lower potential. In both cases, the potential energy of
2. The potential energy increases. When an outside agent the charge will decrease.
makes it move in the direction of the field, the charge moves
to a region of lower electric potential. Then the product 12. (a) The electric potential is the electric potential energy per
of its negative charge with a lower number of volts gives unit charge. The electric potential is a scalar. The electric
a higher number of joules. Keep in mind that a negative field is the electric force per unit charge, and is a vector.
charge feels an electric force in the opposite direction to the (b) Electric potential is the electric potential energy per
field, while the potential is the work done on the charge to unit charge.
move it in a field per unit charge.
13. The electric field lines, shown as solid lines, and the equipo-
3. The proton is moving to a region of higher potential. The
tential surfaces (intersecting the plane of the paper), shown
proton’s electrostatic potential energy is increasing.
as dashed lines, are sketched in the adjacent figure. The
4. The electron is moving to a region of higher electric po- point charge +Q is the point at the right, and the metal
tential. The electron’s electrostatic potential energy is de- sphere with charge −Q is at the left. Near the two charges
creasing. the equipotential surfaces are spheres, and the field lines
are normal to the metal sphere at the sphere’s surface,
5. If V is constant, its gradient is zero; consequently the elec-
tric field is zero throughout the region.

6. Providing you were totally insulated, nothing would hap-


pen to you. But by 100kV , you might find some tingling
and your hair standing out, depending on how dry the air
around you was.

7. The electron-volt is a convenient unit of energy when


considering electrons moving between points at dif-
ferent potentials. The convenience came from having
numerical values which are around or greater than one,
1eV = 1.6 × 10−19 J.

So one perhaps has a better "feel" for the difference


Figure 2.159
between 1 and 100eV than 1.6 × 10−19 J and 1.6 × 10−17 J
and the value in electron volts is easier to write.
256 CHAPTER 2. ELECTRIC POTENTIAL

14. Yes. If the charge on the particle is negative and it moves 17. APPROACH We can use Coulomb’s law and the super-
from a region of low electric potential to a region of high position of fields to find E~ at the origin and the definition
electric potential, its electric potential energy will decrease. of the electric potential due to a point charge to find V at
the origin.
15. The electric field lines, shown as solid lines, and the equipo- Sol. (a) Apply Coulomb’s law and the superposition of
tential surfaces (intersecting the plane of the paper), shown −

fields to find the electric field E at the origin:
as dashed lines, are sketched in the adjacent figure. The
point charge +Q is the point at the right, and the metal −
→ − → −

E = E +q at −a + E +q at +a
sphere with charge +Q is at the left. Near the two charges
kq kq
the equipotential surfaces are spheres, and the field lines are = 2 î − 2 î = 0
a a
normal to the metal sphere at the sphere’s surface. Very
far from both charges, the equipotential surfaces and field The potential V at the origin is given by:
lines approach those of a point charge 2Q located at the
midpoint. V = V+q at −a + V+qat+a
kq kq 2kq
= + =
a a a
and (B) is correct.
(b) Apply Coulomb’s law and the superposition of fields to


find the electric field E at the origin:

~ =E
E ~ +q at −a + E~ −q at +a
     
kq kq 2kq
= î + î = î
a2 a2 a2

The potential V at the origin is given by:

V = V+q at −a + V−qat+a
kq k(−q)
= + =0
a a
Figure 2.160 and (C) is correct.

18. The charge can be moved along any path parallel to the
y-z plane, namely perpendicular to the field.
16. The equipotential surfaces are shown with dashed lines, the
electric field lines are shown with solid lines. Near each
charge, the equipotential surfaces are spheres centered on 19. APPROACH − →
E = − ∂V ∂x î to find the electric field
each charge; far from the charges, the equipotential surface corresponding to the given potential and then compare
is a sphere centered at the midpoint between the charges. its form to those produced by the four alternatives listed.
The electric field lines are perpendicular to the equipoten- SOLUTION (a) Find the electric field corresponding to
tial surfaces. this potential function:


→ ∂V ∂
E =− î = − [4.00|x|+V0 ] î
∂x ∂x

= −4.00 [|x|]î
∂x
If x > 0, then ∂x

[|x|] = 1 and:


E x>0 = −4.00 m î If x < 0, then
V ∂
= −1 and:

∂x [|x|]
 
~ x<0 = V
E 4.00 î
m

A sketch of the electric field in this region follows: (b) (1)


is correct because field lines end on negative charges.

20. No. The local surface charge density is proportional to the


Figure 2.161
normal component of the electric field, not the potential on
the surface.
2.16. ANSWER KEYS AND SOLUTIONS 257

tric potential is constant. The electric field is the gradient


of the potential; if the potential is constant, the gradient is
zero.

26. The Earth’s gravitational equipotential lines are roughly


circular, so the orbit of the satellite would have to be
roughly circular.

Figure 2.162 27. The potential at point P would be unchanged. Each bit of
positive charge will contribute an amount to the potential
based on its charge and its distance from point P. Moving
21. No. Electric potential is the potential energy per unit charges to different locations on the ring does not change
charge at a point in space and electric field is the electric their distance from P, and hence does not change their con-
force per unit charge at a point in space. If one of these tributions to the potential at P.
quantities is zero, the other is not necessarily zero. For ex- The value of the electric field will change. The electric
ample, the point exactly between two charges with equal field is the vector sum of all the contributions to the field
magnitudes and opposite signs will have a zero electric po- from the individual charges. When the charge Q is dis-
tential because the contributions from the two charges will tributed uniformly about the ring, the y-components of
be equal in magnitude and opposite in sign. (Net electric the field contributions cancel, leaving a net field in the x-
potential is a scalar sum.) This point will not have a zero direction. When the charge is not distributed uniformly,
electric field, however, because the electric field contribu- the y-components will not cancel, and the net field will
tions will be in the same direction (towards the negative have both x - and y-components, and will be larger than
and away from the positive) and so will add. (Net elec- for the case of the uniform charge distribution. There is no
tric field is a vector sum.) As another example, consider discrepancy here, because electric potential is a scalar and
the point exactly between two equal positive point charges. electric field is a vector.
The electric potential will be positive since it is the sum
of two positive numbers, but the electric field will be zero 28. The charge density and the electric field strength will be
since the field contributions from the two charges will be greatest at the pointed ends of the football because the
equal in magnitude but opposite in direction. surface there has a smaller radius of curvature than the
middle.
22. (a) V at other points would be lower by 10 V. E would
be unaffected, since E is the negative gradient of V , and a 29. No. You cannot calculate electric potential knowing only
change in V by a constant value will not change the value electric field at a point and you cannot calculate electric
of the gradient. field knowing only electric potential at a point. As an ex-
(b) If V represents an absolute potential, then yes, the fact ample, consider the uniform field between two charged, con-
that the Earth carries a net charge would affect the value ducting plates. If the potential difference between the plates
of V at the surface. If V represents a potential difference, is known, then the distance between the plates must also be
then no, the net charge on the Earth would not affect the known in order to calculate the field. If the field between
choice of V . the plates is known, then the distance to a point of interest
between the plates must also be known in order to calculate
23. If the field lines are not perpendicular to the surface, then
the potential there. In general, to find V , you must know
there is a component parallel to the surface. If there is an
E and be able to integrate it. To find E, you must know V
electric field component parallel to the surface, then a test
and be able to take its derivative. Thus you need E or V in
charge placed on the surface would move along the field
the region around the point, not just at the point, in order
line.
to be able to find the other variable.
But the surface is an equipotential surface, and there is no
force on a charge with no potential difference. This gives 30. (a) Once the two spheres are placed in contact with each
a contradiction which means the initial assumption of field other, they effectively become one larger conductor. They
lines being not perpendicular to the surface is wrong. will have the same potential because the potential every-
24. No. An equipotential line is a line connecting points of where on a conducting surface is constant.
equal electric potential. If two equipotential lines crossed, (b) Because the spheres are identical in size, an amount of
it would indicate that their intersection point has two dif- charge Q/2 will flow from the initially charged sphere to the
ferent values of electric potential simultaneously, which is initially neutral sphere so that they will have equal charges.
impossible. As an analogy, imagine contour lines on a topo- (c) Even if the spheres do not have the same radius, they
graphic map. They also never cross because one point on will still be at the same potential once they are brought
the surface of the Earth cannot have two different values into contact because they still create one larger conductor.
for elevation above sea level. However, the amount of charge that flows will not be ex-
actly equal to half the total charge. The larger sphere will
25. The electric field is zero in a region of space where the elec- end up with the larger charge.
258 CHAPTER 2. ELECTRIC POTENTIAL

31. If the electric field points due north, the change in the po-
tential will be (a) greatest in the direction opposite the field,
south; (b) least in the direction of the field, north; and (c)
zero in a direction perpendicular to the field, east and west.

32. In the vicinity of a pointy protrusion, the electric field can


be very high. This can lead to a spark inside an electronic
device which can make the device to stop functioning.

33. Yes. In regions of space where the equipotential lines are


closely spaced, the electric field is stronger than in regions
of space where the equipotential lines are farther apart.

34. If the electric field in a region of space is uniform, then you Figure 2.164
can infer that the electric potential is increasing or decreas-
ing uniformly in that region. For example, if the electric
field is 10 V/m in a region of space then you can infer that 40. (a) The electric field vector points in the direction of de-
the potential difference between two points 1 meter apart creasing potential. Therefore Va > Vb .
(measured parallel to the direction of the field) is 10 V. If (b) |∆Vab | < |∆Vcd | < |∆Vef | . For a uniform electric field
the electric potential in a region of space is uniform, then |∆V |= |−E∆s|= |E∆s|. The order is determined by the
you can infer that the electric field there is zero. fact that ∆sab < ∆scd < ∆sef
(c) Surface 1 is an equipotential surface because it is per-
35. The electric potential energy of two unlike charges is neg- pendicular to the electric field vectors. Surface 2 is not per-
ative. The electric potential energy of two like charges is pendicular to the electric field so it is not an equipotential
positive. In the case of unlike charges, work must be done surface.
to separate the charges. In the case of like charges, work
must be done to move the charges together. 41. Equal. When a conductor is in electrostatic equilibrium,
the entire conductor is at the same potential.
36. The electric field is the negative of the slope of the V vs. x
graph. 42. (a) V = V . Both spheres and the wire become one con-
1 2
ductor and so are all at the same potential.
(b) Since V1 = V2 , 4π1 Q1
0 r1
1 Q2
= 4π 0 r2
, thus Q
r1 = r2 . Since
1 Q2

r1 > r2 , Q1 > Q2 .
(c) Recall E1 = 4π 1 Q1
2 = r
0 r1
V1
1
and E2 = 4π 1 Q2
0 r2
2 = r . Since
V2
2
r1 > r2 , E1 < E2 .

43. APPROACH : The potential difference is the negative


of the area under the Ex vs. x curve.
Solve: The potential difference between x = 1.0 m and
x = 3.0 m (Fig.2.96) is-

1
∆V = − (200 V)(3.0 m − 1.0 m) = −200 V
Figure 2.163 2
The potential difference is negative since the electric field
points in the direction of decreasing potential.
37. The electric field is the negative of the slope of the V vs. x
graph. If V = constant in some region then E = 0 in that 44. APPROACH The potential difference is the negative of
region. the area under the Ex vs. x curve.
SOLUTION The potential difference between the origin
38. The electron moves to the right. The electric field is the
and x = 3.0 m is
negative of the slope of the electric potential, so the electric
field will be nonzero at x = 2 m and point to the left. The ∆V = V (x = 3.0 m) − V (x = 0.0 m)
electron will therefore experience a net force to the right  
1 1
and accelerate to the right. = − (−100 V)(1.0 m − 0 m) + (200 V)(3.0 m − 1.0 m)
2 2
39. The electron moves to the right. The electric field is the = −150 V
negative of the slope of the electric potential, so the electric
field will be nonzero at x = 2 m and point to the left. The Thus, V (3.0 m) = V (0 m) − 150 V = −50 V − 150 V =
electron will therefore experience a net force to the right −200 V = −0.20kV
and accelerate to the right. The potential decreases from the origin to x = 3.0 m.
2.16. ANSWER KEYS AND SOLUTIONS 259

45. APPROACH The electric field points in the direction of potential is not changing.
decreasing potential and is perpendicular to the equipo-
tential lines. (b) Now, refer to Figure 2.100(b) There are three regions
SOLUTION Please refer to Figure 2.98. The three of different slope. For 0 cm ≤ x < 2 cm,
equipotential surfaces correspond to potentials of
0 V, 200 V, and 400 V. ∆V −50 V − 0 V
=
The electric field component along a direction of constant ∆x 0.02 m − 0 m
potential is Es = −dV /ds = 0 V/m. But, the electric field = −2500 V/m
component perpendicular to the equipotential surface is
⇒ Ex = 2500 V/m
~ ∆V 400 V For 2 cm ≤ x
|E|= = = 20 kV/m
∆s 0.02 m
∆V 50 V − (−50 V)
The direction of the electric field vector is "downhill," per- = = 10, 000 V/m
∆x 0.03 m − 0.02 m
pendicular to the equipotential surfaces. That is, the elec-
tric field is 20 kV/m downward or E ~ = −(20ĵ) kV/m ⇒ Ex = −10, 000 V/m

46. APPROACH The electric field is perpendicular to the


equipotential lines and points "downhill."
SOLUTION Refer to Figure 2.99. Three equipotential
surfaces at potentials of −200V , 0 V, and 200 V are shown.
The electric field is perpendicular to the equipotential sur-
faces:
∆V 400 V
E=− =− = −20kV/m
∆s 0.02 m
The electric field vector is in the third quadrant, 45◦ below
the negative x-axis. That is,

~ = (20kV/m, 45◦ below − x-axis )


E
Figure 2.166
In component
 form,
 this would be
~ 20 20
E = − 2 î − 2 ĵ kV/m.
√ √ ∆V /∆x and Ex are the negative of each other.

47. APPROACH The electric field is the negative of the slope 48. The electric potential difference ∆V between two points in
of the graph of the potential function. a uniform electric field is
SOLUTION (a) Please refer to Figure 2.100(a). There
are three regions of different slope. For 0 cm ≤ x < 10 cm
and 20 cm ≤ x < 30 cm,

∆V
= 0 V/m ⇒ Ex = 0 V/m
∆x
Figure 2.167
For 10 cm ≤ x < 20 cm,
Z
∆V 100 V − (−100 V)
= = 2000 V/m V (xf ) − V (xi ) = − Ex dx = −Ex (xf − xi )
∆x 0.20 m − 0.10 m

⇒ Ex = −2000 V/m Choosing xi = −1.0 m and xf = +1.0 m,

Because Es = −dV /ds, the electric field is zero where the +1000 V − (−1000 V) = −Ex [1.0 m − (−1.0 m)]
⇒ Ex = −1.0kV/m

Alternatively, xi = −1.0 m and xf = −1.0 m. For this


choice,

−1000 V − (+1000 V) = −Ex [−1.0 m − (1.0 m)]


⇒ Ex = −1.0kV/m

The choice of initial and final positions does not change the
physical nature of the electric field or the potential differ-
Figure 2.165 ence.
260 CHAPTER 2. ELECTRIC POTENTIAL

49. APPROACH The electric field is the negative of the SOLUTION Thus, for σ = 0.10 × 10−6 C/m2 and ∆V =
derivative of the potential function. 50 V, we have
SOLUTION (a) The component of the electric field in the
2ε0 ∆V
s-direction is Es = −d V/ds. For the given potential, ∆x =
σ
2 8.85 × 10−12 C2 /N · m2 (50 V)

dV d
100x2 V = 200x V/m = 8.8 × 10−3 m

= =
dx dx 0.10 × 10−6 C/m2
⇒ Ex = −200x V/m
Note: Equipotential surfaces are always perpendicular to
(b) At x = 1 m, Ex = −200 × 1 V/m = −0.2kV/m. the electric field lines.
The potential increases with x, so the electric field must 3. (a) VB − VA = ∆U/q = −W/(−e)
point in the −x-direction. = − 3.94 × 10−19 J / −1.60 × 10−19 C
 

= 2.46 V.
50. (a) Since Ex = −dV /dx, we have
(b) VC − VA = VB − VA = 2.46 V.
d (c) VC − VB = 0 (since C and B are on the same
100e−2x V/m = 200e−2x V/m equipotential line).

Ex = −
dx
4. We connect A to the origin with a line alongR the y axis,
At x = 1.0 m, ~ · d~s = 0
along which there is no change of potential ( E
  ). Then, we connect the origin to B with a line along the
Ex = 200e−2(1.0 m) V/m = 27 V/m
x axis, along which the change in potential is
Z 4
(b) At x = 2.0 m,
Z x=4  2
~ · ds = −4.00 4
∆V = − E xdx = −4.00
  0 0 2
Ex = 200e−2(2.0 m) V/m = 3.7 V/m
which yields VB − VA = −32.0 V.
5. (a) The change in potential is the negative of the "area"
2.16.2 Problems
under the curve. Thus, using the area-of-a-triangle formula,
1. (a) We follow the path from (0, 0) to (20.0 cm, 0) to we have
(20.0 cm, 50.0 cm). Z x=2
V − 10 = − ~ · d~s = 1 (2)(20)
E
∆U = −( work done ) 0 2
which yields V = 30 V. (b) For any region within 0 <
∆U = −( work from origin to (20.0 cm, 0)) − ( work from R
~ · d~s is positive, but for any region for which
x < 3 m, − E
(20.0 cm, 0) to (20.0 cm, 50.0 cm)) Note that the last term
x > 3 m it is negative. Therefore, V = Vmax occurs at
is equal to 0 because the force is perpendicular to the dis-
x=3m
placement.
Z x=3
−6
 −4 V − 10 = − E~ · ds = 1 (3)(20)
∆U = − (qEx ) ∆x = − 12.0 × 10 C (250 V/m)(0.200 m) = −6.00×10 J 2
0

(b) ∆V = ∆U −4
= − 6.00×10 J which yields Vmax = 40 V. (c) In view of our result in part
q 12.0×10−6 C = −50.0 J/C = −50.0 V
(b), we see that now (to find V = 0 ) we are looking for
2. APPROACH The electric field produced by an infinite some X > 3 m such that the "area" from x = 3 m to x = X
sheet of charge is normal to the sheet and is uniform. The is 40 V. Using the formula for a triangle (3 < x < 4) and a
magnitude of the electric field produced by the infinite sheet rectangle (4 < x < X), we require
of charge is E = σ/2ε0 , where σ is the surface charge den-
1
sity. Place the origin of a coordinate system at the sheet (1)(20) + (X − 4)(20) = 40
and take the x axis to be parallel to the field and positive 2
in the direction of the field. Then the electric potential is Therefore, X = 5.5 m.
6. (a) The charge on the sphere is
Z x
V = Vs − Edx = Vs − Ex
0 (200 V)(0.15 m)
q = 4πε0 V R = = 3.3 × 10−9 C
where Vs is the potential at the sheet. The equipotential 8.99 × 109 N · m2 /C2
surfaces are surfaces of constant x; that is, they are planes
(b) The (uniform) surface charge density (charge divided
that are parallel to the plane of charge. If two surfaces are
by the area of the sphere) is
separated by ∆x then their potentials differ in magnitude
by q 3.3 × 10−9 C
∆V = E∆x = (σ/2ε0 ) ∆x σ = 2
= = 1.2 × 10−8 C/m2
4πR 4π(0.15 m)2
2.16. ANSWER KEYS AND SOLUTIONS 261

|∆V | 25.0 × 103 J/C 2


m ddt2x . Let x0 = x − QE
k , or x = x + k , so the equation
0 QE
7. E = = = 1.67 × 106 N/C =
d 1.50 × 10−2 m of motion becomes:
1.67MN/C
d2 (x + QE/k) d2 x0
   
QE k
8. (a) |∆V |= Ed = 5.90 × 103 V/m (0.0100 m) = 59.0 V −k x + 0
+QE = m , or =− x

k dt2 dt 2 m
(b)
1 1 This is the equation for simple harmonic motion ax0 =
mvf2 = |q∆V |: 9.11 × 10−31 vf2 = 1.60 × 10−19 (59.0) −ω 2 x0 with
 
2 2 r
6 k
vf = 4.55 × 10 m/s ω=
m
The period of the motion is then T = 2π k.
pm
ω = 2π
RB RC RB
9. VB − VA = − A E · ds = − A E · ds − C E · ds VB − VA =
0.500 0.400 (d)
(−E cos 180◦ ) −0.300 dy−(E cos 90.0◦ ) −0.200 dx VB −VA =
R R

(325)(0.800) = +260 V (K + U + U ) + ∆E = (K + U + U )
s e i mech s e f
1
0 + 0 + 0 − µk mgxmax = 0 + kx2max − QExmax
2
2 (QE − µk mg)
xmax =
k

12. Arbitrarily take V = 0 at the initial point. Then at distance


d downfield, where L is the rod length, V = −Ed and
Ue = −λLEd. (a)

(K + U )i = (K + U )f
Figure 2.168 1
0 + 0 = µLv 2 − λLEd
2
10. Assume the opposite. Then at some point A on some
s
2λEd
equipotential surface the electric field has a nonzero compo- v=
µ
nent Ep in the plane of the surface. Let a test charge starts
from point A and moves some distance on the surface in the
s
2 (40.0 × 10−6 C/m) (100 N/C)(2.00 m)
RB
direction of the field component. Then ∆V = − A Ẽ · d~s is = = 0.400 m/s
(0.100 kg/m)
nonzero. The electric potential charges across the surface
and it is not an equipotential surface. The contradiction (b) The same.
shows that our assumption is false, that Ep = 0, and that
the field is perpendicular to the equipotential surface.
13. Arbitrarily take V = 0 at point P . Then the potential at
11. (a) Arbitrarily choose V = 0 at 0 . Then at other points ~ · ~s = −EL cos θ.
the original position of the charge is −E
V = −Ex and Ue = QV = −QEx. At the final point a, V = −EL. Suppose the table is fric-
tionless:
(K + U )i = (K + U )f
1
0 − qEL cos θ = mv 2 − qEL
r 2
2qEL(1 − cos θ)
v=
m
s
2 (2.00 × 10−6 C) (300 N/C)(1.50 m) (1 − cos 60.0◦ )
=
Figure 2.169 0.0100 kg
= 0.300 m/s
Between the endpoints of the motion, (K + Us + Ue )i =
(K + Us + Ue )f 0+0+0 = 0+ 12 kx2max −QExmax so xmax = 14. (a) The potential at 1.00 cm is V1 = ke q =
r
2QE (8.99×109 N·m2 /C2 )(1.60×10−19 C) −7
k
1.00×10−2 m = 1.44 × 10 V
(b) At equilibrium, (b) The potential at 2.00 cm is V2 = ke rq =
X (8.99×109 N·m2 /C2 )(1.60×10−19 C)
Fx = −Fs + Fe = 0 or kx = QE 2.00×10−2 m = 0.719 × 10−7 V. Thus,
the difference in potential between the two points is ∆V =
So the equilibrium position is at x = Pk .
QE
V2 − V1 = −7.19 × 10−8 V.
(c) The block’s equation of motion is Fx = −kx + QE = (c) The approach is the same as above except the charge
262 CHAPTER 2. ELECTRIC POTENTIAL

is −1.60 × 10−19 C. This changes the sign of each answer, a distance of a/2 from the center:
with its magnitude remaining the same.
That is, the potential at 1.00 cm is −1.44 × 10−7 V. 1 4q2 1 4q2
V = +
The potential at 2.00 cm is −0.719 × 10−7 V, so ∆V = 4πε0 a/2 4πε0 a/2
V2 − V1 = 7.19 × 10−8 V. 16 8.99 × 109 N · m2 /C2 6.00 × 10−12 C
 
16q2
= =
4πε0 a 0.39 m
15. (a) The potential difference is
= 2.21 V
q q
VA − VB = −
4πε0 rA 4πε0 rB 18. A charge −5q is a distance 2d from P , a charge −5q is a
= 1.0 × 10−6 C 8.99 × 109 N · m2 /C2 distance d from P , and two charges +5q are each a distance
 

1 1
 d from P , so the electric potential at P is

2.0 m 1.0 m q

1 1 1 1

= −4.5 × 103 V V = − − + +
4πε0 2d d d d
8.99 × 109 N · m2 /C2 5.00 × 10−15 C
 
q
(b) Since V (r) depends only on the magnitude of ~r, the = =
result is unchanged. 8πε0 d 2 (4.00 × 10−2 m)
= 5.62 × 10−4 V
16. APPROACH The electric potential for a spherically sym-
metric charge distribution falls off as 1/r, where r is the The zero of the electric potential was taken to be at infinity.
radial distance from the center of the charge distribution.
19. 19. First, we observe that V (x) cannot be equal to zero for
The electric potential V at the surface of a drop of charge x > d. In fact V (x) is always negative for x > d. Now we
q and radius R is given by V = q/4πε0 R. consider the two remaining regions on the x axis: x < 0
SOLUTION (a) With V = 500 V and q = 30 × 10−12 C, and 0 < x < d (a) For 0 < x < d we have d1 = x and
we find the radius to be d2 = d − x. Let
8.99 × 109 N · m2 /C2 30 × 10−12 C
 
q
   
q1 q2 q 1 −3
R= = V (x) = k + = + =0
4πε0 V 500 V d1 d2 4πε0 x d − x
= 5.4 × 10−4 m
and solve: x = d/4. With d = 24.0 cm, we have x =
(b) After the two drops combine to form one big drop, the 6.00 cm. (b) Similarly, for x < 0 the separation between
total volume is twice the volume of an original drop, so the q1 and a point on the x axis whose coordinate is x is given
3
radius R0 of the combined drop is given by (R0 ) = 2R3 and by d1 = −x; while the corresponding separation for q2 is
R = 2 R. The charge is twice the charge of the original
0 1/3 d2 = d − x. We set
drop: q 0 = 2q. Thus,    
q1 q2 q 1 −3
V (x) = k + = + =0
1 q0 1 2q d1 d2 4πε0 −x d − x
V0 = = = 22/3 V = 22/3 (500 V)
4πε0 R0 4πε0 21/3 R
to obtain x = −d/2. With d = 24.0 cm, we have x =
≈ 790 V
−12.0 cm.
Note: A positively charged configuration produces a posi- 20. Since according to the problem statement there is a point
tive electric potential, and a negatively charged configura- in between the two charges on the x axis where the net
tion produces a negative electric potential. Adding more electric field is zero, the fields at that point due to q1 and
charge increases the electric potential. q2 must be directed opposite to each other. This means
that q1 and q2 must have the same sign (i.e., either both
17. The electric potential is given by- are positive or both negative). Thus, the potentials due to
either of them must be of the same sign. Therefore, the net
1 q
V = (2.69) electric potential cannot possibly be zero anywhere except
4πε0 r2
at infinity.
In applying Eq.2.69, we are assuming V → 0 as r → ∞. All
corner particles are equidistant from the center, and since 21. (a) Since the charges are equal and placed symmetrically,
their total charge is F = 0 (b) Since F = qE = 0, E = 0.
(c)
2q1 − 3q1 + 2q1 − q1 = 0
 2.00 × 10−6 C
 
q
V = 2ke = 2 8.99 × 109 N · m2 /C2
then their contribution to Eq.2.69 vanishes. The net poten- r 0.800 m
tial is due, then, to the two +4q2 particles, each of which is 4
V = 4.50 × 10 V = 45.0kV
2.16. ANSWER KEYS AND SOLUTIONS 263

Figure 2.170

22.
|Q|
E=
4π ∈0 r2
Q
V =
4π ∈0 r
|V | 3000 V
r= = = 6.00 m Figure 2.171
|E| 500 V/m
(b)
26.
Q U = U1 + U2 + U3 + U4
V = −3000 V =
4π ∈0 (6.00 m)
U = 0 + U12 + (U13 + U23 ) + (U14 + U24 + U34 )
−3000 V
Q= (6.00 m) = −2.00µC ke Q2 ke Q2 ke Q2
   
1 1
(8.99 × 109 V · m/C) U =0+ + √ +1 + 1+ √ +1
s s 2 s 2
ke Q2 ke Q2
 
23. (a) Ex = kxe q21 + (x−2.00)
ke q2
= 0 becomes Ex = 2
2 U= 4+ √ = 5.41

+q −2q
 s 2 s
ke x2 + (x−2.00)2 = 0. Dividing by ke , 2qx2 = q(x −  
2.00)2 x2 + 4.00x − 4.00 = 0. Therefore E = 0 when An alternate way to get the term 4 + √22 is to recognize

−4.00 ± 16.0 + 16.0
x= = −4.83 m
2
(Note that the positive root does not correspond to a phys-
ically valid situation.) (b) V = kexq1 + 2.00−x
ke q2
= 0
 
+q 2q
or V = ke x − 2.00−x = 0. Again solving for x
2qx = q(2.00 − x) For 0 ≤ x ≤ 2.00 V = 0 when
−2q
x = 0.667 m and |x|
q
= |2−x| For x < 0 x = −2.00 m Figure 2.172
24.
that there are 4 side pairs and 2 face diagonal pairs.
X qi
V = k 27. Consider the two spheres as a system. (a) Conservation
i
ri
  of momentum: 0 = m1 v1 î + m2 v2 (−î) or v2 = mm1 v2 1 By
9 −6 −1 1 1
conservation of energy, 0 = ke (−q 1 )q2
 
V = 8.99 × 10 7.00 × 10 − + = 12 m1 v12 + 12 m2 v22 +
0.0100 0.0100 0.0387 ke (−q1 )q2
d
and
V = −1.10 × 107 V = −11.0MV r1 +r2

ke q1 q2 ke q1 q2 1 1 m21 v12
− = m1 v12 +
(5.00×10−9 C)(−3.00×10−9 C)(8.99×109 V·m/C) r1 + r2 d 2 2 m2
25. (a) U = 4π
qQ
0r
= (0.350 m) = s
−3.86×10−7 J The minus sign means it takes 3.86×10−7 J
 
2m2 ke q1 q2 1 1
v1 = −
to pull the two charges apart from 35 cm to a much larger m1 (m1 + m2 ) r1 + r2 d
separation.
(b) On substituting the given value in above expression, we get-
v1 = 10.8 m/s
Q1 Q2 v m 1 v1
2 = m2 =
0.100 kg(10.8 m/s)
= 1.55 m/s
V = + 0.700 kg
4π0 r1 4π0 r2
(b) If the spheres are metal, electrons will move around
5.00 × 10−9 C 8.99 × 109 V · m/C
 
= on them with negligible energy loss to place the centers of
0.175 m excess charge on the insides of the spheres. Then just before
−3.00 × 10−9 C 8.99 × 109 V · m/C
 
they touch, the effective distance between charges will be
+
0.175 m less than r1 +r2 and the spheres will really be moving faster
V = 103 V than calculated in (a).
264 CHAPTER 2. ELECTRIC POTENTIAL

28. Consider the two spheres as a system. (a) Conservation of Energy of the system of four charged objects is conserved
momentum: 0 = m1 v1 î + m2 v2 (−î) or as the fourth charge flies away:
m1 v1 
1
 
1

v2 = mv 2 + qV = mv 2 + qV
m2 2 2
i f

0 + 40 × 10 C 3.00 × 103 V
−9
 
ke (−q1 )q2
By conservation of energy, 0 = d = 1 2
2 m 1 v1 +
1 2 ke (−q1 )q2
and 1
2 m 2 v2 + 2.00 × 10−13 kg v 2 + 0

r1 +r2 =
s 2
ke q1 q2 ke q1 q2 1 1 m21 v12 2 (1.20 × 10−4 J)
− = m1 v12 + v= = 3.46 × 104 m/s
r1 + r2 d 2 2 m2 2 × 10−13 kg
s  
2m2 ke q1 q2 1 1
v1 = −
m1 (m1 + m2 ) r1 + r2 d 30. The original electrical potential energy is
s
ke q
   
m1 2m1 ke q1 q2 1 1
v2 = v1 = − Ue = qV = q
m2 m2 (m1 + m2 ) r1 + r2 d d
In the final configuration we have mechanical equilibrium.
(b) If the spheres are metal, electrons will move around
The spring and electrostatic forces on each charge are
on them with negligible energy loss to place the centers of ke q 2
2 = 0. Then k = 18d3 . In the final configura-
ke q
excess charge on the insides of the spheres. Then just before −k(2d) + q (3d)
2
they touch, the effective distance between charges will be tion the total potential energy is 12 kx2 + qV = 1 ke q 2
2 18d3 (2d) +
less than r1 +r2 and the spheres will really be moving faster q k3d
eq
= 4 ke q
2
. The missing energy must have become inter-
than calculated in (a). 9 d
ke q 2 4ke q 2
nal energy, as the system is isolated: d = 9d + ∆Eint
29. (a) In an empty universe, the 20nC charge can be placed
2
5 ke q
∆Eint = 9 d
at its location with no energy investment. At a distance of
4 cm, it creates a potential 31. (a)

ke q1 ke Q1 ke Q2 ke (+Q) ke (+Q)
V1 = V (x) = + =√ +p
r r1 r2 x2 + a2 x2 + (−a)2
8.99 × 109 N · m2 /C2 20 × 10−9 C
 
!
= = 4.50kV 2ke Q ke Q 2
0.04 m V (x) = √ = p
x2 + a2 a (x/a)2 + 1
To place the 10 -nC charge there we must put in energy
V (x) 2
=p
U12 = q2 V1 = 10 × 10−9 C 4.5 × 103 V = 4.50 × 10−5 J (ke Q/a)
 
(x/a)2 + 1

Next, to bring up the −20-nC charge requires energy

U23 + U13 = q3 V2 + q3 V1 = q3 (V2 + V1 )


= −20 × 10−9 C 8.99 × 109 N · m2 /C2


10 × 10−9 C 20 × 10−9 C
 
+
0.04 m 0.08 m
= −4.50 × 10−5 J − 4.50 × 10−5 J
= −9.0 × 10−5 J

The total energy of the three charges is


Figure 2.173
U12 + U23 + U13 = −4.50 × 10−5 J

(b) The three fixed charges create this potential at the lo- (b)
cation where the fourth is released:
ke Q1 ke Q2 ke (+Q) ke (−Q)
V = V1 + V2 + V3 V (y) = + = +
r1 r2 |y − a| |y + a|
= 8.99 × 109 N · m2 /C2
  
ke Q 1 1

20 × 10−9 10 × 10−9 20 × 10−9
 V (y) = −
√ + − C/m a |y/a − 1| |y/a + 1|
0.042 + 0.032 0.03 0.05 V (y)

1 1

= −
V = 3.00 × 103 V (ke Q/a) |y/a − 1| |y/a + 1|
2.16. ANSWER KEYS AND SOLUTIONS 265

h q(−2q) (−2q)(3q) (2q)(3q) q(2q) q(3q)


U = ke + + + +√
b a b a a 2 + b2
2q(−2q) i
+√
a 2 + b2
 
2 −2 6 6 2 3 4
U = ke q − + + + −
0.400 0.200 0.400 0.200 0.447 0.447
 
9
 −6 2
 4 4 1
Figure 2.174 U = 8.99 × 10 6.00 × 10 − −
0.400 0.200 0.447
= −3.96 J

ke Q
32. V = r
(8.99×109 N·m2 /C2 )(8.00×10−9 C) 36. Each charge moves off on its diagonal line. All charges have
so r = kVe Q = V = 72.0 V·m
V equal speeds.

For V = 100 V, 50.0 V, and 25.0 V, r = 0.720 m, 1.44 m,


X X
(K + U )i = (K + U )f
and 2.88 m. The radii are inversely proportional to the
4ke q 2 2ke q 2 4ke q 2 2ke q 2
 
potential. 1 2
0+ + √ =4 mv + + √
L 2L 2 2L 2 2L
2
 
1 ke q
2+ √ = 2mv 2
33. Using conservation of energy for the alpha particle-nucleus 2 L
ke qα qgold
s
system, we have Kf +Uf = Ki +Ui But Ui = ke q 2

ri
1
v= 1+ √
and ri ≈ ∞. Thus, Ui = 0. Kf = 0 (vf = 0 at turning 8 mL
ke qα qgold
point), so Uf = Ki or rmin = 21 mα vα2
2
2ke qα qgold 2(8.99×109 N·m2 /C2 )(2)(79)(1.60×10−19 C)
rmin = m 2
α vα
= (6.64×10−27 kg)(2.00×107 m/s)2
=37. A cube has 12 edges and 6 faces. Consequently, there are
−14
2.74 × 10 m = 27.4fm 12 edge pairs separated
√ by s, 2 × 6 = 12 face diagonal pairs
separated
√ by 2s and 4 interior diagonal pairs separated
3s.
34. Using conservation of energy we have: ke eQ
ke q 2 ke q 2
 
= 12 4
r1 U = 12 + √ + √ = 22.8
s s
r
2ke eQ
  2 3
ke qQ
r2 + 1
2 mv 2
which gives: v = m
1
r1 − 1
r2 or v =
q
(2)(8.99×109 N·m2 /C2 )(−1.60×10−19 C)(10−9 C) 1 1
m We can use Wq→ final position = Q∆Vi→f to find the work

9.11×10−31 kg 0.0300 m − 0.020038.
Thus, v = 7.26 × 106 m/s required to move these charges between the given points.

(a) Express the required work in terms of the charge being


moved and the potential due to the charge at x = +a and
simplify to obtain:
P ke qi qj
35. U = rij , summed over all pairs of (i, j) where i 6= j.
W+Q→+a = Q∆V∞→+a = Q[V (a) − V (∞)]
kQ2
 
kQ
= QV (a) = Q =
2a 2a
(b) Express the required work in terms of the charge being
moved and the potentials due to the charges at x = +a and
x = −a and simplify to obtain:

W−Q→0 = −Q∆V∞→0 = −Q[V (0) − V (∞)]


= −QV (0) = −Q [Vcharge + Vcharge ]
−2kQ2
 
kQ kQ
= −Q + =
at a a

(c) Express the required work in terms of the charge being


Figure 2.175 moved and the potentials due to the charges at x = +a and
266 CHAPTER 2. ELECTRIC POTENTIAL

x = −a and simplify to obtain: 41. APPROACH The torque on an electric dipole in an


~ and the potential energy
electric field is given by ~τ = p~ × E
W−Q→2a = −Q∆V0→2a −

of the dipole by U = −~ p · E.
= −Q[V (2a) − V (0)]
= −Q [Vcharge + Vcharge − V (0)] Using its definition, express the torque on a dipole moment
in a uniform electric field:
 
kQ kQ 2kQ
= −Q + − −

3a a a −
→τ =− →p × E ⇒ τ = pE sin θ
2
2kQ
= where θ is the angle between the electric dipole moment
3a
and the electric field.
39. Let q represent the charge being moved from x = 50.0 cm SOLUTION (a) Evaluate τ for θ = 0◦ :
to the origin, Q the ring charge, and a the radius of the
ring. We can use Wq→ final position = q∆Vi→f , where V is τ (0◦ ) = pE sin 0◦ = 0
the expression for the axial field due to a ring charge, to (b) Evaluate τ for θ = 90◦ :
find the work required to move q from x = 50.0 cm to the
τ (90◦ ) = (0.50e · nm) 4.0 × 104 N/C sin 90◦

origin.
Express the required work in terms of the charge being = 3.2 × 10−24 N · m
moved and the potential due to the ring charge at x =
(d) Using its definition, express the potential energy of a
50.0 cm and x = 0 :
dipole in an electric field:
W = q∆V = q[V (0) − V (0.500 m)] −

U = −− →
p · E = −pE cos θ
The potential on the axis of a uniformly charged ring is Evaluate U for θ = 0◦ :
given by:
U (0◦ ) = −(0.50e · nm) 4.0 × 104 N/C cos 0◦

kQ
V (x) = √
x2 + a 2 = −3.2 × 10−24 J

At x = 50.0 cm : Evaluate U for θ = 30◦


U (30◦ ) = −(0.50e · nm) 4.0 × 104 N/C cos 30◦

kQ
V (0.500 m) = p
(0.500 m)2 + a2 = −2.8 × 10−24 J

At x = 0 : 42. ∆V = −14.0 V and Q = −NA e =


− 6.02 × 1023 1.60 × 10−19 = −9.63 × 104 C


kQ kQ ∆V = W
Q, so W = Q∆V =
V (0) = √ = 4

−9.63 × 10 C (−14.0 J/C) = 1.35MJ
a2 a

Substituting for V (0) and V (0.500 m) yields:


43. ∆K = q|∆V | ⇒ 7.37 × 10−17 = q(115)
q = 6.41 × 10−19 C
" #
kQ kQ
W =q −p
a (0.500 m)2 + a2 44. (a) Energy of the proton-field system is conserved as the
" # proton moves from high to low potential, which can be de-
1 1 fined for this problem as moving from 120 V down to 0 V.
= kQq −p
a (0.500 m)2 + a2
Ki + Ui + ∆Emech = Kf + Uf
Substitute numerical values and evaluate W : 1
0 + qV + 0 = mvp2 + 0
 2
 2
9N · m 1
W = 8.988 × 10 (2.00nC)(1.00nC)
1.60 × 10−19 C (120 V) = 1.67 × 10−27 kg vp2
 
C2
" # 2
1 1 vp = 1.52 × 105 m/s
× −p
0.100 m (0.500 m)2 + (0.100 m)2 (b) The electron will gain speed in moving the other way,
= 1.445 × 10−7 J = 1.4 × 10−7 J from Vi = 0 to Vf = 120 V : Ki +Ui +∆Emech = Kf +Uf
1eV 1
= 1.445 × 10−7 J × 0+0+0= mv 2 + qV
1.602 × 10−19 J 2 e
= 9.0 × 1011 eV 1
9.11 × 10−31 kg ve2 + −1.60 × 10−19 C (120 J/C)
 
0=
2
40. ve = 6.49 × 106 m/s
2.16. ANSWER KEYS AND SOLUTIONS 267

45. 51. The potential is


W = ∆K = −q∆V
1 dq 1 −Q
Z Z
1 2
VP = = dq =
9.11 × 10−31 kg 4.20 × 105 m/s

0− 4πε0 rod R 4πε0 R rod 4πε0 R
2
= − −1.60 × 10−19 C ∆V 8.99 × 109 N · m2 /C2 25.6 × 10−12 C
  
=−
From which, ∆V = −0.502 V. 3.71 × 10−2 m
= −6.20 V
46. V = a + bx = 10.0 V + (−7.00 V/m)x (a) At x =
0, V = 10.0 V At x = 3.00 m, V = −11.0 V At We note that the result is exactly what one would expect
x = 6.00 m, V = −32.0 V (b) E = − dV dx = −b = for a point-charge −Q at a distance R. This "coincidence"
−(−7.00 V/m) = 7.00 N/C in the +x direction is due, in part, to the fact that V is a scalar quantity.
47. (a) For r < R
ke Q 52. (a) All the charge is the same distance R from C, so the
V = electric potential at C is
R
dV 1

Q1 6Q1

5Q1
Er = − =0 V = − =−
dr 4πε0 R R 4πε0 R
(b) For r ≥ R
5 8.99 × 109 N · m2 /C2 4.20 × 10−12 C
 
ke Q =− = −2.30 V
V = 8.20 × 10−2 m
r
where the zero was taken to be at infinity. (b) All√the charge
 
dV ke Q ke Q
Er = − =− − 2 = 2 is the same distance from P . That distance is R2 + D2 ,
dr r r
so the electric potential at P is
48. V = 5x − 3x2 y + 2yz 2 Evaluate E at (1, 0, −2)  
1 Q1 6Q1 5Q
∂V V = √ −√ =− √ 1
Ex = − = −5 + 6xy = −5 + 6(1)(0) = −5 4πε0 2
R +D 2 2
R +D 2 4πε0 R2 + D2
∂x 9
5 8.99 × 10 N · m /C2 2
 −12
4.20 × 10 C

∂V =− q
Ey = − = +3x2 − 2z 2 = 3(1)2 − 2(−2)2 = −5 2 2
∂y (8.20 × 10−2 m) + (6.71 × 10−2 m)
∂V = −1.78 V
Ez = − = −4yz = −4(0)(−2) = 0
q ∂z p
E = Ex2 + Ey2 + Ez2 = (−5)2 + (−5)2 + 02 = 7.07 N/C 53. Letting d denote 0.010 m, we have

49. (a) EA > EB since E = ∆V


(b) EB = − ∆V Q1 3Q1 3Q1 Q1
∆s ∆s = V = + − =
4πε0 d 8πε0 d 16πε0 d 8πε0 d
8.99 × 109 N · m2 /C2 30 × 10−9 C
 
= = 1.3 × 104 V
2(0.01 m)

54. (a) [α] = 1


λ C
 C
x = m · m = m2

Figure 2.176

− (6−2)V
2 cm = 200 N/C down (c) The figure is shown to the
right, with sample field lines sketched in.
50. ∆V = V2R − V0
  Figure 2.177
ke Q ke Q ke Q 1
=p − = √ −1
2
R + (2R) 2 R R 5 RL
(b) dq λdx xdx
R R
V = ke = ke = ke α =
ke Q r r 0 d+x
= −0.553 L
 
R ke α L − d ln 1 + d
268 CHAPTER 2. ELECTRIC POTENTIAL

ke dq √ αxdx
Let z = L
− x Then x = 58. Substituting given values into V = ke q
R R
55. V = r = ke 2 r
b2 +(L/2−x)2
L
− z, and dx = −dz
8.99 × 109 N · m2 /C2 q

2
3
Z
(L/2 − z)(−dz) ke αL
Z
dz 7.50 × 10 V =
V = ke α √ =− √ 0.300 m
b2 + z 2 2 b2 + z 2
Z
zdz Substituting q = 2.50 × 10−7 C,
+ ke α √
b2 + z 2 2.50 × 10−7 C
ke αL  p  p N= = 1.56 × 1012 electrons
=− ln z + z 2 + b2 + ke α z 2 + b2 1.60 × 10−19 C/e−
2
 L
 s 2
59. (a)

ke αL  L L
V =− ln −x + − x + b2 
2 2 2
0 E=0
s L
8.99 × 109 26.0 × 10−6
 
L
2 ke q
V = = = 1.67MV

+ ke α − x + b 2
2 R 0.140

" p # 0 (b)
ke αL L/2 − L + (L/2)2 + b2
V =− ln p
2 L/2 + (L/2)2 + b2 8.99 × 109 26.0 × 10−6
 
ke q
E= = = 5.84MN/C away
r2 (0.200)2
s s  
 2 2
L L
+ ke α  − L + b2 − + b2  8.99 × 109 26.0 × 10−6
 
2 2 ke q
V = = = 1.17MV
" "p ## R 0.200
ke αL b2 + (L2 /4) − L/2
V = − ln p (c)
2 b2 + (L2 /4) + L/2
8.99 × 109 26.0 × 10−6
 
ke q
56. E= 2 = = 11.9MN/C away
R (0.140)2
ke dq
dV = √ where dq = σdA = σ2πrdr ke q
r 2 + x2 V = = 1.67MV
Z b R
rdr hp p i
V = 2πσke √ = 2πke σ x2 + b2 − x2 + a2
a r 2 + x2 60. (a) Both spheres must be at the same potential according
to kre1q1 = kre2q2 where also q1 + q2 = 1.20 × 10−6 C. Then

q2 r1
q1 =
r2
q2 r1
+ q2 = 1.20 × 10−6 C
b r2
a 1.20 × 10−6 C
q2 = = 0.300 × 10−6 C on the smaller sphere
P 1 + 6 cm/2 cm
x q1 = 1.20 × 10−6 C − 0.300 × 10−6 C = 0.900 × 10−6 C

8.99 × 109 N · m2 /C2 0.900 × 10−6 C


 
ke q1
V = =
r1 6 × 10−2 m
5
= 1.35 × 10 V
Figure 2.178
(b) Outside the larger sphere,
57. ke q1 V1 1.35 × 105 V
−R E1 = 2 r̂ = r̂ = r̂ = 2.25×106 V/m away
dq λdx λds
Z Z Z
r1 r1 0.06 m
V = ke = ke + ke
all charge r −3R −x semicircle R
3R Outside the smaller sphere,
λdx
Z
+ ke
R x 1.35 × 105 V
E2 = r̂ = 6.74 × 106 V/m away
−R ke λ 3R 0.02 m
V = − ke λ ln(−x)|−3R + πR + ke λ ln x|R
R
3R The smaller sphere carries less charge but creates a much
V = ke ln
R
+ ke λπ + ke ln 3 = ke λ(π + 2 ln 3) stronger electric field than the larger sphere.
2.16. ANSWER KEYS AND SOLUTIONS 269

61. (a)
ke q 2 ke q 2 1 1 1 1
= + mv 2 + mv 2 + mv 2 + mv 2
a 3a 2 2 2 2
   
ke Q ke Q 1 1
Emax = 3.00 × 106 V/m = 2 = = Vmax 2ke q 2
r r r r = 2mv 2
Vmax = Emax r = 3.00 × 106 (0.150) = 450kV 3ar
ke q 2
v=
3am
o
(b) ke Qmax
r2 = Emax { or ke Qmax
r = Vmax Qmax =
Emax r 2 3.00×106 (0.150)2 66. For an element of area which is a ring of radius r and width
= = 7.51µC
ke 8.99×109 dr, dV = √kr2e +x
dq
2
. dq = σdA = Cr(2πrdr) and

r and E = r 2 . Since E = Vr , (b) r =


ke q ke q V
62. V = E = Z R
r2 dr
6.00×105 V V = C (2πke ) √
3.00×106 V/m = 0.200 m and (a) q = Vker = 13.3µC r 2 + x2
 0p  
x
−(8.99×10 )(1.60×10 −19 2 = C (πke ) R R2 + x2 + x2 ln √
9
) R+ R 2 + x2
63. (a) U = ke q1 q2
r = 0.0529×10−9 =
−4.35 × 10−18 J = −27.2eV (b) U = k q q
e 1 2
=
2
r 67. APPROACH The work done by the electric field of the
−(8.99×109 )(1.60×10−19 )
2 −9
2 (0.0529×10 ) = −6.80eV (c) U = ke q1 q2
r = gold nucleus changes the kinetic energy of the proton. We
−ke e2 can apply the work-kinetic energy theorem to derive an
=0
∞ expression for the speed of the proton as a function of
its distance from the gold nucleus. Because the repulsive
64. The potential created by the ring at the electron’s starting Coulomb force F~e varies with distance, we’ll have to eval-
point is ~
uate Fe · d~r in order to find the work done on the proton
R
ke Q ke (2πλa) by this force.
Vi = p 2 =p 2
xi + a2 xi + a2 SOLUTION Apply the work-kinetic energy theorem to
the proton to obtain:
while at the center, it is Vf = 2πke λ. From conservation of
energy, Z ∞


Wnet = F e · dr̂ = ∆K
I0
1
0 + (−eVi ) = me vf2 + (−eVf ) ∞ Z ∞
−→ ke(79e)
Z
2 or, because F e · dr̂ = dr and
!
r0 r0 r2
2e 4πek e λ a
vf2 = (Vf − Vi ) = 1− p 2 Ki = 0,
me me xi + a 2 Z ∞
2 dr 1
−19 9 −7 79ke = mp vf2
  
2 4π 1.60 × 10 8.99 × 10 1.00 × 10 2
vf = i0 r 2
9.11 × 10−31
Evaluating the integral yields:
!
0.200
1− p
2
(0.100) + (0.200) 2  ∞
2 1 79ke2 1
p −79ke = = mp vf2
vf = 1.45 × 10 m/s 7 r r0 r0 2

Solve for vf and simplify to obtain:


65. Take the illustration presented with the problem as an ini-
tial picture. No external horizontal forces act on the set of
s s
158ke2 158k
four balls, so its center of mass stays fixed at the location vf = =e
mp r0 mp r0
of the center of the square. As the charged balls 1 and 2
swing out and away from each other, balls 3 and 4 move Substitute numerical values and evaluate vf :
up with equal y-components of velocity. The maximum-
kineticenergy point is illustrated. System energy is con- vf = 1.602 × 10−19 C ×

served: s
2
158 8.988 × 109 N·m
C2
v v (1.673 × 10−27 kg) (1.00 × 10−13 m)
1 2 = 1.48 × 107 m/s
+ +
CM
3 4 68. APPROACH The work done by the electric field of
v v the gold nucleus changes the kinetic energy of the alpha
particle-eventually bringing it to rest. We can apply the
Figure 2.179 work-kinetic energy theorem to derive an expression for
270 CHAPTER 2. ELECTRIC POTENTIAL

the distance of closest approach. Because the repulsive Level 4


Coulomb
Z force F~e varies with distance, we’ll have to evalu- Section A
Q.No. 1 2 3 4 5 6 7 8 9
ate F~e · d~r in order to find the work done on the alpha Ans. A B D B C A B D D
Q.No. 10 11 12 13 14 15 16 17 18
particles by this force. Ans. D C C D none B D D B
Q.No. 19 20 21 22 23 24 25 26 27
SOLUTION Apply the work-kinetic energy theorem to Ans. C D C B D B B D B
Q.No. 28 29 30 31 32 33 34 35 36
the alpha particle to obtain: Ans. B D C B A C D C D
Q.No. 37 38 39 40
rmin Ans. A B D B


Z
Wnet = F e · d−

r = ∆K Section B
Q.No. 1 2 3 4 5 6 7 8

Ans. B A A D D B B B
or, because Q.No. 9 10 11 12 13 14 15 16
Z rmin rmin Ans. C C A B ABCD C B C

→ k(2e)(79e)
Z
F e · d−

r =− dr and Q.No. 17 18 19 20 21 22 23 24
∞ ∞ r2 Ans. D CD AB 2 6 2 1.73 3.00

Kf = 0
rmin
dr
Z
− 158ke2 = −Ki
∞ r2

Evaluating the integral yields:


 rmiin
1 158ke2
−158ke2 =− = −Ki
r ∞ rmin

Solve for rmin and simplify to obtain:

158ke2
rmin =
Ki

Substitute numerical values and evaluate rmin :


 2
 2
158 8.988 × 109 N·m
C2 1.602 × 10−19 C
rmin = 1.602×10−19 J
5.0Mev × eV
= 4.6 × 10−14 m

2.16.3 Multiple Choice Assignments


Level 1
Q.No. 1 2 3 4 5 6 7 8 9
Ans. A B A C C A B B D
Q.No. 10 11 12 13 14 15 16 17 18
Ans. D B A B A C B D A
Q.No. 19 20 21 22 23 24 25 26 27
Ans. D D B A A C A B B
Q.No. 28 29 30 31 32 33 34 35 36
Ans. A B A B A C B A C
Q.No. 37 38 39
Ans. B B B

Level 2
Q.No. 1 2 3 4 5 6 7 8 9
Ans. B B C D B C B D B
Q.No. 10 11 12 13 14 15 16 17 18
Ans. B D C B D A C

Level 3
Q.No. 1 2 3 4 5 6 7 8 9
Ans. A A B B B B A B A
Q.No. 10 11 12 13 14 15 16 17 18
Ans. A C A C B D B C B
Q.No. 19 20 21 22 23 24 25 26 27
Ans. C A A C

You might also like